You are on page 1of 397

https://yo

utube.co
m/channe
l/UCYa4_J
rOrf8R5Kz
2uOt
https://www.amazo
Click
n.in/dp/B09Q8THP9
Here
1?ref=myi_title_dp
Click
Here
https://unaca
FOLLOW
demy.app.link/
LOOsHfChfkb>
https://unacademy
.com/goal/bank-
exams/RTPSX/subs
SUBSCRIBE
cribe?referral_cod
e=TT10
https://unac
ademy.com/
goal/bank-
exams/RTPS
X/subscribe
?referral_co
de=TT10
1 - 20

21 - 40

41 - 50

51 - 65

66 - 171

172- 237

238 - 375

376 - 386
What will come in the place of question mark (?) in the given expression?
दिए गए व्यंजक में प्रश्नवाचक चचन्ह (?) के स्थान पर क्या आएगा?

1. (?) 2 + 13.053 = 48.982 - 7.95


(A) 15 (B) 14 (C) 17 (D) 16 (E) None of these

2. (?) = 84.92% of 780.05 + 19.98% of 689.99


(A) 819 (B) 822 (C) 801 (D) 831 (E) None of these

3. (?) 2 - 152 = 192 + 212 - 172 - 32


(A) 27 (B) 26 (C) 28 (D) 25 (E) None of these

4. (?) + 576 ÷ 8 - 503 = 564 ÷ 12


(A) 467 (B) 482 (C) 478 (D) 499 (E) None of these

5. (?) = (9 × 32 + 62 - 8) × 2
(A) 218 (B) 214 (C) 216 (D) 212 (E) None of these

6. (?) = 4.98% of 659.89 + 34.98% of 699.95


(A) 241 (B) 213 (C) 278 (D) 298 (E) None of these

7. (?) = 37.96 × 29.02 + 20.98 × 13.04


(A) 1256 (B) 1375 (C) 1145 (D) 1076 (E) None of these

8. (?) + 228.05 = 1215.99 ÷ 37.94 × 17.98


(A) 412 (B) 187 (C) 246 (D) 348 (E) None of these

9. (?) × √2024.92 = 25.02 × √323.99


(A) 10 (B) 16 (C) 13 (D) 14 (E) None of these

10. 0.56 ÷ 0.0007 + 450 ÷ (?) = 950


(A) 4 (B) 6 (C) 3 (D) 2 (E) None of these

11. (?) = 18.962 + 12.992 + 16.022


(A) 792 (B) 774 (C) 786 (D) 768 (E) None of these

https :
//www. https : https
https : //instagra
//youtube.c
facebo m.com/aas :
om/channe hisharoraso
l/UCYa4_Jr ok.com cial(?) //t.m
Orf8R5Kz2u
/aashis utm_mediu
e/stu

01
OtccXQ m=
haroras copy_link
ocial dified
12. (?) + 39.94 = 2729.98 ÷ 64.92
(A) 3 (B) 2 (C) 1 (D) 4 (E) None of these

13. (?) = 10.08% of 520.05 + 15.05% of 560.11


(A) 146 (B) 153 (C) 128 (D) 136 (E) None of these

14. (?) - 150.01 × 8.92 = 102.05 × 11.98


(A) 2574 (B) 2642 (C) 2322 (D) 2448 (E) None of these

15. (?) = 28.952 - 35.022 - 19.982


(A) - 776 (B) - 792 (C) - 784 (D) - 763 (E) None of these

16. (?) = 460.12/9.01 ÷ 194.86/12.93 × 244.74/6.99


(A) 170 (B) 140 (C) 160 (D) 119 (E) None of these

17. (?) = 465.64 + 3441.456 + 487.74 + 895.97


(A) 5040 (B) 5290 (C) 6480 (D) 5950 (E) None of these

18. (?) = 79.85% of 179.85 + 44.83% of 29.91


(A) 195 (B) 142 (C) 160 (D) 158 (E) None of these

19. 23.93% of 739.92 + (?) = (124.83% of 12.12) 2


(A) 75 (B) 59 (C) 22 (D) 47 (E) None of these

20. (?) = 0.2% of 361 × 0.8% of 779


(A) 4.5 (B) 2 (C) 7.5 (D) 12.5 (E) None of these

21. (?) ÷ 9.03 - 111.001 = √1024.89 - √168.88 + 1727.551/3


(A) 1441 (B) 1728 (C) 1278 (D) 1331 (E) None of these

22. (1110 ÷ 10.9 - 21.33) ÷ 16.012 × 2.042 + (?) = 17.44


(A) -6 (B) -3 (C) -9 (D) 7 (E) None of these

23. (?) = √(3974.91 ÷ 15.01 + 15.02% of 159.97)


(A) 17 (B) 39 (C) 67 (D) 5 (E) None of these

24. (?) + 44.89 = 34.94 × 5.95 ÷ 13.97 + 11.042


(A) 81 (B) 41 (C) 91 (D) 21 (E) None of these

25. 4.992 + (?) = 12.49% of 168.1 + 37.51% of 151.94


(A) 95 (B) 53 (C) 76 (D) 68 (E) None of these

https :
//www. https : https
https : //instagra
//youtube.c
facebo m.com/aas :
om/channe hisharoraso
l/UCYa4_Jr ok.com cial(?) //t.m
Orf8R5Kz2u
/aashis utm_mediu
e/stu

02
OtccXQ m=
haroras copy_link
ocial dified
26. (?) 2 + 127.88 × 12.22 ÷ 47.66 ÷ 15.88 = 83.22
(A) 10 (B) 11 (C) 9 (D) 13 (E) None of these

27. (?) × 7 - 11 = 56.22 × 72.44 ÷ 62.99 - 145.011/2


(A) 6 (B) 9 (C) 3 (D) 7 (E) None of these

28. ((?) - 45.033) ÷ 3.92 = 150.022% of 15.999


(A) 141 (B) 139 (C) 167 (D) 105 (E) None of these

29. (?) × 10.06 = 13.952 - 6.03


(A) 81 (B) 41 (C) 19 (D) 20 (E) None of these

30. (?) = 4095.991/3 + √784.34 - 4912.991/3


(A) 95 (B) 27 (C) 76 (D) 68 (E) None of these

31. (?) = 100.03/5.04 - 6.01 × 11.02 + √483.89


(A) -14 (B) 24 (C) -24 (D) 13 (E) None of these

32. 15.01 + (?) = 10.01% of 319.97 + √36.11


(A) 30 (B) 23 (C) 39 (D) 71 (E) None of these

33. (?) 2 + 6.993 = 39.962 - 35.03


(A) 35 (B) 39 (C) 67 (D) 5 (E) None of these

34. (?) - 190.04 × 5.93 = 194.93 × 13.0


(A) 8175 (B) 4120 (C) 3675 (D) 2675 (E) None of these

35. 60.97 × 43.03 - (?) + 20.98 × 31.96 = 39.01 × 67.03


(A) 950 (B) 472 (C) 762 (D) 682 (E) None of these

36. (?) × 13 × 8 = 462 + 380


(A) 20 (B) 17 (C) 24 (D) 13 (E) None of these

37. (?) = √8464 + √5929 + √1521


(A) 110 (B) 208 (C) 280 (D) 700 (E) None of these

38. (?) = 75% of 132 + 33.33% of 69 + 57.14% of 35


(A) 142 (B) 392 (C) 167 (D) 105 (E) None of these

39. 0.01(?) = 0.1111/0.00145


(A) 10 (B) -1 (C) -12 (D) -2 (E) None of these

https :
//www. https : https
https : //instagra
//youtube.c
facebo m.com/aas :
om/channe hisharoraso
l/UCYa4_Jr ok.com cial(?) //t.m
Orf8R5Kz2u
/aashis utm_mediu
e/stu

03
OtccXQ m=
haroras copy_link
ocial dified
40. (?) 2 + 27 = 262 + 242
(A) 95 (B) 35 (C) 76 (D) 68 (E) None of these

41. (?) = 56.28 × 72.33 - 81.39 × 7.88


(A) 3264 (B) 2969 (C) 3384 (D) 4906 (E) None of these

42. (?) × 10.06 = 13.952 - 6.03


(A) 19 (B) 25 (C) 20 (D) 42 (E) None of these

43. (?) + 12.05 × 8.98 ÷ 4.04 = 235


(A) 208 (B) 207 (C) 401 (D) 390 (E) None of these

44. 2(?) = 359.827 ÷ 23.981 ÷ 0.99 × 3.207 + 18.877


(A) 6 (B) 1 (C) 5 (D) 8 (E) None of these

45. (?) + 13.99 = 4793.97 ÷ 94.01 × 57.93


(A) 7410 (B) 4411 (C) 2944 (D) 6451 (E) None of these

46. (?) + 28 = 4346 ÷ 82


(A) 32 (B) 29 (C) 25 (D) 49 (E) None of these

47. (?) = 2161/3 + 33751/3 + 68591/3


(A) 40 (B) 25 (C) 21 (D) 42 (E) None of these

48. (?) = 35% of 420 + 80% of 560


(A) 425 (B) 595 (C) 410 (D) 395 (E) None of these

49. (?) = 87.51% of 168 + 66.66% of 99


(A) 210 (B) 189 (C) 520 (D) 685 (E) None of these

50. (?) = 12.5% of 64 + 28.56% of 98


(A) 70 (B) 14 (C) 36 (D) 16 (E) None of these

51. (?) = 30.952 - 34.052 - 18.952


(A) -524 (B) 512 (C) -556 (D) 543 (E) None of these

52. (?) + 83.05 = 2451.02 ÷ 57.01 × 7.98


(A) 256 (B) 261 (C) 245 (D) 276 (E) None of these

53. (?) = 34.97% of 339.94 - 5.05% of 619.98


(A) 89 (B) 87 (C) 86 (D) 88 (E) None of these

https :
//www. https : https
https : //instagra
//youtube.c
facebo m.com/aas :
om/channe hisharoraso
l/UCYa4_Jr ok.com cial(?) //t.m
Orf8R5Kz2u
/aashis utm_mediu
e/stu

04
OtccXQ m=
haroras copy_link
ocial dified
54. (?) × √4623.98 = 33.99 × √484.02
(A) 11 (B) 16 (C) 13 (D) 14 (E) None of these

55. (?) + 193.05 = 15.94% of 1849.98 + √575.99


(A) 125 (B) 129 (C) 127 (D) 123 (E) None of these

56. (?) = ∛4912.98 - ∛10648.05 + ∛2744


(A) 8 (B) 7 (C) 9 (D) 6 (E) None of these

57. (?) + 23.05 = 4133.99 ÷ 77.95


(A) 32 (B) 30 (C) 36 (D) 34 (E) None of these

58. 2.05 × (?) × 3.98 = 32.052 + 5.953


(A) 131 (B) 147 (C) 123 (D) 155 (E) None of these

59. 23.99 × 32.94 - (?) + 59.08 × 26.97 = 41.02 × 53.99


(A) 171 (B) 177 (C) 173 (D) 175 (E) None of these

60. (?) × 7.97 = 14.93 × 15.95 - 16.05


(A) 26 (B) 22 (C) 28 (D) 24 (E) None of these

61. (?) 2 + 9.153 = 32.042 - 5.95


(A) 15 (B) 17 (C) 18 (D) 16 (E) None of these

62. (?) = 2240.89 ÷ 82.98 × 12.05 - 202.98


(A) 127 (B) 123 (C) 121 (D) 125 (E) None of these

63. 139.98% of 70.11 - (?) = 21.84 × 3.99


(A) 10 (B) 14 (C) 12 (D) 16 (E) None of these

64. (?) = 2184.05 ÷ 90.98 × 17.95 - 48.05


(A) 367 (B) 392 (C) 373 (D) 384 (E) None of these

65. 1938 - (?) = 44.98% of 39.99 × 98


(A) 174 (B) 176 (C) 172 (D) 178 (E) None of these

66. (?) + 442.04 = 40.10% of 2619.98 + √143.92


(A) 615 (B) 618 (C) 617 (D) 616 (E) None of these

67. 43.05 × 31.98 - (?) + 37.98 × 22.97 = 46.98 × 33.95


(A) 647 (B) 663 (C) 652 (D) 673 (E) None of these

https :
//www. https : https
https : //instagra
//youtube.c
facebo m.com/aas :
om/channe hisharoraso
l/UCYa4_Jr ok.com cial(?) //t.m
Orf8R5Kz2u
/aashis utm_mediu
e/stu

05
OtccXQ m=
haroras copy_link
ocial dified
68. (?) × 9.09 = 19.89% of 1350.23 + 1350 ÷ 14.95
(A) 40 (B) 42 (C) 48 (D) 46 (E) None of these

69. (?) 2 - 8.98 = √324.21 of √575.98 ÷ √256.25


(A) 9 (B) 7 (C) 8 (D) 6 (E) None of these

70. 1576 - (?) = 28% of 75 × 54


(A) 442 (B) 418 (C) 433 (D) 429 (E) None of these

71. (?) + 237 = 851.01 ÷ 36.98 × 40.95


(A) 897 (B) 684 (C) 706 (D) 563 (E) None of these

72. (?) = 550.01 ÷ 2.02 - 28.03 × 7.95 + ∛216.98


(A) 56 (B) 57 (C) 55 (D) 58 (E) None of these

73. (?) = 37.95% of 1899.92 + √196.02 - 346.98


(A) 412 (B) 187 (C) 246 (D) 389 (E) None of these

74. (?) + 83.98 = 1288.05 ÷ 45.98 × 33.99


(A) 868 (B) 779 (C) 932 (D) 687 (E) None of these

75. 0.7(?) = 0.49105 ÷ 0.34352


(A) 54 (B) 56 (C) 58 (D) 52 (E) None of these

76. (?) 2 + 35.02 = 544.16 ÷ √256 × √195.98


(A) 15 (B) 21 (C) 17 (D) 16 (E) None of these

77. (?) 1/5 = (1890 ÷ 21) ÷ (15 × 3)


(A) 31 (B) 34 (C) 32 (D) 33 (E) None of these

78. (?) = 280.18 ÷ 7.04 - 12.99 × 16.05 + ∛10647.92


(A) -146 (B) 142 (C) -153 (D) 159 (E) None of these

79. (?) = √2208.95 - √2916.03 + √1521.05


(A) 38 (B) 36 (C) 34 (D) 32 (E) None of these

80. (?) = 80.14% of 339.97 + 15.11% of 819.95


(A) 395 (B) 378 (C) 383 (D) 384 (E) None of these

81. (?) 2 - 1 = 112 × 133 ÷ 49 + √3136


(A) 32 (B) 29 (C) 19 (D) 49 (E) None of these

https :
//www. https : https
https : //instagra
//youtube.c
facebo m.com/aas :
om/channe hisharoraso
l/UCYa4_Jr ok.com cial(?) //t.m
Orf8R5Kz2u
/aashis utm_mediu
e/stu

06
OtccXQ m=
haroras copy_link
ocial dified
82. (?) + (2298 - 7756 + 6683) 1/2 ÷ √81 × 54 = 27
(A) -82 (B) -25 (C) 21 (D) -42 (E) None of these

83. (?) 2 - 2561/4 - √(144 + 112) = 62 + √625


(A) ± 9 (B) ± 8 (C) ± 7 (D) ± 6 (E) None of these

84. (?) = {3/8 of 64 ÷ 6/7 of 14 - (280 ÷ 14 ÷ 4) × 1/4} ÷ 1/3


(A) 9/4 (B) 1/9 (C) 5/2 (D) 8/5 (E) None of these

85. (?) × 38 = 75% of 5700


(A) 112.5 (B) 140.5 (C) 122.5 (D) 160.5 (E) None of these

86. (?) × 29.97 = 26.052 - 16


(A) 32 (B) 29 (C) 22 (D) 49 (E) None of these

87. (?) 2 + 8.943 = 31.012 - 7.02


(A) 15 (B) 25 (C) 21 (D) 42 (E) None of these

88. (?) 2 + 6.013 = 28.992


(A) 30 (B) 25 (C) 41 (D) 15 (E) None of these

89. 17.063 - (?) 2 = 4513.02


(A) 20 (B) 18 (C) 15 (D) 68 (E) None of these

90. 74.02 × 40.03 - (?) + 49.97 × 36.05 = 56.01 × 84.04


(A) 70 (B) 14 (C) 56 (D) 16 (E) None of these

91. (?) = 20.05 × 6.97 - 15.01 × 12.02


(A) 20 (B) 17 (C) -40 (D) -80 (E) None of these

92. (?) × √81795.98 = 25.98 × √483.97


(A) 10 (B) 2 (C) 8 (D) 7 (E) None of these

93. (?) = 45.03% of 420.11 + 4.89% of 739.99


(A) 226 (B) 392 (C) 167 (D) 205 (E) None of these

94. (?) = 25.032 - 1295.941/4 + 999.941/3


(A) 610 (B) 635 (C) 629 (D) 608 (E) None of these

95. 15.983 - (?) 2 = 3975.04


(A) 9 (B) 11 (C) 17 (D) 6 (E) None of these

https :
//www. https : https
https : //instagra
//youtube.c
facebo m.com/aas :
om/channe hisharoraso
l/UCYa4_Jr ok.com cial(?) //t.m
Orf8R5Kz2u
/aashis utm_mediu
e/stu

07
OtccXQ m=
haroras copy_link
ocial dified
96. (?) = √2704 ÷ 21971/3 + 27441/3 × 65611/4
(A) 200 (B) 170 (C) 130 (D) 135 (E) None of these

97. (?)% of 800 = 293 - 22% of 750


(A) 11 (B) 16 (C) 28 (D) 15 (E) None of these

98. 0.64(?) = 0.885 ÷ 0.51271


(A) -64 (B) -39 (C) -67 (D) -51 (E) None of these

99. 648 ÷ (?) + 22 × 2.5 = 112 - 39


(A) 10 (B) 16 (C) 24 (D) 12 (E) None of these

100. (?) × 202 = 244 × 1/2 + 224


(A) 5 (B) 2 (C) 7 (D) 6 (E) None of these

101. (?) 2 + 73 = 212 - 17


(A) 3 (B) 6 (C) 9 (D) 4 (E) None of these

102. 33751/3 of 75/(?) = (?) /3 × 2251/2


(A) 15 (B) 25 (C) 20 (D) 42 (E) None of these

103. (?) = 15% of 680 + 45% of 340


(A) 208 (B) 255 (C) 401 (D) 390 (E) None of these

104. (?) + 6251/2 = 3(3/6) of 132


(A) 437 (B) 411 (C) 512 (D) 428 (E) None of these

105. (?) = 900 ÷ 6 - 14 × 11 + 2251/2


(A) 10 (B) 4 (C) 11 (D) 6 (E) None of these

https :
//www. https : https
https : //instagra
//youtube.c
facebo m.com/aas :
om/channe hisharoraso
l/UCYa4_Jr ok.com cial(?) //t.m
Orf8R5Kz2u
/aashis utm_mediu
e/stu

08
OtccXQ m=
haroras copy_link
ocial dified
1. Ans. (B) 6. Ans. (C)
(?) 2 + 13.053 = 48.982 - 7.95 (?) = 4.98% of 659.89 + 34.98% of
(?) 2 + 133 = 492 - 8 699.95
(?) 2 + 2197 = 2401 - 8 (?) = 5% of 660 + 35% of 700
(?) 2 = 2393 - 2197 = 196 (?) = 0.05 × 660 + 0.35 × 700
(?) = 14 (?) = 33 + 245 = 278

2. Ans. (C) 7. Ans. (B)


(?) = 84.92% of 780.05 + 19.98% of (?) = 37.96 × 29.02 + 20.98 × 13.04
689.99 (?) = 38 × 29 + 21 × 13
(?) = 85% of 780 + 20% of 690 (?) = 1102 + 273 = 1375
(?) = 0.85 × 780 + 0.2 × 690
(?) = 663 + 138 = 801 8. Ans. (D)
(?) + 228.05 = 1215.99 ÷ 37.94 × 17.98
3. Ans. (A) (?) + 228 = 1216 ÷ 38 × 18
(?) 2 - 152 = 192 + 212 - 172 - 32 (?) + 228 = 32 × 18
(?) 2 - 225 = 361 + 441 - 289 - 9 (?) = 576 - 228 = 348
(?) 2 - 225 = 802 - 298
(?) 2 = 504 + 225 = 729 9. Ans. (A)
(?) = 27 (?) × √2024.92 = 25.02 × √323.99
(?) × √2025 = 25 × √324
4. Ans. (C) (?) × 45 = 25 × 18
(?) + 576 ÷ 8 - 503 = 564 ÷ 12 (?) = 5 × 2 = 10
(?) + 72 - 503 = 47
(?) = 47 + 431 = 478 10. Ans. (C)
0.56 ÷ 0.0007 + 450 ÷ (?) = 950
5. Ans. (A) 800 + 450 ÷ (?) = 950
(?) = (9 × 32 + 62 - 8) × 2 450 ÷ (?) = 150
(?) = (9 × 9 + 36 - 8) × 2 (?) = 3
(?) = (81 + 28) × 2
(?) = 109 × 2 = 218

https :
//www. https : https
https : //instagra
//youtube.c
facebo m.com/aas :
om/channe hisharoraso
l/UCYa4_Jr ok.com cial(?) //t.m
Orf8R5Kz2u
/aashis utm_mediu
e/stu

09
OtccXQ m=
haroras copy_link
ocial dified
11. Ans. (C) 16. Ans. (D)
(?) = 18.962 + 12.992 + 16.022 (?) = 460.12/9.01 ÷ 194.86/12.93 ×
(?) = 192 + 132 + 162 244.74/6.99
(?) = 361 + 169 + 256 = 786 (?) = 459/9 ÷ 195/13 × 245/7
(?) = 51 ÷ 15 × 35
12. Ans. (B) (?) = 17 × 7 = 119
(?) + 39.94 = 2729.88 ÷ 64.92
(?) + 40 = 2730 ÷ 65 17. Ans. (B)
(?) = 42 - 40 = 2 (?) = 465.64 + 3441.456 + 487.74 +
895.97
13. Ans. (D) (?) = 5290.806 = 5290
(?) = 10.08% of 520.05 + 15.05% of
560.11 18. Ans. (D)
(?) = 10% of 520 + 15% of 560 (?) = 79.85% of 179.85 + 44.83% of
(?) = 0.1 × 520 + 0.15 × 560 29.91
(?) = 52 + 84 = 136 (?) = 80% of 180 + 45% of 30
(?) = 144 + 13.5 = 157.5 = 158
14. Ans. (A)
(?) - 150.01 × 8.92 = 102.05 × 11.98 19. Ans. (D)
(?) - 150 × 9 = 102 × 12 23.93% of 739.92 + (?) = (124.83% of
(?) = 1224 + 1350 = 2574 12.12) 2
24% of 740 + (?) = (125% of 12) 2
15. Ans. (C) 0.24 × 740 + (?) = (1.25 × 12) 2
(?) = 28.952 - 35.022 - 19.982 177.6 + (?) = 152 = 225
(?) = 292 - 352 - 202 (?) = 47.4 = 47
(?) = 841 - 1225 - 400
(?) = 841 - 1625 = - 784 20. Ans. (A)
(?) = 0.2% of 361 × 0.8% of 779
(?) = 0.2% of 360 × 0.8% of 780
(?) = 0.2 × 3.6 × 0.8 × 7.8 = 4.4928 =
4.5

21. Ans. (C)


(?) ÷ 9.03 - 111.001 = √1024.89 -
√168.88 + 1727.551/3
(?) ÷ 9 - 111 = √1024 - √169 + 17281/3
(?) ÷ 9 - 111 = 32 - 13 + 12

https :
//www. https : https
https : //instagra
//youtube.c
facebo m.com/aas :
om/channe hisharoraso
l/UCYa4_Jr ok.com cial(?) //t.m
Orf8R5Kz2u
/aashis utm_mediu
e/stu

10
OtccXQ m=
haroras copy_link
ocial dified
26. Ans. (C)
(?) ÷ 9 = 31 + 111 = 142 (?) 2 + 127.88 × 12.22 ÷ 47.66 ÷ 15.88
(?) = 1278 = 83.22
(?) 2 + 128 × 12 ÷ 48 ÷ 16 = 83
22. Ans. (B) (?) 2 + 8 ÷ 4 = 83
(1110 ÷ 10.9 - 21.33) ÷ 16.012 × 2.042 (?) 2 = 83 - 2 = 81
+ (?) = 17.44 (?) = 9
(1111 ÷ 11 - 21) ÷ 16 × 22 + (?) = 17
(101 - 21) ÷ 16 × 4 + (?) = 17 27. Ans. (B)
80 ÷ 4 + (?) = 17 (?) × 7 - 11 = 56.22 × 72.44 ÷ 62.99 -
(?) = 17 - 30 = - 3 145.011/2
(?) × 7 - 11 = 56 × 72 ÷ 63 - 1441/2
23. Ans. (A) (?) × 7 - 11 = 8 × 8 - 12
(?) = √(3974.91 ÷ 15.01 + 15.02% of (?) × 7 = 64 - 1 = 63
159.97) (?) = 9
(?) = √(3975 ÷ 15 + 15% of 160)
(?) = √(265 + 0.15 × 160) 28. Ans. (A)
(?) = √(265 + 24) = √289 ((?) - 45.033) ÷ 3.92 = 150.022% of
(?) = 17 15.999
((?) - 45) ÷ 4 = 150% of 16
24. Ans. (C) ((?) - 45) ÷ 4 = 1.5 × 16
(?) + 44.89 = 34.94 × 5.95 ÷ 13.97 + (?) - 45 = 24 × 4
11.042 (?) = 96 + 45 = 141
(?) + 45 = 35 × 6 ÷ 14 + 112
(?) + 45 = 5 × 3 + 121 29. Ans. (C)
(?) = 15 + 76 (?) × 10.06 = 13.952 - 6.03
(?) = 91 (?) × 10 = 142 - 6
(?) × 10 = 196 - 6 = 190
25. Ans. (B) (?) = 19
4.992 + (?) = 12.49% of 168.1 +
37.51% of 151.94 30. Ans. (B)
52 + (?) = 12.5% of 168 + 37.5% of (?) = 4095.991/3 + √784.34 - 4912.991/3
152 (?) = 40961/3 + √784 - 49131/3
25 + (?) = 0.125 × 168 + 0.375 × 152 (?) = 16 + 28 - 17
25 + (?) = 21 + 57 (?) = 28 - 1 = 27
(?) = 78 - 25 = 53

https :
//www. https : https
https : //instagra
//youtube.c
facebo m.com/aas :
om/channe hisharoraso
l/UCYa4_Jr ok.com cial(?) //t.m
Orf8R5Kz2u
/aashis utm_mediu
e/stu

11
OtccXQ m=
haroras copy_link
ocial dified
31. Ans. (C) 36. Ans. (C)
(?) = 100.03/5.04 - 6.01 × 11.02 + (?) × 13 × 8 = 462 + 380
√483.89 (?) × 13 × 8 = 2116 + 380
(?) = 100/5 - 6 × 11 + √484 (?) × 13 × 8 = 2496
(?) = 20 - 66 + 22 (?) × 13 = 312
(?) = 42 - 66 = - 24 (?) = 24

32. Ans. (B) 37. Ans. (B)


15.01 + (?) = 10.01% of 319.97 + (?) = √8464 + √5929 + √1521
√36.11 (?) = 92 + 77 + 39 = 208
15 + (?) = 10% of 320 + √36
15 + (?) = 0.1 × 320 + 6 38. Ans. (A)
(?) = 32 - 9 = 23 (?) = 75% of 132 + 33.33% of 69 +
57.14% of 35
33. Ans. (A) (?) = 0.75 × 132 + 1/3 × 69 + 4/7 × 35
(?) 2 + 6.993 = 39.962 - 35.03 (?) = 99 + 23 + 20 = 142
(?) 2 + 73 = 402 - 35
(?) 2 + 343 = 1600 - 35 39. Ans. (C)
(?) 2 = 1565 - 343 = 1222 0.01(?) = 0.1111/0.00145
(?) = 34.95 = 35 0.12 × (?) = 0.1111/0.13 × 45
2 × (?) = 111 - 3 × 45
34. Ans. (C) 2 × (?) = 111 - 135 = - 24
(?) - 190.04 × 5.93 = 194.93 × 13.0 (?) = - 12
(?) - 190 × 6 = 195 × 13
(?) = 2535 + 1140 = 3675 40. Ans. (B)
(?) 2 + 27 = 262 + 242
35. Ans. (D) (?) 2 + 27 = 676 + 576
60.97 × 43.03 - (?) + 20.98 × 31.96 = (?) 2 = 1252 - 27 = 1225
39.01 × 67.03 (?) = 35
61 × 43 - (?) + 21 × 32 = 39 × 67
2623 - (?) + 672 = 2613
(?) = 3295 - 2613 = 682

https :
//www. https : https
https : //instagra
//youtube.c
facebo m.com/aas :
om/channe hisharoraso
l/UCYa4_Jr ok.com cial(?) //t.m
Orf8R5Kz2u
/aashis utm_mediu
e/stu

12
OtccXQ m=
haroras copy_link
ocial dified
41. Ans. (C) 46. Ans. (C)
(?) = 56.28 × 72.33 - 81.39 × 7.88 (?) + 28 = 4346 ÷ 82
(?) = 56 × 72 - 81 × 8 (?) = 53 - 28 = 25
(?) = 4032 - 648 = 3384
47. Ans. (A)
42. Ans. (A) (?) = 2161/3 + 33751/3 + 68591/3-
(?) × 10.06 = 13.952 - 6.03 (?) = 6 + 15 + 19 = 40
(?) × 10 = 142 - 6
(?) × 10 = 196 - 6 = 190 48. Ans. (B)
(?) = 19 (?) = 35% of 420 + 80% of 560
(?) = 0.35 × 420 + 0.8 × 560
43. Ans. (A) (?) = 147 + 448 = 595
(?) + 12.05 × 8.98 ÷ 4.04 = 235
(?) + 12 × 9 ÷ 4 = 235 49. Ans. (A)
(?) + 3 × 9 = 235 (?) = 87.51% of 168 + 66.66% of 99
(?) = 235 - 27 = 208 (?) = 6/7 × 168 + 2/3 × 99
(?) = 144 + 66 = 210
44. Ans. (A)
2(?) = 359.827 ÷ 23.981 ÷ 0.99 × 3.207 50. Ans. (C)
+ 18.877 (?) = 12.5% of 64 + 28.56% of 98
2(?) = 360 ÷ 24 ÷ 1× 3 + 19 (?) = 0.125 × 64 + 2/7 × 98
2(?) = 15 × 3 + 19 (?) = 8 + 28 = 36
2(?) = 45 + 19 = 64 = 26
(?) = 6

45. Ans. (C)


(?) + 13.99 = 4793.97 ÷ 94.01 × 57.93
(?) + 14 = 4794 ÷ 94 × 58
(?) + 14 = 51 × 58
(?) = 2958 - 14 = 2944

https :
//www. https : https
https : //instagra
//youtube.c
facebo m.com/aas :
om/channe hisharoraso
l/UCYa4_Jr ok.com cial(?) //t.m
Orf8R5Kz2u
/aashis utm_mediu
e/stu

13
OtccXQ m=
haroras copy_link
ocial dified
51. Ans. (C) 56. Ans. (C)
(?) = 30.952 - 34.052 - 18.952 (?) = ∛4912.98 - ∛10648.05 + ∛2744
(?) = 312 - 342 - 192 (?) = ∛4913 - ∛10648 + ∛2744
(?) = 961 - 1156 - 361 (?) = 17 - 22 + 14
(?) = 961 - 1517 = - 556 (?) = 31 - 22 = 9

52. Ans. (B) 57. Ans. (B)


(?) + 83.05 = 2451.02 ÷ 57.01 × 7.98 (?) + 23.05 = 4133.99 ÷ 77.95
(?) + 83 = 2451 ÷ 57 × 8 (?) + 23 = 4134 ÷ 78
(?) + 83 = 43 × 8 (?) = 53 - 23 = 30
(?) = 344 - 83 = 261
58. Ans. (D)
53. Ans. (D) 2.05 × (?) ×3.98 = 32.052 + 5.953
(?) = 34.97% of 339.94 - 5.05% of 2 × (?) × 4 = 322 + 63
619.98 (?) × 8 = 1024 + 216 = 1240
(?) = 35% of 340 - 5% of 620 (?) = 155
(?) = 0.35 × 340 - 0.05 × 620
(?) = 119 - 31 = 88 59. Ans. (A)
23.99 × 32.94 - (?) + 59.08 × 26.97 =
54. Ans. (A) 41.02 × 53.99
(?) × √4623.98 = 33.99 × √484.02 24 × 33 - (?) + 59 × 27 = 41 × 54
(?) × √4624 = 34 × √484 792 - (?) + 1593 = 2214
(?) × 68 = 34 × 22 (?) = 2385 - 2214 = 171
(?) = 11
60. Ans. (C)
55. Ans. (C) (?) × 7.97 = 14.93 × 15.95 - 16.05
(?) + 193.05 = 15.94% of 1849.98 + (?) × 8 = 15 × 16 - 16
√575.99 (?) = 15 × 2 - 2 = 30 - 2 = 28
(?) + 193 = 16% of 1850 + √576
(?) + 193 = 0.16 × 1850 + 24
(?) = 296 - 169 = 127

https :
//www. https : https
https : //instagra
//youtube.c
facebo m.com/aas :
om/channe hisharoraso
l/UCYa4_Jr ok.com cial(?) //t.m
Orf8R5Kz2u
/aashis utm_mediu
e/stu

14
OtccXQ m=
haroras copy_link
ocial dified
61. Ans. (B) 66. Ans. (B)
(?) 2 + 9.153 = 32.042 - 5.95 (?) + 442.04 = 40.10% of 2619.98 +
(?) 2 + 93 = 322 - 6 √143.92
(?) 2 + 729 = 1024 - 6 (?) + 442 = 40% of 2620 + √144
(?) 2 = 1018 - 729 = 289 (?) + 442 = 0.4 × 2620 + 12
(?) = 17 (?) + 430 = 1048
(?) = 618
62. Ans. (C)
(?) = 2240.89 ÷ 82.98 × 12.05 - 202.98 67. Ans. (C)
(?) = 2241 ÷ 83 × 12 - 203 43.05 × 31.98 - (?) + 37.98 × 22.97 =
(?) = 27 × 12 - 203 = 324 - 203 = 121 46.98 × 33.95
43 × 32 - (?) + 38 × 23 = 47 × 34
63. Ans. (A) 1376 - (?) + 874 = 1598
139.98% of 70.11 - (?) = 21.84 × 3.99 (?) = 2250 - 1598 = 652
140% of 70 - (?) = 22 × 4
1.4 × 70 - (?) = 88 68. Ans. (A)
(?) = 98 - 88 = 10 (?) × 9.09 = 19.89% of 1350.23 + 1350
÷ 14.95
64. Ans. (D) (?) × 9 = 20% of 1350 + 1350 ÷ 15
(?) = 2184.05 ÷ 90.98 × 17.95 - 48.05 (?) × 9 = 0.2 × 1350 + 90
(?) = 2184 ÷ 91 × 18 - 48 (?) × 9 = 270 + 90 = 360
(?) = 312 ÷ 13 × 18 - 48 (?) = 40
(?) = 24 × 18 - 48
(?) = 432 - 48 = 384 69. Ans. (D)
(?) 2 - 8.98 = √324.21 of √575.98 ÷
65. Ans. (A) √256.25
1938 - (?) = 44.98% of 39.99 × 98 (?) 2 - 9 = √324 of √576 ÷ √256
1938 - (?) = 45% of 40 × 98 (?) 2 - 9 = 18 × 24 ÷ 16
1938 - (?) = 0.45 × 40 × 98 (?) 2 = 27 + 9 = 36
1938 - (?) = 18 × 98 (?) = 6
(?) = 1938 - 1764 = 174
70. Ans. (A)
1576 - (?) = 28% of 75 × 54
1576 - (?) = 0.28 × 75 × 54
1576 - (?) = 1134
(?) = 442

https :
//www. https : https
https : //instagra
//youtube.c
facebo m.com/aas :
om/channe hisharoraso
l/UCYa4_Jr ok.com cial(?) //t.m
Orf8R5Kz2u
/aashis utm_mediu
e/stu

15
OtccXQ m=
haroras copy_link
ocial dified
71. Ans. (C) 76. Ans. (B)
(?) + 237 = 851.01 ÷ 36.98 × 40.95 (?) 2 + 35.02 = 544.16 ÷ √256 ×
(?) + 237 = 851 ÷ 37 × 41 √195.98
(?) + 237 = 23 × 41 (?) 2 + 35 = 544 ÷ √256 × √196
(?) = 943 - 237 = 706 (?) 2 + 35 = 544 ÷ 16 × 14
(?) 2 + 35 = 34 × 14
72. Ans. (B) (?) 2 = 476 - 35 = 441
(?) = 550.01 ÷ 2.02 - 28.03 × 7.95 + (?) = 21
∛216.98
(?) = 550 ÷ 2 - 28 × 8 + ∛216 77. Ans. (C)
(?) = 275 - 224 + 6 (?) 1/5 = (1890 ÷ 21) ÷ (15 × 3)
(?) = 51 + 6 = 57 (?) 1/5 = 90 ÷ 45 = 2
(?) = 32
73. Ans. (D)
(?) = 37.95% of 1899.92 + √196.02 - 78. Ans. (A)
346.98 (?) = 280.18 ÷ 7.04 - 12.99 × 16.05 +
(?) = 38% of 1900 + √196 - 347 ∛10647.92
(?) = 0.38 × 1900 + 14 - 347 (?) = 280 ÷ 7 - 13 × 16 + ∛10648
(?) = 722 - 333 = 389 (?) = 40 - 208 + 22
(?) = 62 - 208 = - 146
74. Ans. (A)
(?) + 83.98 = 1288.05 ÷ 45.98 × 33.99 79. Ans. (D)
(?) + 84 = 1288 ÷ 46 × 34 (?) = √2208.95 - √2916.03 + √1521.05
(?) + 84 = 28 × 34 (?) = √2209 - √2916 + √1521
(?) = 952 - 84 = 868 (?) = 47 - 54 + 39
(?) = 86 - 54 = 32
75. Ans. (A)
0.7(?) = 0.49105 ÷ 0.34352 80. Ans. (A)
0.7(?) = 0.72 × 105 ÷ 0.73 × 52 (?) = 80.14% of 339.97 + 15.11% of
(?) = 2 × 105 - 3 × 52 819.95
(?) = 210 - 156 = 54 (?) = 80% of 340 + 15% of 820
(?) = 0.8 × 340 + 0.15 × 820
(?) = 272 + 123 = 395

https :
//www. https : https
https : //instagra
//youtube.c
facebo m.com/aas :
om/channe hisharoraso
l/UCYa4_Jr ok.com cial(?) //t.m
Orf8R5Kz2u
/aashis utm_mediu
e/stu

16
OtccXQ m=
haroras copy_link
ocial dified
81. Ans. (C) 86. Ans. (C)
(?) 2 - 1 = 112 × 133 ÷ 49 + √3136 (?) × 29.97 = 26.052 - 16
(?) 2 - 1 = 16 × 19 + 56 (?) × 30 = 262 - 16
(?) 2 = 304 + 57 = 361 (?) × 30 = 676 - 16 = 660
(?) = 19 (?) = 22

82. Ans. (A) 87. Ans. (A)


(?) + (2298 - 7756 + 6683) 1/2 ÷ √81 × (?) 2 + 8.943 = 31.012 - 7.02
54 = 27 (?) 2 + 93 = 312 - 7
(?) + 12251/2 ÷ √81 × 54 = 128 (?) 2 + 729 = 961 - 7
(?) + 35 ÷ 9 × 54 = 128 (?) 2 = 954 - 729 = 225
(?) + 35 × 6 = 128 (?) = 15
(?) = 128 - 210 = - 82
88. Ans. (B)
83. Ans. (A) (?) 2 + 6.013 = 28.992
(?) 2 - 2561/4 - √(144 + 112) = 62 + √625 (?) 2 + 63 = 292
(?) 2 - 4 - √256 = 36 + 25 (?) 2 + 216 = 841
(?) 2 - 4 - 16 = 61 (?) 2 = 841 - 216 = 625
(?) 2 = 61 + 20 = 81 (?) = 25
(?) = ± 9
89. Ans. (A)
84. Ans. (A) 17.063 - (?) 2 = 4513.02
(?) = {3/8 of 64 ÷ 6/7 of 14 - (280 ÷ 14 173 - (?) 2 = 4513
÷ 4) × 1/4} ÷ 1/3 4913 - (?) 2 = 4513
(?) = {(3/8 × 64) ÷ (6/7 × 14) - (280 ÷ (?) 2 = 400
56) × 1/4} ÷ 1/3 (?) = 20
(?) = {24 ÷ 12 - 5 × 1/4} ÷ 1/3
(?) = {2 - 5/4} × 3 90. Ans. (C)
(?) = 3/4 × 3 = 9/4 74.02 × 40.03 - (?) + 49.97 × 36.05 =
56.01 × 84.04
85. Ans. (A) 74 × 40 - (?) + 50 × 36 = 56 × 84
(?) × 38 = 75% of 5700 2960 - (?) + 1800 = 4704
(?) × 38 = 0.75 × 5700 (?) = 4760 - 4704 = 56
(?) = 0.75 × 150 = 112.5

https :
//www. https : https
https : //instagra
//youtube.c
facebo m.com/aas :
om/channe hisharoraso
l/UCYa4_Jr ok.com cial(?) //t.m
Orf8R5Kz2u
/aashis utm_mediu
e/stu

17
OtccXQ m=
haroras copy_link
ocial dified
91. Ans. (C) 96. Ans. (C)
(?) = 20.05 × 6.97 - 15.01 × 12.02 (?) = √2704 ÷ 21971/3 + 27441/3 ×
(?) = 20 × 7 - 15 × 12 65611/4
(?) = 140 - 180 = - 40 (?) = 52 ÷ 13 + 14 × 9
(?) = 4 + 126 = 130
92. Ans. (B)
(?) × √81795.98 = 25.98 × √483.97 97. Ans. (B )
(?) × √81796 = 26 × √484 (?)% of 800 = 293 - 22% of 750
(?) × 286 = 26 × 22 (?) /100 × 800 = 293 - 0.22 × 750
(?) × 13 = 26 (?) × 8 = 293 - 165 = 128
(?) = 2 (?) = 16

93. Ans. (A) 98. Ans. (A)


(?) = 45.03% of 420.11 + 4.89% of 0.64(?) = 0.885 ÷ 0.51271
739.99 0.82 × (?) = 0.885 ÷ 0.83 × 71
(?) = 45% of 420 + 5% of 740 2 × (?) = 85 - 3 × 71
(?) = 0.45 × 420 + 0.05 × 740 2 × (?) = 85 - 213 = - 128
(?) = 189 + 37 = 226 (?) = - 64

94. Ans. (C) 99. Ans. (C)


(?) = 25.032 - 1295.941/4 + 999.941/3 648 ÷ (?) + 22 × 2.5 = 112 - 39
(?) = 252 - 12961/4 + 10001/3 648 ÷ (?) + 55 = 121 - 39
(?) = 625 - 6 + 10 648 ÷ (?) = 82 - 55 = 27
(?) = 625 + 4 = 629 (?) = 24

95. Ans. (B) 100. Ans. (B)


15.983 - (?) 2 = 3975.04 (?) × 202 = 244 × 1/2 + 224
163 - (?) 2 = 3975 (?) × 400 = 242 + 224
4096 - (?) 2 = 3975 (?) × 400 = 576 + 224 = 800
(?) 2 = 121 (?) = 2
(?) = 11

https :
//www. https : https
https : //instagra
//youtube.c
facebo m.com/aas :
om/channe hisharoraso
l/UCYa4_Jr ok.com cial(?) //t.m
Orf8R5Kz2u
/aashis utm_mediu
e/stu

18
OtccXQ m=
haroras copy_link
ocial dified
101. Ans. (C)
(?) 2 + 73 = 212 - 17
(?) 2 + 343 = 441 - 17
(?) 2 = 424 - 343 = 81
(?) = 9

102. Ans. (A)


33751/3 of 75/(?) = (?) /3 × 2251/2
15 × 75/(?) = (?) /3 × 15
(?) 2 = 225
(?) = 15

103. Ans. (B)


(?) = 15% of 680 + 45% of 340
(?) = 0.15 × 680 + 0.45 × 340
(?) = 102 + 153 = 255

104. Ans. (A)


(?) + 6251/2 = 3(3/6) of 132
(?) + 25 = 21/6 × 132
(?) = 21 × 22 - 25
(?) = 462 - 25 = 437

105. Ans. (C)


(?) = 900 ÷ 6 - 14 × 11 + 2251/2
(?) = 150 - 154 + 15
(?) = - 4 + 15 = 11

https :
//www. https : https
https : //instagra
//youtube.c
facebo m.com/aas :
om/channe hisharoraso
l/UCYa4_Jr ok.com cial(?) //t.m
Orf8R5Kz2u
/aashis utm_mediu
e/stu

19
OtccXQ m=
haroras copy_link
ocial dified
https :
//instagram.com/aashisharorasocia
l(?) utm_medium = copy_link

https: //t.me/studified
https : //t.me/studified

https :
//youtube.com/channel/UCYa
4_JrOrf8R5Kz2uOtccXQ

https :
//www.facebook.com/aa
shisharorasocial

https :
//www. https : https
https : //instagra
//youtube.c
facebo m.com/aas :
om/channe hisharoraso
l/UCYa4_Jr ok.com cial(?) //t.m
Orf8R5Kz2u
/aashis utm_mediu
e/stu

20
OtccXQ m=
haroras copy_link
ocial dified
Solve the given equations and choose the correct option.
दिए गए समीकरणों को हल करें और सही ववकल्प चन
ु ें ।

1. I. 3x + 4y = 90
II. 5x + 3y = 95
(A) x > y (B) x < y (C) x ≥ y
(D) x ≤ y (E) x = y or no relation can be established

2. I. x2 = 1521
y3
II. = 4096
(A) x > y (B) x < y (C) x ≥ y
(D) x ≤ y (E) x = y or no relation can be established

3. I. x2 - 57x + 656 = 0
II. y2 - 30y + 224 = 0
(A) x > y (B) x < y (C) x ≥ y
(D) x ≤ y (E) x = y or no relation can be established

4. I. x2 - 23x - 420 = 0
y2
II. - 21y + 108 = 0
(A) x > y (B) x < y (C) x ≥ y
(D) x ≤ y (E) x = y or no relation can be established

5. I. 3x2 - 34x + 63 = 0
II. y2 - 59y + 864 = 0
(A) x > y (B) x < y (C) x ≥ y
(D) x ≤ y (E) x = y or no relation can be established

6. I. 5x2 - 24x + 28 = 0
II. 3y2 - 21y + 36 = 0
(A) x > y (B) x < y (C) x ≥ y
(D) x ≤ y (E) x = y or no relation can be established

https :
//www. https : https
https : //instagra
//youtube.c
facebo m.com/aas :
om/channe hisharoraso
l/UCYa4_Jr ok.com cial(?) //t.m
Orf8R5Kz2u
/aashis utm_mediu
e/stu

21
OtccXQ m=
haroras copy_link
ocial dified
07. I. x2 + 32x + 231 = 0
II. y2 + 3y - 88 = 0
(A) x > y (B) x < y (C) x ≥ y
(D) x ≤ y (E) x = y or no relation can be established

08. I. 3x2 + 31x + 78 = 0


II. 3y2 + 15y + 18 = 0
(A) x > y (B) x < y (C) x ≥ y
(D) x ≤ y (E) x = y or no relation can be established

09. I. 2x2 - 32x + 128 = 0


II. 2y2 - 28y + 98 = 0
(A) x > y (B) x < y (C) x ≥ y
(D) x ≤ y (E) x = y or no relation can be established

10. I. 7x2 - 30x + 27 = 0


II. 9y2 - 30y + 24 = 0
(A) x > y (B) x < y (C) x ≥ y
(D) x ≤ y (E) x = y or no relation can be established

11. I. x2 - 19x + 84 = 0
II. 25y2 - 45y + 18 = 0
(A) x > y (B) x < y (C) x ≥ y
(D) x ≤ y (E) x = y or no relation can be established

12. I. `x2 - 4x - 117 = 0


II. y2 - 29y + 208 = 0
(A) x > y (B) x < y (C) x ≥ y
(D) x ≤ y (E) x = y or no relation can be established

13. I. (x + 25) /11 = - 14/x


II. 3y - (10y - 20) /y = 6
(A) x > y (B) x < y (C) x ≥ y
(D) x ≤ y (E) x = y or no relation can be established

14. I. x2 - 7x - 18 = 0
II. y2 - 9y + 18 = 0
(A) x > y (B) x < y (C) x ≥ y
(D) x ≤ y (E) x = y or no relation can be established

https :
//www. https : https
https : //instagra
//youtube.c
facebo m.com/aas :
om/channe hisharoraso
l/UCYa4_Jr ok.com cial(?) //t.m
Orf8R5Kz2u
/aashis utm_mediu
e/stu

22
OtccXQ m=
haroras copy_link
ocial dified
15. I. x2 + 12x - 45 = 0
II. y2 - 14y + 33 = 0
(A) x > y (B) x < y (C) x ≥ y
(D) x ≤ y (E) x = y or no relation can be established

16. I. x2 - 21x + 104 = 0


II. y2 - 22y - 104 = 0
(A) x > y (B) x < y (C) x ≥ y
(D) x ≤ y (E) x = y or no relation can be established

17. I. 6x(x - 2) - x + 6 = 0
II. y2 - (5/6) y + 1/6 = 0
(A) x > y (B) x < y (C) x ≥ y
(D) x ≤ y (E) x = y or no relation can be established

18. I. 4x(x - 12) + x + 33 = 0


II. y2 - (5/4) y + 3/8 = 0
(A) x > y (B) x < y (C) x ≥ y
(D) x ≤ y (E) x = y or no relation can be established

19. I. x2 - 20x + 99 = 0
II. y2 + 4y - 96 = 0
(A) x > y (B) x < y (C) x ≥ y
(D) x ≤ y (E) x = y or no relation can be established

20. I. 4x - 6 = 3(x + 2) - 3
II. y2 - 33 = 48
(A) x > y (B) x < y (C) x ≥ y
(D) x ≤ y (E) x = y or no relation can be established

21. I. x2 + 21x + 90 = 0
II. y2 + 6y - 91 = 0
(A) x > y (B) x < y (C) x ≥ y
(D) x ≤ y (E) x = y or no relation can be established

22. I. x2 + 25x + 66 = 0
II. y2 - 5y - 14 = 0
(A) x > y (B) x < y (C) x ≥ y
(D) x ≤ y (E) x = y or no relation can be established

https :
//www. https : https
https : //instagra
//youtube.c
facebo m.com/aas :
om/channe hisharoraso
l/UCYa4_Jr ok.com cial(?) //t.m
Orf8R5Kz2u
/aashis utm_mediu
e/stu

23
OtccXQ m=
haroras copy_link
ocial dified
23. I. x2 - 81 = 0
II. y2 + 30y + 189 = 0
(A) x > y (B) x < y (C) x ≥ y
(D) x ≤ y (E) x = y or no relation can be established

24. I. x2 - 14x + 49 = 0
II. y2 - 12y + 35 = 0
(A) x > y (B) x < y (C) x ≥ y
(D) x ≤ y (E) x = y or no relation can be established

25. I. x2 + 19x + 70 = 0
II. y2 - 9y - 22 = 0
(A) x > y (B) x < y (C) x ≥ y
(D) x ≤ y (E) x = y or no relation can be established

26. I. 5x2 - 36x + 63 = 0


II. 7y2 + 6y - 45 = 0
(A) x > y (B) x < y (C) x ≥ y
(D) x ≤ y (E) x = y or no relation can be established

27. I. x2 - 31x + 238 = 0


II. y2 - 22y + 120 = 0
(A) x > y (B) x < y (C) x ≥ y
(D) x ≤ y (E) x = y or no relation can be established

28. I. `x2 + 27x + 140 = 0


II. y2 + 2y - 35 = 0
(A) x > y (B) x < y (C) x ≥ y
(D) x ≤ y (E) x = y or no relation can be established

29. I. x2 + 5x - 36 = 0
II. y2 - 36y + 320 = 0
(A) x > y (B) x < y (C) x ≥ y
(D) x ≤ y (E) x = y or no relation can be established

30. I. x2 + 3x - 40 = 0
II. y2 - 16y + 55 = 0
(A) x > y (B) x < y (C) x ≥ y
(D) x ≤ y (E) x = y or no relation can be established

https :
//www. https : https
https : //instagra
//youtube.c
facebo m.com/aas :
om/channe hisharoraso
l/UCYa4_Jr ok.com cial(?) //t.m
Orf8R5Kz2u
/aashis utm_mediu
e/stu

24
OtccXQ m=
haroras copy_link
ocial dified
31. I. 8x2 + 26x - 7 = 0
II. 3y2 - 35y + 50 = 0
(A) x > y (B) x < y (C) x ≥ y
(D) x ≤ y (E) x = y or no relation can be established

32. I. x2 - 4x - 117 = 0
II. y2 - 29y + 208 = 0
(A) x > y (B) x < y (C) x ≥ y
(D) x ≤ y (E) x = y or no relation can be established

33. I. (x + 25) /11 = - 14/x


II. 3y - (10y - 20) /y = 7
(A) x > y (B) x < y (C) x ≥ y
(D) x ≤ y (E) x = y or no relation can be established

34. I. x2 - 7x - 18 = 0
II. y2 - 9y + 18 = 0
(A) x > y (B) x < y (C) x ≥ y
(D) x ≤ y (E) x = y or no relation can be established

35. I. x2 + 12x - 45 = 0
II. y2 - 14y + 33 = 0
(A) x > y (B) x < y (C) x ≥ y
(D) x ≤ y (E) x = y or no relation can be established

36. I. 2x2 - 17x + 35 = 0


II. 3y2 - 25y + 28 = 0
(A) x > y (B) x < y (C) x ≥ y
(D) x ≤ y (E) x = y or no relation can be established

37. I. x2 + 62x + 952 = 0


II. y2 + 51y + 644 = 0
(A) x > y (B) x < y (C) x ≥ y
(D) x ≤ y (E) x = y or no relation can be established

38. I. x2 + 28x + 192 = 0


II. y2 + 4y - 96 = 0
(A) x > y (B) x < y (C) x ≥ y
(D) x ≤ y (E) x = y or no relation can be established

https :
//www. https : https
https : //instagra
//youtube.c
facebo m.com/aas :
om/channe hisharoraso
l/UCYa4_Jr ok.com cial(?) //t.m
Orf8R5Kz2u
/aashis utm_mediu
e/stu

25
OtccXQ m=
haroras copy_link
ocial dified
39. I. 6x + 5y = 45
II. 4x + 8y = 44
(A) x > y (B) x < y (C) x ≥ y
(D) x ≤ y (E) x = y or no relation can be established

40. I. x2 - 39x + 378 = 0


II. y2 - 5y - 84 = 0
(A) x > y (B) x < y (C) x ≥ y
(D) x ≤ y (E) x = y or no relation can be established

41. I. 3x2 - 29x + 18 = 0


II. y2 - 2y - 8 = 0
(A) x > y (B) x < y (C) x ≥ y
(D) x ≤ y (E) x = y or no relation can be established

42. I. x2 + 4y - 45 = 0
II. y2 + 24y + 119 = 0
(A) x > y (B) x < y (C) x ≥ y
(D) x ≤ y (E) x = y or no relation can be established

43. I. 5x2 + 19x + 18 = 0


II. 5y2 - 22y + 17 = 0
(A) x > y (B) x < y (C) x ≥ y
(D) x ≤ y (E) x = y or no relation can be established

44. I. x2 - 21x + 108 = 0


II. y2 - 29y + 204 = 0
(A) x > y (B) x < y (C) x ≥ y
(D) x ≤ y (E) x = y or no relation can be established

45. I. x2 - 3x - 54 = 0
II. 2y2 - 16y + 24 = 0
(A) x > y (B) x < y (C) x ≥ y
(D) x ≤ y (E) x = y or no relation can be established

46. I. 3x + 4y = 44
II. 7x + 3y = 71
(A) x > y (B) x < y (C) x ≥ y
(D) x ≤ y (E) x = y or no relation can be established

https :
//www. https : https
https : //instagra
//youtube.c
facebo m.com/aas :
om/channe hisharoraso
l/UCYa4_Jr ok.com cial(?) //t.m
Orf8R5Kz2u
/aashis utm_mediu
e/stu

26
OtccXQ m=
haroras copy_link
ocial dified
47. I. x2 = 784
II. y = √1521
(A) x > y (B) x < y (C) x ≥ y
(D) x ≤ y (E) x = y or no relation can be established

48. I. x2 - 40x + 391 = 0


II. y2 - 50y + 621 = 0
(A) x > y (B) x < y (C) x ≥ y
(D) x ≤ y (E) x = y or no relation can be established

49. I. x2 + 9x - 136 = 0
II. y2 - 27y + 152 = 0
(A) x > y (B) x < y (C) x ≥ y
(D) x ≤ y (E) x = y or no relation can be established

50. I. x2 - 17x + 70 = 0
II. y2 - 25y + 84 = 0
(A) x > y (B) x < y (C) x ≥ y
(D) x ≤ y (E) x = y or no relation can be established

51. I. 3x2 - 21x + 36 = 0


II. 5y2 - 16y + 12 = 0
(A) x > y (B) x < y (C) x ≥ y
(D) x ≤ y (E) x = y or no relation can be established

52. I. 3x + 5y = 46
II. 2x + 3y = 29
(A) x > y (B) x < y (C) x ≥ y
(D) x ≤ y (E) x = y or no relation can be established

53. I. x3 = 729
II. y = √144
(A) x > y (B) x < y (C) x ≥ y
(D) x ≤ y (E) x = y or no relation can be established

54. I. x2 + 6x - 91 = 0
II. y2 - 23y + 112 = 0
(A) x > y (B) x < y (C) x ≥ y
(D) x ≤ y (E) x = y or no relation can be established

https :
//www. https : https
https : //instagra
//youtube.c
facebo m.com/aas :
om/channe hisharoraso
l/UCYa4_Jr ok.com cial(?) //t.m
Orf8R5Kz2u
/aashis utm_mediu
e/stu

27
OtccXQ m=
haroras copy_link
ocial dified
55. I. 2x2 - 31x + 84 = 0
II. 3y2 - 36y + 96 = 0
(A) x > y (B) x < y (C) x ≥ y
(D) x ≤ y (E) x = y or no relation can be established

56. I. 4x2 - 32x + 63 = 0


II. 2y2 + 11y - 63 = 0
(A) x > y (B) x < y (C) x ≥ y
(D) x ≤ y (E) x = y or no relation can be established

57. I. x2 + x - 156 = 0
II. y2 = 64
(A) x > y (B) x < y (C) x ≥ y
(D) x ≤ y (E) x = y or no relation can be established

58. I. 21x2 - 13x + 2 = 0


II. 25y2 - 30y + 8 = 0
(A) x > y (B) x < y (C) x ≥ y
(D) x ≤ y (E) x = y or no relation can be established

59. I. 2x2 - 7x + 6 = 0
II. 35y2 - 4y - 15 = 0
(A) x > y (B) x < y (C) x ≥ y
(D) x ≤ y (E) x = y or no relation can be established

60. I. x2 - x - 132 = 0
II. y2 = 1225
(A) x > y (B) x < y (C) x ≥ y
(D) x ≤ y (E) x = y or no relation can be established

61. I. 10x2 - 19x + 6 = 0


II. y2 - 2y + 120 = 0
(A) x > y (B) x < y (C) x ≥ y
(D) x ≤ y (E) x = y or no relation can be established

62. I. x2 + 26x + 48 = 0
II. y2 - 20y - 44 = 0
(A) x > y (B) x < y (C) x ≥ y
(D) x ≤ y (E) x = y or no relation can be established

https :
//www. https : https
https : //instagra
//youtube.c
facebo m.com/aas :
om/channe hisharoraso
l/UCYa4_Jr ok.com cial(?) //t.m
Orf8R5Kz2u
/aashis utm_mediu
e/stu

28
OtccXQ m=
haroras copy_link
ocial dified
63. I. x2 - 4x - 77 = 0
II. y2 + 17y + 70 = 0
(A) x > y (B) x < y (C) x ≥ y
(D) x ≤ y (E) x = y or no relation can be established

64. I. x2 - 9x + 18 = 0
II. y2 + y - 12 = 0
(A) x > y (B) x < y (C) x ≥ y
(D) x ≤ y (E) x = y or no relation can be established

65. I. 15x2 + 7x - 4 = 0
II. y2 - 15y + 36 = 0
(A) x > y (B) x < y (C) x ≥ y
(D) x ≤ y (E) x = y or no relation can be established

https :
//www. https : https
https : //instagra
//youtube.c
facebo m.com/aas :
om/channe hisharoraso
l/UCYa4_Jr ok.com cial(?) //t.m
Orf8R5Kz2u
/aashis utm_mediu
e/stu

29
OtccXQ m=
haroras copy_link
ocial dified
1. Ans. (B) y = 9, 12
3x + 4y = 90 No relation can be established
5x + 3y = 95
y = (90 - 3x) /4 = (95 - 5x) /3 5. Ans. (B)
3 × (90 - 3x) = 4 × (95 - 5x) 3x2 - 34x + 63 = 0
270 - 9x = 380 - 20x (3x - 7) (x - 9) = 0
11x = 110 x = 7/3, 9
x = 10
y = (90 - 3 × 10) /4 y2 - 59y + 864 = 0
y = 60/4 = 15 (y - 27) (y - 32) = 0
x<y y = 27, 32
x<y
2. Ans. (E)
x2 = 1521
x = ± 39 6. Ans. (B)
5x2 - 24x + 28 = 0
y3 = 4096 (x - 2) (5x - 14) = 0
y = 16 x = 2, 14/5
No relation can be established
3y2 - 21y + 36 = 0
3. Ans. (C) 3(y - 3) (y - 4) = 0
x2 - 57x + 656 = 0 y = 3, 4
(x - 16) (x - 41) = 0 x<y
x = 16, 41
7. Ans. (D)
y2 - 30y + 224 = 0 x2 + 32x + 231 = 0
(y - 14) (y - 16) = 0 (x + 21) (x + 11) = 0
y = 14, 16 x = - 21, - 11
x≥y
y2 + 3y - 88 = 0
4. Ans. (E) (x + 11) (x - 8) = 0
x2 - 23x - 420 = 0 x = - 11, 8
(x + 12) (x - 35) = 0 x≤y
x = - 12, 35
8. Ans. (B)
y2 - 21y + 108 = 0 3x2 + 31x + 78 = 0
(y - 9) (y - 12) = 0 (x + 6) (3x + 13) = 0

https :
//www. https : https
https : //instagra
//youtube.c
facebo m.com/aas :
om/channe hisharoraso
l/UCYa4_Jr ok.com cial(?) //t.m
Orf8R5Kz2u
/aashis utm_mediu
e/stu

30
OtccXQ m=
haroras copy_link
ocial dified
x = - 6, - 13/3 12. Ans. (D)
x2 - 4x - 117 = 0
3y2 + 15y + 18 = 0 (x + 9) (x - 13) = 0
3(y + 3) (y + 2) = 0 x = - 9, 13
y = - 3, - 2
x<y y2 - 29y + 208 = 0
(y - 13) (y - 16) = 0
9. Ans. (A) y = 13, 16
2x2 - 32x + 128 = 0 x≤y
2(x - 8) 2 = 0
x=8 13. Ans. (B)
(x + 25) /11 = - 14/x
2y2 - 28y + 98 = 0 x2 + 25x = - 154
2(y - 7) 2 = 0 x2 + 25x + 154 = 0
y=7 (x + 14) (x + 11) = 0
x>y x = - 14, - 11

10. Ans. (E) 3y - (10y - 20) /y = 6


7x2 - 30x + 27 = 0 3y2 - 10y + 20 = 6y
(7x - 9) (x - 3) = 0 3y2 - 16y + 20 = 0
x = 9/7, 3 (y - 2) (3y - 10) = 0
y = 2, 10/3
9y2 - 30y + 24 = 0 x<y
3(3y - 4) (y - 2) = 0
y = 4/3, 2 14. Ans. (E)
No relation can be established x2 - 7x - 18 = 0
(x + 2) (x - 9) = 0
11. Ans. (A) x = - 2, 9
x2 - 19x + 84 = 0
(x - 7) (x - 12) = 0 y2 - 9y + 18 = 0
x = 7, 12 (y - 3) (y - 6) = 0
y = 3, 6
25y2 - 45y + 18 = 0 No relation can be established
(5y - 3) (5y - 6) = 0
y = 3/5, 6/5 15. Ans. (D)
x>y x2 + 12x - 45 = 0
(x + 15) (x - 3) = 0

https :
//www. https : https
https : //instagra
//youtube.c
facebo m.com/aas :
om/channe hisharoraso
l/UCYa4_Jr ok.com cial(?) //t.m
Orf8R5Kz2u
/aashis utm_mediu
e/stu

31
OtccXQ m=
haroras copy_link
ocial dified
x = - 15, 3 y2 - (5/4) y + 3/8 = 0
8y2 - 10y + 3 = 0
y2 - 14y + 33 = 0 (2y - 1) (4y - 3) = 0
(y - 3) (y - 11) = 0 y = 1/2, 3/4
y = 3, 11 x≥y
x≤y
19. Ans. (A)
16. Ans. (E) x2 - 20x + 99 = 0
x2 - 21x + 104 = 0 (x - 9) (x - 11) = 0
(x - 8) (x - 13) = 0 x = 9, 11
x = 8, 13
y2 + 4y - 96 = 0
y2 - 22y - 104 = 0 (y + 12) (y - 8) = 0
(y + 4) (y - 26) = 0 y = - 12, 8
y = - 4, 26 x>y
No relation can be established
20. Ans. (C)
17. Ans. (A) 4x - 6 = 3(x + 2) - 3
6x(x - 2) - x + 6 = 0 4x - 6 = 3x + 6 - 3
6x2 - 12x - x + 6 = 0 4x - 3x = 6 + 3
6x2 - 13x + 6 = 0 x=9
(3x - 2) (2x - 3) = 0
x = 2/3, 3/2 y2 - 33 = 48
y2 = 81
y2 - (5/6) y + 1/6 = 0 y=±9
6y2 - 5y + 1 = 0 x≥y
(3y - 1) (2y - 1) = 0
y = 1/3, 1/2 21. Ans. (E)
x>y x2 + 21x + 90 = 0
(x + 15) (x + 6) = 0
18. Ans. (C) x = - 15, - 6
4x(x - 12) + x + 33 = 0
4x2 - 48x + x + 33 = 0 y2 + 6y - 91 = 0
4x2 - 47x + 33 = 0 (y + 13) (y - 7) = 0
(4x - 3) (x - 11) = 0 y = - 13, 7
x = 3/4, 11 No relationship can be established

https :
//www. https : https
https : //instagra
//youtube.c
facebo m.com/aas :
om/channe hisharoraso
l/UCYa4_Jr ok.com cial(?) //t.m
Orf8R5Kz2u
/aashis utm_mediu
e/stu

32
OtccXQ m=
haroras copy_link
ocial dified
22. Ans. (B) x<y
x2 + 25x + 66 = 0
(x + 22) (x + 3) = 0 26. Ans. (A)
x = - 22, - 3 5x2 - 36x + 63 = 0
(x - 3) (5x - 21) = 0
y2 - 5y - 14 = 0 x = 3, 21/5
(x + 2) (x - 7) = 0
x = - 2, 7 7y2 + 6y - 45 = 0
x<y (y + 3) (7y - 15) = 0
y = - 3, 15/7
23. Ans. (C) x>y
x2 - 81 = 0
x2 = 81 27. Ans. (A)
x=±9 x2 - 31x + 238 = 0
(x - 14) (x - 17) = 0
y2 + 30y + 189 = 0 x = 14, 17
(y + 21) (y + 9) = 0
y = - 21, - 9 y2 - 22y + 120 = 0
x≥y (y - 10) (y - 12) = 0
y = 10, 12
24. Ans. (C) x>y
x2 - 14x + 49 = 0
(x - 7) 2 = 0 28. Ans. (D)
x=7 x2 + 27x + 140 = 0
y2 - 12y + 35 = 0 (x + 20) (x + 7) = 0
(y - 5) (y - 7) = 0 x = - 20, - 7
y = 5, 7
x≥y y2 + 2y - 35 = 0
(y + 7) (y - 5) = 0
25. Ans. (B) y = - 7, 5
x2 + 19x + 70 = 0 x≤y
(x + 14) (x + 5) = 0
x = - 14, - 5 29. Ans. (B)
x2 + 5x - 36 = 0
y2 - 9y - 22 = 0 (x + 9) (x - 4) = 0
(y + 2) (y - 11) = 0 x = - 9, 4
y = - 2, 11

https :
//www. https : https
https : //instagra
//youtube.c
facebo m.com/aas :
om/channe hisharoraso
l/UCYa4_Jr ok.com cial(?) //t.m
Orf8R5Kz2u
/aashis utm_mediu
e/stu

33
OtccXQ m=
haroras copy_link
ocial dified
y2 - 36y + 320 = 0 (x + 25) /11 = - 14/x
(y - 16) (y - 20) = 0 x2 + 25x = - 154
y = 16, 20 x2 + 25x + 154 = 0
x<y (x + 14) (x + 11) = 0
x = - 14, - 11
30. Ans. (D)
x2 + 3x - 40 = 0 3y - (10y - 20) /y = 7
(x + 8) (x - 5) = 0 3y2 - 10y + 20 = 7y
x = - 8, 5 3y2 - 17y + 20 = 0
(3y - 5) (y - 4) = 0
y2 - 16y + 55 = 0 y = 5/3, 4
(y - 5) (y - 11) = 0 x<y
y = 5, 11
x≤y 34. Ans. (E)
x2 - 7x - 18 = 0
31. Ans. (B) (x + 2) (x - 9) = 0
8x2 + 6x - 7 = 0 x = - 2, 9
(2x + 7) (4x - 1) = 0
x = - 7/2, 1/4 y2 - 9y + 18 = 0
(y - 3) (y - 6) = 0
3y2 - 35y + 50 = 0 y = 3, 6
(3y - 5) (y - 10) = 0 No relation can be established
y = 5/3, 10
x<y 35. Ans. (D)
x2 + 12x - 45 = 0
32. Ans. (D) (x + 15) (x - 3) = 0
x2 - 4x - 117 = 0 x = - 15, 3
(x + 9) (x - 13) = 0
x = - 9, 13 y2 - 14y + 33 = 0
(y - 3) (y - 11) = 0
y2 - 29y + 208 = 0 y = 3, 11
(y - 13) (y - 16) = 0 x≤y
y = 13, 16
x≤y

33. Ans. (B)

https :
//www. https : https
https : //instagra
//youtube.c
facebo m.com/aas :
om/channe hisharoraso
l/UCYa4_Jr ok.com cial(?) //t.m
Orf8R5Kz2u
/aashis utm_mediu
e/stu

34
OtccXQ m=
haroras copy_link
ocial dified
36. Ans. (E) y = (45 - 6 × 5) /5
2x2 - 17x + 35 = 0 y = (45 - 30) /5 = 15/5 = 3
(2x - 7) (x - 5) = 0 x>y
x = 7/2, 5
40. Ans. (A)
3y2 - 25y + 28 = 0 x2 - 39x + 378 = 0
(3y - 4) (y - 7) = 0 (x - 18) (x - 21) = 0
y = 4/3, 7 x = 18, 21
No relation can be established
y2 - 5y - 84 = 0
37. Ans. (D) (y + 7) (y - 12) = 0
x2 + 62x + 952 = 0 y = - 7, 12
(x + 34) (x + 28) = 0 x>y
x = - 34, - 28

y2 + 51y + 644 = 0 41. Ans. (E)


(y + 28) (y + 23) = 0 3x2 - 29x + 18 = 0
y = - 28, - 23 (3x - 2) (x - 9) = 0
x≤y x = 2/3, 9

38. Ans. (D) y2 - 2y - 8 = 0


x2 + 28x + 192 = 0 (y + 2) (y - 4) = 0
(x + 16) (x + 12) = 0 y = - 2, 4
x = - 16, - 12 No relation can be established

y2 + 4y - 96 = 0 42. Ans. (E)


(y + 12) (y - 8) = 0 x2 + 4x - 45 = 0
y = - 12, 8 (x + 9) (x - 5) = 0
x≤y x = - 9, 5

39. Ans. (A) y2 + 24y + 119 = 0


6x + 5y = 45 (y + 17) (y + 7) = 0
4x + 8y = 44 y = - 17, - 7
y = (45 - 6x) /5 = (44 - 4x) /8 No relation can be established
360 - 48x = 220 - 20x
28x = 140 43. Ans. (B)
x=5

https :
//www. https : https
https : //instagra
//youtube.c
facebo m.com/aas :
om/channe hisharoraso
l/UCYa4_Jr ok.com cial(?) //t.m
Orf8R5Kz2u
/aashis utm_mediu
e/stu

35
OtccXQ m=
haroras copy_link
ocial dified
5x2 + 19x + 18 = 0 y = (44 - 3 × 8) /4 = (44 - 24) /4 = 20/4
(x + 2) (5x + 9) = 0 =5
x = - 2, - 9/5 x>y

5y2 - 22y + 17 = 0 47. Ans. (B)


(y - 1) (5x - 17) = 0 x2 = 784
y = 1, 17/5 x = ± 28
x<y
y = √1521
44. Ans. (D) y = 39
x2 - 21x + 108 = 0 x<y
(x - 9) (x - 12) = 0
x = 9, 12 48. Ans. (D)
x2 - 40x + 391 = 0
y2 - 29y + 204 = 0 (x - 17) (x - 23) = 0
(y - 12) (y - 17) = 0 x = 17, 23
y = 12, 17
x≤y y2 - 50y + 621 = 0
(y - 23) (y - 27) = 0
45. Ans. (E) y = 23, 27
x2 - 3x - 54 = 0 x≤y
(x + 6) (x - 9) = 0
x = - 6, 9 49. Ans. (D)
x2 + 9x - 136 = 0
2y2 - 16y + 24 = 0 (x + 17) (x - 8) = 0
2(y - 2) (y - 6) = 0 x = - 17, 8
y = 2, 6
No relation can be established y2 - 27y + 152 = 0
(y - 8) (y - 19) = 0
46. Ans. (A) y = 8, 19
3x + 4y = 44 x≤y
7x + 3y = 71
y = (44 - 3x) /4 = (71 - 7x) /3 50. Ans. (E)
3 × (44 - 3x) = 4 × (71 - 7x) x2 - 17x + 70 = 0
132 - 9x = 284 - 28x (x - 7) (x - 10) = 0
19x = 152 x = 7, 10
x=8

https :
//www. https : https
https : //instagra
//youtube.c
facebo m.com/aas :
om/channe hisharoraso
l/UCYa4_Jr ok.com cial(?) //t.m
Orf8R5Kz2u
/aashis utm_mediu
e/stu

36
OtccXQ m=
haroras copy_link
ocial dified
y2 - 25y + 84 = 0 54. Ans. (D)
(y - 4) (y - 21) = 0 x2 + 6x - 91 = 0
y = 4, 21 (x + 13) (x - 7) = 0
No relation can be established x = - 13, 7

51. Ans. (A) y2 - 23y + 112 = 0


3x2 - 21x + 36 = 0 (y - 7) (y - 16) = 0
3(x - 3) (x - 4) = 0 y = 7, 16
x = 3, 4 x≤y

5y2 - 16y + 12 = 0 55. Ans. (E)


(5y - 6) (y - 2) = 0 2x2 - 31x + 84 = 0
y = 6/5, 2 (2x - 7) (x - 12) = 0
x>y x = 7/2, 12

52. Ans. (A) 3y2 - 36y + 96 = 0


3x + 5y = 46 3(y - 4) (y - 8) = 0
2x + 3y = 29 y = 4, 8
y = (46 - 3x) /5 = (29 - 2x) /3 No relation can be established
3 × (46 - 3x) = 5 × (29 - 2x)
138 - 9x = 145 - 10x
x=7 56. Ans. (C)
y = (46 - 3 × 7) /5 = (46 - 21) /5 = 25/5 4x2 - 32x + 63 = 0
=5 (2x - 7) (2x - 9) = 0
x>y x = 7/2, 9/2

53. Ans. (B) 2y2 + 11y - 63 = 0


x3 = 729 (y + 9) (2y - 7) = 0
x=9 y = - 9, 7/2
x≥y
y = √144
y = 12 57. Ans. (E)
x<y x2 + x - 156 = 0
(x + 13) (x - 12) = 0
x = - 13, 12

https :
//www. https : https
https : //instagra
//youtube.c
facebo m.com/aas :
om/channe hisharoraso
l/UCYa4_Jr ok.com cial(?) //t.m
Orf8R5Kz2u
/aashis utm_mediu
e/stu

37
OtccXQ m=
haroras copy_link
ocial dified
y2 = 64 61. Ans. (E)
y=±8 10x2 - 19x + 6 = 0
No relation can be established (5x - 2) (2x - 3) = 0
x = 2/5, 3/2
58. Ans. (B)
21x2 - 13x + 2 = 0 y2 - 2y + 120 = 0
(7x - 2) (3x - 1) = 0 (y + 10) (y - 12) = 0
x = 2/7, 1/3 y = - 10, 12
No relation can be established
25y2 - 30y + 8 = 0
(5y - 2) (5y - 4) = 0 62. Ans. (D)
y = 2/5, 4/5 x2 + 26x + 48 = 0
x<y (x + 24) (x + 2) = 0
x = - 24, - 2
59. Ans. (A)
2x2 - 7x + 6 = 0 y2 - 20y - 44 = 0
(2x - 3) (x - 2) = 0 (y + 2) (y - 22) = 0
x = 3/2, 2 y = - 2, 22
x≤y
35y2 - 4y - 15 = 0
(5y + 3) (7y - 5) = 0 63. Ans. (C)
y = - 3/5, 5/7 x2 - 4x - 77 = 0
x>y (x + 7) (x - 11) = 0
x = - 7, 11
60. Ans. (E)
x2 - x - 132 = 0 y2 + 17y + 70 = 0
(x + 11) (x - 12) = 0 (y + 10) (y + 7) = 0
x = - 11, 12 y = - 10, - 7
x≥y
y2 = 1225
y = ± 35 64. Ans. (C)
No relation can be established x2 - 9x + 18 = 0
(x - 3) (x - 6) = 0
x = 3, 6

https :
//www. https : https
https : //instagra
//youtube.c
facebo m.com/aas :
om/channe hisharoraso
l/UCYa4_Jr ok.com cial(?) //t.m
Orf8R5Kz2u
/aashis utm_mediu
e/stu

38
OtccXQ m=
haroras copy_link
ocial dified
y2 + y - 12 = 0
(y + 4) (y - 3) = 0
y = - 4, 3
x≥y

65. Ans. (B)


15x2 + 7x - 4 = 0
(5x + 4) (3x - 1) = 0
x = - 4/5, 1/3

y2 - 15y + 36 = 0
(y - 3) (y - 12) = 0
y = 3, 12
x<y

https :
//www. https : https
https : //instagra
//youtube.c
facebo m.com/aas :
om/channe hisharoraso
l/UCYa4_Jr ok.com cial(?) //t.m
Orf8R5Kz2u
/aashis utm_mediu
e/stu

39
OtccXQ m=
haroras copy_link
ocial dified
https :
//instagram.com/aashisharorasocia
l(?) utm_medium = copy_link

https: //t.me/studified
https : //t.me/studified

https :
//youtube.com/channel/UCYa
4_JrOrf8R5Kz2uOtccXQ

https :
//www.facebook.com/aa
shisharorasocial

https :
//www. https : https
https : //instagra
//youtube.c
facebo m.com/aas :
om/channe hisharoraso
l/UCYa4_Jr ok.com cial(?) //t.m
Orf8R5Kz2u
/aashis utm_mediu
e/stu

40
OtccXQ m=
haroras copy_link
ocial dified
In each of these questions a number is missing in the series. Find out the
missing number.
इनमें से प्रत्येक प्रश्न श्ख
ंर ला में एक संख्या लुप्त है , लुप्त संख्या ज्ञात कीजजए।

1) 75, 90, 108, ?, 155.52, 186.624


(A) 79 (B) 163 (C) 129.6 (D) 182 (E) None of these

2) 2, 5, 12, 23, ?, 57
(A) 38 (B) 22 (C) 31 (D) 21 (E) None of these

3) 6, 6, 12, 36, 144, ?


(A) 292 (B) 720 (C) 830 (D) 700 (E) None of these

4) 0.5, 1, 3, 15, 105, ?


(A) 1155 (B) 412 (C) 210 (D) 150 (E) None of these

5) 12, 13, 21, 48, ?


(A) 56 (B) 48 (C) 112 (D) 348 (E) None of these

6) 2, 4, 16, 96, ?
(A) 79 (B) 163 (C) 768 (D) 182 (E) None of these

7) 3.5, 7, 21, 84, ?, 2520


(A) 420 (B) 220 (C) 310 (D) 210 (E) None of these

8) 7, 4, 5, 12, 52, ?
(A) 292 (B) 424 (C) 830 (D) 700 (E) None of these

9) 12, 19, 31, 53, 90, ?


(A) 147 (B) 412 (C) 210 (D) 150 (E) None of these

10) 16, 20, - 7, 9, - 116, ?


(A) -56 (B) -48 (C) -80 (D) -48 (E) None of these

11) 411, 434, 464, 501, 545, ?, 654


(A) 326 (B) 296 (C) 596 (D) 490 (E) None of these

https :
//www. https : https
https : //instagra
//youtube.c
facebo m.com/aas :
om/channe hisharoraso
l/UCYa4_Jr ok.com cial(?) //t.m
Orf8R5Kz2u
/aashis utm_mediu
e/stu

41
OtccXQ m=
haroras copy_link
ocial dified
12. 614, 445, 324, 275, ?, 241, 237
(A) 250 (B) 252 (C) 210 (D) 425 (E) None of these

13. 3, 6, 11, 18, ?, 38


(A) 42 (B) 27 (C) 41 (D) 39 (E) None of these

14. 2, 6, 30, 210, 1890, ?


(A) 20790 (B) 1890 (C) 5200 (D) 6850 (E) None of these

15. 7, 4, 5, ?, 20, 52.5, 160.5


(A) 7 (B) 4 (C) 9 (D) 6 (E) None of these

16. 290, ?, 152, 91, 32, - 21


(A) 324 (B) 290 (C) 219 (D) 419 (E) None of these

17. 37, ?, 52, 67, 87, 112, 142


(A) 42 (B) 19 (C) 21 (D) 47 (E) None of these

18. 32, 80, ?, 500, 1250, 3125


(A) 420 (B) 200 (C) 410 (D) 391 (E) None of these

19. 106, 115, ?, 189, 270, 391


(A) 140 (B) 115 (C) 152 (D) 168 (E) None of these

20. 7, 9, 12, 17, 24, ?


(A) 33 (B) 38 (C) 35 (D) 46 (E) None of these

21. 197, 205, ?, 485, 997, 1997, 3725


(A) 79 (B) 163 (C) 269 (D) 182 (E) None of these

22. 217, 345, 515, 733, 1005, ?


(A) 1337 (B) 228 (C) 3112 (D) 2115 (E) None of these

23. 830, 911, 1032, 1201, 1426, ?


(A) 1292 (B) 1715 (C) 1830 (D) 700 (E) None of these

24. 32, 96, 336, 1344, 6048, ?


(A) 30240 (B) 41240 (C) 21050 (D) 15000 (E) None of these

25. 13, 15.1, 19.3, 25.6, 34, ?, 57.1


(A) 56.5 (B) 48.5 (C) 44.5 (D) 34.8 (E) None of these

https :
//www. https : https
https : //instagra
//youtube.c
facebo m.com/aas :
om/channe hisharoraso
l/UCYa4_Jr ok.com cial(?) //t.m
Orf8R5Kz2u
/aashis utm_mediu
e/stu

42
OtccXQ m=
haroras copy_link
ocial dified
26. 3, 9, 36, ?, 1080, 7560
(A) 79 (B) 163 (C) 180 (D) 182 (E) None of these

27. 420, 445, 345, 570, 170, 795, ?


(A) -105 (B) -122 (C) -131 (D) -120 (E) None of these

28. - 18, - 17, - 13, - 4, ?, 37


(A) 22 (B) 12 (C) 20 (D) 70 (E) None of these

29. 2, 5, 10, 17, 26, ?


(A) 37 (B) 41 (C) 21 (D) 15 (E) None of these

30. 136, 138, 132, 144, 124, ?, 112


(A) 56 (B) 48 (C) 154 (D) 348 (E) None of these

31. 11, 47, ?, 152, 242, 422


(A) 65 (B) 83 (C) 92 (D) 78 (E) None of these

32. 1584, 880, 416, ?, 16, - 16


(A) 289 (B) 144 (C) 324 (D) 361 (E) None of these

33. 56, 42, 30, ?, 12, 6


(A) 24 (B) 26 (C) 28 (D) 20 (E) None of these

34. 107, 112, 119, 130, ?, 160


(A) 143 (B) 152 (C) 139 (D) 158 (E) None of these

35. 729, 594, ?, 450, 423, 414


(A) 543 (B) 478 (C) 504 (D) 487 (E) None of these

36. 32, 33.5, ?, 42.5, 2, 123.5


(A) 62 (B) 18 (C) 29 (D) 34 (E) None of these

37. 84, 102, 66, 120, ?, 138


(A) 129 (B) 48 (C) 112 (D) 83 (E) None of these

38. 2731, 683, 171, ?, 11, 3


(A) 82 (B) 96 (C) 122 (D) 43 (E) None of these

39. 5, 9, 21, ?, 165, 489


(A) 57 (B) 89 (C) 65 (D) 74 (E) None of these

https :
//www. https : https
https : //instagra
//youtube.c
facebo m.com/aas :
om/channe hisharoraso
l/UCYa4_Jr ok.com cial(?) //t.m
Orf8R5Kz2u
/aashis utm_mediu
e/stu

43
OtccXQ m=
haroras copy_link
ocial dified
40. 101, 103, ?, 109, 89, 119
(A) 74 (B) 86 (C) 97 (D) 62 (E) None of these

41. 288, 144, 144, ?, 432, 1080


(A) 200 (B) 250 (C) 216 (D) 432 (E) None of these

42. 11, 20, - 44, - 19, - 235, ?


(A) -100 (B) -186 (C) -135 (D) -150 (E) None of these

43. 122, 186, 698, ?, 6522, 14522, 28346


(A) 2426 (B) 698 (C) 2500 (D) 2656 (E) None of these

44. 162, 164, 158, 170, ?, 180, 138


(A) 160 (B) 170 (C) 150 (D) 180 (E) None of these

45. 825, 761, 729, ?, 705, 701, 699


(A) 729 (B) 713 (C) 705 (D) 700 (E) None of these

46. 728, 242, 80, ?, 8, 2


(A) 62 (B) 48 (C) 26 (D) 54 (E) None of these

47. 3, 8, 27, 112, ?, 3396


(A) 418 (B) 565 (C) 369 (D) 641 (E) None of these

48. 94, 98, 114, 178, ?, 1458


(A) 442 (B) 426 (C) 418 (D) 434 (E) None of these

49. 16, 19, 44, ?, 576, 2895


(A) 141 (B) 119 (C) 131 (D) 128 (E) None of these

50. 2, 7, 31, ?, 192, 357


(A) 74 (B) 96 (C) 88 (D) 62 (E) None of these

https :
//www. https : https
https : //instagra
//youtube.c
facebo m.com/aas :
om/channe hisharoraso
l/UCYa4_Jr ok.com cial(?) //t.m
Orf8R5Kz2u
/aashis utm_mediu
e/stu

44
OtccXQ m=
haroras copy_link
ocial dified
1. Ans. (C) 6. Ans. (C)
75 × 1.2 = 90 2×2=4
90 × 1.2 = 108 4 × 4 = 16
108 × 1.2 = 129.6 16 × 6 = 96
129.6 × 1.2 = 155.52 96 × 8 = 768
155.52 × 1.2 = 186.624
7. Ans. (A)
2. Ans. (A) 3.5 × 2 = 7
5-2=3 7 × 3 = 21
12 - 5 = 7 (3 + 4) 21 × 4 = 84
23 - 12 = 11 (7 + 4) 84 × 5 = 420
38 - 23 = 15 (15 + 4) 420 × 6 = 2520
57 - 38 = 19 (15 + 4)
8. Ans. (B)
3. Ans. (B) (7 + 1) × 0.5 = 4
6×1=6 (4 + 1) × 1 = 5
6 × 2 = 12 (5 + 1) × 2 = 12
12 × 3 = 36 (12 + 1) × 4 = 52
36 × 4 = 144 (52 + 1) × 8 = 424
144 × 5 = 720
9. Ans. (A)
4. Ans. (A) 19 - 12 = 7
0.5 × 2 = 1 31 - 19 = 12 (7 + 5 × 1)
1×3=3 53 - 31 = 22 (12 + 5 × 2)
3 × 5 = 15 90 - 53 = 37 (22 + 5 × 3)
15 × 7 = 105 147 - 90 = 57(37 + 5 × 4)
105 × 11 = 1155
10. Ans. (C)
5. Ans. (C) 16 + 22 = 20
12 + 13 = 13 20 - 33 = - 7
13 + 23 = 21 - 7 + 42 = 9
21 + 33 = 48 9 - 53 = - 116
48 + 43 = 112 - 116 + 62 = - 80

https :
//www. https : https
https : //instagra
//youtube.c
facebo m.com/aas :
om/channe hisharoraso
l/UCYa4_Jr ok.com cial(?) //t.m
Orf8R5Kz2u
/aashis utm_mediu
e/stu

45
OtccXQ m=
haroras copy_link
ocial dified
11. Ans. (C) 16. Ans. (C)
411 + 23 = 434 290 - 71 = 219
434 + 30 = 464 219 - 67 = 152
464 + 37 = 501 152 - 61 = 91
501 + 44 = 545 91 - 59 = 32
545 + 51 = 596 32 - 53 = - 21
596 + 58 = 654
17. Ans. (A)
12. Ans. (A) 37 + 5 × 1 = 42
614 - 132 = 445 42 + 5 × 2 = 52
445 - 112 = 324 52 + 5 × 3 = 67
324 - 72 = 275 67 + 5 × 4 = 87
275 - 52 = 250 87 + 5 × 5 = 112
250 - 32 = 241 112 + 5 × 6 = 142
241 - 22 = 237
18. Ans. (B)
13. Ans. (B) 32 × 2.5 = 80
3+3=6 80 × 2.5 = 200
6 + 5 = 11 200 × 2.5 = 500
11 + 7 = 18 500 × 2.5 = 1250
18 + 9 = 27 1250 × 2.5 = 3125
27 + 11 = 38
19. Ans. (A)
14. Ans. (A) 106 + 32 = 115
2×3=6 115 + 52 = 140
6 × 5 = 30 140 + 72 = 189
30 × 7 = 210 189 + 92 = 270
210 × 9 = 1890 270 + 112 = 391
1890 × 11 = 20790
20. Ans. (C)
15. Ans. (C) 7+2=9
(7 + 1) × 0.5 = 4 9 + 3 = 12
(4 + 1) × 1 = 5 12 + 5 = 17
(5 + 1) × 1.5 = 9 17 + 7 = 24
(9 + 1) × 2 = 20 24 + 11 = 35
(20 + 1) × 2.5 = 52.5
(52.5 + 1) × 3 = 160.5

https :
//www. https : https
https : //instagra
//youtube.c
facebo m.com/aas :
om/channe hisharoraso
l/UCYa4_Jr ok.com cial(?) //t.m
Orf8R5Kz2u
/aashis utm_mediu
e/stu

46
OtccXQ m=
haroras copy_link
ocial dified
21. Ans. (C) 26. Ans. (C)
197 + 23 = 205 3×3=9
205 + 43 = 269 9 × 4 = 36
269 + 63 = 485 36 × 5 = 180
485 + 83 = 997 180 × 6 = 1080
997 + 103 = 1997 1080 × 7 = 7560
1997 + 123 = 3725
27. Ans. (A)
22. Ans. (A) 420 + 52 = 445
63 + 1 = 217 445 - 102 = 345
73 + 2 = 345 345 + 152 = 570
83 + 3 = 515 570 - 202 = 170
93 + 4 = 733 170 + 252 = 795
103 + 5 = 1005 795 - 302 = - 105
113 + 6 = 1337
28. Ans. (B)
23. Ans. (B) - 18 + 12 = - 17
830 + 92 = 911 - 17 + 22 = - 13
911 + 112 = 1032 - 13 + 32 = - 4
1032 + 132 = 1201 - 4 + 42 = 12
1201 + 152 = 1426 12 + 52 = 37
1426 + 172 = 1715
29. Ans. (A)
24. Ans. (A) 2+3=5
32 × 3 = 96 5 + 5 = 10
96 × 3.5 = 336 10 + 7 = 17
336 × 4 = 1344 17 + 9 = 26
1344 × 4.5 = 6048 26 + 11 = 37
6048 × 5 = 30240
30. Ans. (C)
25. Ans. (C) 136 + 1 × 2 = 138
13 + 2.1 = 15.1 138 - 2 × 3 = 132
15.1 + 4.2 = 19.3 132 + 3 × 4 = 144
19.3 + 6.3 = 25.6 144 - 4 × 5 = 124
25.6 + 8.4 = 34 124 + 5 × 6 = 154
35 + 10.5 = 44.5 154 - 6 × 7 = 112
44.5 + 12.6 = 57.1

https :
//www. https : https
https : //instagra
//youtube.c
facebo m.com/aas :
om/channe hisharoraso
l/UCYa4_Jr ok.com cial(?) //t.m
Orf8R5Kz2u
/aashis utm_mediu
e/stu

47
OtccXQ m=
haroras copy_link
ocial dified
31. Ans. (C) 36. Ans. (C)
11 + 180 ÷ 5 = 47 32 + 1.5 × 30 = 33.5
47 + 180 ÷ 4 = 92 33.5 - 1.5 × 31 = 29
92 + 180 ÷ 3 = 152 29 + 1.5 × 32 = 42.5
152 + 180 ÷ 2 = 242 42.5 - 1.5 × 33 = 2
242 + 180 ÷ 1 = 422 2 + 1.5 × 34 = 123.5

32. Ans. (B) 37. Ans. (B)


123 - 12 × 12 = 1584 84 + 18 × 1 = 102
103 - 12 × 10 = 880 102 - 18 × 2 = 66
83 - 12 × 8 = 416 66 + 18 × 3 = 120
63 - 12 × 6 = 144 120 - 18 × 4 = 48
43 - 12 × 4 = 16 48 + 18 × 5 = 138
23 - 12 × 2 = - 16
38. Ans. (D)
33. Ans. (D) (2731 + 1) ÷ 4 = 683
8 × 7 = 56 (683 + 1) ÷ 4 = 171
7 × 6 = 42 (171 + 1) ÷ 4 = 43
6 × 5 = 30 (43 + 1) ÷ 4 = 11
5 × 4 = 20 (11 + 1) ÷ 4 = 3
4 × 3 = 12
3×2=6 39. Ans. (A)
(5 - 2) × 3 = 9
34. Ans. (A) (9 - 2) × 3 = 21
107 + 5 = 112 (21 - 2) × 3 = 57
112 + 7 = 119 (57 - 2) × 3 = 165
119 + 11 = 130 (165 - 2) × 3 = 489
130 + 13 = 143
143 + 17 = 160 40. Ans. (C)
101 + 1 × 2 = 103
35. Ans. (C) 103 - 2 × 3 = 97
729 - 594 = 135 97 + 3 × 4 = 109
594 - 504 = 90 (135 - 5 × 9) 109 - 4 × 5 = 89
594 - 450 = 54 (90 - 4 × 9) 89 + 5 × 6 = 119
450 - 423 = 27 (54 - 3 × 9)
423 - 414 = 9 (27 - 2 × 9)

https :
//www. https : https
https : //instagra
//youtube.c
facebo m.com/aas :
om/channe hisharoraso
l/UCYa4_Jr ok.com cial(?) //t.m
Orf8R5Kz2u
/aashis utm_mediu
e/stu

48
OtccXQ m=
haroras copy_link
ocial dified
41. Ans. (C) 46. Ans. (C)
288 × 0.5 = 144 (728 - 2) ÷ 3 = 242
144 × 1 = 144 (242 - 2) ÷ 3 = 80
144 × 1.5 = 216 (80 - 2) ÷ 3 = 26
216 × 2 = 432 (26 - 2) ÷ 3 = 8
432 × 2.5 = 1080 (8 - 2) ÷ 3 = 2

42. Ans. (B) 47. Ans. (B)


11 + 32 = 20 (3 + 1) × 2 = 8
20 - 43 = - 44 (8 + 1) × 3 = 27
- 44 + 52 = - 19 (27 + 1) × 4 = 112
- 19 - 63 = - 235 (112 + 1) × 5 = 565
- 235 + 72 = - 186 (565 + 1) × 6 = 3396

43. Ans. (A) 48. Ans. (D)


122 + 43 = 186 94 + 22 = 98
186 + 83 = 698 98 + 42 = 114
698 + 123 = 2426 114 + 82 = 178
2426 + 163 = 6522 178 + 162 = 434
6522 + 203 = 14522 434 + 322 = 1458
14522 + 243 = 28346
49. Ans. (A)
44. Ans. (C) (16 + 3) × 1 = 19
162 + 1 × 2 = 164 (19 + 3) × 2 = 44
164 - 2 × 3 = 158 (44 + 3) × 3 = 141
158 + 3 × 4 = 170 (141 + 3) × 4 = 576
170 - 4 × 5 = 150 (576 + 3) × 5 = 2895
150 + 5 × 6 = 180
180 - 6 × 7 = 138 50. Ans. (C)
2 + 3 2 - 22 = 7
45. Ans. (B) 7 + 72 - 52 = 31
825 - 26 = 761 31 + 112 - 82 = 88
761 - 25 = 729 88 + 152 - 112 = 192
729 - 24 = 713 192 + 192 - 142 = 357
713 - 23 = 705
705 - 22 = 701
701 - 21 = 699

https :
//www. https : https
https : //instagra
//youtube.c
facebo m.com/aas :
om/channe hisharoraso
l/UCYa4_Jr ok.com cial(?) //t.m
Orf8R5Kz2u
/aashis utm_mediu
e/stu

49
OtccXQ m=
haroras copy_link
ocial dified
https :
//instagram.com/aashisharorasocia
l(?) utm_medium = copy_link

https: //t.me/studified
https : //t.me/studified

https :
//youtube.com/channel/UCYa
4_JrOrf8R5Kz2uOtccXQ

https :
//www.facebook.com/aa
shisharorasocial

https :
//www. https : https
https : //instagra
//youtube.c
facebo m.com/aas :
om/channe hisharoraso
l/UCYa4_Jr ok.com cial(?) //t.m
Orf8R5Kz2u
/aashis utm_mediu
e/stu

50
OtccXQ m=
haroras copy_link
ocial dified
In each of these questions a number is wrong in the series) Find out the
wrong number.
इन प्रश्नों में प्रत्येक में श्ंखला में एक संख्या गलत है । गलत संख्या बताये।

1) 2, 4, 9, 23, 59, 166


(A) 4 (B) 23 (C) 59 (D) 166 (E) None of these

2) - 19, - 18, - 14, - 9, 11, 3


(A) 36 (B) -14 (C) -9 (D) 11 (E) None of these

3) 7, 8, 16, 41, 107, 232


(A) 41 (B) 16 (C) 107 (D) 8 (E) None of these

4) 4, 9, 20, 48, 90, 185


(A) 180 (B) 20 (C) 90 (D) 48 (E) None of these

5) 20, 90, 230, 380, 600


(A) 230 (B) 90 (C) 380 (D) 600 (E) None of these

6) 170, 227, 292, 379, 448, 540


(A) 227 (B) 292 (C) 379 (D) 448 (E) None of these

7) 8, 3, 3, 9, 26, 53
(A) 8 (B) 9 (C) 26 (D) 53 (E) None of these

8) 15, 34, 79, 132, 223, 350


(A) 223 (B) 34 (C) 132 (D) 79 (E) None of these

9) 12, 16, 32, 96, 383, 1376


(A) 383 (B) 96 (C) 1376 (D) 32 (E) None of these

10) 36, 18, 18, 27, 58, 135


(A) 36 (B) 18 (C) 58 (D) 135 (E) None of these

11) 148, 106, 71, 39, 22, 8, 1


(A) 71 (B) 39 (C) 148 (D) 106 (E) None of these

https :
//www. https : https
https : //instagra
//youtube.c
facebo m.com/aas :
om/channe hisharoraso
l/UCYa4_Jr ok.com cial(?) //t.m
Orf8R5Kz2u
/aashis utm_mediu
e/stu

51
OtccXQ m=
haroras copy_link
ocial dified
12) 120, 60, 92, 225, 787.5, 3543.75
(A) 120 (B) 60 (C) 92 (D) 225 (E) None of these

13) 42, 49, 54, 65, 78, 95


(A) 49 (B) 42 (C) 54 (D) 78 (E) None of these

14) 11, 19, 42, 110, 235, 451


(A) 451 (B) 110 (C) 19 (D) 42 (E) None of these

15) 12, 9, 12, 27, 91, 465


(A) 91 (B) 27 (C) 12 (D) 9 (E) None of these

16) 144, 72, 72, 108, 202, 540


(A) 144 (B) 72 (C) 202 (D) 108 (E) None of these

17) 13, 9, 12, 34, 89, 213


(A) 12 (B) 89 (C) 9 (D) 34 (E) None of these

18) 5, 7, 13, 25, 42, 75


(A) 75 (B) 13 (C) 25 (D) 42 (E) None of these

19) 72, 144, 216, 360, 584, 936


(A) 584 (B) 360 (C) 216 (D) 936 (E) None of these

20 ) 10800, 1800, 360, 80, 30, 15


(A) 15 (B) 360 (C) 80 (D) 30 (E) None of these

21) 12, 72, 360, 1440, 4320, 8640, 8600


(A) 8600 (B) 8640 (C) 12 (D) 72 (E) None of these

22) 24, 13, 14, 22, 45, 112.5


(A) 112.5 (B) 45 (C) 14 (D) 13 (E) None of these

23) 14, 29, 89, 359, 1798, 10799


(A) 29 (B) 89 (C) 1798 (D) 10799 (E) None of these

24) 13, 15, 19, 25, 33, 45


(A) 15 (B) 19 (C) 25 (D) 45 (E) None of these

25) 747, 736, 714, 680, 637, 582


(A) 680 (B) 736 (C) 637 (D) 747 (E) None of these

https :
//www. https : https
https : //instagra
//youtube.c
facebo m.com/aas :
om/channe hisharoraso
l/UCYa4_Jr ok.com cial(?) //t.m
Orf8R5Kz2u
/aashis utm_mediu
e/stu

52
OtccXQ m=
haroras copy_link
ocial dified
26) 77, 367, 593, 763, 887, 967
(A) 77 (B) 976 (C) 887 (D) 367 (E) None of these

27) 1023, 1082, 1155, 1224, 1295, 1368


(A) 1082 (B) 1368 (C) 1225 (D) 1295 (E) None of these

28) 58, 64, 76, 108, 118, 148


(A) 58 (B) 108 (C) 64 (D) 148 (E) None of these

29) 108, 162, 243, 364.5, 564.75, 820.125


(A) 564.75 (B) 243 (C) 108 (D) 162 (E) None of these

30) 5, 12, 23, 39, 53, 72


(A) 72 (B) 23 (C) 39 (D) 12 (E) None of these

31) 12, 19, 36, 54, 82, 117


(A) 12 (B) 36 (C) 19 (D) 117 (E) None of these

32) 17, 18, 20, 24, 32, 48, 86


(A) 32 (B) 17 (C) 96 (D) 24 (E) None of these

33) 9, 19, 39, 79, 159, 327, 639


(A) 327 (B) 639 (C) 159 (D) 39 (E) None of these

34 ) - 22, - 6, 58, - 74, 138, 154


(A) -6 (B) -22 (C) 58 (D) -74 (E) None of these

35) 17, 18, 26, 59, 117, 242


(A) 59 (B) 117 (C) 26 (D) 18 (E) None of these

36) 17, 22, 31, 48, 61, 82


(A) 82 (B) 48 (C) 31 (D) 22 (E) None of these

37) 21, 25, 34, 49, 73, 101


(A) 25 (B) 49 (C) 73 (D) 101 (E) None of these

38) 4, 15, 37, 70, 118, 169


(A) 118 (B) 70 (C) 37 (D) 169 (E) None of these

39) - 6, - 3, 2, 9, 19, 29
(A) 29 (B) -3 (C) -6 (D) 19 (E) None of these

https :
//www. https : https
https : //instagra
//youtube.c
facebo m.com/aas :
om/channe hisharoraso
l/UCYa4_Jr ok.com cial(?) //t.m
Orf8R5Kz2u
/aashis utm_mediu
e/stu

53
OtccXQ m=
haroras copy_link
ocial dified
40) 2, 4, 9, 23, 64, 182
(A) 182 (B) 64 (C) 4 (D) 23 (E) None of these

41) 2, 4, 11, 39, 145, 572


(A) 11 (B) 145 (C) 39 (D) 572 (E) None of these

42) 81, 106, 142, 191, 256, 336


(A) 336 (B) 256 (C) 142 (D) 191 (E) None of these

43) 493, 485, 458, 394, 267, 53


(A) 493 (B) 458 (C) 394 (D) 267 (E) None of these

44) 420, 120, 45, 40, 75, 295


(A) 120 (B) 45 (C) 420 (D) 75 (E) None of these

45) 4, 10, 25, 63.5, 156.25, 390.625


(A) 156.25 (B) 25 (C) 63.5 (D) 10 (E) None of these

46) 32, 16, 18, 24, 48, 120


(A) 48 (B) 32 (C) 18 (D) 24 (E) None of these

47) 82, 123, 172, 236, 296, 372


(A) 172 (B) 236 (C) 123 (D) 82 (E) None of these

48) 1213, 413, 133, 43, 13, 3


(A) 43 (B) 133 (C) 1213 (D) 413 (E) None of these

49) 6, 9, 12, 36, 144, 720


(A) 9 (B) 36 (C) 12 (D) 144 (E) None of these

50) 12, 24, 36, 57, 78, 102


(A) 12 (B) 24 (C) 36 (D) 78 (E) None of these

51) 300, 302, 305, 310, 319, 328


(A) 305 (B) 319 (C) 302 (D) 328 (E) None of these

52) 24, 36, 60, 89, 132


(A) 36 (B) 60 (C) 89 (D) 24 (E) None of these

53) 62, 76, 92, 110, 134, 152


(A) 134 (B) 110 (C) 152 (D) 76 (E) None of these

https :
//www. https : https
https : //instagra
//youtube.c
facebo m.com/aas :
om/channe hisharoraso
l/UCYa4_Jr ok.com cial(?) //t.m
Orf8R5Kz2u
/aashis utm_mediu
e/stu

54
OtccXQ m=
haroras copy_link
ocial dified
54) 9, 7, 11, 29, 113, 549
(A) 9 (B) 29 (C) 549 (D) 113 (E) None of these

55) 7, 9, 13, 21, 34, 69


(A) 34 (B) 69 (C) 13 (D) 21 (E) None of these

56) 84, 168, 252, 420, 679, 1092


(A) 420 (B) 679 (C) 252 (D) 168 (E) None of these

57) 10, 12, 68, 30, 226, 56


(A) 12 (B) 56 (C) 226 (D) 68 (E) None of these

58) 124, 62, 62, 93, 186, 469, 1395


(A) 469 (B) 186 (C) 1395 (D) 93 (E) None of these

59) 7920, 1320, 268, 66, 22, 11


(A) 7920 (B) 1320 (C) 22 (D) 268 (E) None of these

60) 9, 6, 10, 33, 97, 214


(A) 97 (B) 33 (C) 214 (D) 10 (E) None of these

61) 123, 140, 174, 228, 293, 378


(A) 293 (B) 228 (C) 174 (D) 378 (E) None of these

62 ) 3072, 768, 192, 46, 12, 3


(A) 768 (B) 12 (C) 46 (D) 3072 (E) None of these

63) 2, 6, 14, 26, 38, 62


(A) 38 (B) 62 (C) 14 (D) 26 (E) None of these

64) 16, 100, 167, 219, 238, 286


(A) 167 (B) 286 (C) 219 (D) 238 (E) None of these

65) 4073, 1007, 239, 51, 5, - 3


(A) 1007 (B) 239 (C) 4073 (D) 51 (E) None of these

66) 2, 6, 21, 86, 378, 1890


(A) 378 (B) 86 (C) 21 (D) 6 (E) None of these

67) 5, 12, 35, 96, 199, 390


(A) 390 (B) 199 (C) 96 (D) 12 (E) None of these

https :
//www. https : https
https : //instagra
//youtube.c
facebo m.com/aas :
om/channe hisharoraso
l/UCYa4_Jr ok.com cial(?) //t.m
Orf8R5Kz2u
/aashis utm_mediu
e/stu

55
OtccXQ m=
haroras copy_link
ocial dified
68) 4, 8, 23, 99, 564, 3947
(A) 564 (B) 99 (C) 23 (D) 3947 (E) None of these

69) 142, 61, 163, 40, 184, 15


(A) 142 (B) 15 (C) 184 (D) 163 (E) None of these

70) 20, 42, 76, 110, 156


(A) 76 (B) 42 (C) 110 (D) 20 (E) None of these

71) 1580, 527, 176, 59, 22, 7


(A) 59 (B) 176 (C) 22 (D) 1580 (E) None of these

72) 256, 128, 128, 194, 384, 960


(A) 256 (B) 960 (C) 256 (D) 384 (E) None of these

73) 233, 237, 229, 241, 221, 251, 209


(A) 229 (B) 221 (C) 209 (D) 237 (E) None of these

74) 1577, 848, 605, 526, 497, 488, 485


(A) 526 (B) 605 (C) 1577 (D) 485 (E) None of these

75) 86, 103, 131, 176, 236, 311


(A) 176 (B) 86 (C) 103 (D) 311 (E) None of these

76 ) 33, 70, 131, 224, 349, 518


(A) 70 (B) 131 (C) 224 (D) 518 (E) None of these

77) 37, 40, 49, 76, 157, 398


(A) 398 (B) 37 (C) 157 (D) 398 (E) None of these

78) 31, 68, 129, 220, 349, 516


(A) 31 (B) 349 (C) 516 (D) 220 (E) None of these

79) 12, 16, 34, 68, 132, 232


(A) 34 (B) 68 (C) 132 (D) 232 (E) None of these

80) 45, 90, 30, 120, 24, 140


(A) 45 (B) 24 (C) 140 (D) 120 (E) None of these

https :
//www. https : https
https : //instagra
//youtube.c
facebo m.com/aas :
om/channe hisharoraso
l/UCYa4_Jr ok.com cial(?) //t.m
Orf8R5Kz2u
/aashis utm_mediu
e/stu

56
OtccXQ m=
haroras copy_link
ocial dified
1. Ans. (B) 6. Ans. (C)
2×3-2=4 132 + 1 = 170
4×3-3=9 152 + 2 = 227
9 × 3 - 5 = 22 172 + 3 = 292
22 × 3 - 7 = 59 192 + 5 = 366
59 × 3 - 11 = 166 212 + 7 = 448
232 + 11 = 540
2. Ans. (C)
- 19 + 12 = - 18 7. Ans. (B)
- 18 + 22 = - 14 8 - 9 + 22 = 3
- 14 + 32 = - 5 3 - 9 + 32 = 3
- 5 + 42 = 11 3 - 9 + 42 = 10
11 + 52 = 36 10 - 9 + 52 = 26
26 - 9 + 62 = 53
3. Ans. (A)
7 + 13 = 8 8. Ans. (D)
8 + 23 = 16 23 + 7 = 15
16 + 33 = 43 33 + 7 = 34
43 + 43 = 107 43 + 7 = 71
107 + 53 = 232 53 + 7 = 132
63 + 7 = 223
4. Ans. (D) 73 + 7 = 350
4×2+1=9
9 × 2 + 2 = 20 9. Ans. (A)
20 × 2 + 3 = 43 12 + 41 = 16
43 × 2 + 4 = 90 16 + 42 = 32
90 × 2 + 5 = 185 32 + 43 = 96
96 + 44 = 352
5. Ans. (A) 352 + 45 = 1376
4 × 5 = 20
9 × 10 = 90 10. Ans. (C)
14 × 15 = 210 36 × 0.5 = 18
19 × 20 = 380 18 × 1 = 18
24 × 24 = 600 18 × 1.5 = 27
27 × 2 = 54
54 × 2.5 = 135

https :
//www. https : https
https : //instagra
//youtube.c
facebo m.com/aas :
om/channe hisharoraso
l/UCYa4_Jr ok.com cial(?) //t.m
Orf8R5Kz2u
/aashis utm_mediu
e/stu

57
OtccXQ m=
haroras copy_link
ocial dified
11. Ans. (B) 16. Ans. (C)
148 - 7 × 6 = 106 144 × 0.5 = 72
106 - 7 × 5 = 71 72 × 1 = 72
71 - 7 × 4 = 43 72 × 1.5 = 1008
43 - 7 × 3 = 22 108 × 2 = 216
22 - 7 × 2 = 8 216 × 2.5 = 540
8-7×1=1
17. Ans. (B)
12. Ans. (C) 13 - 5 + 13 = 9
120 × 0.5 = 60 9 - 5 + 23 = 12
60 × 1.5 = 90 12 - 5 + 33 = 34
90 × 2.5 = 225 34 - 5 + 43 = 93
225 × 3.5 = 787.5 93 - 5 + 53 = 213
787.5 × 4.5 = 3543.75
18. Ans. (D)
13. Ans. (A) 5+1×2=7
42 + 5 = 47 7 + 2 × 3 = 13
47 + 7 = 54 13 + 3 × 4 = 25
54 + 11 = 65 25 + 4 × 5 = 45
65 + 13 = 78 45 + 5 × 6 = 75
78 + 17 = 95
19. Ans. (A)
14.Ans. (D) 72 + 144 = 216
11 + 23 = 19 144 + 216 = 360
19 + 33 = 46 216 + 360 = 576
46 + 43 = 110 360 + 576 = 936
110 + 53 = 235
235 + 63 = 451 20. Ans. (C)
10800 ÷ 6 = 1800
15. Ans. (A) 1800 ÷ 5 = 360
(12 - 3) × 1 = 9 360 ÷ 4 = 90
(9 - 3) × 2 = 12 90 ÷ 3 = 30
(12 - 3) × 3 = 27 30 ÷ 2 = 15
(27 - 3) × 4 = 96
(96 - 3) × 5 = 465

https :
//www. https : https
https : //instagra
//youtube.c
facebo m.com/aas :
om/channe hisharoraso
l/UCYa4_Jr ok.com cial(?) //t.m
Orf8R5Kz2u
/aashis utm_mediu
e/stu

58
OtccXQ m=
haroras copy_link
ocial dified
21. Ans. (A) 26. Ans. (C)
12 × 6 = 72 77 + 172 + 1 = 367
72 × 5 = 360 367 + 152 + 1 = 593
360 × 4 = 1440 593 + 132 + 1 = 763
1440 × 3 = 4320 763 + 112 + 1 = 885
4320 × 2 = 8640 885 + 92 + 1 = 967
8640 × 1 = 8640
27. Ans. (A)
22. Ans. (A) 322 - 1 = 1023
24 × 0.5 + 1 = 13 332 - 1 = 1088
13 × 1 + 1 = 14 342 - 1 = 1155
14 × 1.5 + 1 = 22 352 - 1 = 1224
22 × 2 + 1 = 45 362 - 1 = 1295
45 × 2.5 + 1 = 113.5 372 - 1 = 1368

23. Ans. (C) 28. Ans. (B)


14 × 2 + 1 = 29 58 + 6 × 1 = 64
29 × 3 + 2 = 89 64 + 6 × 2 = 76
89 × 4 + 3 = 359 76 + 6 × 3 = 94
359 × 5 + 4 = 1799 94 + 6 × 4 = 118
1799 × 6 + 5 = 10799 118 + 6 × 5 = 148

24. Ans. (D) 29. Ans. (A)


13 + 2 × 1 = 15 108 × 1.5 = 162
15 + 2 × 2 = 19 162 × 1.5 = 243
19 + 2 × 3 = 25 243 × 1.5 = 364.5
25 + 2 × 4 = 33 364.5 × 1.5 = 546.75
33 + 2 × 5 = 43 546.75 × 1.5 = 820.125

25. Ans. (A) 30. Ans. (C)


747 - 11 = 736 5 + 7 = 12
736 - 22 = 714 12 + 11 = 23
714 - 33 = 681 23 + 13 = 36
681 - 44 = 637 36 + 17 = 53
637 - 55 = 582 53 + 19 = 72

https :
//www. https : https
https : //instagra
//youtube.c
facebo m.com/aas :
om/channe hisharoraso
l/UCYa4_Jr ok.com cial(?) //t.m
Orf8R5Kz2u
/aashis utm_mediu
e/stu

59
OtccXQ m=
haroras copy_link
ocial dified
31. Ans. (B) 36. Ans. (B)
12 + 7 × 1 = 19 17 + 5 = 22
19 + 7 × 2 = 33 22 + 9 = 31
33 + 7 × 3 = 54 31 + 13 = 44
54 + 7 × 4 = 82 44 + 17 = 61
82 + 7 × 5 = 117 61 + 21 = 82

32. Ans. (C) 37. Ans. (C)


17 + 20 = 18 25 - 21 = 4
18 + 21 = 20 34 - 25 = 9 (4 + 5 = 9)
20 + 22 = 24 49 - 34 = 15 (9 + 6 = 15)
24 + 23 = 32 71 - 49 = 22 (15 + 7 = 22)
32 + 24 = 48 101 - 71 = 30(22 + 8 = 30)
48 + 25 = 80
38. Ans. (A)
33. Ans. (A) 4 + 11 × 1 = 15
9 × 2 + 1 = 19 15 + 11 × 2 = 37
19 × 2 + 1 = 39 37 + 11 × 3 = 70
39 × 2 + 1 = 79 70 + 11 × 4 = 114
79 × 2 + 1 = 159 114 + 11 × 5 = 169
159 × 2 + 1 = 319
319 × 2 + 1 = 639 39. Ans. (D)
-6+3=-3
34. Ans. (D) -3+5=2
- 22 + 42 = - 6 2+7=9
- 6 + 82 = 54 9 + 9 = 18
54 + 42 = 74 18 + 11 = 29
74 + 82 = 138
138 + 42 = 154 40. Ans. (A)
2×3-2=4
35. Ans. (A) 4×3-3=9
17 + 13 = 18 9 × 3 - 4 = 23
18 + 23 = 26 23 × 3 - 5 = 64
26 + 33 = 53 64 × 3 - 6 = 186
53 + 43 = 117
117 + 53 = 242

https :
//www. https : https
https : //instagra
//youtube.c
facebo m.com/aas :
om/channe hisharoraso
l/UCYa4_Jr ok.com cial(?) //t.m
Orf8R5Kz2u
/aashis utm_mediu
e/stu

60
OtccXQ m=
haroras copy_link
ocial dified
41. Ans. (C) 46. Ans. (C)
2×4-4=4 32 × 0.5 = 16
4 × 4 - 5 = 11 16 × 1 = 16
11 × 4 - 6 = 38 16 × 1.5 = 24
38 × 4 - 7 = 145 24 × 2 = 48
145 × 4 - 8 = 572 48 × 2.5 = 120

42. Ans. (B) 47. Ans. (B)


81 + 52 = 106 92 + 1 = 82
106 + 62 = 142 112 + 2 = 123
142 + 72 = 191 132 + 3 = 172
191 + 82 = 255 152 + 5 = 230
255 + 92 = 336 172 + 7 = 296
192 + 11 = 372
43. Ans. (D)
493 - 23 = 485 48. Ans. (D)
485 - 33 = 458 (1213 - 4) ÷ 3 = 403
458 - 43 = 394 (403 - 4) ÷ 3 = 133
394 - 53 = 269 (133 - 4) ÷ 3 = 43
269 - 63 = 53 (43 - 4) ÷ 3 = 13
(13 - 4) ÷ 3 = 3
44. Ans. (A)
420 × 1/4 - 5 = 100 49. Ans. (A)
100 × 1/2 - 5 = 45 6×1=6
45 × 1 - 5 = 40 6 × 2 = 12
40 × 2 - 5 = 75 12 × 3 = 36
75 × 4 - 5 = 295 36 × 4 = 144
144 × 5 = 720
45. Ans. (C)
4 × 2.5 = 10 50. Ans. (C)
10 × 2.5 = 25 24 - 12 = 12
25 × 2.5 = 62.5 39 - 24 = 15 (12 + 3)
62.5 × 2.5 = 156.25 57 - 39 = 18 (15 + 3)
156.25 × 2.5 = 390.625 78 - 57 = 21 (18 + 3)
102 - 78 = 24(21 + 3)

https :
//www. https : https
https : //instagra
//youtube.c
facebo m.com/aas :
om/channe hisharoraso
l/UCYa4_Jr ok.com cial(?) //t.m
Orf8R5Kz2u
/aashis utm_mediu
e/stu

61
OtccXQ m=
haroras copy_link
ocial dified
51. Ans. (B) 56. Ans. (B)
300 + 2 = 302 84 + 168 = 252
302 + 3 = 305 268 + 252 = 420
305 + 5 = 310 252 + 420 = 672
310 + 7 = 317 420 + 672 = 1092
317 + 11 = 328
57. Ans. (C)
52. Ans. (C) 23 + 2 = 10
12 × 2 = 24 32 + 3 = 12
12 × 3 = 36 43 + 4 = 68
12 × 5 = 60 52 + 5 = 30
12 × 7 = 84 63 + 6 = 222
12 × 11 = 132 72 + 7 = 56

53. Ans. (A) 58. Ans. (A)


62 + 14 = 76 124 × 0.5 = 62
76 + 16 = 92 62 × 1 = 62
92 + 18 = 110 62 × 1.5 = 93
110 + 20 = 130 93 × 2 = 186
130 + 22 = 152 186 × 2.5 = 465
465 × 3 = 1395
54.Ans. (D)
9×1-2=7 59. Ans. (D)
7 × 2 - 3 = 11 7920 ÷ 6 = 1320
11 × 3 - 4 = 29 1320 ÷ 5 = 264
29 × 4 - 5 = 111 264 ÷ 4 = 66
111 × 5 - 6 = 549 66 ÷ 3 = 22
22 ÷ 2 = 11
55. Ans. (A)
7 + 21 = 9 60. Ans. (A)
9 + 22 = 13 9 + 13 - 4 = 6
13 + 23 = 21 6 + 23 - 4 = 10
21 + 24 = 37 10 + 33 - 4 = 33
37 + 25 = 69 33 + 43 - 4 = 93
93 + 53 - 4 = 214

https :
//www. https : https
https : //instagra
//youtube.c
facebo m.com/aas :
om/channe hisharoraso
l/UCYa4_Jr ok.com cial(?) //t.m
Orf8R5Kz2u
/aashis utm_mediu
e/stu

62
OtccXQ m=
haroras copy_link
ocial dified
61. Ans. (B) 66. Ans. (B)
123 + 17 = 140 2×3=6
140 + 34 = 174 6 × 3.5 = 21
174 + 51 = 225 21 × 4 = 84
225 + 68 = 293 84 × 4.5 = 378
293 + 85 = 378 378 × 5 = 1890

62. Ans. (C) 67. Ans. (C)


3072 ÷ 4 = 768 5 + 23 - 12 = 12
768 ÷ 4 = 192 12 + 33 - 22 = 35
192 ÷ 4 = 48 35 + 43 - 32 = 90
48 ÷ 4 = 12 90 + 53 - 42 = 199
12 ÷ 4 = 3 199 + 63 - 52 = 390

63. Ans. (A) 68. Ans. (A)


2+4×1=6 4 × 3 - 22 = 8
6 + 4 × 2 = 14 8 × 4 - 32 = 23
14 + 4 × 3 = 26 23 × 5 - 42 = 99
26 + 4 × 4 = 42 99 × 6 - 52 = 569
42 + 4 × 5 = 62 569 × 7 - 62 = 3947

64. Ans. (D) 69. Ans. (D)


16 + 92 + 3 = 100 142 - 92 = 61
100 + 82 + 3 = 167 61 + 102 = 162
167 + 72 + 3 = 219 161 - 112 = 40
219 + 62 + 3 = 258 40 + 122 = 184
258 + 52 + 3 = 286 184 - 132 = 15

65. Ans. (A) 70. Ans. (A)


46 - 23 = 4073 4 × 5 = 20
45 - 19 = 1005 6 × 7 = 42
44 - 17 = 239 8 × 9 = 72
43 - 13 = 51 10 × 11 = 110
42 - 11 = 5 12 × 13 = 156
41 - 7 = - 3

https :
//www. https : https
https : //instagra
//youtube.c
facebo m.com/aas :
om/channe hisharoraso
l/UCYa4_Jr ok.com cial(?) //t.m
Orf8R5Kz2u
/aashis utm_mediu
e/stu

63
OtccXQ m=
haroras copy_link
ocial dified
71. Ans. (C) 76. Ans. (C)
(1580 + 1) ÷ 3 = 527 33 + 6 = 33
(527 + 1) ÷ 3 = 176 43 + 6 = 70
(176 + 1) ÷ 3 = 59 53 + 6 = 131
(59 + 1) ÷ 3 = 20 63 + 6 = 222
(20 + 1) ÷ 3 = 7 73 + 6 = 349
83 + 6 = 518
72. Ans. (A)
256 × 0.5 = 128 77. Ans. (D)
128 × 1 = 128 37 + 31 = 40
128 × 1.5 = 192 40 + 32 = 49
192 × 2 = 384 49 + 33 = 76
384 × 2.5 = 960 76 + 34 = 157
157 + 35 = 400
73. Ans. (D)
233 + 1 × 2 = 235 78. Ans. (B)
235 - 2 × 3 = 229 33 + 4 = 31
229 + 3 × 4 = 241 43 + 4 = 68
241 - 4 × 5 = 221 53 + 4 = 129
221 + 5 × 6 = 251 63 + 4 = 22
251 - 6 × 7 = 209 73 + 4 = 347
83 + 4 = 516
74. Ans. (A)
1577 - 36 = 848 79. Ans. (A)
848 - 35 = 605 12 + 22 = 16
605 - 34 = 524 16 + 42 = 32
524 - 33 = 497 32 + 62 = 68
497 - 32 = 488 68 + 82 = 132
488 - 31 = 485 132 + 102 = 232

75. Ans. (C) 80. Ans. (C)


86 + 15 × 1 = 101 45 × 2 = 90
101 + 15 × 2 = 131 90 ÷ 3 = 30
131 + 15 × 3 = 176 30 × 4 = 120
176 + 15 × 4 = 236 120 ÷ 5 = 24
236 + 15 × 5 = 311 24 × 6 = 144

https :
//www. https : https
https : //instagra
//youtube.c
facebo m.com/aas :
om/channe hisharoraso
l/UCYa4_Jr ok.com cial(?) //t.m
Orf8R5Kz2u
/aashis utm_mediu
e/stu

64
OtccXQ m=
haroras copy_link
ocial dified
https :
//instagram.com/aashisharorasocia
l(?) utm_medium = copy_link

https: //t.me/studified
https : //t.me/studified

https :
//youtube.com/channel/UCYa
4_JrOrf8R5Kz2uOtccXQ

https :
//www.facebook.com/aa
shisharorasocial

https :
//www. https : https
https : //instagra
//youtube.c
facebo m.com/aas :
om/channe hisharoraso
l/UCYa4_Jr ok.com cial(?) //t.m
Orf8R5Kz2u
/aashis utm_mediu
e/stu

65
OtccXQ m=
haroras copy_link
ocial dified
1. P and Q started a business with investment of Rs. 16500 and Rs. 15600
respectively for X months and (X + 2) months. At the end of the year, out of
total profit of Rs. 17280, profit share of earned by Q is Rs. 9360, then find the
value of X.
P और Q ने क्रमशः X महीने और (X + 2) महीने के ललए 16500 रुपये और 15600 रुपये के ननवेश
के साथ एक व्यवसाय शुरू ककया। वर्ष के अंत में , 17280 रुपये के कुल लाभ में से Q द्वारा अजजषत
लाभ का दहस्सा 9360 रुपये है , तो X का मान ज्ञात कीजजए।
(A) 7 (B) 8 (C) 5
(D) 6 (E) None of these

2. Two container X and Y contains mixture of soda and water. Container X


contain 78% of water and container Y contain 62% of soda. In what ratio
should mixture of container X and Y mixed to get a mixture that contain 50%
of water?
िो कंटे नर X और Y में सोडा और पानी का लमश्ण है । कंटे नर X में 78% पानी है और कंटे नर Y में
62% सोडा है । 50% पानी वाला लमश्ण प्राप्त करने के ललए कंटे नर X और Y के लमश्ण को ककस
अनुपात में लमलाया जाना चादहए?
(A) 5 : 9 (B) 6 : 7 (C) 3 : 7
(D) 4 : 5 (E) None of these

3. A bus covers 1080 km with speed of (X + 15) km/hr in 24 hours. If speed of


bus is increased by (4X/3)%, then find the distance travelled by bus in 22
hours.
एक बस 24 घंटे में (X + 15) ककमी/घंटा की गनत से 1080 ककमी की िरू ी तय करती है । यदि बस की
गनत (4X/3)% बढा िी जाती है , तो 22 घंटे में बस द्वारा तय की गई िरू ी ज्ञात कीजजए।
(A) 1373 km (B) 1345 km (C) 1322 km
(D) 1386 km (E) None of these

4. A shopkeeper sold an article Rs. 840. If shopkeeper allowed two successive


discount 14.28% and 20% respectively and earns 20% profit. Find the sum of
markup price and cost price of book.
एक िकु ानिार ने एक वस्तु को 840 रुपये में बेचा। यदि िक
ु ानिार िो क्रलमक छूट क्रमशः 14.28%
और 20% िे ता है और 20% लाभ अजजषत करता है । पुस्तक के माकषअप मूल्य और लागत मूल्य का
योग ज्ञात कीजजए।
(A) Rs. 1926 (B) Rs. 1925 (C) Rs. 1927
(D) Rs. 1924 (E) None of these

https :
//www. https : https
https : //instagra
//youtube.c
facebo m.com/aas :
om/channe hisharoraso
l/UCYa4_Jr ok.com cial(?) //t.m
Orf8R5Kz2u
/aashis utm_mediu
e/stu

66
OtccXQ m=
haroras copy_link
ocial dified
5. A certain sum of money at R% simple interest becomes Rs. 7936 in 3 years
and Rs. 10496 in 8 years. Find the value of R.
R% साधारण ब्याज पर एक ननजश्चत रालश 3 साल में 7936 रुपये और 8 साल में 10496 रुपये हो
जाती है । R का मान ज्ञात कीजजए।
(A) 8 (B) 7 (C) 6
(D) 9 (E) None of these

6. Average of 5 consecutive even numbers is 42 and sum of 6 consecutive


odd numbers is 252. Find the largest number among all 11 numbers.
5 क्रमागत सम संख्याओं का औसत 42 है और 6 क्रमागत ववर्म संख्याओं का योग 252 है । सभी
11 संख्याओं में से सबसे बडी संख्या ज्ञात कीजजए।
(A) 47 (B 41 (C) 45
(D) 43 (E) None of these

7. A shopkeeper sold a book and a bag at Rs. X and the selling price of the
book is 28.57% less than the selling price of the bag. If the selling price of
book and bag is increased by Rs. 120 and Rs. 240, then the ratio of selling
price of book to bag becomes 3 : 5. Find the value of X.
एक िक ु ानिार ने एक ककताब और एक बैग X रुपये में बेचा और ककताब का ववक्रय मूल्य बैग के
ववक्रय मल्
ू य से 28.57% कम है । यदि पस् ु तक और बैग के ववक्रय मल् ू य में 120 रुपये और 240
रुपये की वद्
र चध की जाती है , तो पुस्तक और बैग के बबक्री मूल्य का अनुपात 3 : 5 हो जाता है ।X का
मान ज्ञात कीजजए।
(A) 270 (B) 330 (C) 360
(D) 320 (E) None of these

8. The sum of age of Divya and Priya 3 years ago is 60 years and the ratio of
age of Divya and Priya 6 years hence is 7 : 6. Find the age of Priya.
3 वर्ष पव
ू ष दिव्या और वप्रया की आयु का योग 60 वर्ष है और 6 वर्ष बाि दिव्या और वप्रया की आयु
का अनुपात 7 : 6 है । वप्रया की आयु ज्ञात कीजजए।
(A) 20 years (B) 40 years (C) 30 years
(D) 10 years (E) None of these

9. Tap A can fill a tank in 35 hours while tap B can empty the tank in 63 hours.
If tap A opened for 25 hours, then find the time taken by B to empty the tank.
नल A एक टं की को 35 घंटे में भर सकता है जबकक नल B टं की को 63 घंटे में खाली कर सकता है ।
यदि नल A को 25 घंटे के ललए खोला जाता है , तो टैंक को खाली करने में B द्वारा ललया गया
समय ज्ञात कीजजए।
(A) 50 hours (B) 45 hours (C) 55 hours
(D) 40 hours (E) None of these

https :
//www. https : https
https : //instagra
//youtube.c
facebo m.com/aas :
om/channe hisharoraso
l/UCYa4_Jr ok.com cial(?) //t.m
Orf8R5Kz2u
/aashis utm_mediu
e/stu

67
OtccXQ m=
haroras copy_link
ocial dified
10. A boat can travel 240 km upstream in 'A' hours while 66.67% less than
time taken to cover same distance in downstream. If the speed of boat in still
water in 20 km/hr, then find the downstream speed.
एक नाव 'A' घंटे में धारा के प्रनतकूल 240 ककमी की यात्रा कर सकती है जबकक धारा के अनुकूल
समान िरू ी तय करने में लगने वाले समय से 66.67% कम समय लेती है । यदि शांत जल में नाव
की गनत 20 ककमी/घंटा है , तो धारा के अनुकूल गनत ज्ञात कीजजए।
(A) 40 km/hr (B) 20 km/hr (C) 10 km/hr
(D) 30 km/hr (E) None of these

11. Average salary of Kashvi, Mitali and Sanvi is Rs. 66000 and average salary
of Mitali, Sanvi and Pratik is Rs. 52000. What is the difference between salary
of Kashvi and Pratik?
काशवी, लमताली और सान्वी का औसत वेतन 66000 रुपये है और लमताली, सान्वी और प्रतीक का
औसत वेतन 52000 रुपये है । काशवी और प्रतीक के वेतन में ककतना अंतर है ?
(A) Rs. 48000 (B) Rs. 42000 (C) Rs. 46000
(D) Rs. 44000 (E) None of these

12. Vanya purchased a purse and a jeans for Rs. Y and (Y + 12000)
respectively and sold both item for Rs. 68820. She gains of 9% on purse and
14% loss on jeans. Find the value of (Y + 12000).
वान्या ने क्रमशः Y और (Y + 12000) रुपये में एक पसष और एक जींस खरीिी और िोनों वस्तुओं
को 68820 रुपये में बेच दिया। उसे पसष पर 9% का लाभ और जींस पर 14% की हानन हुई। (Y +
12000) का मान ज्ञात कीजजए।
(A) 23000 (B) 38000 (C) 42000
(D) 51000 (E) None of these

13. Find the cost of the wire require to fence a rectangular field having length
and breadth 32 m and 17 m respectively. The cost of the wire is Rs. 4.25/m.
एक आयताकार क्षेत्र की लंबाई और चौडाई क्रमशः 32 मीटर और 17 मीटर है , बाड लगाने के ललए
आवश्यक तार की लागत ज्ञात कीजजए। तार की लागत रुपये 4.25/m है ।
(A) Rs. 419.3 (B) Rs. 414.7 (C) Rs. 412.8
(D) Rs. 416.5 (E) None of these

14. Vinay travels from city A to city B. He covers 3/5 distance by car and rest
covers by bike at 2/7 times of the speed of car. If average speed is 55 km/hr,
then find the speed of car.
ववनय शहर A से शहर B तक यात्रा करता है । वह कार द्वारा 3/5 िरू ी तय करता है और शेर् िरू ी
कार की गनत के 2/7 गन
ु ा बाइक से तय करता है । यदि औसत गनत 55 ककमी/घंटा है , तो कार की
गनत ज्ञात कीजजए।
(A) 120 km/hr (B) 110 km/hr (C) 140 km/hr
(D) 130 km/hr (E) None of these

https :
//www. https : https
https : //instagra
//youtube.c
facebo m.com/aas :
om/channe hisharoraso
l/UCYa4_Jr ok.com cial(?) //t.m
Orf8R5Kz2u
/aashis utm_mediu
e/stu

68
OtccXQ m=
haroras copy_link
ocial dified
15. A sum of Rs. 9500 becomes Rs. 14630 at rate of simple interest R% in 3
years. What will be the interest, If the rate of interest is (R + 4)%?
9500 रुपये की रालश 3 साल में साधारण ब्याज R% की िर से 14630 रुपये हो जाती है । ब्याज
क्या होगा, यदि ब्याज की िर (R + 4)% है ?
(A) Rs. 6270 (B) Rs. 5290 (C) Rs. 4980
(D) Rs. 3940 (E) None of these

16. Aarohi started a business with Rs. 8000 and after 3 months, Rohil joined
her with 6500. At the end of the year, out of the total profit Aarohi received
total Rs. 3200, then find the share of Rohil.
आरोही ने 8000 रुपये के साथ एक व्यवसाय शरू ु ककया और 3 महीने बाि, रोदहल 6500 के साथ
उसके साथ जुड गया। वर्ष के अंत में , कुल लाभ में से आरोही को कुल 3200 रुपये प्राप्त हुए, तो
रोदहल का दहस्सा ज्ञात कीजजए।
(A) Rs. 1950 (B) Rs. 1830 (C) Rs. 1690
(D) RS. 1720 (E) None of these

17. The ratio of the age of Mohan to that of Vinod, 2 years ago was 19 : 18. 7
years hence, then age of Mohan will be 3 years more than the age of Vinod at
that time. Find the present age of Vinod.
2 वर्ष पहले मोहन और ववनोि की आयु का अनुपात 19 : 18 था। 7 वर्ष बाि, मोहन की आयु उस
समय ववनोि की आयु से 3 वर्ष अचधक होगी। ववनोि की वतषमान आयु ज्ञात कीजजए।
(A) 48 years (B) 62 years (C) 56 years
(D) 34 years (E) None of these

18. A mixture of 1200 liters contains juice and water in the ratio of 3 : 4
respectively. If 80 liters of mixture is taken out and replaced with 80 liters of
juice, then what will be the ratio of juice and water in the final mixture?
1200 लीटर के एक लमश्ण में रस और पानी क्रमशः 3 : 4 के अनुपात में हैं। यदि 80 लीटर लमश्ण
ननकाल ललया जाता है और 80 लीटर रस से बिल दिया जाता है , तो अंनतम लमश्ण में रस और पानी
का अनुपात क्या होगा?
(A) 5 : 6 (B) 2 : 5 (C) 7 : 8
(D) 9 : 4 (E) None of these

19. A ship sails 72 km in a river in upstream in 18 hours. How much time will
it take to cover the 78 km distance in downstream in the same river, If the
speed of the current is (1/5) th of the speed of boat in still water?
एक जहाज 18 घंटे में धारा के प्रनतकूल निी में 72 ककमी की िरू ी तय करता है । उसी निी में धारा के
अनुकूल 78 ककमी की िरू ी तय करने में ककतना समय लगेगा, यदि धारा की गनत जस्थर जल में नाव
की गनत का (1/5) वां है ?
(A) 15 hours (B) 13 hours (C) 14 hours
(D) 12 hours (E) None of these

https :
//www. https : https
https : //instagra
//youtube.c
facebo m.com/aas :
om/channe hisharoraso
l/UCYa4_Jr ok.com cial(?) //t.m
Orf8R5Kz2u
/aashis utm_mediu
e/stu

69
OtccXQ m=
haroras copy_link
ocial dified
20. Efficiency of Rohini is 20% more than Sagar. Time taken by Sagar to
finish a work is what percent more than time taken by Rohini to finish the
same work?
रोदहणी की िक्षता सागर से 20% अचधक है । सागर द्वारा ककसी कायष को समाप्त करने में ललया
गया समय, रोदहणी द्वारा उसी कायष को समाप्त करने में ललए गए समय से ककतने प्रनतशत
अचधक है ?
(A) 40% (B) 30% (C) 10%
(D) 20% (E) None of these

21. P can complete a piece of work in 30 days. While R can complete the
whole work in 40 days. If Q is 20% more efficient than P, then find the time
taken by all of them together to complete work.
P एक कायष को 30 दिनों में पूरा कर सकता है । जबकक R पूरे काम को 40 दिनों में पूरा कर सकता
है । यदि Q, P से 20% अचधक कुशल है , तो उन सभी द्वारा लमलकर कायष को परू ा करने में ललया
गया समय ज्ञात कीजजए।
(A) 7(11/31) days (B) 10(10/59) days (C) 81(3/37) days
(D) 11(17/41) days (E) None of these

22. A boat can cover 'D' km upstream and downstream distance in 16 hours.
The ratio of downstream speed to upstream speed is 9 : 7 and speed of boat
in still water is 40 km/hr. If upstream speed is decreased by 20%, then find
the time taken by boat to cover 420 km in upstream.
एक नाव धारा के प्रनतकूल और धारा के अनुकूल 'D' ककमी की िरू ी 16 घंटे में तय कर सकती है ।
धारा के अनकु ू ल गनत का धारा के प्रनतकूल गनत से अनप
ु ात 9 : 7 है और शांत जल में नाव की गनत
40 ककमी/घंटा है । यदि धारा के प्रनतकूल गनत 20% कम हो जाती है , तो नाव द्वारा धारा के
प्रनतकूल 420 ककमी की िरू ी तय करने में लगने वाला समय ज्ञात कीजजए।
(A) 13 hours (B) 12 hours (C) 15 hours
(D) 14 hours (E) None of these

23. Siya and Jiya starts travelling from point A to point B respectively
towards each other. Speed of Siya is 37.5% more than speed of Jiya. If they
starts travelling at same time and Siya travels 81 km more than Jiya, then find
the distance between point A to point B.
लसया और जजया क्रमशः बबंि ु A से बबंि ु B तक एक िस
ू रे की ओर यात्रा करना शरू ु करती हैं। लसया
की गनत जजया की गनत से 37.5% अचधक है । यदि वे एक ही समय पर यात्रा करना शुरू करते हैं
और लसया जजया से 81 ककमी अचधक यात्रा करती है , तो बबंि ु A से बबंि ु B के बीच की िरू ी ज्ञात
कीजजए।
(A) 454 km (B) 589 km (C) 624 km
(D) 513 km (E) None of these

24. A shopkeeper marked an article 80% above the cost price and sold it after

https :
//www. https : https
https : //instagra
//youtube.c
facebo m.com/aas :
om/channe hisharoraso
l/UCYa4_Jr ok.com cial(?) //t.m
Orf8R5Kz2u
/aashis utm_mediu
e/stu

70
OtccXQ m=
haroras copy_link
ocial dified
giving a discount of Rs. 455 and still earned a profit of 15%. Find the selling
price of the article.
एक िक ु ानिार ने एक वस्तु पर लागत मल्
ू य से 80% अचधक मल्
ू य अंककत ककया और उसे 455
रुपये की छूट िे कर बेच दिया और किर भी 15% का लाभ अजजषत ककया। वस्तु का ववक्रय मूल्य
ज्ञात कीजजए।
(A) Rs. 769 (B) Rs. 805 (C) Rs. 643
(D) Rs. 567 (E) None of these

25. The numbers 'R' and 'S' are 80% less and 55% less respectively than the
third number 'T'. 'R' is how much present more/less than 'S'?
संख्या 'R' और 'S', तीसरी संख्या 'T' से क्रमशः 80% कम और 55% कम है । 'R', 'S' से ककतना
अचधक/कम उपजस्थत है ?
(A) 55.55% (B) 52.78% (C) 53.89%
(D) 54.61% (E) None of these

26. Average marks scored by a students in 6 subjects is 28. If average marks


scored by him excluding his highest and lowest marks is 20 and lowest
marks scored by him is 18, then find the highest marks scored by him.
एक छात्र द्वारा 6 ववर्यों में प्राप्त ककए गए औसत अंक 28 हैं। यदि उसके उच्चतम और ननम्नतम
अंकों को छोडकर उसके द्वारा बनाए गए औसत अंक 20 हैं और उसके द्वारा बनाए गए न्यूनतम
अंक 18 हैं, तो उसके द्वारा बनाए गए उच्चतम अंक ज्ञात कीजजए।
(A) 70 (B) 80 (C) 90
(D) 60 (E) None of these

27. Rs. 7500 is invested at R% p.a. simple interest and Rs. 9000 is invested at
(R + 3)% p.a. simple interest in two different schemes. Total interest of Rs.
6360 earned at the end of four years from both the schemes. Find the value of
R.
7500 रुपये प्रनत वर्ष साधारण ब्याज पर R% पर ननवेश ककया जाता है और 9000 रुपये (R + 3)%
प्रनतवर्ष साधारण ब्याज पर िो अलग-अलग योजनाओं में ननवेश ककया जाता है । िोनों योजनाओं से
चार साल के अंत में अजजषत कुल 6360 रुपये का ब्याज। R का मान ज्ञात कीजजए।
(A) 9 (B) 7 (C) 8
(D) 6 (E) None of these

28. After 10 years, the ratio of ages of 'X' and 'Y' will be 7 : 5 respectively.
After 9 years, 'X' will be 8 years elder than 'Z'. If 'Y' is 4 years younger than
'Z', then find the ratio of present age of 'Y' and 'Z'.
10 वर्ष बाि, 'X' और 'Y' की आयु का अनप ु ात क्रमशः 7 : 5 होगा। 9 वर्ष बाि 'X', 'Z' से 8 वर्ष बडा
होगा। यदि 'Y', 'Z' से 4 वर्ष छोटा है , तो 'Y' और 'Z' की वतषमान आयु का अनुपात ज्ञात कीजजए।

https :
//www. https : https
https : //instagra
//youtube.c
facebo m.com/aas :
om/channe hisharoraso
l/UCYa4_Jr ok.com cial(?) //t.m
Orf8R5Kz2u
/aashis utm_mediu
e/stu

71
OtccXQ m=
haroras copy_link
ocial dified
(A) 6 : 7 (B) 2 : 3 (C) 5 : 6
(D) 8 : 9 (E) None of these

29. Akshay started a business with an initial investment of Rs. 1800. After 'X +
3' months, Akshay added Rs. 400 more while Manshi entered into the
business by investing Rs. 6525. At the end of the year the ratio of profit share
of Akshay and Manshi is 8 : 9. Find the value of 'X'.
अक्षय ने 1800 रुपये के शुरुआती ननवेश के साथ एक व्यवसाय शुरू ककया। 'X + 3' महीनों के बाि,
अक्षय ने 400 रुपये और जोडे, जबकक मानशी ने 6525 रुपये का ननवेश करके व्यापार में प्रवेश
ककया। वर्ष के अंत में अक्षय और लाभ के दहस्से का अनुपात मानशी का अनुपात 8 : 9 है । 'X' का
मान ज्ञात कीजजए।
(A) 7 (B) 5 (C) 4
(D) 6 (E) None of these

30. Vessel A contains mixture of juice and water in the ratio of 5 : 8


respectively while vessel B contains mixture of juice and water in the ratio of
4 : 3 respectively. Total quantity of mixture in vessel A and vessel B is 346 L
and the difference between water of vessel A and juice of vessel A is 12 L.
Find the total quantity of juice in both vessels.
बतषन A में रस और पानी का लमश्ण क्रमशः 5 : 8 के अनुपात में है जबकक पात्र B में रस और पानी
का लमश्ण क्रमशः 4 : 3 के अनुपात में है । बतषन A और बतषन B में लमश्ण की कुल मात्रा 346
लीटर है और बतषन A के पानी और बतषन A के रस के बीच का अंतर 12 लीटर है । िोनों बतषनों में
रस की कुल मात्रा ज्ञात कीजजए।
(A) 188 L (B) 174 L (C) 154 L
(D) 168 L (E) None of these

31. Aman and Tiya together can complete 55.55% of a work in 12 days while
Tiya alone can complete 60% of the work in 32.4 days. Find the time taken by
Aman to complete 83.33% of the total work.
अमन और दटया एक साथ 12 दिनों में 55.55% काम पूरा कर सकते हैं जबकक दटया अकेले 32.4
दिनों में 60% काम पूरा कर सकती है । अमन द्वारा कुल कायष का 83.33% पूरा करने में ललया
गया समय ज्ञात कीजजए।
(A) 20 days (B) 30 days (C) 40 days
(D) 10 days (E) None of these

32. LCM of two numbers (which are not more than three digits) is 2261 and
their HCF is 7. Find the sum of both numbers.
िो संख्याओं का लघुत्तम समापवत्यष (जो तीन अंकों से अचधक नहीं हैं) 2261 है और उनका म.स.प.
7 है । िोनों संख्याओं का योग ज्ञात कीजजए।
(A) 322 (B) 448 (C) 252
(D) 514 (E) None of these

https :
//www. https : https
https : //instagra
//youtube.c
facebo m.com/aas :
om/channe hisharoraso
l/UCYa4_Jr ok.com cial(?) //t.m
Orf8R5Kz2u
/aashis utm_mediu
e/stu

72
OtccXQ m=
haroras copy_link
ocial dified
33. 660 ml of mixture P containing alcohol and water in the ratio of 8 : 3 is
mixed with 780 ml of mixture Q containing alcohol and water in the ratio of 7 :
6 respectively. Find the ratio of the quantities of alcohol to water in the final
mixture.
8 : 3 के अनपु ात में अल्कोहल और पानी वाले लमश्ण P के 660 लमलीलीटर को क्रमशः 7 : 6 के
अनुपात में अल्कोहल और पानी वाले लमश्ण Q के 780 लमलीलीटर के साथ लमलाया जाता है ।
अंनतम लमश्ण में अल्कोहल की मात्रा का पानी से अनुपात ज्ञात कीजजए।
(A) 4 : 3 (B) 5 : 9 (C) 6 : 7
(D) 5 : 3 (E) None of these

34. A shopkeeper marked his article 20% above its cost price and offered a
discount of X%. If cost price of the article is Rs. 840 and the shopkeeper gets
loss Rs. 84 on selling the article, then find the value of X.
एक िकु ानिार ने अपनी वस्तु को लागत मूल्य से 20% अचधक अंककत ककया और X% की छूट की
पेशकश की। यदि वस्तु का क्रय मल् ू य 840 रुपये है और िक
ु ानिार को वस्तु बेचने पर 84 रुपये की
हानन होती है , तो X का मान ज्ञात कीजजए।
(A) 35% (B) 25% (C) 45%
(D) 55% (E) None of these

35. Tiya and Piya started a business investing Rs. 15000 for 'M' months and
Rs. 18000 for 6 months, respectively. Out of total profit of Rs. 5400, profit
share of Piya is Rs. 2400. Find the value of 'M'.
दटया और वपया ने क्रमशः 'M' महीने के ललए 15000 रुपये और 6 महीने के ललए 18000 रुपये का
ननवेश कर एक व्यवसाय शुरू ककया। 5400 रुपये के कुल लाभ में से वपया का लाभ दहस्सा 2400
रुपये है । 'M' का मान ज्ञात कीजजए।
(A) 9 (B) 5 (C) 8
(D) 7 (E) None of these

36. Rohit spent 50% of his monthly income on rent, 35% of the remaining of
travelling, 20% of the remaining on shopping. If amount spent on travelling is
Rs. 1400, then find the amount spent of shopping.
रोदहत अपनी मालसक आय का 50% ककराए पर, शेर् का 35% यात्रा पर, शेर् का 20% खरीिारी पर
खचष करता है । यदि यात्रा पर खचष की गई रालश 1400 रुपये है , तो खरीिारी पर खचष की गई रालश
ज्ञात कीजजए।
(A) Rs. 520 (B) Rs. 480 (C) Rs. 690
(D) Rs. 430 (E) None of these

37. Sahil travels first 330 km at a speed of 55 km/hr, next 288 km at speed of
72 km/hr and rest of the 335 km at a speed of 67 km/hr. Find the average
speed of the Sahil in the whole journey.

https :
//www. https : https
https : //instagra
//youtube.c
facebo m.com/aas :
om/channe hisharoraso
l/UCYa4_Jr ok.com cial(?) //t.m
Orf8R5Kz2u
/aashis utm_mediu
e/stu

73
OtccXQ m=
haroras copy_link
ocial dified
सादहल पहले 330 ककमी की यात्रा 55 ककमी/घंटा की गनत से, अगले 288 ककमी की यात्रा 72
ककमी/घंटा की गनत से और शेर् 335 ककमी की यात्रा 67 ककमी/घंटा की गनत से करता है । पूरी यात्रा
में सादहल की औसत गनत ज्ञात कीजजए।
(A) 42.10 km/hr (B) 59.56 km/hr (C) 63.53 km/hr
(D) 32.09 km/hr (E) None of these

38. The perimeter of rectangle is 62 cm less than perimeter of a square whose


side is 48 cm. Find the difference between the areas of rectangle and square,
If breadth of rectangle is 5 cm less than its length.
आयत का पररमाप उस वगष के पररमाप से 62 सेमी कम है जजसकी भुजा 48 सेमी है । आयत और
वगष के क्षेत्रिल के बीच अंतर ज्ञात कीजजए, यदि आयत की चौडाई उसकी लंबाई से 5 सेमी कम है ।
(A) 1229 cm2 (B) 1236 cm2 (C) 1254 cm2
(D) 1241 cm2 (E) None of these

39. Average marks scored by Vinay in X subjects of first term was (X + 16). In
16 subjects of second and third term he scored marks with an average of 54.
After that his overall average marks increased by 2. Find the value of X.
पहले सत्र के X ववर्यों में ववनय द्वारा प्राप्त औसत अंक (X + 16) थे। िस
ू रे और तीसरे के 16
ववर्यों में उसने 54 के औसत से अंक प्राप्त ककए। उसके बाि उसके कुल औसत अंकों में 2 की
वद्
र चध हुई। X का मान ज्ञात कीजजए।
(A) 34 (B) 32 (C) 38
(D) 376 (E) None of these

40. 28 men can complete a piece of work in 44 days. After 20 days 'X' more
men joined the work and the whole work was completed in 8 days less than
its stipulated time. Find the value of 'X‘.
28 पुरुर् एक कायष को 44 दिनों में पूरा कर सकते हैं। 20 दिनों के बाि 'X' और आिमी काम में
शालमल हो गए और परू ा काम ननधाषररत समय से 8 दिन कम में परू ा हो गया। 'X' का मान ज्ञात
कीजजए।
(A) 16 (B) 17 (C) 15
(D) 14 (E) None of these

41. The ratio of juice to water in a vessel is 7 : 5 respectively. If 24 liters of


mixture is taken out and 28 liters of juice is added to mixture, then the ratio
of juice to water becomes 7 : 4. Find the initial quantity of mixture.
एक बतषन में जस
ू का पानी से अनप ु ात क्रमशः 7 : 5 है । यदि 24 लीटर लमश्ण ननकाल ललया जाता
है और लमश्ण में 28 लीटर रस लमला दिया जाता है , तो रस का पानी से अनुपात 7 : 4 हो जाता है ।
लमश्ण की आरं लभक मात्रा ज्ञात कीजजए।
(A) 289 L (B) 216 L (C) 256 L
(D) 225 L (E) None of these

https :
//www. https : https
https : //instagra
//youtube.c
facebo m.com/aas :
om/channe hisharoraso
l/UCYa4_Jr ok.com cial(?) //t.m
Orf8R5Kz2u
/aashis utm_mediu
e/stu

74
OtccXQ m=
haroras copy_link
ocial dified
42. If 4 women and 12 men can complete a work in 18 days, then find the time
taken by 7 women and 15 men to complete the work.
यदि 4 मदहलाएं और 12 परु ु र् एक कायष को 18 दिनों में परू ा कर सकते हैं, तो 7 मदहलाओं और 15
पुरुर्ों द्वारा कायष को पूरा करने में ललया गया समय ज्ञात कीजजए।
(A) 3 days (B) 5 days (C) 6 days
(D) 4 days (E) None of these

43. Nisha and Misti started a business with investment Rs. 5400 and Rs. 6900.
After 7 month Nisha added Rs. X and Misti added Rs. (X + 200). If at the end
of the year profit share of Misti is Rs. 19700 out of total profit of Rs. 35460,
then find the value of X.
ननशा और लमष्टी ने 5400 रुपये और 6900 रुपये के ननवेश के साथ एक व्यवसाय शुरू ककया। 7
महीने के बाि ननशा ने X रुपये जोडे और लमस्टी ने (X + 200) रुपये जोडे। यदि वर्ष के अंत में
लमस्टी का लाभ दहस्सा 35460 रुपये के कुल लाभ में से 19700 रुपये है , तो X का मान ज्ञात
कीजजए।
(A) 2520 (B) 2480 (C) 2360
(D) 2240 (E) None of these

44. Monthly income of Varsha is Rs. (X + 3500). She spent her monthly
income such that 12% in entertainment, 19% of travelling, 21% on education,
23% on food, 9% on rent and rest deposits in bank. If the difference between
sum of travelling and education expenses and amount deposited in bank is
Rs. 4320, then find the value of 'X'.
वर्ाष की मालसक आय (X + 3500) रुपये है । उसने अपनी मालसक आय इस तरह खचष की कक 12%
मनोरं जन में , 19% यात्रा पर, 21% लशक्षा पर, 23% भोजन पर, 9% ककराए पर और बाकी बैंक में
जमा रालश पर। यदि यात्रा और लशक्षा व्यय के योग और बैंक में जमा रालश के बीच का अंतर 4320
रुपये है , तो 'X' का मान ज्ञात कीजजए।
(A) 13800 (B) 14500 (C) 12700
(D) 15600 (E) None of these

45. The ratio of cost price of a bag and a book is 4 : 5. A shopkeeper sold the
bag at 20% profit and book at 20% less. If the difference between their selling
price is Rs. 120, then find the cost price of book.
एक बैग और एक ककताब के क्रय मूल्य का अनुपात 4 : 5 है । एक िक ु ानिार ने बैग को 20% लाभ
पर बेचा और 20% कम पर बक ु ककया। यदि उनक े ववक्रय म ल्
ू य के बीच का अंतर 120 रुपये है , तो
पुस्तक का क्रय मूल्य ज्ञात कीजजए।
(A) Rs. 750 (B) Rs. 520 (C) Rs. 490
(D) Rs. 630 (E) None of these

https :
//www. https : https
https : //instagra
//youtube.c
facebo m.com/aas :
om/channe hisharoraso
l/UCYa4_Jr ok.com cial(?) //t.m
Orf8R5Kz2u
/aashis utm_mediu
e/stu

75
OtccXQ m=
haroras copy_link
ocial dified
46. A boat covers Y km downstream and upstream distance in 20 hours.
Speed of boat in still water is 2X km/hr and speed of stream is (X - 15) km/hr.
If upstream is 40 km/hr, then find the time taken by boat to cover (Y + 180) km
downstream distance.
एक नाव धारा के अनुकूल और धारा के प्रनतकूल Y ककमी की िरू ी 20 घंटे में तय करती है । शांत जल
में नाव की गनत 2X ककमी/घंटा है और धारा की गनत (X - 15) ककमी/घंटा है । यदि धारा के प्रनतकूल
40 ककमी/घंटा है , तो नाव द्वारा धारा के अनुकूल (Y + 180) ककमी की िरू ी तय करने में लगने
वाला समय ज्ञात कीजजए।
(A) 11 hours (B) 13 hours (C) 14 hours
(D) 12 hours (E) None of these

47. The average of runs scored in 28 match is 45. If the average runs scored
in first 15 match is 48 and average runs scored in next 7 matches is 41, then
find the total runs scored in remaining matches.
28 मैचों में बनाए गए रनों का औसत 45 है । यदि पहले 15 मैचों में बनाए गए औसत रन 48 हैं
और अगले 7 मैचों में बनाए गए औसत रन 41 हैं, तो शेर् मैचों में बनाए गए कुल रन ज्ञात
कीजजए।
(A) 232 (B) 248 (C) 253
(D) 227 (E) None of these

48. Rs. 3840 is invested at X% p.a. simple interest for 5 years in scheme 'M'.
The interest received from scheme 'M' is then invested at 37.5% p.a.
compounded annually in scheme 'N' for 2 years. If the interest received from
scheme 'N' is Rs. 4275, then find the value of 'X'.
योजना 'M' में 5 साल के ललए X% प्रनत वर्ष साधारण ब्याज पर 3840 रुपये का ननवेश ककया जाता
है । योजना 'M' से प्राप्त ब्याज को 2 साल के ललए योजना 'N' में सालाना 37.5% सालाना
चक्रवद्र चध पर ननवेश ककया जाता है । यदि योजना 'N' से प्राप्त ब्याज 4275 रुपये है , तो 'X' का
मान ज्ञात कीजजए।
(A) 23 (B) 26 (C) 25
(D) 24 (E) None of these

49. Speed of car A is 90% more than that of car B. Find the distance travelled
by the car B in 4 hours 24 minutes, If time taken by the car A to cover 798 km
in 6 hours.
कार A की गनत कार B की गनत से 90% अचधक है । कार B द्वारा 4 घंटे 24 लमनट में तय की गई
िरू ी ज्ञात कीजजए, यदि कार A को 6 घंटे में 798 ककमी की िरू ी तय करने में समय लगता है ।
(A) 256 km (B) 308 km (C) 489 km
(D) 518 km (E) None of these

50. The length and breadth of a rectangle is 30 cm and 24 cm, respectively. If

https :
//www. https : https
https : //instagra
//youtube.c
facebo m.com/aas :
om/channe hisharoraso
l/UCYa4_Jr ok.com cial(?) //t.m
Orf8R5Kz2u
/aashis utm_mediu
e/stu

76
OtccXQ m=
haroras copy_link
ocial dified
the length of the rectangle is decreased by 15% and its breadth is increased
by 5%, then find the percentage change in its perimeter.
एक आयत की लंबाई और चौडाई क्रमशः 30 सेमी और 24 सेमी है । यदि आयत की लंबाई में 15%
की कमी की जाती है और इसकी चौडाई में 5% की वद्
र चध की जाती है , तो इसके पररमाप में प्रनतशत
पररवतषन ज्ञात कीजजए।
(A) 5.46% (B) 7.34% (C) 4.89%
(D) 6.11% (E) None of these

51. If we multiply numerator of a fraction by 4 and subtract 4 from it then it


becomes equal to denominator of that fraction. When the numerator is
doubled and denominator is increased by 50% then it becomes 1/2. What will
be four times of that fraction?
यदि हम ककसी लभन्न के अंश को 4 से गुणा करें और उसमें से 4 घटा िें तो वह उस लभन्न के हर के
बराबर हो जाता है । जब अंश को िोगुना और हर में 50% की वद्
र चध की जाती है तो यह 1/2 हो जाता
है । उस लभन्न का चार गन
ु ा क्या होगा?
(A) 4/5 (B) 3/2 (C) 8/5
(D) 5/2 (E) 5/3

52. Train A and B are travelling at x km/hr and (x + 36) km/hr respectively.
Train B crosses train A when running in the same direction in 31.5 seconds.
Find the sum of speed of both the trains, If they pass each other in 9 seconds
while running in opposite direction.
ट्रे न A और B क्रमशः x ककमी/घंटा और (x + 36) ककमी/घंटा की गनत से यात्रा कर रही हैं। ट्रे न B
समान दिशा में चलते हुए ट्रे न A को 31.5 सेकंड में पार करती है । िोनों ट्रे नों की गनत का योग ज्ञात
कीजजए यदि वे ववपरीत दिशा में िौडते हुए एक िस ू रे को 9 सेकंड में पार करती हैं
(A) 126 km/hr (B) 120 km/hr (C) 116 km/hr
(D) 136 km/hr (E) None of these

53. The difference between two numbers is 500. On dividing the larger
number by the smaller, we get 3 as quotient and 0 as remainder. What is the
smaller number?
िो संख्याओं का अंतर 500 है । बडी संख्या को छोटी संख्या से भाग िे ने पर भागिल 3 और शेर्िल
0 प्राप्त होता है । छोटी संख्या क्या है ?
(A) 250 (B) 200 (C) 100
(D) 300 (E) None of these

54. Anubhav had some money. He invested 55% of this amount in scheme X
offering simple interest of 27% pa and rest in scheme Y offering compound
interest of 20% pa compounded annually. At the end of 2 years, total interest

https :
//www. https : https
https : //instagra
//youtube.c
facebo m.com/aas :
om/channe hisharoraso
l/UCYa4_Jr ok.com cial(?) //t.m
Orf8R5Kz2u
/aashis utm_mediu
e/stu

77
OtccXQ m=
haroras copy_link
ocial dified
received by him for both the schemes is Rs. 2970. Find the amount that
Anubhav had initially?
अनुभव के पास कुछ पैसे थे। उसने इस रालश का 55% योजना X में 27% प्रनतवर्ष के साधारण
ब्याज की पेशकश में ननवेश ककया और शेर् योजना Y में 20% वावर्षक चक्रवद् र चध ब्याज की पेशकश
की। 2 वर्ष के अंत में , िोनों योजनाओं के ललए उसके द्वारा प्राप्त कुल ब्याज 2970 रुपये है ।
अनुभव के पास शुरू में ककतनी रालश थी?
(A) Rs. 9000 (B) Rs. 7000 (C) Rs. 4000
(D) Rs. 6000 (E) None of these

55. The marked price of the laptop is Rs. 24000 which is 25% more than its
cost price. If the shopkeeper offers two successive discounts 10% and x%
respectively on the marked price of the laptop, while he gets the profit of
1.25%, then find the value of x.
लैपटॉप का अंककत मल्ू य 24000 रुपये है जो इसके लागत मल् ू य से 25% अचधक है । यदि िक
ु ानिार
लैपटॉप के अंककत मूल्य पर क्रमशः 10% और x% की िो क्रलमक छूट प्रिान करता है , जबकक उसे
1.25% का लाभ प्राप्त होता है , तो x का मान ज्ञात कीजजए।
(A) 10 (B) 12 (C) 8
(D) 24 (E) None of these

56. There are two persons having speeds of work, the faster one can finish a
work 4 days earlier than slower one alone. They both together can finish the
same work in 8(8/9) days. In how many days faster one can finish the work?
िो व्यजक्तयों की कायष करने की गनत है , धीमी गनत वाले व्यजक्त की तुलना में 4 दिन पहले तेजी से
काम परू ा कर सकता है । वे िोनों लमलकर उसी कायष को 8(8/9) दिनों में परू ा कर सकते हैं। कोई
कायष ककतने दिनों में तेजी से पूरा कर सकता है ?
(A) 24 days (B) 16 days (C) 18 days
(D) 20 days (E) None of these

57. In a mix of milk and water in which water is 25% of milk. 20 liters of the
mix is taken out from ‘Y’ liters of the mix. If 12 liters water is added to the
remaining mixture, then the ratio of milk and water becomes 8 : 5. Find the
value of Y.
िध
ू और पानी के लमश्ण में जजसमें पानी िध ू का 25% है । लमश्ण के 'Y' लीटर में से 20 लीटर
लमश्ण ननकाल ललया जाता है । यदि शेर् लमश्ण में 12 लीटर पानी लमला दिया जाए, तो िध ू और
पानी का अनुपात 8 : 5 हो जाता है । Y का मान ज्ञात कीजजए।
(A) 25 L (B) 48 L (C) 20 L
(D) 60 L (E) None of these

58. The ratio of the present age of Rajat and Deepak is 4 : x. Rajat is 6 years
younger than Prateek. Prateek’s age after 4 years will be 34 years. The
difference between Deepak’s age and Rajat’s age is the same as present age

https :
//www. https : https
https : //instagra
//youtube.c
facebo m.com/aas :
om/channe hisharoraso
l/UCYa4_Jr ok.com cial(?) //t.m
Orf8R5Kz2u
/aashis utm_mediu
e/stu

78
OtccXQ m=
haroras copy_link
ocial dified
Prateek. Deepak is older than Rajat. What should come in place of x?
रजत और िीपक की वतषमान आयु का अनुपात 4 : x है । रजत प्रतीक से 6 साल छोटा है । 4 वर्ष
बाि प्रतीक की आयु 34 वर्ष होगी। िीपक और रजत की उम्र के बीच का अंतर प्रतीक की वतषमान
उम्र के बराबर है । िीपक रजत से बडा है । x के स्थान पर क्या आना चादहए?
(A) 7 (B) 9 (C) 10
(D) 8 (E) None of these

59. The sum of the volume of cylinder and volume of cone is 2190π cm3 &
height of both cylinder and cone is 10 cm each. If the radius of the cone is 15
cm, then find the ratio of radius of cylinder to radius of cone.
बेलन के आयतन और शंकु के आयतन का योग 2190π सेमी3 है और बेलन और शंकु
दोनों की ऊँचाई प्रत्येक 10 सेमी है । यदद शंकु की त्रिज्या 15 सेमी है , तो बेलन की त्रिज्या का
शंकु की त्रिज्या से अनुपात ज्ञात कीजिए।
(A) 3 : 7 (B) 4 : 7 (C) 4 : 5
(D) 2 : 3 (E) None of these

60. A boat covers (X + 50) km downstream in 5 hours, (Y + 24) km upstream in


5 hours, then find time taken by boat to travel (4X + Y) km upstream distance,
If speed of boat in still water is thrice that of stream which is 4 km/hr.
एक नाव धारा के अनुकूल (X + 50) ककमी 5 घंटे में , (Y + 24) ककमी धारा के प्रनतकूल 5 घंटे में
तय करती है , तो नाव द्वारा धारा के प्रनतकूल (4X + Y) ककमी की िरू ी तय करने में लगने वाला
समय ज्ञात कीजजए, यदि शांत जल में नाव की गनत है धारा का तीन गुना जो 4 ककमी/घंटा है ।
(A) 15 hours (B) 17 hours (C) 18 hours
(D) 20 hours (E) None of these

61. The ratio between the number of mangoes that P and Q have is 9 : 11.
After eating same number of mangoes the ratio of the mangoes left for P and
Q is 2 : 3. The total number of mangoes left with P and Q after eating is 50.
Find the number of mangoes P had earlier.
P और Q के पास आमों की संख्या का अनुपात 9 : 11 है । समान संख्या में आम खाने के बाि P
और Q के ललए बचे आमों का अनप ु ात 2 : 3 है । खाने के बाि P और Q के पास बचे आमों की कुल
संख्या 50 है । P के पास पहले आमों की संख्या ज्ञात कीजजए।
(A) 45 (B) 30 (C) 25
(D) 20 (E) None of these

62. In a survey of Laptops users on four major companies it is found that 20%
use Lenevo, 50% of remaining use HP and 30% of the rest use Dell and rest X
use Asus. If the total number of Laptop users are 22500. Find the value of X.
चार प्रमुख कंपननयों पर लैपटॉप उपयोगकताषओं के एक सवेक्षण में यह पाया गया है कक 20%
Lenovo का उपयोग करते हैं, शेर् के 50% HP का उपयोग करते हैं और शेर् 30% Dell का

https :
//www. https : https
https : //instagra
//youtube.c
facebo m.com/aas :
om/channe hisharoraso
l/UCYa4_Jr ok.com cial(?) //t.m
Orf8R5Kz2u
/aashis utm_mediu
e/stu

79
OtccXQ m=
haroras copy_link
ocial dified
उपयोग करते हैं और शेर् X Asus का उपयोग करते हैं। यदि लैपटॉप उपयोगकताषओं की कुल
संख्या 22500 है । X का मान ज्ञात कीजजए।
(A) 7000 (B) 6300 (C) 3600
(D) 4500 (E) None of these

63. In a shop, IPL jerseys are sold which are of teams : KKR, CSK, RCB and
RR. The number of jerseys present are 4, 5, 7 and 9 of KKR, CSK, RCB and
RR respectively. Find the probability of either KKR or CSK jerseys being sold.
एक िक ु ान में आईपीएल की जसी बेची जाती है जो टीमों की हैं : KKR, CSK, RCB और RR ।
मौजूि जसी की संख्या क्रमशः KKR, CSK, RCB और RR की 4, 5, 7 और 9 है । या तो KKR या
CSK की जसी बेचे जाने की प्रानयकता ज्ञात कीजजए।
(A) 9/25 (B) 5/25 (C) 12/25
(D) 4/25 (E) None of these

64. Three cooks P, Q and R can cook a dish in 16, 20 and 24 hours
respectively. They all started the work together but after (X - 8) hours, P left
the cooking and X hours before completion of dish Q also left. The whole
dish is cooked in (X + 5) hours. Find the value of X.
तीन रसोइया P, Q और R एक व्यंजन को क्रमशः 16, 20 और 24 घंटे में पका सकते हैं। वे सभी
एक साथ काम करना शुरू करते हैं लेककन (X - 8) घंटों के बाि, P खाना बनाना छोड िे ता है और
पकवान परू ा होने से X घंटे पहले Q भी काम छोड िे ता है । परू ा व्यंजन (X + 5) घंटों में पक जाता
है । X का मान ज्ञात कीजजए।
(A) 20 (B) 50 (C) 10
(D) 30 (E) None of these

65. Out of two parallel sides of Trapezium, larger side is 16 cm and the
distance between them is 2 cm less than the smaller side. If the area of the
trapezium is 104 cm2, then find the distance between the two parallel sides.
समलंब की िो समानांतर भुजाओं में से बडी भुजा 16 सेमी है और उनके बीच की िरू ी छोटी भुजा से
2 सेमी कम है । यदि समलंब का क्षेत्रिल 104 सेमी2 हो, तो िोनों समान्तर भुजाओं के बीच की िरू ी
ज्ञात कीजजए।
(A) 3 cm (B) 8 cm (C) 4 cm
(D) 2 cm (E) None of these

66. A boat can travel 63 km in upstream and return back to starting point in
total of P hours and speed of the boat in still water is 8 km/hr, then time taken
by the boat to cover 54 km in downstream and 21 km in upstream together is
9 hours. Find the value of P.
एक नाव धारा के प्रनतकूल 63 ककमी की यात्रा कर सकती है और कुल P घंटे में वापस प्रारं लभक बबंि ु
पर वापस आ सकती है और जस्थर पानी में नाव की गनत 8 ककमी/घंटा है , तो

https :
//www. https : https
https : //instagra
//youtube.c
facebo m.com/aas :
om/channe hisharoraso
l/UCYa4_Jr ok.com cial(?) //t.m
Orf8R5Kz2u
/aashis utm_mediu
e/stu

80
OtccXQ m=
haroras copy_link
ocial dified
नाव द्वारा धारा के अनुकूल 54 ककमी और धारा के प्रनतकूल 21 ककमी की िरू ी तय करने में ललया
गया समय साथ में 9 घंटे हैं। P का मान ज्ञात कीजजए।
(A) 14 km (B) 12 km (C) 16 km
(D) 10 km (E) None of these

67. Ashok borrowed sum of money of Rs. X at the rate of 20% p.a.
compounded annually. After one year, If the rate of interest is reduced to
15%, then at the end of two years he paid Rs. 470 less interest. What is the
value of X?
अशोक ने सालाना चक्रवद्र चध 20% प्रनत वर्ष की िर से X रुपये की धनरालश उधार ली। एक वर्ष के
बाि, यदि ब्याज की िर को घटाकर 15% कर दिया जाता है , तो िो वर्ष के अंत में उसने 470 रुपये
कम ब्याज का भग ु तान ककया। X का मल् ू य क्या है ?
(A) 4000 (B) 1100 (C) 1000
(D) 1200 (E) None of these

68. A shopkeeper has a total of 450 items (pencils + pens) in which 200 are
pens. He sold total number of pens at Rs. 35 per pen and total number of
pencils at Rs. P per pencil. If the mean price of all items taken together is Rs.
20 per item, then what is the value of P?
एक िक ु ानिार के पास कुल 450 वस्तुएँ (पें लसल + पेन) हैं जजनमें 200 पेन हैं। उसने पेन की कुल
संख्या 35 रुपये प्रनत पेन और पें लसल की कुल संख्या P रुपये प्रनत पें लसल की िर से बेची। यदि सभी
वस्तओु ं को लमलाकर औसत मल् ू य 20 रुपये प्रनत वस्तु है , तो P का मल्ू य क्या है ?
(A) 4 (B) 8 (C) 5
(D) 7 (E) None of these

69. Total distance of a journey is 500 km. P covered the first 20 km at a speed
of 40 km/hr, the next 140 km at 70 km/hr and remaining journey at 34 km/hr.
Find the average speed to complete the journey.
एक यात्रा की कुल िरू ी 500 ककमी है । P ने पहले 20 ककमी को 40 ककमी/घंटा की गनत से, अगले
140 ककमी को 70 ककमी/घंटा की गनत से और शेर् यात्रा को 34 ककमी/घंटा की गनत से तय ककया।
यात्रा को पूरा करने के ललए औसत गनत ज्ञात कीजजए।
(A) 40 kn/hr (B) 20 km/hr (C) 19 km/hr
(D) 25 km/hr (E) None of these

70. Rajat, Ram and Ritesh started a business. Rajat invested four times as
much as Ram’s investment and the investment of Ritesh is 40% less than that
of Ram. At the end of one year total profit of 7000 was earned, in which
Rajat’s share was 5000. What is the difference between share of Ram and
Ritesh?

https :
//www. https : https
https : //instagra
//youtube.c
facebo m.com/aas :
om/channe hisharoraso
l/UCYa4_Jr ok.com cial(?) //t.m
Orf8R5Kz2u
/aashis utm_mediu
e/stu

81
OtccXQ m=
haroras copy_link
ocial dified
रजत, राम और ररतेश ने एक व्यवसाय शुरू ककया। रजत ने राम के ननवेश से चार गुना ननवेश
ककया और ररतेश का ननवेश राम के ननवेश से 40% कम है । एक वर्ष के अंत में 7000 का कुल लाभ
अजजषत ककया गया, जजसमें रजत का दहस्सा 5000 था। राम और ररतेश के दहस्से के बीच ककतना
अंतर है ?
(A) Rs. 400 (B) Rs. 500 (C) Rs. 600
(D) Rs. 700 (E) None of these

71. A Jewelry shop owner has a bag, which contains jewels. It contains 4
necklaces and 8 rings. There is another bag which contains 5 necklaces and
5 rings. One jewel is to be drawn from either of the two bags. What is the
probability of drawing a necklace?
एक आभूर्ण की िक ु ान के माललक के पास एक थैला है , जजसमें गहने हैं। इसमें 4 हार और 8
अंगदू ियां हैं। एक और थैला है जजसमें 5 हार और 5 अंगदू ियां हैं। िोनों थैललयों में से ककसी एक में से
एक रत्न ननकालना है । एक हार ननकालने की प्रानयकता क्या है ?
(A) 15/21 (B) 7/22 (C) 5/12
(D) 20/21 (E) None of these

72. Aditi gets Rs. 700 money as a prize in a contest. She spends 25% of the
amount in buying chocolates and X% of the remaining in buying fruits and
rest of it, she gives to her brother. If the difference between the amount she
gives to her brother and amount she spent on buying chocolates is 140, then
find the value of X.
एक प्रनतयोचगता में अदिनत को परु स्कार के रूप में 700 रुपये लमलते हैं। वह रालश का 25% चॉकलेट
खरीिने में और शेर् का X% िल खरीिने में खचष करती है और शेर् रालश वह अपने भाई को िे ती है ।
यदि उसके द्वारा अपने भाई को िी गई रालश और उसके द्वारा चॉकलेट खरीिने पर खचष की गई
रालश के बीच का अंतर 140 है , तो X का मान ज्ञात कीजजए।
(A) 70 (B) 40 (C) 36
(D) 45 (E) None of these

73. The average weight of some boys in a group is 25.5 kg. If 4 boys with an
average of 23.2 kg leave the group and 5 boys with an average weight of
31.66 kg join the group, the average of the group increases by 2 kg. What is
the number of boys initially?
एक समूह में कुछ लडकों का औसत भार 25.5 ककग्रा है । यदि 23.2 ककग्रा के औसत वाले 4
लडकेसमह ू छोड िे ते हैं और 31.66 ककग्रा के औसत वजन वाले 5 लडके समह ू में शालमल हो जाते हैं,
तो समूह का औसत 2 ककग्रा बढ जाता है । प्रारं भ में लडकों की संख्या ककतनी है ?
(A) 19 (B) 25 (C) 12
(D) 4 (E) None of these

74. Two pipes P and Q can fill a tank in 30 min and 35 min respectively and a

https :
//www. https : https
https : //instagra
//youtube.c
facebo m.com/aas :
om/channe hisharoraso
l/UCYa4_Jr ok.com cial(?) //t.m
Orf8R5Kz2u
/aashis utm_mediu
e/stu

82
OtccXQ m=
haroras copy_link
ocial dified
valve A can empty that tank in 70 min. All of the three start their work and
after 8 min valve A has been closed. What is the total time taken to fill the
tank from beginning?
िो पाइप P और Q एक टैंक को क्रमशः 30 लमनट और 35 लमनट में भर सकते हैं और एक वाल्व A
उस टैंक को 70 लमनट में खाली कर सकता है । तीनों अपना काम शरू ु करते हैं और 8 लमनट के बाि
वाल्व A को बंि कर दिया जाता है । शुरुआत से टैंक को भरने में कुल ककतना समय लगता है ?
(A) 20 min (B) 50 min (C) 18 min
(D) 30 min (E) None of these

75. A woman sells mobile covers each of Rs. X after giving a discount of 22%
on the marked price. Had she not given any discount, she would have earned
a profit of 25%. If the cost price of mobile cover is decreased by 30% and she
wants to earn 25% profit, she sells it for Rs. 350. Find the value of X.
एक मदहला अंककत मूल्य पर 22% की छूट िे ने के बाि X रुपये के प्रत्येक मोबाइल कवर को बेचती
है । यदि उसने कोई छूट नहीं िी होती, तो उसे 25% का लाभ होता। यदि मोबाइल कवर का लागत
मूल्य 30% कम कर दिया जाता है और वह 25% लाभ अजजषत करना चाहती है , तो वह इसे 350
रुपये में बेचती है । X का मान ज्ञात कीजजए।
(A) 390 (B) 440 (C) 450
(D) 240 (E) None of these

76. Three persons P, Q and R with their cars start from the same place in
same direction. P and R start at 8 : 00 AM and Q starts at 11 : 00 AM. Q
overtakes P at 12 : 00 PM and then doubles the speed of his car and
overtakes R at 2 : 00 PM. Find the ratio between the speed of car P and car R.
तीन व्यजक्त P, Q और R अपनी कारों के साथ एक ही स्थान से एक ही दिशा में चलना शरू ु करते
हैं। P और R सुबह 8 : 00 बजे शुरू होते हैं और Q सुबह 11 : 00 बजे शुरू होते हैं। Q िोपहर 12 :
00 बजे P से आगे ननकल जाता है और किर अपनी कार की गनत को िोगुना कर िे ता है और िोपहर
2 : 00 बजे R से आगे ननकल जाता है । कार P और कार R की गनत के बीच अनप ु ात ज्ञात कीजजए।
(A) 5 : 3 (B) 4 : 3 (C) 3 : 10
(D) 4 : 5 (E) None of these

77. The average price of a t - shirt in a garment store is Rs. A. There are three
kinds of t - shirts in the store namely full sleeve, half sleeve and no sleeve.
The ratio between the number of full sleeve, number of half sleeve and
number of no sleeve is 3 : 4 : 5 respectively. If the average price o a full
sleeve t - shirt is Rs. (A + 1) and the average price of a no sleeve t - shirt is
Rs. (A - 1) , then find the average price of a half sleeve t - shirt in the store.

https :
//www. https : https
https : //instagra
//youtube.c
facebo m.com/aas :
om/channe hisharoraso
l/UCYa4_Jr ok.com cial(?) //t.m
Orf8R5Kz2u
/aashis utm_mediu
e/stu

83
OtccXQ m=
haroras copy_link
ocial dified
एक गारमें ट स्टोर में एक टी-शटष की औसत कीमत A रुपये है । स्टोर में तीन प्रकार अथाषत ् पूरी
आस्तीन, आधी आस्तीन और बबना आस्तीन की टी-शटष हैं। परू ी आस्तीन की संख्या, आधी
आस्तीन की संख्या और बबना आस्तीन की संख्या के बीच का अनुपात क्रमशः 3 : 4 : 5 है । यदि
पूरी आस्तीन की टी-शटष की औसत कीमत रुपये (A + 1) है और बबना आस्तीन की टी-शटष की
औसत कीमत रुपये (A - 1) है , तो आधी आस्तीन की टी-शटष की औसत कीमत ज्ञात कीजजए।
(A) (2A + 1) /2 (B) (A + 1) /2 (C) (2A-1) /2
(D) (2A + 3) /2 (E) None of these

78. Aman and Barkha are working on the assignment. Aman takes 6 hours to
type 34 pages while Barkha takes 5 hours to type 45 pages. Find the time
taken by them to type an assignment of 220 pages, If they are working
together.
अमन और बरखा असाइनमें ट पर काम कर रहे हैं। अमन को 34 पेज टाइप करने में 6 घंटे लगते हैं
जबकक बरखा को 45 पेज टाइप करने में 5 घंटे लगते हैं। 220 पष्र िों का एक असाइनमें ट टाइप
करने के ललए उनके द्वारा ललया गया समय ज्ञात कीजजए, यदि वे एक साथ काम कर रहे हैं।
(A) 15 (B) 85 (C) 55
(D) 75 (E) None of these

79. A sum of money is divided among P, Q, R and S in the ratio 5 : 4 : 7 : 9


respectively. If the share of P is Rs. 2300 more than that of Q, then what is the
total amount of R and S together?
एक धनरालश P, Q, R और S के बीच क्रमशः 5 : 4 : 7 : 9 के अनप ु ात में बांटी जाती है । यदि P का
दहस्सा Q की तुलना में 2300 रुपये अचधक है , तो R और S की लमलाकर कुल रालश ककतनी है ?
(A) Rs. 36800 (B) Rs. 36000 (C) Rs. 35000
(D) Rs. 37200 (E) None of these

80. Total surface area of cuboid of length 20 m and height 6 m is 656 m2.
What is the difference between length and breadth of cuboid?
20 मीटर लंबाई और 6 मीटर ऊंचाई वाले घनाभ का कुल सतह क्षेत्रिल 656 मीटर2 है । घनाभ की
लंबाई और चौडाई में ककतना अंतर है ?
(A) 15 m (B) 10 m (C) 12 m
(D) 21 m (E) None of these

81. Average income of P for the first 6 months is Rs. 6000 and for the last 6
months is Rs. 6500 respectively. If his average expenditure for the year is Rs.
5000, then what is the savings of P for the whole year?

https :
//www. https : https
https : //instagra
//youtube.c
facebo m.com/aas :
om/channe hisharoraso
l/UCYa4_Jr ok.com cial(?) //t.m
Orf8R5Kz2u
/aashis utm_mediu
e/stu

84
OtccXQ m=
haroras copy_link
ocial dified
पहले 6 महीनों के ललए P की औसत आय 6000 रुपये है और वपछले 6 महीनों के ललए क्रमशः
6500 रुपये है । यदि वर्ष के ललए उसका औसत व्यय 5000 रुपये है , तो पूरे वर्ष के ललए P की बचत
ककतनी है ?
(A) Rs. 20000 (B) Rs. 22000 (C) Rs. 15000
(D) Rs. 23000 (E) None of these

82. Aman purchased 150 carpets at Rs. X each. He spends Rs. 7000 on
transportation. He fixed the label price of each carpet as Rs. 400 and decides
to give a discount of 5% on labeled price. If his gain percentage is 42.5%,
then find the value of X.
अमन ने 150 कालीन X रुपये में खरीिे । वह पररवहन पर 7000 रुपये खचष करता है । उसने प्रत्येक
कालीन का लेबल मूल्य 400 रुपये तय ककया और लेबल मूल्य पर 5% की छूट िे ने का िैसला
ककया। यदि उसका लाभ प्रनतशत 42.5% है , तो X का मान ज्ञात कीजजए।
(A) 50 (B) 60 (C) 70
(D) 80 (E) None of these

83. The bucket P has the capacity four times than the capacity of bucket Q.
Bucket P and bucket Q takes 24 turns to fill the cistern together. Find the
number of turns taken by P to fill the cistern.
बाल्टी P की क्षमता बाल्टी Q की क्षमता से चार गुना है । बाल्टी P और बाल्टी Q को एक साथ टं की
को भरने में 24 बार लगते हैं। P द्वारा टं की को भरने में ललए गए िेरों की संख्या ज्ञात कीजजए।
(A) 30 (B) 20 (C) 10
(D) 40 (E) None of these

84. An article bought at Rs. X and after allowing a discount of 10% on marked
price of Rs. 1440, it is sold at 8% profit. Find the value of X.
एक वस्तु को X रुपये में खरीिा गया और 1440 रुपये के अंककत मूल्य पर 10% की छूट िे ने के
बाि इसे 8% लाभ पर बेचा गया। X का मान ज्ञात कीजजए।
(A) 4700 (B) 6300 (C) 1200
(D) 5500 (E) None of these

85. A distributer mixes two types of rice A and B in two sacks(S1 and S2). The
ratio of the quantity of Type A rice to that of type B rice in S1 is 3 : 7 and the
ratio of the quantity of Type A rice to that of type B rice in S2 is 4 : 3. After
that he takes 20 kg of mixture from S1 and 21 kg from S2 and some additional
mixture of some weight of type A rice and mixes all of them together. The
mixture he obtained has ratio of rice of type A to the rice of type B is 21 : 23.
Find the weight of mixture obtained.

https :
//www. https : https
https : //instagra
//youtube.c
facebo m.com/aas :
om/channe hisharoraso
l/UCYa4_Jr ok.com cial(?) //t.m
Orf8R5Kz2u
/aashis utm_mediu
e/stu

85
OtccXQ m=
haroras copy_link
ocial dified
एक ववतरक िो प्रकार के चावल A और B को िो बोररयों (S1 और S2) में लमलाता है । S1 में टाइप
A चावल की मात्रा का B चावल से अनप ु ात 3 : 7 है और S2 में टाइप A चावल की मात्रा का B
चावल से अनुपात 4 : 3 है । उसके बाि वह S1 से 20 ककग्रा और S2 से 21 ककग्रा लमश्ण लेता है
और A प्रकार के चावल के कुछ वजन का कुछ अनतररक्त लमश्ण लेता है और उन सभी को एक
साथ लमला िे ता है । उसे प्राप्त लमश्ण में A प्रकार के चावल का B प्रकार के चावल से अनुपात 21 :
23 है । प्राप्त लमश्ण का भार ज्ञात कीजजए।
(A) 36 kg (B) 44 kg (C) 38 kg
(D) 34 kg (E) None of these

86. A and B start a business with an investment of Rs. 10,000 and Rs. 16,000
respectively. After 5 months from the start of the business B quits and C
joins the business with an investment of 20,000. If the total profit at the end
of one year business is Rs. 8500, then find the share of C.
A और B क्रमशः 10,000 रुपये और 16,000 रुपये के ननवेश के साथ एक व्यवसाय शरू ु करते हैं।
व्यवसाय शुरू होने के 5 महीने बाि B छोड िे ता है और C 20,000 के ननवेश के साथ व्यवसाय में
शालमल हो जाता है । यदि एक वर्ष के अंत में कुल लाभ 8500 रुपये है , तो C का दहस्सा ज्ञात
कीजजए।
(A) Rs. 1950 (B) Rs. 1930 (C) Rs. 3500
(D) Rs. 1900 (E) None of these

87. A boat is running in a river to a place A which is X km from B. The boat


with the speed 10 km/hr covered the distance in 20 hours in downstream and
return to the same place in 30 hours. Find the value of (2X - 80).
एक नाव निी में एक स्थान A के ललए िौड रही है जो B से X ककमी है । नाव 10 ककमी/घंटा की गनत
से धारा के अनुकूल 20 घंटे में िरू ी तय करती है और उसी स्थान पर 30 घंटे में वापस आती है । (2X
- 80) का मान ज्ञात कीजजए।
(A) 400 (B) 124 (C) 132
(D) 134 (E) None of these

88. The average weight of 8 players in a team is 67 kg. When two more
players (P and Q) are added to the team , the average weight of the team gets
reduced by 6 kg. If the weight of the P exceeds that of Q by 26 kg, then find
the weight of Q.
एक टीम में 8 खखलाडडयों का औसत भार 67 ककग्रा है । जब टीम में िो और खखलाडी (P और Q)
जोडे जाते हैं, तो टीम का औसत वजन 6 ककग्रा कम हो जाता है । यदि P का भार Q के भार से 26
ककग्रा अचधक है , तो Q का भार ज्ञात कीजजए।
(A) 14 kg (B) 24 kg (C) 18 kg
(D) 12 kg (E) None of these

https :
//www. https : https
https : //instagra
//youtube.c
facebo m.com/aas :
om/channe hisharoraso
l/UCYa4_Jr ok.com cial(?) //t.m
Orf8R5Kz2u
/aashis utm_mediu
e/stu

86
OtccXQ m=
haroras copy_link
ocial dified
89. Rohan invested Rs. 1460 in a scheme A at 10% simple interest for 5 years.
The amount received from scheme A is invested in scheme B at 10% simple
interest for T years. If the difference between the interests received from both
the schemes is Rs. 1022, then find the value of T.
रोहन ने 5 साल के ललए 10% साधारण ब्याज पर एक योजना A में 1460 रुपये का ननवेश ककया।
योजना A से प्राप्त रालश को योजना B में T वर्ष के ललए 10% साधारण ब्याज पर ननवेश ककया
जाता है । यदि िोनों योजनाओं से प्राप्त ब्याज के बीच का अंतर 1022 रुपये है , तो T का मान ज्ञात
कीजजए।
(A) 8 years (B) 9 years (C) 7 years
(D) 6 years (E) None of these

90. In a total time of 30 sec, a boat covers a certain distance in downstream


direction and comes back to the starting point. The downstream speed and
the upstream speed of boat are 18 m/s and 12 m/s respectively. Find the total
distance covered by the boat.
30 सेकंड के कुल समय में , एक नाव धारा के अनुकूल दिशा में एक ननजश्चत िरू ी तय करती है और
प्रारं लभक बबंि ु पर वापस आती है । नाव की धारा के अनुकूल गनत और धारा के प्रनतकूल गनत क्रमशः
18 मी/से और 12 मी/से है । नाव द्वारा तय की गई कुल िरू ी ज्ञात कीजजए।
(A) 680 m (B) 620 m (C) 432 m
(D) 640 m (E) None of these

91. Efficiency of Tap P is 4/3rd of efficiency of Tap Q. Tap Q can fill the tank in
20 hours and tap R can empty the tank in 30 hours. Find the time taken by tap
P, Tap Q and tap R together to fill the tank.
नल P की क्षमता, नल Q की क्षमता की 4/3 है । नल Q टं की को 20 घंटे में भर सकता है और नल
R टं की को 30 घंटे में खाली कर सकता है । टैंक को भरने के ललए टै प P, टै प Q और टै प R द्वारा
एक साथ ललया गया समय ज्ञात कीजजए।
(A) 15 hours (B) 18 hours (C) 12 hours
(D) 16 hours (E) None of these

92. A invests Rs. 2400 in a scheme which offers simple interest at 18% p.a. for
3 years and B invests Rs. X in the same scheme at 12% interest rate for 4
years. If the ratio of interest received by A to that by B is 6 : 5, then find the
value of X.
A एक योजना में 2400 रुपये का ननवेश करता है जो 3 साल के ललए 18% प्रनतवर्ष पर साधारण
ब्याज प्रिान करता है और B उसी योजना में 4 साल के ललए 12% ब्याज िर पर X रुपये का ननवेश
करता है । यदि A द्वारा प्राप्त ब्याज का B द्वारा प्राप्त ब्याज से अनुपात 6 : 5 है , तो X का मान
ज्ञात कीजजए।
(A) 2500 (B) 2250 (C) 3000
(D) 5000 (E) None of these

https :
//www. https : https
https : //instagra
//youtube.c
facebo m.com/aas :
om/channe hisharoraso
l/UCYa4_Jr ok.com cial(?) //t.m
Orf8R5Kz2u
/aashis utm_mediu
e/stu

87
OtccXQ m=
haroras copy_link
ocial dified
93. A mixture contains alcohol and water in the ratio of 2 : 3. If 25 litres of
mixture is replaced with same amount of alcohol, the ratio of alcohol and
water becomes 13 : 12. Find the quantity of alcohol in the original mixture.
एक लमश्ण में अल्कोहल और पानी का अनुपात 2 : 3 है । यदि 25 लीटर लमश्ण को अल्कोहल की
समान मात्रा से बिल दिया जाता है , तो अल्कोहल और पानी का अनपु ात 13 : 12 हो जाता है । मल

लमश्ण में अल्कोहल की मात्रा ज्ञात कीजजए।
(A) 50 L (B) 20 L (C) 30 L
(D) 40 L (E) None of these

94. The total cost price of a mobile charger and an earphone is Rs. 375.
Earphone is sold at profit of 37.5% whereas charger is at loss of 4%. Find the
selling price of the charger, If the selling price of earphone is Rs. 275.
एक मोबाइल चाजषर और एक ईयरिोन का कुल लागत मूल्य 375 रुपये है । ईयरिोन को 37.5%
के लाभ पर बेचा जाता है जबकक चाजषर को 4% की हानन पर बेचा जाता है । यदि ईयरिोन का
ववक्रय मल्
ू य 275 रुपये है , तो चाजषर का ववक्रय मल्
ू य ज्ञात कीजजए।
(A) Rs. 147 (B) Rs. 163 (C) Rs. 168
(D) Rs. 155 (E) None of these

95. A certain sum of money is invested in a bank for 2.5 years at 16(2/3)%
simple interest. If interest amount received is Rs. 6250, then what will be the
total amount, If the sum invested for only 2 years?
एक ननजश्चत रालश को बैंक में 2.5 वर्ष के ललए 16(2/3)% साधारण ब्याज पर ननवेश ककया जाता
है । यदि प्राप्त ब्याज रालश 6250 रुपये है , तो कुल रालश ककतनी होगी, यदि रालश को केवल 2 वर्ष के
ललए ननवेश ककया जाता है ?
(A) Rs. 10000 (B) Rs. 20000 (C) Rs.15000
(D) Rs. 25000 (E) None of these

96. The sum of the two numbers is 18. When the smaller number is decreased
by 10% and the larger number increased by 10%, then their difference
becomes 12. What is the product of these two numbers (approx. value)
िो संख्याओं का योग 18 है । जब छोटी संख्या में 10% की कमी और बडी संख्या में 10% की वद् र चध
होती है , तो उनका अंतर 12 हो जाता है । इन िोनों संख्याओं का गण
ु निल (अन ु म ाननत मान) क्या
है ?
(A) 60 (B) 35 (C) 55
(D) 45 (E) None of these

97. The average of bats distributed to players in a week is 53. The week
started from Sunday and average of the bats is distributed on the first three
days is 50 and the number of bats distributed on last 2 days are 85. Find the
average of the bats distributed on Wednesday and Thursday.

https :
//www. https : https
https : //instagra
//youtube.c
facebo m.com/aas :
om/channe hisharoraso
l/UCYa4_Jr ok.com cial(?) //t.m
Orf8R5Kz2u
/aashis utm_mediu
e/stu

88
OtccXQ m=
haroras copy_link
ocial dified
एक सप्ताह में खखलाडडयों को बांटे गए बल्लों का औसत 53 है । सप्ताह रवववार से शुरू हुआ और
पहले तीन दिनों में बांटे गए बल्लों का औसत 50 है और अंनतम 2 दिनों में बांटे गए बल्लों की
संख्या 85 है । बुधवार और गुरुवार को बांटे गए बल्लों का औसत ज्ञात कीजजए।
(A) 68 (B) 52 (C) 45
(D) 30 (E) None of these

98. In commonwealth games the average number of medals distributed to


India and China together was 225. If Australia replaces India then the average
becomes 205 and If Australia replaces China then average becomes 180. Find
the number of medals won by India.
राष्ट्रमंडल खेलों में भारत और चीन को लमलाकर बांटे गए पिकों की औसत संख्या 225 थी। यदि
ऑस्ट्रे ललया भारत की जगह लेता है तो औसत 205 हो जाता है और यदि ऑस्ट्रे ललया चीन की जगह
ले लेता है तो औसत 180 हो जाता है । भारत द्वारा जीते गए पिकों की संख्या ज्ञात कीजजए।
(A) 100 (B) 200 (C) 400
(D) 500 (E) None of these

99. The circular path is running around a circular field from outside. The
perimeter of the circular field is 176 m and the perimeter of the outermost
edge of the road is 264 m. Find the area of the road.
वत्त
र ाकार पथ बाहर से एक वत्त
र ाकार मैिान के चारों ओर चल रहा है । वत्त
र ाकार क्षेत्र का पररमाप 176
मीटर है और सडक के सबसे बाहरी ककनारे का पररमाप 264 मीटर है । सडक का क्षेत्रिल ज्ञात
कीजजए।
(A) 3080 cm2 (B) 2500 cm2 (C) 1474 cm2
(D) 1278 cm2 (E) None of these

100. Vishal and Akhil started a work together and after ‘n’ days Vishal left the
work and remaining work is completed by Akhil in 15 days. Time taken by
Akhil and Vishal alone are in the ratio 3 : 2 respectively. If Akhil can complete
the whole work in 30 days, then calculate the value of n/2.
ववशाल और अखखल ने एक साथ एक कायष शुरू ककया और 'n' दिनों के बाि ववशाल ने कायष छोड
दिया और शेर् कायष अखखल द्वारा 15 दिनों में पूरा ककया गया। अखखल और ववशाल द्वारा अकेले
ललया गया समय क्रमशः 3 : 2 के अनप ु ात में है । यदि अखखल परू े कायष को 30 दिनों में परू ा कर
सकता है , तो n/2 के मान की गणना कीजजए।
(A) 3 (B) 4 (C) 6
(D) 8 (E) None of these

101. In a family of 8 adults and X minors, the average consumption of rice per
head per month is 10.8 kg, while the average consumption for adult is 15
kg/head and for minors it is 6 kg/head. Find the value of X.

https :
//www. https : https
https : //instagra
//youtube.c
facebo m.com/aas :
om/channe hisharoraso
l/UCYa4_Jr ok.com cial(?) //t.m
Orf8R5Kz2u
/aashis utm_mediu
e/stu

89
OtccXQ m=
haroras copy_link
ocial dified
8 वयस्कों और X अवयस्कों के एक पररवार में , प्रनत माह प्रनत व्यजक्त चावल की औसत खपत
10.8 ककलोग्राम है , जबकक वयस्कों के ललए औसत खपत 15 ककलोग्राम/लसर और नाबाललगों के
ललए यह 6 ककलोग्राम/लसर है । X का मान ज्ञात कीजजए।
(A) 20 (B) 12 (C) 7
(D) 2 (E) None of these

102. A sum of money becomes Rs. 8322 in 16 years at 8% simple interest and
another sum of money becomes Rs. 10556 in 12 years at the rate of 11% at
simple interest. What is the difference between both the sum?
एक धनरालश 16 वर्ों में 8% साधारण ब्याज पर 8322 रुपये हो जाती है और िस ू री धनरालश 12
वर्ों में साधारण ब्याज पर 11% की िर से 10556 रुपये हो जाती है । िोनों के योग में ककतना अंतर
है ?
(A) Rs. 990 (B) Rs. 900 (C) Rs. 945
(D) Rs. 855 (E) None of these

103. Aman’s present age is 20% more than Bheem’s present age. 24 years
ago, Aman was 6 times as old as Bheem. What is the present age of Bheem?
अमन की वतषमान आयु भीम की वतषमान आयु से 20% अचधक है । 24 वर्ष पहले, अमन भीम से 6
गुना बडा था। भीम की वतषमान आयु क्या है ?
(A) 25 years (B) 20 years (C) 10 years
(D) 40 years (E) None of these

104. Mahesh earned 25% as share from profit and Naresh earned 45% as
share from profit and rest is earned by Om. Mahesh invest for 6 months,
Naresh invest for 4 months and Om invest for 5 months and Mahesh’s
investment was Rs. 12000. What was Om’s investment?
महे श ने लाभ से शेयर के रूप में 25% अजजषत ककया और नरे श ने लाभ से शेयर के रूप में 45%
अजजषत ककया और शेर् ओम द्वारा अजजषत ककया गया। महे श 6 महीने के ललए ननवेश करता है ,
नरे श 4 महीने के ललए ननवेश करता है और ओम 5 महीने के ललए ननवेश करता है और महे श का
ननवेश 12000 रुपये था। ओम का ननवेश ककतना था?
(A) Rs. 47500 (B) Rs. 6300 (C) Rs. 17280
(D) Rs. 15500 (E) None of these

105. Rahul obtained 88% in examination. Rachit obtained 5 more marks than
Rahul and obtained 90% marks in that exam. If Ranvir obtained 190 marks in
the exam, then what is his percentage of marks?
राहुल ने परीक्षा में 88% अंक प्राप्त ककए। रचचत ने राहुल से 5 अंक अचधक प्राप्त ककए और उस
परीक्षा में 90% अंक प्राप्त ककए। यदि रणवीर ने परीक्षा में 190 अंक प्राप्त ककए, तो उसके अंकों का
प्रनतशत क्या है ?
(A) 80% (B) 76% (C) 67%
(D) 82% (E) None of these

https :
//www. https : https
https : //instagra
//youtube.c
facebo m.com/aas :
om/channe hisharoraso
l/UCYa4_Jr ok.com cial(?) //t.m
Orf8R5Kz2u
/aashis utm_mediu
e/stu

90
OtccXQ m=
haroras copy_link
ocial dified
106. A cloth merchant bought a bundle of cloth of length 100 m and width 1 m
and another bundle of same cloth of length 150 m and width 3/4 m, at the rate
of Rs. 78 per square meter. What would be the difference between cost of
both bundles?
एक कपडा व्यापारी ने 100 मीटर लंबाई और 1 मीटर चौडाई वाले कपडे का एक बंडल और 150
मीटर लंबाई और 3/4 मीटर चौडाई वाले कपडे का एक और बंडल 78 रुपये प्रनत वगष मीटर की िर से
खरीिा। िोनों बंडलों की लागत के बीच ककतना अंतर होगा?
(A) Rs. 950 (B) Rs. 930 (C) Rs. 975
(D) Rs. 900 (E) None of these

107. A cost price of a mixture of two oils A and B is Rs. 60/liter. The cost price
of 12 liters of oil A is Rs. 40 per liter. If the cost price of oil B is Rs. 64 per
liter, then how many liters of oil B should be mixed?
िो तेलों A और B के लमश्ण का लागत मूल्य 60 रुपये/लीटर है । 12 लीटर तेल A का क्रय मूल्य 40
रुपये प्रनत लीटर है । यदि तेल B का लागत मल् ू य 64 रुपये प्रनत लीटर है , तो ककतने लीटर तेल B
लमलाया जाना चादहए?
(A) 60 liters (B) 20 liters (C) 32 liters
(D) 40 liters (E) None of these

108. The upstream speed of boat and stream speed are in the ratio 2 : 1
respectively and the difference between the upstream speed and the speed of
the boat in still water is 9 km/hr. If the speed of boat is increased by 4 km/hr,
then what is the approximate time taken by the boat to cover 324 km
downstream?
नाव की धारा के प्रनतकूल गनत और धारा की गनत क्रमशः 2 : 1 के अनप ु ात में है और धारा के
प्रनतकूल गनत और जस्थर पानी में नाव की गनत के बीच का अंतर 9 ककमी/घंटा है । यदि नाव की
गनत 4 ककमी/घंटा बढा िी जाती है , तो नाव द्वारा धारा के अनुकूल 324 ककमी की िरू ी तय करने में
ककतना समय लगता है ?
(A) 1.4 hours (B) 8.1 hours (C) 5.8 hours
(D) 12 hours (E) None of these

109. Pipes A and B can fill a tank in 4 hours and 6 hours respectively,
however there is a leak in the tank which can empty the half filled tank in 6
hours. If pipes A and B are open together, then in how much time will they
take to fill the tank?
पाइप A और B एक टैंक को क्रमशः 4 घंटे और 6 घंटे में भर सकते हैं, हालांकक टैंक में एक छे ि है
जो आधे भरे टैंक को 6 घंटे में खाली कर सकता है । यदि पाइप A और B एक साथ खल ु े हैं, तो वे
टैंक को भरने में ककतना समय लेंगे?
(A) 3 hours (B) 9 hours (C) 4 hours
(D) 6 hours (E) None of these

https :
//www. https : https
https : //instagra
//youtube.c
facebo m.com/aas :
om/channe hisharoraso
l/UCYa4_Jr ok.com cial(?) //t.m
Orf8R5Kz2u
/aashis utm_mediu
e/stu

91
OtccXQ m=
haroras copy_link
ocial dified
110. Atul sold 3/5 of a share at a profit of 35% and the rest at a loss of 25%. If
he got and overall profit of Rs. 858, then what is actual cost of share?
अतुल ने 3/5 दहस्से को 35% के लाभ पर और शेर् को 25% की हानन पर बेच दिया। यदि उसे 858
रुपये का कुल लाभ प्राप्त हुआ, तो शेयर की वास्तववक लागत क्या है ?
(A) Rs. 6800 (B) Rs. 6200 (C) Rs. 7800
(D) Rs. 6400 (E) None of these

111. When an article is sold at 3/5th of the marked price, the loss percentage
was 10%. The article should be sold at what fraction of marked price to gain
20%?
जब एक वस्तु अंककत मूल्य के 3/5 पर बेची जाती है , तो हानन प्रनतशत 10% था। 20% लाभ प्राप्त
करने के ललए वस्तु को अंककत मल्
ू य के ककतने भाग पर बेचा जाना चादहए?
(A) 1/5 (B) 3/5 (C) 4/5
(D) 2/5 (E) None of these

112. A sum amounts to Rs.. 5760 in 2 years and Rs.. 6912 in 3 years at
compound interest. Find the principal.
चक्रवद्
र चध ब्याज पर एक रालश 2 साल में 5760 रुपये और 3 साल में 6912 रुपये हो जाती है ।
मूलधन ज्ञात कीजजए।
(A) Rs. 7000 (B) Rs. 4000 (C) Rs.3600
(D) Rs. 4500 (E) None of these

113. Abhishek’s age is 60% more than what it was 12 years ago. After how
many years will 45% of his age will be equal to his age 14 years ago?
अलभर्ेक की आयु 12 वर्ष पहले की आयु से 60% अचधक है । ककतने वर्ों के बाि उसकी आयु का
45% उसकी 14 वर्ष पूवष की आयु के बराबर होगा?
(A) 8 years (B) 5 years (C) 12 years
(D) 4 years (E) None of these

114. A can do a job in 12 days and B is 20% less efficient than A. They start
working in alternate days starting from A. In how many days the work will be
finished?
A ककसी काम को 12 दिनों में कर सकता है और B, A से 20% कम कुशल है । वे A से शुरू करते
हुए वैकजल्पक दिनों में काम करना शुरू करते हैं। काम ककतने दिनों में समाप्त हो जाएगा?
(A) 21(7/4) days (B) 5 days (C) 13(1/4) days
(D) 3(1/4) days (E) None of these

115. A liter of water weighs 1 kg and a liter of another liquid weighs 1250
grams. A mixture of the two weigh 1050 gram per liter. What is the fraction of
volume of water and liquid respectively in a liter of mixture?

https :
//www. https : https
https : //instagra
//youtube.c
facebo m.com/aas :
om/channe hisharoraso
l/UCYa4_Jr ok.com cial(?) //t.m
Orf8R5Kz2u
/aashis utm_mediu
e/stu

92
OtccXQ m=
haroras copy_link
ocial dified
एक लीटर पानी का वजन 1 ककलो और एक लीटर िस ू रे तरल का वजन 1250 ग्राम होता है । िोनों
के लमश्ण का वजन 1050 ग्राम प्रनत लीटर है । एक लीटर लमश्ण में क्रमशः पानी और तरल के
आयतन का अंश ककतना है ?
(A) 3/5 ,3/2 (B) 4/5 , 1/5 (C) 2/5 ,4/5
(D) 3/7 ,6/7 (E) None of these

116. Varun started a business with an investment 3/5 times the investment of
Atul. Atul joined Varun after 9 months. Varun left after 3 months of Atul
joining. If at the end of 1.5 years, profit received by Atul is Rs. 1000, then
what will be the difference between profit received by both of them?
वरुण ने अतुल के ननवेश के 3/5 गुना ननवेश के साथ एक व्यवसाय शुरू ककया। अतुल 9 महीने बाि
वरुण से लमला। अतुल के जुडने के 3 महीने बाि वरुण ने छोड दिया। यदि 1.5 वर्ष के अंत में , अतुल
द्वारा प्राप्त लाभ 1000 रुपये है , तो उन िोनों द्वारा प्राप्त लाभ के बीच ककतना अंतर होगा?
(A) Rs. 400 (B) Rs. 100 (C) Rs. 200
(D) Rs. 300 (E) None of these

117. 25% of Amit’s monthly salary is spent on food, 20% in spend on rent and
30% of the remaining is spent on entertainment, then he is left with Rs.
16940. If the monthly salary of Amar is 20% more than that of Amit, then what
is the monthly salary of Amar?
अलमत के मालसक वेतन का 25% भोजन पर खचष होता है , 20% ककराए पर खचष होता है और शेर्
का 30% मनोरं जन पर खचष होता है , तो उसके पास 16940 रुपये बचते हैं। यदि अमर का मालसक
वेतन अलमत के मालसक वेतन से 20% अचधक है , तो अमर का मालसक वेतन ककतना है ?
(A) Rs. 40000 (B) Rs. 11000 (C) Rs. 51000
(D) Rs. 52800 (E) None of these

118. Father distributed Rs. 1200 among his four sons A, B, C and D such that
A got Rs. 50 more than B and ratio of the amount received by C and D is 6 : 7
respectively. Amount received by D is Rs. 100 more than the amount received
by B. What is the average share of A and D?
वपता ने अपने चार पुत्रों A, B, C और D में 1200 रुपये इस प्रकार बांटे कक A को B से 50 रुपये
अचधक लमले और C और D को प्राप्त रालश का अनुपात क्रमशः 6 : 7 है । D द्वारा प्राप्त रालश B
द्वारा प्राप्त रालश से 100 रुपये अचधक है । A और D का औसत दहस्सा क्या है ?
(A) Rs. 425 (B) Rs. 325 (C) Rs. 500
(D) Rs. 725 (E) None of these

119. Two pipes A and B can fill a tank in 24 min and 32 in respectively. If both
the pipes are opened simultaneously, then after how much time should B be
closed so that tank is full in 18 min?

https :
//www. https : https
https : //instagra
//youtube.c
facebo m.com/aas :
om/channe hisharoraso
l/UCYa4_Jr ok.com cial(?) //t.m
Orf8R5Kz2u
/aashis utm_mediu
e/stu

93
OtccXQ m=
haroras copy_link
ocial dified
िो पाइप A और B एक टैंक को क्रमशः 24 लमनट और 32 लमनट में भर सकते हैं। यदि िोनों पाइपों
को एक साथ खोला जाता है , तो ककतने समय बाि B को बंि कर िे ना चादहए ताकक टैंक 18 लमनट
में भर जाए?
(A) 8 min (B) 5 min (C) 6 min
(D) 2 min (E) None of these

120. A boat can cover 132 km upstream in 26.4 hours and the speed of
current is 3 km/hr, then what is the difference between the time taken by boat
to cover 46.2 km in downstream and time taken by boat to cover 86 km in
upstream?
एक नाव 26.4 घंटे में धारा के प्रनतकूल 132 ककमी की िरू ी तय कर सकती है और धारा की गनत 3
ककमी/घंटा है , तो नाव द्वारा धारा के अनक
ु ू ल 46.2 ककमी की िरू ी तय करने में ललए गए समय और
धारा के प्रनतकूल 86 ककमी की िरू ी तय करने में नाव द्वारा ललए गए समय के बीच का अंतर क्या
है ?
(A) 14 hours (B) 7 hours (C) 13 hours
(D) 6 hours (E) None of these

121. Mitesh and Nitesh invested Rs. 2400 and Rs. 3600 respectively for
different number of months. The sum of their time periods of investments is 7
months. If out of a profit of Rs. 17000, Mitesh's share is Rs. 8000, then for
how many months has Mitesh invested his amount?
लमतेश और ननतेश ने अलग - अलग महीनों के ललए क्रमशः 2400 रुपये और 3600 रुपये का
ननवेश ककया। उनके ननवेश की समय अवचध का योग 7 महीने है । यदि 17000 रुपये के लाभ में से
लमतेश का दहस्सा 8000 रुपये है , तो लमतेश ने ककतने महीनों के ललए अपनी रालश का ननवेश ककया
है ?
(A) 5 months (B) 6 months (C) 4 months
(D) 3 months (E) None of these

122. Sum of present ages of Prakash and Pankaj is 76 years and sum of
present ages of Prakash and Suresh is 64 years. If the average age of all
three persons is 32 years, then what is the ratio of present age of Pankaj,
Prakash and Suresh respectively?
प्रकाश और पंकज की वतषमान आयु का योग 76 वर्ष है और प्रकाश और सुरेश की वतषमान आयु का
योग 64 वर्ष है । यदि तीनों व्यजक्तयों की औसत आयु 32 वर्ष है , तो क्रमशः पंकज, प्रकाश और
सरु े श की वतषमान आयु का अनप ु ात क्या है ?
(A) 7 : 11 : 5 (B) 8 : 11 : 5 (C) 2 : 3 : 5
(D) 1 : 1 : 1 (E) None of these

123. The length of a train is (X - 10) m. If it is running at the speed of 64 km/hr


and crosses a (X + 30) m long bridge in 18 seconds, then find the value of X.

https :
//www. https : https
https : //instagra
//youtube.c
facebo m.com/aas :
om/channe hisharoraso
l/UCYa4_Jr ok.com cial(?) //t.m
Orf8R5Kz2u
/aashis utm_mediu
e/stu

94
OtccXQ m=
haroras copy_link
ocial dified
एक रे लगाडी की लम्बाई (X - 10) मीटर है । यदि वह 64 ककमी/घंटा की गनत से चल रही है और एक
(X + 30) मीटर लंबे पुल को 18 सेकंड में पार करती है , तो X का मान ज्ञात कीजजए।
(A) 150 (B) 200 (C) 250
(D) 300 (E) None of these

124. Prakash invested money in a scheme which claims to provide 18% per
annual simple interest but it actually provides 10% per annum compound
interest. If the difference between Prakash's actual and expected interest
after 2 years is Rs. 2100, then what was the money invested by Prakhar?
प्रकाश ने एक योजना में पैसा ननवेश ककया जो 18% वावर्षक साधारण ब्याज प्रिान करने का िावा
करती है लेककन यह वास्तव में 10% प्रनत वर्ष चक्रवद्
र चध ब्याज प्रिान करती है । यदि 2 वर्ष बाि
प्रकाश के वास्तववक और अपेक्षक्षत ब्याज के बीच का अंतर 2100 रुपये है , तो प्रखर द्वारा ननवेश
की गई रालश ककतनी थी?
(A) Rs. 20000 (B) Rs. 50000 (C) Rs. 14000
(D) Rs. 30000 (E) None of these

125. In a family three sons were born after every 3 years on after the other. If
the ratio of present ages of father and the eldest son 15 : 7 and the average of
present ages of three sons is 18 years, then what will be the ratio of the ages
of the father and the youngest son after 5 years?
एक पररवार में हर 3 साल बाि तीन बेटे पैिा होते हैं। यदि वपता और सबसे बडे पुत्र की वतषमान आयु
का अनुपात 15 : 7 है और तीन पुत्रों की वतषमान आयु का औसत 18 वर्ष है , तो 5 वर्ष बाि वपता
और सबसे छोटे पत्र
ु की आयु का अनप ु ात क्या होगा?
(A) 3 : 2 (B) 5 : 2 (C) 2 : 5
(D) 7 : 5 (E) None of these

126. Three pipes A, B and C can fill a tank completely in 20 min, 30 min and
40 min respectively. The tank is empty and all the three pipes are opened. All
the three pipes are used to fill three different liquids in a such a way that pipe
A fills milk, pipe B fills tea and pipe C fills water. What will be the amount of
milk after 4 minutes from the start?
तीन पाइप A, B और C एक टैंक को क्रमशः 20 लमनट, 30 लमनट और 40 लमनट में पूरी तरह से
भर सकते हैं। टं की खाली है और तीनों पाइपों को खोल दिया गया है । तीनों पाइपों का उपयोग तीन
अलग - अलग तरल पिाथों को इस प्रकार भरने के ललए ककया जाता है कक पाइप A िध ू , पाइप B
चाय और पाइप C पानी भरता है । शरू ु करने के 4 लमनट बाि िधू की मात्रा क्या होगी?
(A) 14 units (B) 12 units (C) 24 units
(D) 10 units (E) None of these

https :
//www. https : https
https : //instagra
//youtube.c
facebo m.com/aas :
om/channe hisharoraso
l/UCYa4_Jr ok.com cial(?) //t.m
Orf8R5Kz2u
/aashis utm_mediu
e/stu

95
OtccXQ m=
haroras copy_link
ocial dified
127. From a well shuffled deck of 52 cards, four cards have been chosen at
random without replacement. What is the probability that all four chosen
cards are Red?
52 पत्तों की एक अच्छी तरह से िेटी गई गड्डी में से चार पत्ते बबना प्रनतस्थापन के यादृजच्छक रूप
से चन
ु े गए हैं। क्या प्रानयकता है कक चनु े गए चारों पत्ते लाल हैं?
(A) 46/833 (B) 11/833 (C) 10/833
(D) 12/833 (E) None of these

128. A rectangular board is decorated by posting 'n' square shaped coloured


papers of 144 cm2 surface area each on it. Perimeter of rectangular board is
600 cm and length and breadth of the board are in the ratio 3 : 2. What is the
value of n?
एक आयताकार बोडष को उस पर 144 सेमी2 सतह क्षेत्र के 'n' चौकोर आकार के रं गीन कागज
चचपका कर सजाया गया है । आयताकार बोडष का पररमाप 600 सेमी है और बोडष की लंबाई और
चौडाई का अनपु ात 3 : 2 है । n का मान क्या है ?
(A) 100 (B) 150 (C) 120
(D) 140 (E) None of these

129. The ratio of two numbers is 2 : 5. When X is added to both the numbers,
the ratio becomes 5 : 11 and when X is subtracted from both the numbers the
ratio becomes 1 : 3. What is the value of X?
िो संख्याओं का अनुपात 2 : 5 है । जब िोनों संख्याओं में X जोडा जाता है तो अनुपात 5 : 11 हो
जाता है और जब िोनों संख्याओं में X घटाया जाता है तो अनुपात 1 : 3 हो जाता है । X का मूल्य
क्या है ?
(A) 50 (B) 60 (C) 30
(D) 70 (E) None of these

130. Marked price and cost price of watch are in the ratio 6 : 5 respectively
and marked price and selling price of watch are in the ratio 5 : 4 respectively.
If shopkeeper had a loss of Rs. 12 on selling the watch, then what is the
discount percent given on the marked price?
घडी का अंककत मूल्य और क्रय मूल्य क्रमशः 6 : 5 के अनुपात में हैं और घडी के अंककत मूल्य और
बबक्री मूल्य क्रमशः 5 : 4 के अनुपात में हैं। यदि िक
ु ानिार को घडी बेचने पर 12 रुपये की हानन
होती है , तो अंककत मूल्य पर िी गई छूट प्रनतशत क्या है ?
(A) 40% (B) 30% (C) 20%
(D) 50% (E) None of these

https :
//www. https : https
https : //instagra
//youtube.c
facebo m.com/aas :
om/channe hisharoraso
l/UCYa4_Jr ok.com cial(?) //t.m
Orf8R5Kz2u
/aashis utm_mediu
e/stu

96
OtccXQ m=
haroras copy_link
ocial dified
131. A invests some money in a scheme. After four months, B invests Rs.
7000 in the same scheme. After 18 months from the time of investment of A,
the ratio of profit of B to the profit of A is 14 : 27. Calculate the investment of
A.
A एक योजना में कुछ धन ननवेश करता है । चार महीने बाि, B उसी योजना में 7000 रुपये का
ननवेश करता है । A के ननवेश के 18 महीने बाि, B के लाभ का A के लाभ से अनुपात 14 : 27 है । A
के ननवेश की गणना कीजजए।
(A) Rs. 15000 (B) Rs. 25000 (C) Rs. 10500
(D) Rs. 11500 (E) None of these

132. Circumference of a circle of radius 105 cm is equal to the perimeter of a


rectangle. If the breadth of the rectangle is 10 cm less than its length, then
find the area of the rectangle.
105 सेमी बत्रज्या वाले एक वत्त
र की पररचध एक आयत के पररमाप के बराबर है । यदि आयत की
चौडाई उसकी लम्बाई से 10 सेमी कम है , तो आयत का क्षेत्रिल ज्ञात कीजजए।
(A) 24200 cm2 (B) 27200 cm2 (C) 3200 cm2
(D) 42400 cm2 (E) None of these

133. Rohit lent a sum of money to his friend Mohit at simple interest 5% per
annum. If Mohit has to return his money with interest of Rs. 2700 after 4
years, then find the sum of money given to Mohit.
रोदहत ने अपने िोस्त मोदहत को 5% प्रनत वर्ष के साधारण ब्याज पर एक रालश उधार िी। यदि
मोदहत को 4 वर्ष बाि 2700 रुपये के ब्याज के साथ अपना पैसा वापस करना है , तो मोदहत को िी
गई रालश ज्ञात कीजजए।
(A) Rs. 13500 (B) Rs. 20000 (C) Rs. 10500
(D) Rs. 40000 (E) None of these

134. The speed of a man is 8 km/hr and he covers a certain distance in 6


hours 45 min. If he covers the same distance on bike at the speed of 45
km/hr, then find the time taken by him.
एक आिमी की गनत 8 ककमी/घंटा है और वह 6 घंटे 45 लमनट में एक ननजश्चत िरू ी तय करता है ।
यदि वह समान िरू ी बाइक पर 45 ककमी/घंटा की गनत से तय करता है , तो उसके द्वारा ललया गया
समय ज्ञात कीजजए।
(A) 47 min (B) 60 min (C) 72 min
(D) 45 min (E) None of these

135. A group of 400 men are involved in a task while the food is sufficient
only for 31 days. After 28 days, 280 men leave the task. For how many days
will the rest of the food last for the rest of the men?
400 परु
ु र्ों का एक समह
ू एक कायष में शालमल है जबकक भोजन केवल 31 दिनों के ललए पयाषप्त है ।

https :
//www. https : https
https : //instagra
//youtube.c
facebo m.com/aas :
om/channe hisharoraso
l/UCYa4_Jr ok.com cial(?) //t.m
Orf8R5Kz2u
/aashis utm_mediu
e/stu

97
OtccXQ m=
haroras copy_link
ocial dified
28 दिनों के बाि, 280 पुरुर् कायष छोड िे ते हैं। बाकी आिलमयों के ललए बचा हुआ खाना ककतने दिनों
तक चलेगा?
(A) 5 days (B) 10 days (C) 15 days
(D) 20 days (E) None of these

136. The average age of father, mother and son is 32 years. At present the
mother is 3 times as old as the son. 6 years ago, the ratio of the ages of
father to son was 7 : 1. Find the age of son 8 years from now.
वपता, माता और पुत्र की औसत आयु 32 वर्ष है । वतषमान में माता की आयु पत्र ु की आयु की 3 गुनी
है । 6 वर्ष पहले, वपता और पत्र
ु की आय ु का अन प
ु ात 7 : 1 था। अब से 8 वर्ष बाि पत्र
ु की आयु ज्ञात
कीजजए।
(A) 40 years (B) 12 years (C) 20 years
(D) 30 years (E) None of these

137. Wheat costing Rs. 48/kg is mixed with wheat costing Rs. 56/kg. If the
mixture is sold at Rs. 66/kg, it makes the seller a profit 25%. In what ratio
were the two varieties of wheat is mixed?
48 रुपये/ककग्रा की कीमत वाले गेहूं को 56 रुपये/ककग्रा की कीमत वाले गेहूं के साथ लमलाया जाता
है । यदि लमश्ण को 66 रुपये/ककग्रा पर बेचा जाता है , तो इससे ववक्रेता को 25% का लाभ होता है ।
गेहूँ की िो ककस्मों को ककस अनप
ु ात में लमलाया गया?
(A) 2 : 3 (B) 3 : 2 (C) 6 : 1
(D) 4 : 3 (E) None of these

138. 20 men can complete a work in 15 days and 15 women can complete the
same work in 30 days. If 10 women work for first 15 days, then the remaining
work will be completed by 10 men in how many days?
20 पुरुर् एक काम को 15 दिनों में पूरा कर सकते हैं और 15 मदहलाएं उसी काम को 30 दिनों में
परू ा कर सकती हैं। यदि 10 मदहलाएँ पहले 15 दिन कायष करती हैं, तो शेर् कायष 10 परु
ु र् ककतने
दिनों में पूरा करें गे?
(A) 10 days (B) 20 days (C) 30 days
(D) 15 days (E) None of these

139. A certain distance is covered by three cars A, B and C in total 12 hours.


Car A whose speed is 40 km/hr covers 1/4th distance, car B covers 40% of the
remaining distance and Car C covers remaining distance. If speed of car A, B
and C are in the ratio 4 : 8 : 3 respectively, then find the total distance
covered.
तीन कारों A, B और C द्वारा कुल 12 घंटे में एक ननजश्चत िरू ी तय की जाती है । कार A जजसकी
गनत 40 ककमी/घंटा है , 1/4 िरू ी तय करती है , कार B शेर् िरू ी का 40% तय करती है और कार C

https :
//www. https : https
https : //instagra
//youtube.c
facebo m.com/aas :
om/channe hisharoraso
l/UCYa4_Jr ok.com cial(?) //t.m
Orf8R5Kz2u
/aashis utm_mediu
e/stu

98
OtccXQ m=
haroras copy_link
ocial dified
शेर् िरू ी तय करती है । यदि कार A, B और C की गनत क्रमशः 4 : 8 : 3 के अनुपात में है , तो तय
की गई कुल िरू ी ज्ञात कीजजए।
(A) 480 km (B) 400 km (C) 500 km
(D) 650 km (E) None of these

140. A student got 93 marks in Math and yet failed by 13 marks. If passing
marks in Math are 42.4%, then find the maximum marks.
एक छात्र ने गखणत में 93 अंक प्राप्त ककए और किर भी 13 अंकों से अनत्त
ु ीणष हो गया। यदि गखणत
में उत्तीणष अंक 42.4% हैं, तो अचधकतम अंक ज्ञात कीजजए।
(A) 200 marks (B) 300 marks (C) 250 marks
(D) 400 marks (E) None of these

141. Ratio of A’s salary to A’s savings is 8 : 3 and his expenditure is Rs.
20000. If A’s expenditure is 62.5% of A’s salary, then find A’s savings.
A के वेतन का A की बचत से अनप ु ात 8 : 3 है और उसका व्यय 20000 रुपये है । यदि A का व्यय
A के वेतन का 62.5% है , तो A की बचत ज्ञात कीजजए।
(A) Rs. 20000 (B) Rs. 22000 (C) Rs. 12000
(D) Rs.23000 (E) None of these

142. Total surface area of a cuboid of length 20 m and height 6 m is 656 m2.
What is the difference between length and breadth of the cuboid?
20 मीटर लंबाई और 6 मीटर ऊंचाई वाले घनाभ का कुल सतह क्षेत्रिल 656 मी2 है । घनाभ की
लंबाई और चौडाई में ककतना अंतर है ?
(A) 30 m (B) 12 m (C) 10 m
(D) 40 m (E) None of these

143. Himanshu started a business with Rs. 7200 and after ‘m’ months from
the start of business, Pratik joined with Rs. 8400. At the end of the year
profits were divided in the ratio of 12 : 7 respectively. Find the value of 'm'.
दहमांशु ने 7200 रुपये के साथ एक व्यवसाय शरू ु ककया और व्यापार की शरु ु आत से 'm' महीनों के
बाि, प्रतीक 8400 रुपये के साथ जुड गया। वर्ष के अंत में लाभ क्रमशः 12 : 7 के अनुपात में
ववभाजजत ककया गया। 'm' का मान ज्ञात कीजजए।
(A) 6 (B) 5 (C) 10
(D) 12 (E) None of these

144. In how many days can person A alone complete the work, If A alone take
half number of days as compared to person B alone and 1/3rd number of
days as compared to person C alone to complete the work?It is given that it
will take 6 days to finish the work, If all three work together.

https :
//www. https : https
https : //instagra
//youtube.c
facebo m.com/aas :
om/channe hisharoraso
l/UCYa4_Jr ok.com cial(?) //t.m
Orf8R5Kz2u
/aashis utm_mediu
e/stu

99
OtccXQ m=
haroras copy_link
ocial dified
व्यजक्त A अकेले कायष को ककतने दिनों में परू ा कर सकता है , यदि A अकेले व्यजक्त B की तल ु ना
में आधे दिन और अकेले व्यजक्त C की तुलना में 1/3 दिनों की संख्या में कायष पूरा करता है ?यह
दिया गया है कक यदि तीनों एक साथ कायष करते हैं तो कायष को परू ा करने में 6 दिन लगें गे।
(A) 15 days (B) 12 days (C) 11 days
(D) 10 days (E) None of these

145. Manju is 5 years older to Sanju and ratio of ages of Sanju to Anju after 8
years will be 7 : 6. 6 years ago, Age of Manju was 90% more than age of Anju
at that time. Find the value of P, If age of Manju after 5 years will be P% more
than Sanju.
मंज,ू संजू से 5 वर्ष बडी है और 8 वर्ष बाि संजू की आयु का अंजू से अनप
ु ात 7 : 6 होगा। 6 वर्ष
पूव,ष मंजू की आयु उस समय अंजू की आयु से 90% अचधक थी। P का मान ज्ञात कीजजए, यदि 5
वर्ष बाि मंजू की आयु संजू से P% अचधक होगी।
(A) 36% (B) 20% (C) 25%
(D) 45% (E) None of these

146. An article was sold at a profit of 15%. If it was sold for Rs. 12.5 more,
then profit percentage would have been 20%. What is the cost price of the
article?
एक वस्तु को 15% लाभ पर बेचा गया। यदि इसे 12.5 रुपये अचधक में बेचा जाता, तो लाभ
प्रनतशत 20% होता। वस्तु का क्रय मल्
ू य क्या है ?
(A) Rs. 200 (B) Rs. 300 (C) Rs. 250
(D) Rs. 500 (E) None of these

147. A milk vendor has two buckets of milk. In which the first is containing
25% of water and rest is milk while the second is containing equal
proportion of milk and water. How much mixture should be taken from 1st
bucket so that the resultant mixture contains 12 liter in which the ratio of
milk to water is 5 : 3?
एक िध ू वाले के पास िो बाल्टी िधू है । जजसमें पहले में 25% पानी है और बाकी में िध
ू है जबकक
िस
ू रे में बराबर अन प
ु ात में िधू और पानी है । पहली बाल्टी से ककतना लमश्ण ननकाला जाए कक
पररणामी लमश्ण में 12 लीटर हो जजसमें िध ू का पानी से अनुपात 5 : 3 हो?
(A) 6 liters (B) 8 liters (C) 10 liters
(D) 18 liters (E) None of these

148. The perimeter of a rectangle is 5/2 times the perimeter of a square.


Length of rectangle is 50% more than its breadth. Find the ratio of the area
of the square to the area of the rectangle.
एक आयत का पररमाप एक वगष के पररमाप का 5/2 गुना है । आयत की लंबाई उसकी चौडाई से
50% अचधक है । वगष के क्षेत्रिल का आयत के क्षेत्रिल से अनुपात ज्ञात कीजजए।

https :
//www. https : https
https : //instagra
//youtube.c
facebo m.com/aas :
om/channe hisharoraso
l/UCYa4_Jr ok.com cial(?) //t.m
Orf8R5Kz2u
/aashis utm_mediu
e/stu

100
OtccXQ m=
haroras copy_link
ocial dified
(A) 1 : 4 (B) 1 : 6 (C) 1 : 8
(D) 1 : 2 (E) None of these

149. A merchant purchased two articles each of Rs. 46000. He sold one article
at 17% profit and other at 9% loss. Find the total gain percentage in the
transaction for the merchant.
एक व्यापारी ने 46000 रुपये की िो वस्तुएँ खरीिीं। उसने एक वस्तु को 17% लाभ पर और िस
ू री
को 9% हानन पर बेचा। व्यापारी के ललए लेन-िे न में कुल लाभ प्रनतशत ज्ञात कीजजए।
(A) 4% (B) 5% (C) 6%
(D) 7% (E) None of these

150. Arpan and Darpan invested Rs. 6000 and Rs. 7500 respectively in the
business. Arpan withdrew Rs. 1500 after 3 months and Darpan withdrew Rs.
2000 after 5 months. Find the profit share of Arpan, after one year of Darpan
earns a profit of Rs. 2280.
अपषण और िपषण ने व्यापार में क्रमशः 6000 रुपये और 7500 रुपये का ननवेश ककया। अपषण ने 3
महीने बाि 1500 रुपये ननकाले और िपषण ने 5 महीने बाि 2000 रुपये ननकाले। अपषण का लाभ
दहस्सा ज्ञात कीजजए, िपषण के एक वर्ष के बाि 2280 रुपये का लाभ अजजषत करता है ।
(A) Rs. 1680 (B) Rs. 1755 (C) Rs. 1170
(D) Rs. 1000 (E) None of these

151. The average of present age of Aashi and Sara is 40 years. If the ratio of
the age of Aashi 6 years ago from now to the age of Sara 2 years hence from
now is 8 : 11, respectively, then find the difference between their present
ages.
आशी और सारा की वतषमान आयु का औसत 40 वर्ष है । यदि 6 वर्ष पहले आशी की आयु का कैसे से
2 वर्ष बाि सारा की आयु का अनुपात क्रमशः 8 : 11 है , तो उनकी वतषमान आयु के बीच का अंतर
ज्ञात कीजजए।
(A) 3 years (B) 4 years (C) 5 years
(D) 6 years (E) None of these

152. Roshan is 6 times as efficient as Mohsin and both can complete a work
together in 18 days. If efficiency of Manav is equal to average of efficiencies
of Roshan and Mohsin, then find the time taken by Manav to do the same
work alone.
रोशन, मोहलसन से 6 गनु ा कुशल है और िोनों लमलकर एक कायष को 18 दिनों में परू ा कर सकते हैं।
यदि मानव की िक्षता रोशन और मोहलसन की औसत क्षमता के बराबर है , तो मानव द्वारा अकेले
समान कायष को करने में ललया गया समय ज्ञात कीजजए।
(A) 34 days (B) 38 days (C) 36 days
(D) 32 days (E) None of these

https :
//www. https : https
https : //instagra
//youtube.c
facebo m.com/aas :
om/channe hisharoraso
l/UCYa4_Jr ok.com cial(?) //t.m
Orf8R5Kz2u
/aashis utm_mediu
e/stu

101
OtccXQ m=
haroras copy_link
ocial dified
153. In a election between two candidates winner got 40% of total votes cast.
30% of total votes cost declared invalid. If votes margin between winner and
loser was 120, then find the total votes cast in the election.
िो उम्मीिवारों के बीच एक चन ु ाव में ववजेता को डाले गए कुल मतों का 40% प्राप्त हुआ। कुल
लागत का 30% वोट अमान्य घोवर्त ककया गया। यदि ववजेता और हारने वाले के बीच मतों का
अंतर 120 था, तो चन ु ाव में डाले गए कुल मत ज्ञात कीजजए।
(A) 1260 (B) 1240 (C) 1220
(D) 1200 (E) None of these

154. Diya started a business with an initial investment of Rs. 4250. After 4
months, Manya joined her investing Rs. '4250 + x'. If the ratio of profit of Diya
and Manya is 3 : 5, then find the value of '4250 + x'.
िीया ने 4250 रुपये के शरु
ु आती ननवेश के साथ एक व्यवसाय शरूु ककया। 4 महीने के बाि, मान्या
'4250 + x' रुपये के ननवेश के साथ जुड गई। यदि िीया और मान्या के लाभ का अनुपात 3 : 5 है ,
तो '4250 + x' का मान ज्ञात कीजजए।
(A) 10585 (B) 10625 (C) 10785
(D) 10465 (E) None of these

155. Navnit saves 85% of his income. If his savings is decreased by 30%
while his expenditure is increased by 58%, then find the percentage
increase/decrease in his monthly income.
नवनीत अपनी आय का 85% बचाता है । यदि उसकी बचत 30% कम हो जाती है जबकक उसका
व्यय 58% बढ जाता है , तो उसकी मालसक आय में प्रनतशत वद्
र चध/कमी ज्ञात कीजजए।
(A) 16.8% (B) 15.7% (C) 17.4%
(D) 18.9% (E) None of these

156. The ratio of the speed of a boat in still water to the speed of the stream
is 8 : 3. The boat takes 12 hours more to cover the distance of 220 km in
upstream than to cover the same distance downstream. Find the speed of the
boat in still water.
शांत जल में एक नाव की गनत का धारा की गनत से अनप ु ात 8 : 3 है । नाव को धारा के प्रनतकूल
220 ककमी की िरू ी तय करने में समान िरू ी को धारा के अनुकूल तय करने में 12 घंटे अचधक लगते
हैं। जस्थर जल में नाव की गनत ज्ञात कीजजए।
(A) 16 km/hr (B) 18 km/hr (C) 14 km/hr
(D) 12km/hr (E) None of these

157. Train 'P' moving with a speed of 72 km/hr can cross a pole in 22 seconds
while train 'P' can cross train 'Q' coming from the opposite side with a speed
of 63 km/hr in 28 seconds. Find the length of train 'Q'.
ट्रे न 'P' 72 ककमी/घंटा की गनत से चलते हुए एक खंभे को 22 सेकंड में पार कर सकती है जबकक
ट्रे न'P' ववपरीत दिशा से आ रही ट्रे न 'Q' को 28 सेकंड में 63 ककमी/घंटा की गनत से

https :
//www. https : https
https : //instagra
//youtube.c
facebo m.com/aas :
om/channe hisharoraso
l/UCYa4_Jr ok.com cial(?) //t.m
Orf8R5Kz2u
/aashis utm_mediu
e/stu

102
OtccXQ m=
haroras copy_link
ocial dified
पार कर सकती है । ट्रे न 'Q' की लंबाई ज्ञात कीजजए।
(A) 630 m (B) 620 m (C) 610 m
(D) 640 m (E) None of these

158. Varun invests Rs. X at rate of 20% p.a. compound interest for 3 years. At
the end of 3 years, he received Rs. 5460 as interest. If he invest Rs. 'X + 1500'
at the rate of 12% simple interest for 2 years, then find simple interest
received by him.
वरुण 3 साल के ललए 20% वावर्षक चक्रवद् र चध ब्याज की िर से X रुपये का ननवेश करता है । 3 वर्ष
के अंत में , उसे ब्याज के रूप में 5460 रुपये प्राप्त हुए। यदि वह 2 वर्ष के ललए 12% साधारण
ब्याज की िर से 'X + 1500' रुपये का ननवेश करता है , तो उसे प्राप्त साधारण ब्याज ज्ञात कीजजए।
(A) Rs. 2140 (B) Rs. 2130 (C) Rs. 2160
(D) Rs. 2150 (E) None of these

159. The curved surface area of a conical container whose slant height is 19
meters is 1914 m2. Find the volume of a cylindrical container whose radius of
the base and the height is equal to that of the conical container.
19 मीटर त्रतरछी ऊंचाई वाले एक शंक्वाकार बततन का वक्र पृष्ठीय क्षेिफल 1914 मी2 है ।
एक बेलनाकार पाि का आयतन ज्ञात कीजिए जिसके आधार की त्रिज्या और ऊँचाई
शंक्वाकार पाि के बराबर है ।
(A) 27560 m3 (B) 27720 m3 (C) 27630 m3
(D) 27840 m 3 (E) None of these

160. A vessel contains mixture of soda and water in the ratio of 9 : 7


respectively. If 64 liters of mixture is taken out and 81 liter of soda and 63
liters of water is added to the mixture, then find the ratio of soda and water in
final mixture.
एक बतषन में सोडा और पानी का लमश्ण क्रमशः 9 : 7 के अनुपात में है । यदि लमश्ण का 64 लीटर
ननकाल ललया जाता है और लमश्ण में 81 लीटर सोडा और 63 लीटर पानी लमला दिया जाता है , तो
अंनतम लमश्ण में सोडा और पानी का अनुपात ज्ञात कीजजए।
(A) 4 : 3 (B) 6 : 5 (C) 7 : 8
(D) 9 : 7 (E) None of these

161. Rohit can complete a piece of work in 48 days while Rohit and Manas
together can complete the same work in 30 days. If Rohit and Manas together
started the work and Rohit left after 18 days, then find the time required to
complete the rest of the work by Manas.
रोदहत एक कायष को 48 दिनों में पूरा कर सकता है जबकक रोदहत और मानस लमलकर उसी कायष को

https :
//www. https : https
https : //instagra
//youtube.c
facebo m.com/aas :
om/channe hisharoraso
l/UCYa4_Jr ok.com cial(?) //t.m
Orf8R5Kz2u
/aashis utm_mediu
e/stu

103
OtccXQ m=
haroras copy_link
ocial dified
30 दिनों में पूरा कर सकते हैं। यदि रोदहत और मानस ने एक साथ कायष शुरू ककया और रोदहत 18
दिनों के बाि काम छोड िे ता है , तो मानस द्वारा शेर् कायष को पूरा करने में लगने वाला समय ज्ञात
कीजजए।
(A) 34 days (B) 32 days (C) 38 days
(D) 36 days (E) None of these

162. 540 ml of mixture contains juice and water in the ratio of 11 : 7


respectively. If 'X' ml of water and 'X + 130' ml of juice is added into it, then
ratio of juice to water in the resultant mixture become 2 : 1. Find the value of
'X'.
540 लमली लमश्ण में रस और पानी क्रमशः 11 : 7 के अनुपात में हैं। यदि इसमें 'X' लमली पानी
और 'X + 130' लमली जूस लमला दिया जाए, तो पररणामी लमश्ण में जूस का पानी से अनुपात 2 :
1 हो जाता है । 'X' का मान ज्ञात कीजजए।
(A) 20 (B) 30 (C) 40
(D) 50 (E) None of these

163. The length of a rectangular field is 40% more than its breadth. If the cost
of fencing the rectangular field at the rate of Rs. 6 meter is Rs. 1728, then find
the length of the rectangular field.
एक आयताकार मैिान की लम्बाई उसकी चौडाई से 40% अचधक है । यदि 6 मीटर की िर से
आयताकार क्षेत्र में बाड लगाने की लागत 1728 रुपये है , तो आयताकार क्षेत्र की लंबाई ज्ञात
कीजजए।
(A) 96 m (B) 72 m (C) 68 m
(D) 84 m (E) None of these

164. Rishabh is 4 years elder to Aayushi. 28 years hence from now the ratio
of the age of Rishabh to that of Aayushi will be 10 : 9. 3 years ago from now,
the age of Rishabh was how much percent more/less than that of Aayushi?
ऋर्भ आयुर्ी से 4 साल बडा है । अब से 28 वर्ष बाि, ऋर्भ की आयु का आयुर्ी से अनुपात 10 : 9
होगा। अब से 3 वर्ष पहले, ऋर्भ की आय,ु आयर् ु ी की आयु से ककतने प्रनतशत अचधक/कम थी?
(A) 70% (B) 80% (C) 50%
(D) 60% (E) None of these

165. Monthly savings of Raman is Rs. 600 more than his monthly expenditure
and his total monthly income is Rs. 4200. If his monthly income is increased
by 60% and his monthly savings is increased by 20%, then find the increase
in his monthly expenditure.
रमन की मालसक बचत उसके मालसक व्यय से 600 रुपये अचधक है और उसकी कुल मालसक आय
4200 रुपये है । यदि उसकी मालसक आय में 60% की वद् र चध होती है और उसकी मालसक बचत में
20% की वद्
र चध होती है , तो उसके मालसक व्यय में व द्
र चध ज्ञात कीजजए।

https :
//www. https : https
https : //instagra
//youtube.c
facebo m.com/aas :
om/channe hisharoraso
l/UCYa4_Jr ok.com cial(?) //t.m
Orf8R5Kz2u
/aashis utm_mediu
e/stu

104
OtccXQ m=
haroras copy_link
ocial dified
(A) Rs. 2070 (B) Rs. 2230 (C) Rs. 2040
(D) Rs. 2380 (E) None of these

166. Madhur started a business with an initial investment of Rs. 2800. After X
months Aarush entered into the business investing Rs. 2000. After 1 years,
Ratio of profit of Madhur and Aarush will be 21 : 10. Find the value of 'X'.
मधरु ने 2800 रुपये के शुरुआती ननवेश के साथ एक व्यवसाय शुरू ककया। आरुर् ने X महीने के
बाि 2000 रुपये का ननवेश करके व्यापार में प्रवेश ककया। 1 साल बाि, मधरु और आरुर् के लाभ
का अनुपात 21 : 10 होगा। 'X' का मान ज्ञात कीजजए।
(A) 4 (B) 7 (C) 6
(D) 8 (E) None of these

167. An article whose cost price is Rs. 6800 is marked 36% above its cost
price and sold after giving Rs. 'X' discount. If there is a profit of 8% in the
transaction, then find the value of 'X'.
एक वस्तु जजसका क्रय मूल्य 6800 रुपये है , उसके लागत मूल्य से 36% अचधक अंककत ककया
जाता है और 'X' रुपये की छूट िे ने के बाि बेचा जाता है । यदि सौिे में 8% का लाभ होता है , तो 'X'
का मान ज्ञात कीजजए।
(A) 2091 (B) 1866 (C) 1904
(D) 1789 (E) None of these

168. Average of 18 numbers is 68. If the average of first 15 numbers is 62


and the ratio of sixteenth, seventeenth and eighteenth number is 2 : 3 : 1
respectively, then find the seventeenth number.
18 संख्याओ ं का औसत 68 है । यदद पहली 15 संख्याओ ं का औसत 62 है और सोलहवीं,
सिहवीं और अठारहवीं संख्याओ ं का अनुपात क्रमशः 2 : 3 : 1 है , तो सिहवीं संख्या ज्ञात
कीजिए।
(A) 153 (B) 161 (C) 147
(D) 175 (E) None of these

169. A certain sum of money at a certain rate of simple interest becomes Rs.
7200 in 4 years and becomes Rs. 7950 in 7 years. Find the sum of money.
साधारण ब्याज की एक ननजश्चत िर पर एक ननजश्चत रालश 4 साल में 7200 रुपये हो जाती है और
7 साल में 7950 रुपये हो जाती है । धनरालश ज्ञात कीजजए।
(A) Rs. 6300 (B) Rs. 6200 (C) Rs. 6400
(D) Rs. 6100 (E) None of these

170. Train P can cross a pole and a 324 meters long platform in 4.8 seconds
and 26.4 seconds respectively. If the length of train P is 50% less than that of
train Q and speed of train Q is 20% more that of train P, then find the time
taken by the train Q to cross the same platform.

https :
//www. https : https
https : //instagra
//youtube.c
facebo m.com/aas :
om/channe hisharoraso
l/UCYa4_Jr ok.com cial(?) //t.m
Orf8R5Kz2u
/aashis utm_mediu
e/stu

105
OtccXQ m=
haroras copy_link
ocial dified
ट्रे न P एक खंबे और 324 मीटर लंबे प्लेटिॉमष को क्रमशः 4.8 सेकंड और 26.4 सेकंड में पार कर
सकती है । यदि ट्रे न P की लंबाई ट्रे न Q की लंबाई से 50% कम है और ट्रे न Q की गनत ट्रे न P की
गनत से 20% अचधक है , तो समान प्लेटिॉमष को पार करने के ललए ट्रे न Q द्वारा ललया गया समय
ज्ञात कीजजए।
(A) 28 seconds (B) 24 seconds (C) 22 seconds
(D) 26 seconds (E) None of these

171. A box contains 8 Red, 7 Blue and X Black pens. Probability of drawing
one Red pen is 1/3, then find the value of X.
एक बॉक्स में 8 लाल, 7 नीले और X काले पेन हैं। एक लाल पेन ननकालने की प्रानयकता 1/3 है , तो
X का मान ज्ञात कीजजए।
(A) 8 (B) 9 (C) 7
(D) 6 (E) None of these

172. Ratio of the present ages of Samayra and Raghav is P : Q respectively.


At the time of birth of Anaya ratio of their ages was 14 : 17 respectively. If
Anaya is 2 years old now and the age of Raghav at the time of birth of Anaya
is 34 years, then find the value of 4P : 3Q.
समायरा और राघव की वतषमान आयु का अनप ु ात क्रमशः P : Q है । अनाया के जन्म के समय
उनकी आयु का अनुपात क्रमशः 14 : 17 था। यदि अनाया अभी 2 वर्ष की है और अनाया के जन्म
के समय राघव की आयु 34 वर्ष है , तो 4P : 3Q का मान ज्ञात कीजजए।
(A) 7 : 6 (B) 3 : 4 (C) 8 : 5
(D) 9 : 2 (E) None of these

173. Rs. 7500 and Rs. 9000 are invested at a rate of (R + 5)% p.a. simple
interest and (R - 4)% p.a. simple interest respectively for 3 years. If total
interest received at the end of 3 years is Rs. 4995, then find the value of ‘R’.
7500 रुपये और 9000 रुपये 3 साल के ललए क्रमशः (R + 5)% प्रनतवर्ष साधारण ब्याज और (R -
4)% प्रनतवर्ष साधारण ब्याज की िर से ननवेश ककए जाते हैं। यदि 3 वर्ष के अंत में प्राप्त कुल ब्याज
4995 रुपये है , तो 'R' का मान ज्ञात कीजजए।
(A) 9 (B) 12 (C) 8
(D) 10 (E) None of these

174. The speed of boat ‘R’ in still water is 3 km/hr less than that of boat ‘S’
and 20% more than the speed of the stream. If boat ‘S’ takes 8.5 hours to
travel 306 km downstream, then find the upstream speed of boat ‘R’.
शांत जल में नाव 'R' की गनत नाव 'S' की गनत से 3 ककमी/घंटा कम है और धारा की गनत से 20%
अचधक है । यदि नाव 'S' को धारा के अनुकूल 306 ककमी की यात्रा करने में 8.5 घंटे लगते हैं, तो
नाव 'R' की धारा के प्रनतकूल गनत ज्ञात कीजजए।

https :
//www. https : https
https : //instagra
//youtube.c
facebo m.com/aas :
om/channe hisharoraso
l/UCYa4_Jr ok.com cial(?) //t.m
Orf8R5Kz2u
/aashis utm_mediu
e/stu

106
OtccXQ m=
haroras copy_link
ocial dified
(A) 2 km/hr (B) 3 km/hr (C) 4 km/hr
(D) 5 km/hr (E) None of these

175. The ratio of the height to the radius of the base of a cylinder is 3 : 4. If
the curved surface area of the cylinder is 14784 cm2, then find the area of its
base.
एक बेलन के आधार की बत्रज्या से ऊँचाई का अनुपात 3 : 4 है । यदि बेलन का वक्र पष्र िीय क्षेत्रिल
14784 सेमी2 है , तो इसके आधार का क्षेत्रिल ज्ञात कीजजए।
(A) 9856 cm2 (B) 9247 cm2 (C) 9684 cm2
(D) 9426 cm2 (E) None of these

176. On selling 56 articles for Rs. 3360 is a loss of equal to the cost price of 8
articles. Find the selling price of 20 articles, If the shopkeeper wants to earn a
profit of 40%.
56 वस्तओु ं को 3360 रुपये में बेचने पर 8 वस्तओु ं के क्रय मल्
ू य के बराबर की हानन होती है । 20
वस्तुओं का ववक्रय मूल्य ज्ञात कीजजए, यदि िक ु ानिार 40% का लाभ कमाना चाहता है ।
(A) Rs. 1960 (B) Rs. 1830 (C) Rs. 1690
(D) Rs. 1720 (E) None of these

177. The ratio of the investment of Kaira to Saira is 5 : 3 and the ratio of time
of investment is 7 : 10 respectively. Find the profit share of Saira out of the
total profit Rs. 3315.
कायरा और सायरा के ननवेश का अनप ु ात 5 : 3 है और ननवेश के समय का अनप ु ात क्रमशः 7 : 10
है । 3315 रुपये के कुल लाभ में से सायरा का लाभ दहस्सा ज्ञात कीजजए।
(A) Rs. 1480 (B) Rs. 1620 (C) Rs. 1530
(D) Rs. 1340 (E) None of these

178. Ratio of the monthly income of Ravi to Satish is 8 : 9. Monthly


expenditure of Ravi is Rs. 8000 while that of Satish is Rs. 2000 more than
Ravi. If the ratio of the savings of Ravi to Satish is 12 : 13, then find the
monthly income of Ravi.
रवव की मालसक आय का सतीश से अनुपात 8 : 9 है । रवव का मालसक खचष 8000 रुपये है जबकक
सतीश का खचष रवव से 2000 रुपये अचधक है । यदि रवव की बचत का सतीश से अनप ु ात 12 : 13 है ,
तो रवव की मालसक आय ज्ञात कीजजए।
(A) Rs. 34000 (B) Rs. 33000 (C) Rs. 32000
(D) Rs. 35000 (E) None of these

179. Parth and Rohan together can complete a work in 30 days while Parth
takes 48 days to complete the same work alone. If Sohit is 33.33% more
efficient than Rohan, then find the time taken by Sohit alone to complete the
whole work.

https :
//www. https : https
https : //instagra
//youtube.c
facebo m.com/aas :
om/channe hisharoraso
l/UCYa4_Jr ok.com cial(?) //t.m
Orf8R5Kz2u
/aashis utm_mediu
e/stu

107
OtccXQ m=
haroras copy_link
ocial dified
पाथष और रोहन लमलकर एक कायष को 30 दिनों में पूरा कर सकते हैं जबकक पाथष अकेले उसी कायष
को परू ा करने में 48 दिन का समय लेता है । यदि सोदहत, रोहन की तल ु ना में 33.33% अचधक
कुशल है , तो अकेले सोदहत द्वारा पूरे कायष को पूरा करने में ललया गया समय ज्ञात कीजजए।
(A) 40 days (B) 60 days (C) 30 days
(D) 50 days (E) None of these

180. A shopkeeper mixed (2X + 1) kg of rice flour of price Rs. 45/kg with 15 kg
of corn flour of price Rs. 80/kg. If shopkeeper earns 16.66% profit from selling
the flour mixture at Rs. 70 per kg, then find the value of X.
एक िक ु ानिार ने (2X + 1) ककलो चावल का आटा, जजसकी कीमत 45 रुपये/ककलो है , को 15 ककलो
मक्के के आटे , जजसकी कीमत 80 रुपये/ककलो है , के साथ लमला दिया। यदि िक
ु ानिार आटे के
लमश्ण को 70 रुपये/ककलो की िर से बेचने पर 16.66% लाभ अजजषत करता है , तो X का मान ज्ञात
कीजजए।
(A) 7.4 (B) 8.7 (C) 10
(D) 9.5 (E) None of these

181. 380 liters of a mixture contains 65% juice and rest water. If 'X' liters of
juice is added in to the mixture, then quantity of water in the final mixture
becomes 20% of total quantity of mixture. Find the value of 'X'.
380 लीटर लमश्ण में 65% रस और शेर् पानी है । यदि लमश्ण में 'X' लीटर रस लमलाया जाता है ,
तो अंनतम लमश्ण में पानी की मात्रा लमश्ण की कुल मात्रा का 20% हो जाती है । 'X' का मान ज्ञात
कीजजए।
(A) 293 (B) 285 (C) 279
(D) 268 (E) None of these

182. Mohini and Raghav started a business by investing Rs. 7800 and Rs.
7200 respectively. After 5 months Mohini withdrew Rs. 1800 while Raghav
invested Rs. 1800 more. If the annual profit earned by them was Rs. 5000,
then find the profit share of Raghav.
मोदहनी और राघव ने क्रमशः 7800 रुपये और 7200 रुपये ननवेश करके एक व्यवसाय शुरू ककया।
5 महीने बाि मोदहनी ने 1800 रुपये ननकाले जबकक राघव ने 1800 रुपये और ननवेश ककए। यदि
उनके द्वारा अजजषत वावर्षक लाभ 5000 रुपये था, तो राघव का लाभ दहस्सा ज्ञात कीजजए।
(A) Rs. 2680 (B) Rs. 2820 (C) Rs. 2750
(D) Rs. 2570 (E) None of these

183. Mehul usually covers 240 km in 4 hours. If he covers 345 km with 15%
more speed than his usual speed and rest 96 km with 80% of his usual speed,
then find the time taken by Mehul to cover his journey.

https :
//www. https : https
https : //instagra
//youtube.c
facebo m.com/aas :
om/channe hisharoraso
l/UCYa4_Jr ok.com cial(?) //t.m
Orf8R5Kz2u
/aashis utm_mediu
e/stu

108
OtccXQ m=
haroras copy_link
ocial dified
मेहुल आमतौर पर 4 घंटे में 240 ककमी की िरू ी तय करता है । यदि वह अपनी सामान्य गनत से
15% अचधक गनत से 345 ककमी की िरू ी तय करता है और अपनी सामान्य गनत से 80% अचधक
गनत से 96 ककमी की िरू ी तय करता है , तो मेहुल द्वारा अपनी यात्रा को तय करने में ललया गया
समय ज्ञात कीजजए।
(A) 9 hours (B) 6 hours (C) 8 hours
(D) 7 hours (E) None of these

184. Ajay deposited Rs. 3200 at R% rate of interest compound annually. After
two years he received Rs. 850 as interest. Find the value of R.
अजय ने सालाना चक्रवद् र चध चक्रवद्
र चध ब्याज िर पर 3200 रुपये जमा ककए। िो वर्ष बाि उसे ब्याज
के रूप में 850 रुपये प्राप्त हुए। R का मान ज्ञात कीजजए।
(A) 15.6 % (B) 12.5% (C) 14.08%
(D) 13.12% (E) None of these

185. The average weight the five students 'P', 'Q', 'R', 'S', and 'T' is 54 kg and
the average of 'R' and 'S' is 50 kg. If the ratio of 'P', 'Q' and 'T' is 5 : 3 : 2
respectively, then find the average of 'P' and 'T'.
पांच छात्रों 'P', 'Q', 'R', 'S' और 'T' का औसत वजन 54 ककलो है और 'R' और 'S' का औसत
वजन 50 ककलो है । यदि 'P', 'Q' और 'T' का अनुपात क्रमशः 5 : 3 : 2 है , तो 'P' और 'T' का
औसत ज्ञात कीजजए।
(A) 59.5 kg (B) 52.4 kg (C) 57.1 kg
(D) 54.8 kg (E) None of these

186. Monthly income of Mitali is Rs. 24000 out of which he spent 35%, X%
and 15% of her total income in travelling, rent and shopping respectively. If
he saves rest amount is Rs. 7200, then find the value of 'X'.
लमताली की मालसक आय 24000 रुपये है जजसमें से वह अपनी कुल आय का क्रमशः 35%, X%
और 15% यात्रा, ककराए और खरीिारी पर खचष करता है । यदि वह शेर् रालश 7200 रुपये बचाता है ,
तो 'X' का मान ज्ञात कीजजए।
(A) 20 (B) 30 (C) 50
(D) 40 (E) None of these

187. A trader sold an article at Rs. 6664. If he marked the price 60% above of
its cost price and sold it after giving two successive discounts of 15% and
30% respectively, then find the cost price of article.
एक व्यापारी ने एक वस्तु को 6664 रुपये में बेचा। यदि वह मूल्य को उसके लागत मूल्य से 60%
अचधक अंककत करता है और क्रमशः 15% और 30% की िो क्रलमक छूट िे ने के बाि उसे बेचता है ,
तो वस्तु का क्रय मल्
ू य ज्ञात कीजजए।
(A) Rs. 7300 (B) Rs. 7200 (C) Rs. 7000
(D) Rs. 7100 (E) None of these

https :
//www. https : https
https : //instagra
//youtube.c
facebo m.com/aas :
om/channe hisharoraso
l/UCYa4_Jr ok.com cial(?) //t.m
Orf8R5Kz2u
/aashis utm_mediu
e/stu

109
OtccXQ m=
haroras copy_link
ocial dified
188. 4 years ago from now age of Meeta is 62.5% of the age of Nukul. If 6
years hence from now, age Meeta will be 70% of the age of Nukul, then find
the present age of Meeta.
अब से 4 वर्ष पहले मीता की आयु नुकुल की आयु का 62.5% है । यदि अब से 6 वर्ष बाि, मीता की
आयु नुकुल की आयु का 70% होगी, तो मीता की वतषमान आयु ज्ञात कीजजए।
(A) 28 years (B) 27 years (C) 29 years
(D) 26 years (E) None of these

189. Vibha alone can complete a work in 30 days. Vibha and Varun starts
working together but after 6 days Varun left the work and remaining work is
completed by Vibha in 19 days. Efficiency of Vibha is how much percent
more/less than efficiency of Varun?
ववभा अकेले एक कायष को 30 दिनों में परू ा कर सकती है । ववभा और वरुण एक साथ कायष करना
शुरू करते हैं लेककन 6 दिनों के बाि वरुण कायष छोड िे ता है और शेर् कायष ववभा द्वारा 19 दिनों में
पूरा ककया जाता है । ववभा की क्षमता वरुण की िक्षता से ककतने प्रनतशत अचधक/कम है ?
(A) 30% (B) 20% (C) 40%
(D) 10% (E) None of these

190. A boat can cover 840 km downstream distance in X hours and the boat
can cover 360 km in still water in 8 hours. If the speed of stream is 44.44%
less than speed of boat in still water, then find the value of 'X'.
एक नाव धारा के अनुकूल X घंटे में 840 ककमी की िरू ी तय कर सकती है और शांत पानी में नाव 8
घंटे में 360 ककमी की िरू ी तय कर सकती है । यदि धारा की गनत शांत जल में नाव की गनत से
44.44% कम है , तो 'X' का मान ज्ञात कीजजए।
(A) 16 hours (B) 14 hours (C) 10 hours
(D) 12 hours (E) None of these

191. 40 women can complete a work in 18 days. If 25 women started the same
work and after 10 days 'X' more women joined the work such that the
remaining work got completed in 5 days, then find the value of 'X'. (All
women's has same efficiency)
40 मदहलाएं एक कायष को 18 दिनों में पूरा कर सकती हैं। यदि 25 मदहलाएं समान कायष शुरू करती
हैं और 10 दिनों के बाि 'X' और मदहलाएं कायष में शालमल हो जाती हैं, तो शेर् कायष 5 दिनों में परू ा
हो जाता है , तो 'X' का मान ज्ञात कीजजए। (सभी मदहलाओं की िक्षता समान होती है )
(A) 67 (B) 69 (C) 65
(D) 63 (E) None of these

192. The sum of three numbers is 424. If the ratio of 1st to 2nd number is 3 : 7
and 2nd to 3rd number is 5 : 8, then find the difference between 2nd and 3rd

https :
//www. https : https
https : //instagra
//youtube.c
facebo m.com/aas :
om/channe hisharoraso
l/UCYa4_Jr ok.com cial(?) //t.m
Orf8R5Kz2u
/aashis utm_mediu
e/stu

110
OtccXQ m=
haroras copy_link
ocial dified
number.
तीन संख्याओं का योग 424 है । यदि पहली से िस ू री संख्या का अनुपात 3 : 7 है और िसू री से
तीसरी संख्या का अनुपात 5 : 8 है , तो िस
ू री और तीसरी संख्या के बीच का अंतर ज्ञात कीजजए।
(A) 88 (B) 82 (C) 84
(D) 86 (E) None of these

193. Aman invested Rs. 3000 in a business which is 20% less than the sum
invested by Piya. Aman invested his sum for 9 months and Piya invested her
sum for 4 months. If the profit received by Aman is Rs. 3600, then find the
total profit received by Aman and Piya together.
अमन ने एक व्यवसाय में 3000 रुपये का ननवेश ककया जो वपया द्वारा ननवेश की गई रालश से
20% कम है । अमन ने अपनी रालश 9 महीने के ललए ननवेश की और वपया ने अपनी रालश 4 महीने
के ललए ननवेश की। यदि अमन द्वारा प्राप्त लाभ 3600 रुपये है , तो अमन और वपया द्वारा प्राप्त
कुल लाभ ज्ञात कीजजए।
(A) Rs.5800 (B) Rs. 5400 (C) Rs. 5200
(D) Rs. 5600 (E) None of these

194. The incomes of Vihan and Ruhan is Rs. 6800 and Rs. 7200 respectively.
If their expenditures is Rs. 5300 and Rs. 4700 respectively, then find the ratio
of their savings.
ववहान और रुहान की आय क्रमशः 6800 रुपये और 7200 रुपये है । यदि उनका व्यय क्रमशः 5300
रुपये और 4700 रुपये है , तो उनकी बचत का अनुपात ज्ञात कीजजए।
(A) 2 : 9 (B) 3 : 5 (C) 8 : 7
(D) 4 : 7 (E) None of these

195. In a town, there are 30500 people out of which 52% are males. 10540
males are literate which is 1560 more than the number of females who
literate. How much percent of people are illiterate?
एक कस्बे में 30500 लोग हैं जजनमें से 52% परु
ु र् हैं। 10540 परु
ु र् साक्षर हैं जो साक्षर मदहलाओं
की संख्या से 1560 अचधक है । ककतने प्रनतशत लोग ननरक्षर हैं?
(A) 36% (B) 32% (C) 34%
(D) 38% (E) None of these

196. Rs. 'X' when invested at R% Per annum compound interest,


compounded annually amounts to Rs. 4880 in 5 years and Rs. 7320 in 6
years. Find the value of 'R'.
R% प्रनत वर्ष चक्रवद्र चध ब्याज पर ननवेश ककए जाने पर 'X' रुपये, वावर्षक रूप से संयोजजत होकर 5
वर्ों में 4880 रुपये और 6 वर्ों में 7320 रुपये हो जाते हैं। 'R' का मान ज्ञात कीजजए।
(A) 50 (B) 30 (C) 40
(D) 60 (E) None of these

https :
//www. https : https
https : //instagra
//youtube.c
facebo m.com/aas :
om/channe hisharoraso
l/UCYa4_Jr ok.com cial(?) //t.m
Orf8R5Kz2u
/aashis utm_mediu
e/stu

111
OtccXQ m=
haroras copy_link
ocial dified
197. Tanu travelling with a speed of 'X + 10' km/hr takes 4 hours less time to
cover a distance of 150 km than the time taken by Rita who is travelling with
a speed of X km/hr, to cover the same distance. Find the ratio of speed of
Tanu to that of Rita.
तनु 'X + 10' ककमी/घंटा की गनत से यात्रा करते हुए 150 ककमी की िरू ी तय करने में रीता द्वारा
ललए गए समय की तुलना में 4 घंटे कम समय लेती है , जो समान िरू ी को तय करने के ललए X
ककमी/घंटा की गनत से यात्रा कर रही है । तनु की गनत का रीता की गनत से अनुपात ज्ञात कीजजए।
(A) 4 : 9 (B) 6 : 7 (C) 5 : 3
(D) 8 : 5 (E) None of these

198. Length and width of a rectangular room is 120 m and 88 m respectively.


If the length and width of a tiles is 2 m and 1.5 m, then find the number of
tiles required to cover the room floor.
एक आयताकार कमरे की लंबाई और चौडाई क्रमशः 120 मीटर और 88 मीटर है । यदि एक टाइल
की लंबाई और चौडाई 2 मीटर और 1.5 मीटर है , तो कमरे के िशष को ढकने के ललए आवश्यक
टाइलों की संख्या ज्ञात कीजजए।
(A) 3210 (B) 3380 (C) 3520
(D) 3470 (E) None of these

199. A container contains mixture soda and water mixed in certain ratio.
When 16.67% of the mixture is taken out and replaced with same quantity of
water, then the ratio of soda to water becomes 1 : 2. Find the ratio of soda to
water in the initial mixture.
एक कंटे नर में सोडा और पानी का लमश्ण एक ननजश्चत अनुपात में है । जब लमश्ण का 16.67%
ननकाल ललया जाता है और पानी की समान मात्रा से बिल दिया जाता है , तो सोडा का पानी से
अनुपात 1 : 2 हो जाता है । आरं लभक लमश्ण में सोडा का पानी से अनुपात ज्ञात कीजजए।
(A) 5 : 4 (B) 2 : 3 (C) 6 : 7
(D) 8 : 9 (E) None of these

200. If the area of a sector of a circle is 154 cm2, then find the perimeter of
circle. (π = 22/7, ϴ = 90°)
यदि ककसी वत्त
र के एक बत्रज्यखंड का क्षेत्रिल 154 सेमी2 है , तो वत्त
र का पररमाप ज्ञात कीजजए। (π =
22/7, ϴ = 90°)
(A) 84 cm (B) 82 cm (C) 86 cm
(D) 88 cm (E) None of these

https :
//www. https : https
https : //instagra
//youtube.c
facebo m.com/aas :
om/channe hisharoraso
l/UCYa4_Jr ok.com cial(?) //t.m
Orf8R5Kz2u
/aashis utm_mediu
e/stu

112
OtccXQ m=
haroras copy_link
ocial dified
201. Sunil covered 132 km by car and rest 36 km by cycle. If the speed of the
car is 175% more than that of the cycle and whole distance is covered in 3
hours 30 minutes, then find the speed of the car.
सुनील ने कार से 132 ककमी और साइककल से 36 ककमी की िरू ी तय की। यदि कार की गनत
साइककल की गनत से 175% अचधक है और परू ी िरू ी 3 घंटे 30 लमनट में तय की जाती है , तो कार
की गनत ज्ञात कीजजए।
(A) 64 km/hr (B) 66 km/hr (C) 63 km/hr
(D) 65 km/hr (E) None of these

202. Speed of boat in downstream is 20% more than speed of boat in still
water. If speed of stream is decreased by 10%, then the boat can travel a
distance of 246 km is upstream in 12 hours. Find the time taken by boat to
cover the same distance in downstream with the original speed of stream.
धारा के अनुकूल नाव की गनत जस्थर जल में नाव की गनत से 20% अचधक है । यदि धारा की गनत
10% कम कर िी जाए, तो नाव 12 घंटे में धारा के प्रनतकूल 246 ककमी की िरू ी तय कर सकती है ।
धारा की मूल गनत के साथ धारा के अनुकूल समान िरू ी तय करने में नाव द्वारा ललया गया समय
ज्ञात कीजजए।
(A) 6.8 hours (B) 7.4 hours (C) 8.2 hours
(D) 9.5 hours (E) None of these

203. On selling 39 articles for Rs. 7803 there is a loss equal to the cost price
of 12 articles. Find the cost price of each article.
39 वस्तुओं को 7803 रुपये में बेचने पर 12 वस्तुओं के क्रय मूल्य के बराबर हानन होती है । प्रत्येक
वस्तु का क्रय मूल्य ज्ञात कीजजए।
(A) Rs. 216 (B) Rs. 225 (C) Rs. 256
(D) Rs. 289 (E) None of these

204. The income of Ritesh is Rs. 3000 more than that of Mahak. If both of
them spends 65% of their respective incomes, then find the difference
between the savings of Ritesh and Mahak.
ररतेश की आय महक की आय से 3000 रुपये अचधक है । यदि वे िोनों अपनी संबंचधत आय का
65% खचष करते हैं, तो ररतेश और महक की बचत के बीच का अंतर ज्ञात कीजजए।
(A) Rs. 1020 (B) Rs. 1050 (C) Rs. 1030
(D) Rs. 1040 (E) None of these

205. In a business there are two partners Robin and Shubham. Robin started
the business with the capital of Rs. 15000 and after X months Shubham
joined him the capital of Rs. 10000. If at the end of the year the profit is Rs.
1440 share of Robin out of the total profit of Rs. 2240, then find the value of
'X'.

https :
//www. https : https
https : //instagra
//youtube.c
facebo m.com/aas :
om/channe hisharoraso
l/UCYa4_Jr ok.com cial(?) //t.m
Orf8R5Kz2u
/aashis utm_mediu
e/stu

113
OtccXQ m=
haroras copy_link
ocial dified
एक व्यवसाय में िो साझेिार रॉबबन और शभ ु म हैं। रॉबबन ने 15000 रुपये की पंज
ू ी के साथ व्यापार
शुरू ककया और X महीने के बाि शुभम 10000 रुपये की पूंजी के साथ जुड गया। यदि वर्ष के अंत में
लाभ 2240 रुपये के कुल लाभ में रॉबबन का 1440 रुपये का दहस्सा है , तो 'X' का मान ज्ञात
कीजजए।
(A) 2 (B) 5 (C) 4
(D) 3 (E) None of these

206. The present age of Ravi is 10% less than that of Vipul. 15 years from
now, the ratio of their ages will be 14 : 15, respectively. Find the ratio of their
ages 21 years ago.
रवव की वतषमान आयु ववपल ु की आयु से 10% कम है । अब से 15 वर्ष बाि, उनकी आयु का अनप
ु ात
क्रमशः 14 : 15 होगा। 21 वर्ष पूवष उनकी आयु का अनुपात ज्ञात कीजजए।
(A) 3 : 2 (B) 4 : 7 (C) 8 : 9
(D) 6 : 5 (E) None of these

207. When 162 is added to a three digit number, the number becomes 140%
of itself. Find the sum of the digits of the three digit number.
जब 162 को तीन अंकों की संख्या में जोडा जाता है , तो संख्या स्वयं का 140% हो जाती है । तीन
अंकों की संख्या के अंकों का योग ज्ञात कीजजए।
(A) 8 (B) 6 (C) 9
(D) 7 (E) None of these

208. 'P' and 'Q' together can complete a work in 24 days while 'P' alone can
complete the same work in 32 days. If 'Q' starts the work and is joined by 'P'
after 60 days, then in how many days the total work will be completed?
'P' और 'Q' लमलकर एक काम को 24 दिनों में परू ा कर सकते हैं जबकक 'P' अकेले उसी काम को
32 दिनों में पूरा कर सकता है । यदि 'Q' ने कायष शुरू ककया और 60 दिनों के बाि 'P' से जुड गया,
तो कुल कायष ककतने दिनों में पूरा होगा?
(A) 65 days (B) 63 days (C) 69 days
(D) 67 days (E) None of these

209. The radius and height of a cone is 'X' and 'X + 7' cm. If the curved
surface area of the cone is 65π cm2 and slant height of the cone is 13 cm.
Find the volume of the cone.
एक शंकु की त्रिज्या और ऊंचाई 'X' और 'X + 7' सेमी है । यदद शंकु का वक्र पृष्ठीय क्षेिफल
65π सेमी2 है और शंकु की त्रतयतक ऊंचाई 13 सेमी है । शंकु का आयतन ज्ञात कीजिए।
(A) 110 π cm3 (B) 100 π cm3 (C) 120 π cm3
(D) 130 π cm3 (E) None of these

210. Vinita invests 50% of Rs. 'X + 1500' for 2 year at 10% compound interest
and rest invested for 2 years at 14.28% simple interest. If she receives Rs.

https :
//www. https : https
https : //instagra
//youtube.c
facebo m.com/aas :
om/channe hisharoraso
l/UCYa4_Jr ok.com cial(?) //t.m
Orf8R5Kz2u
/aashis utm_mediu
e/stu

114
OtccXQ m=
haroras copy_link
ocial dified
3000 as simple interest at the end of 2 years, then find the compound interest
received by Vinita.
ववनीता 'X + 1500' रुपये का 50% 10% चक्रवद् र चध ब्याज पर 2 साल के ललए ननवेश करती है और
बाकी 2 साल के ललए 14.28% साधारण ब्याज पर ननवेश करती है । यदि उसे 2 वर्ष के अंत में
साधारण ब्याज के रूप में 3000 रुपये प्राप्त होते हैं, तो ववनीता द्वारा प्राप्त चक्रवद्
र चध ब्याज ज्ञात
कीजजए।
(A) Rs. 11400 (B) Rs. 1230 (C) Rs. 1150
(D) Rs. 2205 (E) None of these

211. In a 1080 liters mixture, the ratio of the juice to water is 5 : 4. How much
of the quantity of water should be added to the mixture so that the
percentage of the quantity of juice in the mixture becomes 30%?
1080 लीटर के लमश्ण में रस का पानी से अनप ु ात 5 : 4 है । लमश्ण में पानी की ककतनी मात्रा
लमलाई जाए कक लमश्ण में रस की मात्रा का प्रनतशत 30% हो जाए?
(A) 840 L (B) 920 L (C) 780 L
(D) 660 L (E) None of these

212. The age of three girls is in arithmetic progression. The sum of their
present ages is 48 years and the product of ages of eldest and youngest girls
is 192. Find the age of the youngest girl.
तीन लडककयों की उम्र अंकगखणतीय प्रगनत में है । उनकी वतषमान आयु का योग 48 वर्ष है और
सबसे बडी और सबसे छोटी लडककयों की आयु का गुणनिल 192 है । सबसे छोटी लडकी की आयु
ज्ञात कीजजए।
(A) 6 years (B) 7 years (C) 8 years
(D) 5 years (E) None of these

213. In a company, 15% of the total employees are below the age of 22 years,
70% of the rest employees are of the age 22 to 40 years. If 153 employees are
above the age of 40 years, then find the total employees in the company.
एक कंपनी में , कुल कमषचाररयों में से 15% की आयु 22 वर्ष से कम है , शेर् कमषचाररयों में से 70%
की आयु 22 से 40 वर्ष है । यदि 153 कमषचाररयों की आयु 40 वर्ष से अचधक है , तो कंपनी में कुल
कमषचाररयों की संख्या ज्ञात कीजजए।
(A) 700 (B) 500 (C) 400
(D) 600 (E) None of these

214. A shopkeeper sold a bag after giving 40% discount and earned 7.14%
profit. He marked the bag Rs. 1012 above cost price which is Rs. (X + 200).
Find the value of X.
एक िक ु ानिार ने 40% छूट िे ने के बाि एक बैग बेचा और 7.14% का लाभ अजजषत ककया। उसने
बैग पर लागत मल् ू य से 1012 रुपये अचधक अंककत ककया जो कक (X + 200) रुपये है । । X का मान
ज्ञात कीजजए।

https :
//www. https : https
https : //instagra
//youtube.c
facebo m.com/aas :
om/channe hisharoraso
l/UCYa4_Jr ok.com cial(?) //t.m
Orf8R5Kz2u
/aashis utm_mediu
e/stu

115
OtccXQ m=
haroras copy_link
ocial dified
(A) 1082 (B) 1088 (C) 1086
(D) 1284 (E) None of these

215. Rohini saves 28% of her monthly income. If monthly income of Rohini is
increased by 32.4% and she still saves the same amounts as before, then find
percentage increase (approximate) in his monthly expenditure.
रोदहणी अपनी मालसक आय का 28% बचाती है । यदि रोदहणी की मालसक आय में 32.4% की
वद्र चध हो जाती है और वह अभी भी पहले की तरह ही बचत करती है , तो उसके मालसक व्यय में
प्रनतशत वद्र चध (अनुमाननत) ज्ञात कीजजए।
(A) 45% (B) 52% (C) 63%
(D) 38% (E) None of these

216. 28 male can complete a piece of work in 60 days. 'X' male started the
work. After 18 days, 46 male took 24 days to complete the remaining work.
Find the value of 'X'.
28 पुरुर् एक कायष को 60 दिनों में पूरा कर सकते हैं। 'X' पुरुर् ने काम शुरू ककया। 18 दिनों के
बाि, शेर् कायष को परू ा करने में 46 परु
ु र्ों को 24 दिन लगे। 'X' का मान ज्ञात कीजजए।
(A) 32 (B) 38 (C) 36
(D) 34 (E) None of these

217. The volume of a cone is 3080 cm3. If radius of the cone is 14 cm, then
find the height of cone.
एक शंकु का आयतन 3080 सेमी3 है । यदि शंकु की बत्रज्या 14 सेमी है , तो शंकु की ऊंचाई ज्ञात
कीजजए।
(A) 18 cm (B) 16 cm (C) 15 cm
(D) 17 cm (E) None of these

218. A boat can travel a distance of 210 km in upstream as well as in


downstream in 24 hours. Find the speed of boat in still water, If speed of
stream is 3 km/hr.
एक नाव 24 घंटे में धारा के प्रनतकूल और साथ ही धारा के अनुकूल 210 ककमी की िरू ी तय कर
सकती है । शांत जल में नाव की गनत ज्ञात कीजजए, यदि धारा की गनत 3 ककमी/घंटा है ।
(A) 14 km/hr (B) 16 km/hr (C) 18 km/hr
(D) 12 km/hr (E) None of these

219. Abhay invested Rs. (x + 800) in two schemes. In scheme A he invests


sum for 5 years at 15% simple interest and in scheme B he invests sum for 2
years at 20% compound interest. If difference between interest received from
both schemes Rs. 5084 , then find the value of (2x + 700).

https :
//www. https : https
https : //instagra
//youtube.c
facebo m.com/aas :
om/channe hisharoraso
l/UCYa4_Jr ok.com cial(?) //t.m
Orf8R5Kz2u
/aashis utm_mediu
e/stu

116
OtccXQ m=
haroras copy_link
ocial dified
अभय ने िो योजनाओं में (x + 800) रुपये का ननवेश ककया। योजना A में वह 5 वर्ष के ललए 15%
साधारण ब्याज पर ननवेश करता है और योजना B में वह 2 वर्ष के ललए 20% चक्रवद् र चध ब्याज पर
रालश ननवेश करता है । यदि िोनों योजनाओं से प्राप्त ब्याज का अंतर 5084 रुपये है , तो (2x +
700) का मान ज्ञात कीजजए।
(A) 32500 (B) 31900 (C) 28500
(D) 32400 (E) None of these

220. In a examination total (X + 40) students appeared. Out of them 40%


students scored 60% marks, 20% students scored marks between 60% to
80% and rest are scored above 80% marks. If 144 students scored above 80%
marks, then find the value of X.
एक परीक्षा में कुल (X + 40) छात्र उपजस्थत हुए। उनमें से 40% छात्रों ने 60% अंक प्राप्त ककए,
20% छात्रों ने 60% से 80% के बीच अंक प्राप्त ककए और शेर् 80% से अचधक अंक प्राप्त ककए।
यदि 144 छात्रों ने 80% से अचधक अंक प्राप्त ककए हैं, तो X का मान ज्ञात कीजजए।
(A) 380 (B) 340 (C) 360
(D) 320 (E) None of these

221. The ratio of the incomes of Ritu and Piya is 8 : 5 respectively. Ritu
spends 25% of his income on rent, 10% of rest on shopping and saves half of
the remaining. Piya spends 48.16% of his income and saves the rest. The
saving of Piya is how much percent more/less than that of Ritu?
ररतु और वपया की आय का अनुपात क्रमशः 8 : 5 है । ररतु अपनी आय का 25% ककराए पर, शेर् का
10% खरीिारी पर खचष करती है और शेर् का आधा बचाती है । वपया अपनी आय का 48.16% खचष
करती है और शेर् बचाती है । वपया की बचत, ररतु की बचत से ककतने प्रनतशत अचधक/कम है ?
(A) 4.2% (B) 4% (C) 4.6%
(D) 4.4% (E) None of these

222. 270 ml of mixture contains alcohol and soda in the ratio 7 : 11


respectively. If 25 ml of alcohol and 35 ml of soda is added into the mixture,
then what will be the ratio of soda to alcohol in the final mixture?
270 लमली लमश्ण में अल्कोहल और सोडा क्रमशः 7 : 11 के अनप
ु ात में है । यदि लमश्ण में 25
लमली अल्कोहल और 35 लमली सोडा लमलाया जाता है , तो अंनतम लमश्ण में सोडा का अल्कोहल से
अनुपात क्या होगा?
(A) 11 : 18 (B) 14 : 17 (C) 20 : 13
(D) 15 : 16 (E) None of these

223. Sum of the digits of a two digits number is 9. Ten’s digit of the number is
25% more than the unit’s digit. Find the difference of the digits of the number.

https :
//www. https : https
https : //instagra
//youtube.c
facebo m.com/aas :
om/channe hisharoraso
l/UCYa4_Jr ok.com cial(?) //t.m
Orf8R5Kz2u
/aashis utm_mediu
e/stu

117
OtccXQ m=
haroras copy_link
ocial dified
िो अंकों की एक संख्या के अंकों का योग 9 है । संख्या का िहाई का अंक इकाई के अंक से 25%
अचधक है । संख्या के अंकों का अंतर ज्ञात कीजजए।
(A) 3 (B) 4 (C) 2
(D) 1 (E) None of these

224. The present age of Rohan is x years and Sohan is 8x years. 21 years
hence from now the ratio of their ages will be 23 : 9 respectively. Find the
value of 8x.
रोहन की वतषमान आयु x वर्ष है और सोहन की आयु 8x वर्ष है । अब से 21 वर्ष बाि उनकी आयु का
अनुपात क्रमशः 23 : 9 होगा। 8x का मान ज्ञात कीजजए।
(A) 42 (B) 48 (C) 46
(D) 44 (E) None of these

225. Rs. 'P + 2000' when invested at 10% per annum, Rs. 'P + 2000' becomes
Rs. 3500 when it is invested at simple interest for 4 years. Find the interest
received when same sum is invested at 15% simple interest for 3 years.
'P + 2000' रुपये जब 10% प्रनत वर्ष पर ननवेश ककया जाता है , तो 'P + 2000' रुपये 3500 रुपये
हो जाता है जब इसे 4 साल के ललए साधारण ब्याज पर ननवेश ककया जाता है । प्राप्त ब्याज ज्ञात
कीजजए जब समान रालश को 3 वर्ष के ललए 15% साधारण ब्याज पर ननवेश ककया जाता है ।
(A) Rs. 1125 (B) Rs. 1089 (C) Rs. 1228
(D) Rs. 1336 (E) None of these

226. A shopkeeper sold an article for Rs. 13464 after giving two successive
discount X% and 15% respectively and marked the article 20% above the cost
price which is Rs. 22000, then find the value of ‘X’.
एक िकु ानिार ने क्रमशः िो क्रलमक छूट X% और 15% िे ने के बाि एक वस्तु को 13464 रुपये में
बेचा और वस्तु को लागत मूल्य से 20% अचधक अंककत ककया जो कक 22000 रुपये है , तो 'X' का
मान ज्ञात कीजजए।
(A) 40% (B) 20% (C) 30%
(D) 10% (E) None of these

227. Mihir can complete a piece of work in 50 days while Anil and Mihir
together can complete the same work in 30 days. Find the time taken by
Shubhi who is 25% more efficient than Anil to complete the whole work.
लमदहर एक काम को 50 दिनों में पूरा कर सकता है जबकक अननल और लमदहर लमलकर उसी काम
को 30 दिनों में परू ा कर सकते हैं। शभ
ु ी द्वारा ललया गया समय ज्ञात कीजजए जो परू े कायष को परू ा
करने में अननल से 25% अचधक कुशल है ।
(A) 64 days (B) 62 days (C) 60 days
(D) 66 days (E) None of these

https :
//www. https : https
https : //instagra
//youtube.c
facebo m.com/aas :
om/channe hisharoraso
l/UCYa4_Jr ok.com cial(?) //t.m
Orf8R5Kz2u
/aashis utm_mediu
e/stu

118
OtccXQ m=
haroras copy_link
ocial dified
228. Naman and Yash entered into a business with initial investment of Rs. X
and Rs. (X + 4000) and ratio of time for which they made their investment is 5
: 8 respectively. If profit share of Naman out of total profit of Rs. 1360 is Rs.
400, then find the value of X.
नमन और यश ने X रुपये और (X + 4000) रुपये के शुरुआती ननवेश के साथ एक व्यवसाय में
प्रवेश ककया और समय का अनप ु ात जजसके ललए उन्होंने अपना ननवेश क्रमशः 5 : 8 ककया। यदि
1360 रुपये के कुल लाभ में से नमन का लाभ दहस्सा 400 रुपये है , तो X का मान ज्ञात कीजजए।
(A) 6000 (B) 7000 (C) 8000
(D) 5000 (E) None of these

229. Rashi can make 3 rounds of square garden along its boundary in 3.2
minutes. Each side of the square garden is 24 meters. Find the time taken by
her to make 6 rounds of a circular field whose radius is 7 meters.
रालश 3.2 लमनट में वगाषकार बगीचे की सीमा के साथ-साथ 3 चक्कर लगा सकती है । वगाषकार
बगीचे की प्रत्येक भुजा 24 मीटर की है । 7 मीटर बत्रज्या वाले एक वत्त
र ाकार मैिान के 6 चक्कर
लगाने में उसके द्वारा ललया गया समय ज्ञात कीजजए।
(A) 152 sec (B) 176 sec (C) 148 sec
(D) 139 sec (E) None of these

230. A bag contain 8 Red, 9 Green and 12 Black Balls. 2 Balls are drawn
simultaneously from the bag. Find the probability of getting 1 Red and 1
Black Ball.
एक थैले में 8 लाल, 9 हरी और 12 काली गें िें हैं। थैले से एक साथ 2 गें िें ननकाली जाती हैं। 1 लाल
और 1 काली गें ि प्राप्त करने की प्रानयकता ज्ञात कीजजए।
(A) 42/209 (B) 44/201 (C) 46/205
(D) 48/203 (E) None of these

231. A container contains 896 kg of mixture of Rice flour and Wheat flour in
the ratio of 7 : 9 respectively. If 'X' kg of Rice flour and 'X + 88' kg of Wheat
flour is added into the mixture and the ratio of Rice flour to Wheat flour in the
final mixture is 2 : 3, then find the value of 'X + 88'.
एक कंटे नर में 896 ककग्रा चावल का आटा और गेहूं का आटा क्रमशः 7 : 9 के अनुपात में है । यदि
लमश्ण में 'X' ककलो चावल का आटा और 'X + 88' ककलो गेहूं का आटा लमलाया जाता है और
अंनतम लमश्ण में चावल के आटे का गेहूं के आटे से अनुपात 2 : 3 है , तो 'X + 88' का मान ज्ञात
कीजजए।
(A) 92 (B) 96 (C) 98
(D) 94 (E) None of these

232. If 15 females working 10 hours a day can make 9 articles in 12 days, then
in how many days 16 females working 7 hours per day will make 14 articles?

https :
//www. https : https
https : //instagra
//youtube.c
facebo m.com/aas :
om/channe hisharoraso
l/UCYa4_Jr ok.com cial(?) //t.m
Orf8R5Kz2u
/aashis utm_mediu
e/stu

119
OtccXQ m=
haroras copy_link
ocial dified
यदि 15 मदहलाएँ प्रनतदिन 10 घंटे कायष करके 12 दिन में 9 वस्तुएँ बना सकती हैं, तो 16 मदहलाएँ
प्रनतदिन 7 घंटे कायष करके 14 वस्तुएँ ककतने दिनों में बना लेंगी?
(A) 21 days (B) 23 days (C) 25 days
(D) 27 days (E) None of these

233. Average weight of 18 workers a factory 'X' kg. Three workers whose
weight is 66 kg, 62 kg and 83 kg respectively leave the factory and three more
workers whose weight is 85 kg, 78 kg and 84 kg respectively joined the
factory so the final average weight becomes 64 kg, then find the value of 'X'.
एक कारखाने में 18 श्लमकों का औसत भार 'X' ककग्रा. तीन श्लमक जजनका वजन क्रमशः 66
ककग्रा, 62 ककग्रा और 83 ककग्रा है , िैक्ट्री छोड िे ते हैं और तीन और श्लमक जजनका वजन क्रमशः
85 ककग्रा, 78 ककग्रा और 84 ककग्रा है , कारखाने में शालमल हो जाते हैं, इसललए अंनतम औसत वजन
64 ककग्रा हो जाता है , तो 'X' का मान ज्ञात कीजजए।
(A) 68 (B) 64 (C) 66
(D) 62 (E) None of these

234. Kanta bought an article for Rs.. 10750 from a shop at a discount of 14%
while Sonam bought same article from the same shop and paid Rs.. 8750.
Find the discount % given to Sonam.
कांता ने एक िक
ु ान से 14% की छूट पर 10750 रुपये में एक वस्तु खरीिी, जबकक सोनम ने उसी
िक
ु ान से समान वस्तु खरीिी और 8750 रुपये का भुगतान ककया। सोनम को िी गई छूट% ज्ञात
कीजजए।
(A) 20% (B) 30% (C) 50%
(D) 40% (E) None of these

235. Sum of the population of city 'P' and 'Q' is 1800000. Population of city 'P'
is 45% of the sum of population of city 'P' and 'Q'. If 48% of people in city 'Q'
are females, then find the number of males in city 'Q'.
शहर 'P' और 'Q' की जनसंख्या का योग 1800000 है । शहर 'P' की जनसंख्या शहर 'P' और 'Q'
की जनसंख्या के योग का 45% है । यदि शहर 'Q' में 48% लोग मदहलाएं हैं, तो शहर 'Q' में परु
ु र्ों
की संख्या ज्ञात कीजजए।
(A) 514800 (B) 512800 (C) 513800
(D) 515800 (E) None of these

236. Ranu spent 15.5% of her monthly income on education and 30% of the
remaining on rent. If amount saved by her after spending on education and
rent is Rs.. 70980, then find the monthly income of Ranu.
रानू ने अपनी मालसक आय का 15.5% लशक्षा पर और शेर् का 30% ककराए पर खचष ककया।
यदिलशक्षा और ककराए पर खचष करने के बाि उसके द्वारा बचाई गई रालश 70980 रुपये है , तो रानू
की मालसक आय ज्ञात कीजजए।

https :
//www. https : https
https : //instagra
//youtube.c
facebo m.com/aas :
om/channe hisharoraso
l/UCYa4_Jr ok.com cial(?) //t.m
Orf8R5Kz2u
/aashis utm_mediu
e/stu

120
OtccXQ m=
haroras copy_link
ocial dified
(A) Rs. 120000 (B) Rs. 110000 (C) Rs. 100000
(D) Rs. 130000 (E) None of these

237. Kartik runs from city X to city Y with a speed of 24 km/h and returns back
to city X from city Y with a speed of 16 km/h. If total distance travelled by him
is 61.44 km, then find the total time taken to complete the whole journey.
कानतषक शहर X से शहर Y तक 24 ककमी/घंटा की गनत से िौडता है और शहर Y से वापस शहर X
में 16 ककमी/घंटा की गनत से वापस आता है । यदि उसके द्वारा तय की गई कुल िरू ी 61.44 ककमी
है , तो पूरी यात्रा को पूरा करने में लगा कुल समय ज्ञात कीजजए।
(A) 4 hours 10 min (B) 6 hours 11 min (C) 3 hours 12 min
(D) 5 hours 13 min (E) None of these

238. Priti invested Rs.. 4200 at (R + 6)% p.a. simple interest while Rs.. 5400 at
(R + 9)% p.a. simple interest. If at the end of 4 years total amount received by
Priti is Rs.. 14856, then find the value of R.
प्रीनत ने 4200 रुपये का ननवेश (R + 6)% प्रनत वर्ष साधारण ब्याज पर 5400 रुपये (R + 9)%
प्रनत वर्ष साधारण ब्याज पर ककया। यदि 4 वर्ष के अंत में प्रीनत द्वारा प्राप्त कुल रालश 14856 रुपये
है , तो R का मान ज्ञात कीजजए।
(A) 5 (B) 7 (C) 6
(D) 8 (E) None of these

239. Ratio of speed of a boat in still water and speed of stream is 5 : 2


respectively. If a boat can travel 126 km in upstream as well as downstream in
20 hours, then find the time taken by the boat to cover 45 km in upstream and
63 km in downstream.
जस्थर जल में एक नाव की गनत और धारा की गनत का अनुपात क्रमशः 5 : 2 है । यदि एक नाव 20
घंटे में धारा के प्रनतकूल और धारा के अनक
ु ू ल 126 ककमी की यात्रा कर सकती है , तो नाव द्वारा
धारा के प्रनतकूल 45 ककमी और धारा के अनुकूल 63 ककमी की िरू ी तय करने में लगने वाला समय
ज्ञात कीजजए।
(A) 7 hours (B) 8 hours (C) 5 hours
(D) 6 hours (E) None of these

240. Area of a square is 400 cm2 and side of the square is equal to width of a
rectangle. If length of diagonal of the rectangle is 29 cm, then find the
perimeter of rectangle.
एक वगत का क्षेिफल 400 सेमी2 है और वगत की भुिा एक आयत की चौडाई के बराबर है ।
यदद आयत के ववकर्त की लम्बाई 29 सेमी है , तो आयत का पररमाप ज्ञात कीजिए।
(A) 84 cm (B) 81 cm (C) 83 cm
(D) 82 cm (E) None of these

https :
//www. https : https
https : //instagra
//youtube.c
facebo m.com/aas :
om/channe hisharoraso
l/UCYa4_Jr ok.com cial(?) //t.m
Orf8R5Kz2u
/aashis utm_mediu
e/stu

121
OtccXQ m=
haroras copy_link
ocial dified
241. An article is sold for Rs. 2160 after earning a profit of 20%. If marked
price of article is Rs. 600 more than its cost price, then find the discount
percentage.
एक वस्तु को 20% का लाभ अजजषत करने के बाि 2160 रुपये में बेचा जाता है । यदि वस्तु का
अंककत मूल्य इसके क्रय मूल्य से 600 रुपये अचधक है , तो छूट प्रनतशत ज्ञात कीजजए।
(A) 20% (B) 10% (C) 30%
(D) 40% (E) None of these

242. Rs. 7585 is divided among Sonu, Mohit and Mona such that (3/4) times of
share of Sonu is equal to (2/5) times of share of Mohit is equal to (3/7) times
share of Mona. Find the share of Mohit.
7585 रुपये को सोनू, मोदहत और मोना के बीच इस प्रकार बांटा गया है कक सोनू के दहस्से का (3/4)
गुना मोदहत के दहस्से के (2/5) गुना के बराबर है (3/7) गुणा मोना के दहस्से के बराबर है । मोदहत
का दहस्सा ज्ञात कीजजए।
(A) Rs. 3045 (B) Rs. 3065 (C) Rs. 3075
(D) Rs. 3055 (E) None of these

243. Rita and Mita alone can do a piece of work in 45 days and 30 days
respectively. The efficiency of Teena is 4/5th of the total efficiency of Rita and
Mita together. In how many days, Rita, Mita and Teena together can do the
whole work?
रीता और मीता अकेले एक काम को क्रमशः 45 दिन और 30 दिन में कर सकती हैं। टीना की
िक्षता, रीता और मीता की लमलाकर कुल िक्षता की 4/5 है । रीता, मीता और टीना लमलकर पूरा
काम ककतने दिनों में कर सकती हैं?
(A) 13 days (B) 12 days (C) 11 days
(D) 10 days (E) None of these

244. The ratio of the incomes of Rashi to Pawan is 8 : 11 and Rashi and
Pawan spend Rs. 2800 and Rs. 4000 respectively. Find the total income of
Rashi and Pawan, If saving of Pawan is Rs. 3600 more than that of Rashi.
रालश और पवन की आय का अनप ु ात 8 : 11 है और रालश और पवन क्रमशः 2800 रुपये और 4000
रुपये खचष करते हैं। रालश और पवन की कुल आय ज्ञात कीजजए, यदि पवन की बचत रालश की बचत
से 3600 रुपये अचधक है ।
(A) Rs. 30400 (B) Rs. 30400 (C) Rs. 30800
(D) Rs. 30600 (E) None of these

https :
//www. https : https
https : //instagra
//youtube.c
facebo m.com/aas :
om/channe hisharoraso
l/UCYa4_Jr ok.com cial(?) //t.m
Orf8R5Kz2u
/aashis utm_mediu
e/stu

122
OtccXQ m=
haroras copy_link
ocial dified
245. Present ages of A and B are in the ratio 5 : 4 respectively. After 20 years
the ratio of their ages becomes 15 : 13. Find the age of C who is 6 years
younger than B.
A और B की वतषमान आयु क्रमशः 5 : 4 के अनुपात में है । 20 वर्ष बाि उनकी आयु का अनुपात 15
: 13 हो जाता है । C की आयु ज्ञात कीजजए जो B से 6 वर्ष छोटा है ।
(A) 26 years (B) 28 years (C) 22 years
(D) 24 years (E) None of these

246. Pulkit sold his watch of cost Rs. 6800 at a profit of 15% to Neeta, who
spent Rs. 680 for repairing it and than sold it to Sonal at profit of Rs. ’X’ for
Rs. 11475. Find the value of ‘X’.
पलु ककत ने अपनी 6800 रुपये की लागत वाली घडी को 15% के लाभ पर नीता को बेच दिया,
जजसने इसकी मरम्मत के ललए 680 रुपये खचष ककए और सोनल को 'X' रुपये के लाभ पर 11475
रुपये में बेच दिया। 'X' का मान ज्ञात कीजजए।
(A) 2975 (B) 2756 (C) 2891
(D) 3012 (E) None of these

247. Compound interest on a certain sum of money for 2 years is Rs. 4536
while the simple interest on the same sum for same time period is Rs. 4200.
Find the rate of interest.
एक ननजश्चत रालश पर 2 वर्ों का चक्रवद्
र चध ब्याज 4536 रुपये है जबकक उसी रालश पर समान
समय अवचध के ललए साधारण ब्याज 4200 रुपये है । ब्याज की िर ज्ञात कीजजए।
(A) 19% (B) 18% (C) 16%
(D) 17% (E) None of these

248. A started a business with investment Rs. (X + 3000). After 4 months B


joined the business with Rs. (X + 9000). At the end of the year total profit
earned by them is Rs. 4750. If profit earned by A and B is Rs. Y and Rs. 'Y +
250', then find the initial investment of B.
A ने ननवेश (X + 3000) रुपये के साथ एक व्यवसाय शरू ु ककया। 4 महीने बाि B (X + 9000)
रुपये के साथ व्यापार में शालमल हो गया। वर्ष के अंत में उनके द्वारा अजजषत कुल लाभ 4750 रुपये
है । यदि A और B द्वारा अजजषत लाभ Y रुपये और 'Y + 250' रुपये है , तो B का प्रारं लभक ननवेश
ज्ञात कीजजए।
(A) Rs. 17000 (B) Rs. 16000 (C) Rs. 15000
(D) Rs. 14000 (E) None of these

249. A container contain (X + 50) ltr mixture of water and alcohol in the ratio
of 4 : 5. If 40 ltr alcohol is added to the mixture, then the ratio of water to
alcohol becomes 2 : 3. Find the value of X.

https :
//www. https : https
https : //instagra
//youtube.c
facebo m.com/aas :
om/channe hisharoraso
l/UCYa4_Jr ok.com cial(?) //t.m
Orf8R5Kz2u
/aashis utm_mediu
e/stu

123
OtccXQ m=
haroras copy_link
ocial dified
एक कंटे नर में (X + 50) लीटर पानी और अल्कोहल का लमश्ण 4 : 5 के अनुपात में है । यदि लमश्ण
में 40 लीटर अल्कोहल लमलाया जाता है , तो पानी का अल्कोहल से अनप
ु ात 2 : 3 हो जाता है । X
का मान ज्ञात कीजजए।
(A) 320 (B) 310 (C) 330
(D) 340 (E) None of these

250. Find the height of a cylinder whose volume is 2112 cm3 and radius is 8
cm.
एक बेलन की ऊँचाई ज्ञात कीजिए जिसका आयतन 2112 सेमी3 और त्रिज्या 8 सेमी है ।
(A) 12.4 cm (B) 13.9 cm (C) 11.7 cm
(D) 10.5 cm (E) None of these

251. Prateek invested Rs. (X + 2000) , Qamar invested Rs. (X + 3000) in a


partnership. If profit at the end of the year is Rs. 48,000 and value of Qamar’s
share is Rs. 28,000, then what is the value of X?
प्रतीक ने (X + 2000) रुपये का ननवेश ककया, कमर ने एक साझेिारी में (X + 3000) रुपये का
ननवेश ककया। यदि वर्ष के अंत में लाभ 48,000 रुपये है और कमर के दहस्से का मल्
ू य 28,000
रुपये है , तो X का मूल्य क्या है ?
(A) 2000 (B) 400 (C) 500
(D) 2300 (E) None of these

252. The ratio of simple interest on a sum at R% rate of interest per annum
for 6 years to the compound interest on 25% rate of interest, compounded
annually, for 2 years is 8 : 5 respectively. What is the value of R?
एक रालश पर 6 वर्ष के ललए R% वावर्षक ब्याज की िर से साधारण ब्याज का 25% वावर्षक
चक्रवद्
र चध ब्याज िर से 2 वर्ष के ललए चक्रवद्
र चध ब्याज का अनुपात क्रमशः 8 : 5 है । R का मान
क्या है ?
(A) 25 (B) 15 (C) 12
(D) 18 (E) None of these

253. A bag P contains some quantity of sugar. Bag Q contains 80 kg of salt.


20% of sugar from bag P is taken out and mixed with same quantity of salt
from bag Q. If the remaining quantity of salt in bag Q is 70% of its actual
quantity, then what is the initial quantity of sugar in bag P?
एक बैग P में कुछ मात्रा में चीनी है । बैग Q में 80 ककलो नमक है । बैग P से 20% चीनी ननकाल ली
जाती है और बैग Q से उतनी ही मात्रा में नमक लमलाया जाता है । यदि बैग Q में नमक की शेर्
मात्रा वास्तववक मात्रा का 70% है , तो बैग P में चीनी की प्रारं लभक मात्रा ककतनी है ?
(A) 120 kg (B) 150 kg (C) 110 kg
(D) 100 kg (E) None of these

https :
//www. https : https
https : //instagra
//youtube.c
facebo m.com/aas :
om/channe hisharoraso
l/UCYa4_Jr ok.com cial(?) //t.m
Orf8R5Kz2u
/aashis utm_mediu
e/stu

124
OtccXQ m=
haroras copy_link
ocial dified
254. Time taken to complete a work by B alone is 50% more than the time
taken by both B and C to complete the work. C is twice as efficient as A. A
and B together takes 15 days to complete the same work. C will take how
many days to complete the work?
अकेले B द्वारा ककसी कायष को परू ा करने में ललया गया समय B और C िोनों द्वारा कायष को परू ा
करने में ललए गए समय से 50% अचधक है । C, A से िोगुना कुशल है । A और B लमलकर समान
कायष को परू ा करने में 15 दिन लेते हैं। C कायष को परू ा करने में ककतने दिन लेगा?
(A) 40 days (B) 38 (2/3) days (C) 37 (1/2) days
(D) 36 days (E) None of these

255. 6 years ago, the ratio of the age of Chandan to that of Chaman was 9 : 7.
Chandan’s age after 6 years will be equal to the age of Chaman after 10 years.
Find the age of Chaman after 12 years.
6 वर्ष पहले, चंिन की आयु का चमन की आयु से अनुपात 9 : 7 था। 6 वर्ष बाि चंिन की आयु 10
वर्ष बाि चमन की आयु के बराबर होगी। 12 वर्ष बाि चमन की आयु ज्ञात कीजजए।
(A) 28 days (B) 32 days (C) 26 days
(D) 30 days (E) None of these

256. A train has 12 bogies. The length of each bogie of the train is 15 m and
the length of the engine of the train is 10 m. Distance between each bogie is 1
m and the distance between engine and the first bogie is 0.8 m. If the train is
moving with a speed of 70 m/sec, then how long it will take to cross a
platform of length 29.2 m?
एक ट्रे न में 12 डडब्बे होते हैं। ट्रे न की प्रत्येक डडब्बे की लंबाई 15 मीटर है और ट्रे न के इंजन की
लंबाई 10 मीटर है । प्रत्येक डडब्बे के बीच की िरू ी 1 मीटर है और इंजन और पहली डडब्बे के बीच की
िरू ी 0.8 मीटर है । यदि रे लगाडी 70 मीटर/सेकेण्ड की गनत से चल रही है , तो 29.2 मीटर लम्बे
प्लेटिामष को पार करने में उसे ककतना समय लगेगा?
(A) 4.2 sec (B) 1.8 sec (C) 3.3 sec
(D) 1. 0sec (E) None of these

257. A person’s monthly expenditure on house rent is 24(4/5)% of his savings


and his saving is 28(2/3)% of his monthly income. If his annual expenditure
on house rent is Rs. 31992, then find the annual income of the person.
मकान ककराए पर एक व्यजक्त का मालसक खचष उसकी बचत का 24(4/5)% है और उसकी बचत
उसकी मालसक आय का 28(2/3)% है । यदि मकान ककराए पर उसका वावर्षक व्यय 31992 रुपये है ,
तो व्यजक्त की वावर्षक आय ज्ञात कीजजए।
(A) 4.5 lakh (B) 2 lakh (C) 3.5 lakh
(D) 4.8 lakh (E) None of these

https :
//www. https : https
https : //instagra
//youtube.c
facebo m.com/aas :
om/channe hisharoraso
l/UCYa4_Jr ok.com cial(?) //t.m
Orf8R5Kz2u
/aashis utm_mediu
e/stu

125
OtccXQ m=
haroras copy_link
ocial dified
258. A solid metallic sphere of diameter 6 cm recast into a cylinder of
diameter 4 cm. Find the height of the cylinder.
6 सेमी व्यास वाले धातु के एक िोस गोले को 4 सेमी व्यास वाले एक बेलन में ढाला जाता है । बेलन
की ऊँचाई ज्ञात कीजजए।
(A) 5 cm (B) 9 cm (C) 6 cm
(D) 8 cm (E) None of these

259. A car and a bike travel from Delhi to Lucknow at a speed of 50 km/hr and
80 km/hr respectively. If bike takes 6 hours less than car to reach Lucknow,
then find the distance between Delhi and Lucknow.
एक कार और एक बाइक दिल्ली से लखनऊ की यात्रा क्रमशः 50 ककमी/घंटा और 80 ककमी/घंटा की
गनत से करती है । यदि लखनऊ पहुँचने में बाइक कार से 6 घंटे कम लेती है , तो दिल्ली और लखनऊ
के बीच की िरू ी ज्ञात कीजजए
(A) 500 km (B) 600 km (C) 800 km
(D) 400 km (E) None of these

260. A shopkeeper bought two items A and B together at Rs. 2500. He sold
the items together at Rs. 3025 and eared a profit of 15% and 25% on items A
and B respectively. Cost price of item A is Rs. ___ less than cost price of item
B.
एक िक ु ानिार ने िो वस्तुएं A और B को एक साथ 2500 रुपये में खरीिा। उसने उन वस्तुओं को
एक साथ 3025 रुपये में बेचा और वस्तुओं A और B पर क्रमशः 15% और 25% का लाभ अजजषत
ककया। वस्तु A का क्रय मल्ू य वस्तु B के क्रय मल्
ू य से ____ रुपये कम है ।
(A) 600 (B) 400 (C) 500
(D) 200 (E) None of these

261. Average of age of Renu and Rekha after 3 years will be 30 years and
average of age of Priya and Renu is 32 years. If average age of Srishti and
Priya before three years is 36.5 years and Rekha is presently 24 years old,
then what is the ratio of present ages Srishti and Renu?
3 वर्ष बाि रे नू और रे खा की औसत आयु 30 वर्ष होगी और वप्रया और रे नू की औसत आयु 32 वर्ष
होगी। यदि तीन वर्ष से पहले सजर ष्ट और वप्रया की औसत आयु 36.5 वर्ष है और रे खा वतषमान में 24
वर्ष की है , तो सजर ष्ट और रे णु की वतषमान आयु का अनप
ु ात ककतना है ?
(A) 4 : 3 (B) 2 : 3 (C) 3 : 2
(D) 5 : 1 (E) None of these

262. P alone started a business with investment of Rs. 8000 and after 'x' years
Q joins him with investment Rs. 1500 more than that of P. If at the end of 6
years, Out of total profit of Rs. 10750 share of Q is Rs. 4750, then what is the
value of 'x'?

https :
//www. https : https
https : //instagra
//youtube.c
facebo m.com/aas :
om/channe hisharoraso
l/UCYa4_Jr ok.com cial(?) //t.m
Orf8R5Kz2u
/aashis utm_mediu
e/stu

126
OtccXQ m=
haroras copy_link
ocial dified
P अकेले 8000 रुपये के ननवेश के साथ एक व्यवसाय शुरू करता है और 'x' वर्ों के बाि Q, P से
1500 रुपये अचधक ननवेश के साथ उसके साथ जड ु जाता है । यदि 6 वर्ष के अंत में , 10750 रुपये के
कुल लाभ में Q का दहस्सा 4750 रुपये है , तो 'x' का मान क्या है ?
(A) 1 (B) 2 (C) 3
(D) 4 (E) None of these

263. A certain sum is to be divided between two persons A and B. A received


15% more than the average sum received by both of them. If B received Rs.
2880 less than that of A, then what is the value of that certain sum?
एक ननजश्चत रालश को िो व्यजक्तयों A और B के बीच ववभाजजत ककया जाना है । A को उन िोनों
द्वारा प्राप्त औसत रालश से 15% अचधक प्राप्त होता है । यदि B को A से 2880 रुपये कम प्राप्त
होते हैं, तो उस ननजश्चत रालश का मल्
ू य क्या है ?
(A) Rs. 19200 (B) Rs. 14400 (C) Rs. 21600
(D) Rs. 26800 (E) None of these

264. Mohan appeared in SBI PO EXAM. When Mohan got 36% of the marks
then he just failed by 9 marks and also when Mohan scored 44% of the marks
then he scored 3 marks more than what was required to qualify. What are the
minimum marks that Mohan needs in order to qualify the exam?
मोहन SBI PO परीक्षा में उपजस्थत हुए। जब मोहन को 36% अंक प्राप्त हुए तो वह केवल 9 अंकों
से अनुत्तीणष हो गया और साथ ही जब मोहन ने 44% अंक प्राप्त ककए तो उसने अहषता प्राप्त करने
के ललए आवश्यक अंकों से 3 अंक अचधक प्राप्त ककए। परीक्षा उत्तीणष करने के ललए मोहन को
न्यूनतम ककतने अंक चादहए?
(A) 65 (B) 52 (C) 63
(D) 70 (E) None of these

265. Price of Kashmiri rice and Basmati rice is Rs. 95 and Rs. 75 respectively.
In what ratio two varieties of rice i.e. 'Kashmiri Rice' and 'Basmati Rice' are
mixed with each other, when the mixture of these two rice is sold at Rs. 100,
then there is a profit of 25%?
कश्मीरी चावल और बासमती चावल की कीमत क्रमशः 95 रुपये और 75 रुपये है । चावल की िो
ककस्मों अथाषत 'कश्मीरी चावल' और 'बासमती चावल' को ककस अनुपात में लमलाया जाता है , जब
इन िोनों चावलों के लमश्ण को 100 रुपये में बेचा जाता है , तो 25% का लाभ होता है ?
(A) 1 : 2 (B) 1 : 3 (C) 2 : 5
(D) 4 : 1 (E) None of these

266. Find the time taken by the boat to travel 64 km downstream and 40 km
upstream, If speed of the boat in still water is three times the speed of the
stream and the boat can travel 48 km downstream and 16 km upstream in 5
hours.

https :
//www. https : https
https : //instagra
//youtube.c
facebo m.com/aas :
om/channe hisharoraso
l/UCYa4_Jr ok.com cial(?) //t.m
Orf8R5Kz2u
/aashis utm_mediu
e/stu

127
OtccXQ m=
haroras copy_link
ocial dified
नाव द्वारा धारा के अनक ु ू ल 64 ककमी और धारा के प्रनतकूल 40 ककमी की िरू ी तय करने में लगने
वाला समय ज्ञात कीजजए, यदि जस्थर पानी में नाव की गनत धारा की गनत से तीन गुना है और नाव
5 घंटे में धारा के अनक
ु ू ल 48 ककमी और धारा के प्रनतकूल 16 ककमी की यात्रा कर सकती है ।
(A) 8 hours (B) 5 hours (C) 9 hours
(D) 6 hours (E) None of these

267. Garima, Prashant and Rinkesh can do a work in 30 days, 24 days and 20
days respectively. If Garima started the work and after three days Prashant
join Garima and after another two days Rinkesh join both, then in how many
days the whole work will be completed?
गररमा, प्रशांत और ररंकेश एक कायष को क्रमशः 30 दिन, 24 दिन और 20 दिन में कर सकते हैं।
यदि गररमा कायष शुरू करती है और तीन दिनों के बाि प्रशांत गररमा के साथ जुड जाता है और िो
दिनों के बाि ररंकेश िोनों के साथ जडु जाता है , तो परू ा कायष ककतने दिनों में परू ा होगा?
(A) 11 days (B) 9 days (C) 6 days
(D) 5 days (E) None of these

268. A rectangular sheet with dimensions 22 m × 10 m is rolled into a cylinder


so that the smaller side becomes the height of the cylinder. What is the
volume of the cylinder so formed?
22 मीटर × 10 मीटर ववमाओं वाली एक आयताकार शीट को एक बेलन में इस प्रकार लपेटा जाता
है कक छोटी भज
ु ा बेलन की ऊंचाई बन जाए। इस प्रकार बने बेलन का आयतन क्या है ?
(A) 165 cm 3 (B) 385 cm3 (C) 410 cm3
(D) 220 cm3 (E) None of these

269. Time taken by a boat to cover 76.8 km downstream is 4 hours 48 minutes


and ratio of speed of boat in still water to the speed of stream is 7 : 1, then
what is the upstream speed of boat?
एक नाव द्वारा धारा के अनक ु ू ल 76.8 ककमी की िरू ी तय करने में ललया गया समय 4 घंटे 48
लमनट है और जस्थर पानी में नाव की गनत का धारा की गनत से अनुपात 7 : 1 है , तो नाव की धारा
के प्रनतकूल गनत क्या है ?
(A) 12 km/hr (B) 9 km/hr (C) 10 km/hr
(D) 15 km/hr (E) None of these

270. Two units of an item is sold at Rs. 'x' and Rs. '2x' profit amount and total
profit percent earned after selling both the items is 36%, then what is the
value of 'x', If the cost price of each unit is Rs. 500?
ककसी वस्तु की िो इकाइयों को 'x' रुपये और '2x' रुपये की लाभ रालश पर बेचा जाता है और िोनों
वस्तओ
ु ं को बेचने के बाि अजजषत कुल लाभ प्रनतशत 36% है , तो 'x' का मल्
ू य क्या है , यदि प्रत्येक
इकाई का लागत मूल्य 500 रुपये है ?

https :
//www. https : https
https : //instagra
//youtube.c
facebo m.com/aas :
om/channe hisharoraso
l/UCYa4_Jr ok.com cial(?) //t.m
Orf8R5Kz2u
/aashis utm_mediu
e/stu

128
OtccXQ m=
haroras copy_link
ocial dified
(A) 100 (B) 90 (C) 120
(D) 150 (E) None of these

271. Ankita has 1200 saris, some saris sells at 10% profit and the rest at 20%
profit. If her overall profit percentage is 16%, then how many saris she sold at
20% profit?
अंककता के पास 1200 साडडयाँ हैं, कुछ साडडयाँ 10% लाभ पर और शेर् 20% लाभ पर बेचती हैं।
यदि उसका कुल लाभ प्रनतशत 16% है , तो उसने 20% लाभ पर ककतनी साडडयाँ बेचीं?
(A) 640 (B) 360 (C) 720
(D) 540 (E) None of these

272. P and Q invested their amounts in the ratio 3 : 4, Q and R invested their
amount in the ratio 6 : 7 and R and S invested their amounts in the ratio 14 :
15. If the profit at the end of the year is Rs. 75,000, then what is the value of
R’s share?
P और Q ने अपनी रालश को 3 : 4 के अनुपात में ननवेश ककया, Q और R ने अपनी रालश को 6 : 7
के अनुपात में ननवेश ककया और R और S ने अपनी रालश को 14 : 15 के अनुपात में ननवेश ककया।
यदि वर्ष के अंत में लाभ 75,000 रुपये है , तो R के दहस्से का मल्
ू य क्या है ?
(A) Rs. 20,000 (B) Rs. 21,000 (C) Rs. 22,000
(D) Rs, 23,000 (E) None of these

273. A mixture contains 45% milk and remaining water. When 75 liters milk is
added to the mixture, then the ratio of milk and water becomes 3 : 2
respectively. What is the initial quantity of milk in the mixture?
एक लमश्ण में 45% िधू और बचा हुआ पानी है । जब लमश्ण में 75 लीटर िध ू लमला दिया जाता है ,
तो िधू और पानी का अनुपात क्रमशः 3 : 2 हो जाता है । लमश्ण में िध
ू की आरं लभक मात्रा ककतनी
है ?
(A) 90 liters (B) 84 liters (C) 60 liters
(D) 108 liters (E) None of these

274. The difference between the simple interest earned for 3 years and the
compound interest earned for 2 years on the same principal amount when
both are invested at the same rate of interest of 20% per annum, is Rs. 1,120.
What is the principal amount?
3 वर्ष के ललए अजजषत साधारण ब्याज और समान मल ू रालश पर 2 वर्ष के ललए अजजषत चक्रवद्
र चध
ब्याज के बीच का अंतर, जब िोनों को 20% प्रनत वर्ष की समान ब्याज िर पर ननवेश ककया जाता
है , 1,120 रुपये है । मूल रालश क्या है ?
(A) Rs. 8000 (B) Rs, 5000 (C) Rs. 7000
(D) Rs. 3000 (E) None of these

https :
//www. https : https
https : //instagra
//youtube.c
facebo m.com/aas :
om/channe hisharoraso
l/UCYa4_Jr ok.com cial(?) //t.m
Orf8R5Kz2u
/aashis utm_mediu
e/stu

129
OtccXQ m=
haroras copy_link
ocial dified
275. Average marks obtained by A, B and C in a Physics is 67 and average
marks obtained by B, C and D in same subject is 73. If the sum of marks
obtained by A and D is 158, then how many marks obtained by A?
एक भौनतकी में A, B और C द्वारा प्राप्त औसत अंक 67 हैं और B, C और D द्वारा समान ववर्य
में प्राप्त औसत अंक 73 हैं। यदि A और D द्वारा प्राप्त अंकों का योग 158 है , तो A द्वारा प्राप्त
अंकों का योग ककतना है ?
(A) 76 (B) 70 (C) 72
(D) 60 (E) None of these

276. The distance cover by a boat in upstream in 4 hours is 11 km less than


distance covered by the boat in downstream in 3 hours. If the speed of the
stream is 3 km/ hr, then in how much time it will cover 75 km in still water?
एक नाव द्वारा धारा के प्रनतकूल 4 घंटे में तय की गई िरू ी, नाव द्वारा धारा के अनुकूल 3 घंटे में
तय की गई िरू ी से 11 ककमी कम है । यदि धारा की गनत 3 ककमी/घंटा है , तो शांत जल में वह ककतने
समय में 75 ककमी की िरू ी तय करे गी?
(A) 6.5 hours (B) 5.5 hours (C) 7.5 hours
(D) 6 hours (E) None of these

277. Find the total surface area of a rectangular parallelepiped body whose
sum of length, breadth and height is 25 cm and length of body diagonal is 15
cm.
एक आयताकार समांतर चतुभज ुष ननकाय का कुल सतह क्षेत्र ज्ञात कीजजए, जजसकी लंबाई, चौडाई
और ऊंचाई का योग 25 सेमी है और शरीर के ववकणष की लंबाई 15 सेमी है ।
(A) 400 cm2 (B) 800 cm2 (C) 500 cm2
(D) 200 cm 2 (E) None of these

278. Population (male, female and children) of a town is 75000 out of which
children are 20% and ratio of male to female is 3 : 2. If out of total population
40% are illiterate and ratio of illiterate male, female and children is 27 : 13 :
10, then what is the number of literate female population in the town?
एक कस्बे की जनसंख्या (परु
ु र्, मदहला और बच्चे) 75000 है , जजसमें से बच्चे 20% हैं और परु ु र्ों
का मदहलाओं से अनुपात 3 : 2 है । यदि कुल जनसंख्या में से 40% ननरक्षर हैं और ननरक्षर पुरुर्,
मदहला और बच्चों का अनुपात 27 : 13 : 10 है , तो शहर में साक्षर मदहला जनसंख्या की संख्या
ककतनी है ?
(A) 15800 (B) 16200 (C) 15500
(D) 18500 (E) None of these

279. After 5 years, sum of age of Jitu and Tushar is 8 years less than the sum
of age of Mahesh and Shital at that time and Tushar is 8 years younger than
Mahesh. If the ratio of present age of Jitu and Mahesh is 4 : 5 and present age

https :
//www. https : https
https : //instagra
//youtube.c
facebo m.com/aas :
om/channe hisharoraso
l/UCYa4_Jr ok.com cial(?) //t.m
Orf8R5Kz2u
/aashis utm_mediu
e/stu

130
OtccXQ m=
haroras copy_link
ocial dified
of Shital is 40 years, then what is the present age of Tushar?
5 वर्ष बाि, जीतू और तर्
ु ार की आयु का योग उस समय महे श और शीतल की आयु के योग से 8
वर्ष कम है और तुर्ार महे श से 8 वर्ष छोटा है । यदि जीतू और महे श की वतषमान आयु का अनुपात 4
: 5 है और शीतल की वतषमान आयु 40 वर्ष है , तो तर् ु ार की वतषमान आयु ककतनी है ?
(A) 42 years (B) 40 years (C) 18 years
(D) 54 years (E) None of these

280. To complete a piece of work, A has worked alone for 4 hours and B has
worked alone for some amount of time. If A and B together can complete the
work in 8 hours and A alone can complete the work in 12 hours, then how
many hours B actually worked for?
एक कायष को परू ा करने के ललए, A ने 4 घंटे अकेले कायष ककया है और B ने कुछ समय के ललए
अकेले कायष ककया है । यदि A और B लमलकर उस काम को 8 घंटे में पूरा कर सकते हैं और A
अकेला उस काम को 12 घंटे में परू ा कर सकता है , तो B ने वास्तव में ककतने घंटे काम ककया?
(A) 8 hours (B) 12 hours (C) 16 hours
(D) 10 hours (E) None of these

281. A is 20% more efficient than B and A alone can complete the whole work
in 30 days. If A, B and C together can complete the work in 9(27/37) days,
then in how many days C alone can complete the work?
A, B से 20% अचधक कुशल है और A अकेले पूरे कायष को 30 दिनों में पूरा कर सकता है । यदि A,
B और C लमलकर उस कायष को 9(27/37) दिनों में परू ा कर सकते हैं, तो C अकेला उस कायष को
ककतने दिनों में पूरा कर सकता है ?
(A) 20 days (B) 36 days (C) 24 days
(D) 25 days (E) None of these

282. Pawan, Shikha and Suresh started a business with their investments in
the ratio 7 : 6 : 8 respectively. After 3 years, Shikha doubles his investment.
What will be the difference between profit share of Shikha and Suresh, If
profit share of Pawan at the end of 5 years is Rs. 12250?
पवन, लशखा और सुरेश ने क्रमशः 7 : 6 : 8 के अनुपात में अपने ननवेश के साथ एक व्यवसाय शुरू
ककया। 3 साल बाि लशखा अपने ननवेश को िोगन ु ा कर िे ती है । यदि 5 वर्ष के अंत में पवन का लाभ
दहस्सा 12250 रुपये है , तो लशखा और सुरेश के लाभ में ककतना अंतर होगा?
(A) Rs. 500 (B) Rs. 700 (C) Rs. 600
(D) Rs. 800 (E) None of these

283. Ranjit sold his bike at a loss of 40%. If he had sold it for Rs. 25,000 more,
he would have made a percentage profit of 10%. What is the actual selling
price of his bike?

https :
//www. https : https
https : //instagra
//youtube.c
facebo m.com/aas :
om/channe hisharoraso
l/UCYa4_Jr ok.com cial(?) //t.m
Orf8R5Kz2u
/aashis utm_mediu
e/stu

131
OtccXQ m=
haroras copy_link
ocial dified
रं जीत ने अपनी बाइक 40% की हानन पर बेच िी। यदि उसने इसे 25,000 रुपये अचधक में बेचा
होता, तो उसे 10% का प्रनतशत लाभ होता। उसकी बाइक का वास्तववक ववक्रय मूल्य क्या है ?
(A) Rs. 30000 (B) Rs. 32000 (C) Rs. 25000
(D) Rs. 45000 (E) None of these

284. Ratio of the income of Kusal, Priya and Vivek is 2 : 3 : 4. Kusal saves one
- fifth of his income and ratio of expenditure of Kusal, Priya and Vivek is 3 : 4
: 5. What is the ratio of their saving?
कुसल, वप्रया और वववेक की आय का अनप ु ात 2 : 3 : 4 है । कुसल अपनी आय का एक - पांचवां
दहस्सा बचाता है और कुसल, वप्रया और वववेक के व्यय का अनुपात 3 : 4 : 5 है । उनकी बचत का
अनप ु ात क्या है ?
(A) 5 : 12 : 13 (B) 6 : 13 : 12
(C) 6 : 13 : 20
(D) 6 : 7 : 13 (E) None of these

285. In a college 500 students, ratio of girls to boys is 2 : 3 and ratio of


number of boys passed to number of boys failed is 5 : 1. If total 30 girls got
failed, then what is the ratio of number of boys who passed to the total
students?
एक कॉलेज में 500 छात्र हैं, लडककयों का लडकों से अनुपात 2 : 3 है और उत्तीणष लडकों की संख्या
का अनत्तु ीणष लडकों की संख्या से अनपु ात 5 : 1 है । यदि कुल 30 लडककयां अनत्त
ु ीणष हो जाती हैं, तो
उत्तीणष होने वाले लडकों की संख्या का कुल छात्रों से अनुपात ककतना है ?
(A) 3 : 2 (B) 1 : 2 (C) 2 : 5
(D) 7 : 5 (E) None of these

286. What is the normal time taken by Rashmi (in minutes) to reach her final
destination, If it is known that when she reduces her speed by 2/5th, then she
reaches her destination 24 minutes late?
रजश्म द्वारा अपने अंनतम गंतव्य तक पहुँचने में ललया जाने वाला सामान्य समय (लमनटों में )
ककतना है , यदि यह ज्ञात है कक जब वह अपनी गनत 2/5 कम करती है , तो वह 24 लमनट िे र से
अपने गंतव्य पर पहुँचती है ?
(A) 14 min (B) 10 min (C) 16 min
(D) 15 min (E) None of these

287. A and B started a business by investing Rs. 6440 and Rs. 5750
respectively and at the end of 12 months, the ratio of the shares of profit
received by A and B was 14 : 15 respectively. For how many months, A was
not in the business? (consider B invested for 12 months).
A और B ने क्रमशः 6440 रुपये और 5750 रुपये का ननवेश करके एक व्यवसाय शरू ु ककया और
12 महीने के अंत में , A और B द्वारा प्राप्त लाभ के शेयरों का अनुपात क्रमशः 14 : 15

https :
//www. https : https
https : //instagra
//youtube.c
facebo m.com/aas :
om/channe hisharoraso
l/UCYa4_Jr ok.com cial(?) //t.m
Orf8R5Kz2u
/aashis utm_mediu
e/stu

132
OtccXQ m=
haroras copy_link
ocial dified
था। A ककतने महीनों से व्यवसाय में नहीं था? (मान लें कक B ने 12 महीने के ललए ननवेश ककया है )

(A) 2 months (B) 9 months (C) 6 months
(D) 8 months (E) None of these

288. Mixture A contains 72 liters of milk and water. 3 liters of milk and 13
liters of water are added into mixture A. The milk in final mixture in A is 20%
more than that of water in final mixture. Find the ratio of milk and water in
initial mixture.
लमश्ण A में 72 लीटर िध ू और पानी है । लमश्ण A में 3 लीटर िधू और 13 लीटर पानी लमलाया
जाता है । A में अंनतम लमश्ण में िधू अंनतम लमश्ण में पानी की तुलना में 20% अचधक है ।
आरं लभक लमश्ण में िध ू और पानी का अनप ु ात ज्ञात कीजजए।
(A) 5 : 4 (B) 5 : 3 (C) 5 : 2
(D) 7 : 3 (E) None of these

289. The ratio of P’s age after 6 years and Q’s present age is 10 : 7 and the
ratio of P’s age 6 years ago and R’s age three years ago is 3 : 4. If Q is 3 years
younger than P, then what is the sum of age of P and Q?
6 वर्ष बाि P की आयु और Q की वतषमान आयु का अनुपात 10 : 7 है और 6 वर्ष पहले P की आयु
और तीन वर्ष पहले R की आयु का अनप ु ात 3 : 4 है । यदि Q, P से 3 वर्ष छोटा है , तो P और Q की
आयु का योग ककतना है ?
(A) 45 years (B) 50 years (C) 60 years
(D) 30 years (E) None of these

290. A sum of money becomes two times in 8 years at simple interest and it
becomes four times in 2 years at compound interest. Find the ratio of rate of
simple interest to rate of compound interest.
कोई धनरालश साधारण ब्याज पर 8 वर्ष में िगु ुनी हो जाती है और चक्रवद्
र चध ब्याज पर 2 वर्ष में
चार गुनी हो जाती है । साधारण ब्याज की िर का चक्रवद्
र चध ब्याज की िर से अनुपात ज्ञात कीजजए।
(A) 5 : 3 (B) 6 : 1 (C) 1 : 8
(D) 4 : 5 (E) None of these

291. A and B start a business with total capital of Rs. 9000. If their capitals are
interchanged, A would have received 50% more than of what he received out
of the total profit. What was the capital invested by B?
A और B 9000 रुपये की कुल पूंजी के साथ एक व्यवसाय शुरू करते हैं। यदि उनकी पूंजी आपस में
बिल िी जाती है , तो A को कुल लाभ से 50% अचधक प्राप्त होता है । B द्वारा ननवेश की गई पंज
ू ी
क्या थी?
(A) Rs. 5500 (B) Rs. 5000 (C) Rs. 5400
(D) Rs. 6500 (E) None of these

https :
//www. https : https
https : //instagra
//youtube.c
facebo m.com/aas :
om/channe hisharoraso
l/UCYa4_Jr ok.com cial(?) //t.m
Orf8R5Kz2u
/aashis utm_mediu
e/stu

133
OtccXQ m=
haroras copy_link
ocial dified
292. Initial price of a movie ticket was Rs. 250 and after sale of 82% tickets,
the price per ticket was increased by 12%. If total number of ticket sold was
450, then find the total earnings on ticket sold after increment in price.
एक मूवी दटकट की शुरुआती कीमत 250 रुपये थी और 82% दटकटों की बबक्री के बाि, प्रनत दटकट
कीमत में 12% की वद्र चध हुई। यदि बेचे गए दटकटों की कुल संख्या 450 थी, तो कीमत में वद्
र चध के
बाि बेचे गए दटकटों पर कुल आय ज्ञात कीजजए।
(A) Rs. 22400 (B) Rs. 22680 (C) Rs. 22960
(D) Rs. 22120 (E) None of these

293. Ram distributed 60% of his monthly income among A, B and C such that
A got Rs. 220 less than amount received by C and the respective ratio of
amount received by A and B is 3 : 4. If B got Rs. 7280, then find monthly
income of Ram.
राम ने अपनी मालसक आय का 60% A, B और C के बीच इस प्रकार ववतररत ककया कक A को C
द्वारा प्राप्त रालश से 220 रुपये कम लमले और A और B द्वारा प्राप्त रालश का संबंचधत अनप
ु ात 3
: 4 है यदि B को 7280 रुपये लमलते हैं, तो राम की मालसक आय ज्ञात कीजजए।
(A) Rs. 30700 (B) Rs. 32500 (C) Rs. 30000
(D) Rs. 12000 (E) None of these

294. A chemist mixed two chemicals L and Q. 960 ml mixture of chemical L


and chemical Q in the ratio of 7 : 3 mixed with 480 ml mixture of chemical L
and chemical Q in the ratio 13 : 7, then what will be the ratio of chemical L
and chemical Q in the final mixture?
एक रसायनज्ञ ने िो रसायनों L और Q को लमलाया। रासायननक L और रासायननक Q के 960
लमली लमश्ण को 7 : 3 के अनप ु ात में रासायननक L और रासायननक Q के 480 लमली लमश्ण को
13 : 7 के अनुपात में लमलाया, तो अंनतम लमश्ण में रासायननक L और रासायननक Q का अनुपात
क्या होगा?
(A) 45 : 16 (B) 43 : 17 (C) 41 : 19
(D) 44 : 17 (E) None of these

295. A shopkeeper earned a profit equal to cost price of 6 items while selling
30 items together. Each item is marked at 25% above the cost price and Rs.
28 discount is given on each item. What is the selling price of each item?
एक िक ु ानिार ने 30 वस्तुओं को एक साथ बेचकर 6 वस्तुओं के लागत मूल्य के बराबर लाभ
अजजषत ककया। प्रत्येक वस्तु पर लागत मल् ू य से 25% अचधक मल् ू य अंककत है और प्रत्येक वस्तु पर
28 रुपये की छूट िी जाती है । प्रत्येक वस्तु का ववक्रय मूल्य क्या है ?
(A) Rs. 696 (B) Rs. 672 (C) Rs. 636
(D) Rs. 618 (E) None of these

https :
//www. https : https
https : //instagra
//youtube.c
facebo m.com/aas :
om/channe hisharoraso
l/UCYa4_Jr ok.com cial(?) //t.m
Orf8R5Kz2u
/aashis utm_mediu
e/stu

134
OtccXQ m=
haroras copy_link
ocial dified
296. The amount received on a certain sum of money at rate of 15% per
annum compounded interest for two years is Rs. 8620 less than the amount
received on same sum of money at rate of 20% per annum simple interest for
7 years. What is the sum of money?
एक ननजश्चत रालश पर 15% प्रनत वर्ष की िर से िो वर्ों के ललए चक्रवद्
र चध ब्याज पर प्राप्त रालश
समान रालश पर 7 वर्ों के ललए साधारण ब्याज पर 20% प्रनत वर्ष की िर से प्राप्त रालश से 8620
रुपये कम है । धन रालश क्या है ?
(A) Rs. 6000 (B) Rs. 5000 (C) Rs. 8000
(D) Rs. 70000 (E) None of these

297. Kabir and Ronak together can complete the whole work in 16 days and
Ronak and Mehta together can complete the whole work in 24 days. If Ronak
alone can complete the whole work in 48 days, then how much part of the
work done by Kabir and Mehta together in 12 days?
कबीर और रौनक लमलकर परू े काम को 16 दिनों में परू ा कर सकते हैं और रौनक और मेहता
लमलकर पूरे काम को 24 दिनों में पूरा कर सकते हैं। यदि रौनक अकेले पूरे कायष को 48 दिनों में पूरा
कर सकता है , तो कबीर और मेहता द्वारा लमलकर 12 दिनों में ककए गए कायष का ककतना भाग है ?
(A) 3/4 (B) 4/5 (C) 7/8
(D) 4/7 (E) None of these

298. In a school there are total 160 students and average marks of the class
in a test is 75. Average marks of 50 students of the class is 80 while average
marks of 60 other students is 40, then what is the average marks obtained by
remaining 50 students of the class?
एक स्कूल में कुल 160 छात्र हैं और एक परीक्षा में कक्षा के औसत अंक 75 हैं। कक्षा के 50 छात्रों के
औसत अंक 80 हैं जबकक अन्य 60 छात्रों के औसत अंक 40 हैं, तो कक्षा के शेर् 50 छात्रों द्वारा
प्राप्त औसत अंक ककतने हैं?
(A) 122 marks (B) 112 marks (C) 116 marks
(D) 120 marks (E) None of these

299. A boat can cover 120 km upstream and 165 km downstream in 39 hours
and it can cover 90 km upstream and 132 km downstream in 30 hours. In how
much time the boat can cover 220 km downstream and 80 km in upstream?
एक नाव धारा के प्रनतकूल 120 ककमी और धारा के अनुकूल 165 ककमी की िरू ी 39 घंटे में तय कर
सकती है और यह 90 ककमी धारा के प्रनतकूल और 132 ककमी धारा के अनक ु ू ल 30 घंटे में तय कर
सकती है । ककतने समय में नाव धारा के अनुकूल 220 ककमी और धारा के प्रनतकूल 80 ककमी की
िरू ी तय कर सकती है ?
(A) 36 hours (B) 25 hours (C) 16 hours
(D) 30 hours (E) None of these

https :
//www. https : https
https : //instagra
//youtube.c
facebo m.com/aas :
om/channe hisharoraso
l/UCYa4_Jr ok.com cial(?) //t.m
Orf8R5Kz2u
/aashis utm_mediu
e/stu

135
OtccXQ m=
haroras copy_link
ocial dified
300. Amit got 81% marks in an exam and Nitin got 72% marks in the same
exam. If the difference between the marks obtained by Amit and Nitin is 117,
then what is the marks obtained by Nitin in the exam?
अलमत ने एक परीक्षा में 81% अंक प्राप्त ककए और उसी परीक्षा में नननतन ने 72% अंक प्राप्त
ककए। यदि अलमत और नननतन द्वारा प्राप्त अंकों के बीच का अंतर 117 है , तो परीक्षा में नननतन
द्वारा प्राप्त अंक ककतने हैं
(A) 962 marks (B) 360 marks (C) 936 marks
(D) 840 marks (E) None of these

https :
//www. https : https
https : //instagra
//youtube.c
facebo m.com/aas :
om/channe hisharoraso
l/UCYa4_Jr ok.com cial(?) //t.m
Orf8R5Kz2u
/aashis utm_mediu
e/stu

136
OtccXQ m=
haroras copy_link
ocial dified
1. Ans. (B) 6. Ans. (A)
(16500 × X) /{15600 × (X + 2) } = Largest even number = 42 + 2 × 2 =
(17280 - 9360) /9360 42 + 4 = 46
(55 × X) /{52 × (X + 2) } = 7920/9360 Largest odd number = 252/6 + 2 × (6
X/(X + 2) = 11/13 × 52/55 = 4/5 - 1) /2 = 42 + 5 = 47
5x = 4x + 8 Largest number = 47
X=8
7. Ans. (C)
2. Ans. (C) CP of Bag = X/(1 + 5/7) = Rs. 7X/12
2. Mixture X : Mixture Y = (50 - 38) : CP of Book = 7X/12 × 5/7 = Rs, 5X/12
(78 - 50) (5X/12 + 120) /(7X/12 + 240) = 3/5
= 12 : 28 = 3 : 7 25X/12 + 600 = 21X/12 + 720
4X/12 = 120
3. Ans. (D) X = 360
X + 15 = 1080/24
X + 15 = 45 8. Ans. (C)
X = 30 At present, Age of Divya = (7x - 6)
Increase in Speed = 4/3 × 30 = 40% years
Distance travelled in 22 hours = 45 × Age of Priya = (6x - 6) years
1.4 × 22 = 1386 km (7x - 6 - 3) + (6x - 6 - 3) = 60
13x - 18 = 60
4. Ans. (B) 13x = 78
CP of Article = 840/1.2 = Rs. 700 x=6
MP of Article = 840/(0.8 × 6/7) = Age of Priya = 6 × 6 - 6 = 36 - 6 = 30
980/0.8 = Rs. 1225 years
Sum of CP and MP = 700 + 1225 =
Rs. 1925 9. Ans. (B)
A ➔ 35 9
5. Ans. (A) } 315 {
Interest in 1 year = (10496 - 7936) /5 B ➔ 63 5
= 2560/5 = Rs. 512 B empty the tank in = (25 × 9) /5 = 45
Principal = 7936 - 512 × 3 = 7936 - hours
1536 = Rs. 6400
R = (512 × 100) /6400 = 8

https :
//www. https : https
https : //instagra
//youtube.c
facebo m.com/aas :
om/channe hisharoraso
l/UCYa4_Jr ok.com cial(?) //t.m
Orf8R5Kz2u
/aashis utm_mediu
e/stu

137
OtccXQ m=
haroras copy_link
ocial dified
10. Ans. (D) 15. Ans. (A)
Downstream Speed : Upstream R = (14630 - 9500) × 100/(9500 × 3)
Speed = 3 : 1 R = 5130/(95 × 3) = 18
Downstream Speed = 3x km/hr R + 4 = 18 + 4 = 22
Upstream Speed = x km/hr Interest = 9500 × 0.22 × 3 = Rs. 6270
3x + x = 2 × 20
4x = 40 16. Ans. (A)
x = 10 Profit Ratio, Arohi : Rohil
Downstream Speed = 3 × 10 = 30 = (8000 × 12) : (6500 × 9) = 64 : 39
km/hr Profit Share of Rohil = 3200 × 39/64 =
Rs. 1950
11. Ans. (B)
Difference between Income of 17. Ans. (C)
Kashvi& Pratik At present, Age of Vinod = 3 × 18/(19
= 3 × (66000 - 52000) = 3 × 14000 = - 18) + 2
Rs. 42000 = 3 × 18 + 2 = 54 + 2 = 56 years

12. Ans. (C) 18. Ans. (C)


Y × 1.09 + (Y + 12000) × 0.86 = 68820 In final mixture, Juice : Water
1.09Y + 0.86Y + 10320 = 68820 = {(1200 - 80) × 3/7 + 80} : (1200 - 80)
1.95Y = 58500 × 4/7
Y = 30000 = (1120 × 3/7 + 80) : (1120 × 4/7)
Y + 12000 = 30000 + 12000 = 42000 = (480 + 80) : 640 = 560 : 640 = 7 : 8

13. Ans. (D) 19. Ans. (B)


Cost of Wire = 2 × (32 + 17) × 4.25 Upstream Speed = 72/18 = 4 km/hr
= 8.5 × 49 = Rs. 416.5 Speed of Boat in Still water = 5x
km/hr
14. Ans. (B) Speed of Steam = x km/hr
Speed of Car = 7x km/hr Upstream Speed : Downstream
Speed of Bike = 7x × 2/7 = 2x km/hr Speed
3/7x + 2/2x = 5/55 = (5 - 1) : (5 + 1) = 4 : 6 = 2 : 3
10/7x = 1/11 Downstream Speed = 4 × 3/2 = 6
7x = 110 km/hr km/hr
Time taken in Downstream = 78/6 =
13 hours

https :
//www. https : https
https : //instagra
//youtube.c
facebo m.com/aas :
om/channe hisharoraso
l/UCYa4_Jr ok.com cial(?) //t.m
Orf8R5Kz2u
/aashis utm_mediu
e/stu

138
OtccXQ m=
haroras copy_link
ocial dified
20. Ans. (D) CP of Article = 455/(1.8 - 1.15) =
Efficiency of Sagar = 5 455/0.65 = Rs. 700
Efficiency of Rohini = 5 × 1.2 = 6 SP of Article = 700 × 1.15 = Rs. 805
Time taken by Rohini : Time taken
by Sagar = 1/5 : 1/6 = 6 : 5 25. Ans. (A)
Required % = (6 - 5) /5 × 100 = 20% R is less than S by = (0.45 - 0.2) /0.45
× 100 = 55.55%
21. Ans. (B)
P ➔ 30 4 26. Ans. (A)
} 120 { Highest marks scored = 28 × 6 - 20 ×
R ➔ 40 3 4 - 18
Efficiency of Q = 4 × 1.2 = 4.8 = 168 - 80 - 18 = 168 - 98 = 70
Together they all complete the work
in 27. Ans. (C)
= 120/(4 + 4.8 + 3) = 120/11.8 = 7500 × R/100 × 4 + 9000 × (R + 3) /100
10(10/59) days × 4 = 6360
75R + 90(R + 3) = 1590
22. Ans. (C) 75R + 90R + 270 = 1590
Speed of Boat in still water = 40 165R = 1320
km/hr R=8
Speed of Stream = 40 × (9 - 7) /(9 + 7)
= 40 × 2/16 = 5 km/hr 28. Ans. (C)
Upstream Speed = 45 - 5 = 35 km/hr X is elder to Y by = 8 + 4 = 12 years
Time taken to cover 420 km in A present, Age of Y = 12 × 5/(7 - 5) -
reduced upstream speed 10
= 420/(35 × 0.8) = 420/28 = 15 hours = 60/2 - 10 = 30 - 10 = 20 years
Age of Z = 20 + 4 = 24 years
23. Ans. (D) Age of Y : Age of Z = 20 : 24 = 5 : 6
Distance between A & B = 81 ×
(1.375 + 1) /0.375 29. Ans. (B)
= 216 × 2.375 = 513 km Investment of Akshay = 1800 × 12 +
400 × (12 - X - 3)
24. Ans. (B) = 21600 + 400(9 - X) = 21600 + 3600 -
400X = 400(63 - X)
Investment of Manshi = 6525 × (12 -
X - 3) = 6525(9 - X)

https :
//www. https : https
https : //instagra
//youtube.c
facebo m.com/aas :
om/channe hisharoraso
l/UCYa4_Jr ok.com cial(?) //t.m
Orf8R5Kz2u
/aashis utm_mediu
e/stu

139
OtccXQ m=
haroras copy_link
ocial dified
400(63 - X) /6525(9 - X) = 8/9 33. Ans. (D)
(63 - X) /(9 - X) = 29/2 In Final mixture, Alcohol : Water
126 - 2X = 261 - 29X = (660 × 8/11 + 780 × 7/13) : (660 ×
27X = 135 3/11 + 780 × 6/13)
X=5 = (480 + 420) : (180 + 360) = 900 :
540 = 5 : 3
30. Ans. (A)
In Vessel A, Juice = 12 × 5/(8 - 5) = 34. Ans. (B)
60/3 = 20 L SP = 840 - 84 = Rs. 756
Water = 12 × 8/(8 - 3) = 96/3 = 32 L MP = 840 × 1.2 = Rs. 1008
Total = 20 + 32 = 52 L X% = (1008 - 756) /1008 × 100 = 25%
In Vessel B, Total = 346 - 52 = 294 L
Juice = 294 × 4/7 = 168 L 35. Ans. (A)
Juice in both vessels = 20 + 168 = (15000 × M) /(18000 × 6) = (5400 -
188 L 2400) /2400
5M/36 = 3000/2400 = 5/4
31. Ans. (B) M=9
Aman and Tiya can complete work
in = 12/(5/9) = 21.6 days 36. Ans. (A)
Tiya can complete work in = 32.4/0.6 Monthly Income = 1400/(0.35 × 0.5) =
= 54 days Rs. 8000
Aman + Tiya ➔ 21.6 5 Amount spent on shopping = 8000 ×
} 108 { 0.5 × 0.65 × 0.2 = Rs. 520
Tiya ➔ 54 2
Aman complete 83.33% of work in 37. Ans. (C)
= (108 × 5/6) /(5 - 2) = 90/3 = 30 days Average Speed = x km/hr
(330 + 288 + 335) /x = 330/55 + 288/72
32. Ans. (C) + 335/67
2 numbers are = 7x and 7y 953/x = 6 + 4 + 5
LCM (7x, 7y) = 2261 x = 953/15 = 63.53 km/hr
7xy = 2261
xy = 323 = 17 × 19 38. Ans. (C)
7x = 7 × 17 = 119 Perimeter of Square = 4 × 48 = 192
7y = 7 × 19 = 133 cm
Sum = 7x + 7y = 119 + 133 = 252 Length of Rectangle = x cm
Breadth of Rectangle = (x - 5) cm
2 × (x + x - 5) = 192 - 62

https :
//www. https : https
https : //instagra
//youtube.c
facebo m.com/aas :
om/channe hisharoraso
l/UCYa4_Jr ok.com cial(?) //t.m
Orf8R5Kz2u
/aashis utm_mediu
e/stu

140
OtccXQ m=
haroras copy_link
ocial dified
2 × (2x - 5) = 130 43. Ans. (D)
2x - 5 = 65 Investment of Nisha = 5400 × 12 + X
2x = 70 × 5 = Rs. 5(12960 + X)
x = 35 Investment of Misti = 6900 × 12 + (X
Area of Square = 482 = 2304 cm2 + 200) × 5 = Rs. 5(16760 + X)
Area of Rectangle = 35 × 30 = 1050 5(16760 + X) /5(12960 + X) =
cm2 19700/(35460 - 19700)
Difference = 2304 - 1050 = 1254 cm2 (16760 + X) /(12960 + X) =
19700/15760 = 5/4
39. Ans. (B) 67040 + 4x = 64800 + 5x
X × (X + 16) + 16 × 54 = (X + 16) × (X x = 2240
+ 16 + 2)
X2 + 16X + 864 = X2 + 34X + 288 44. Ans. (B)
18X = 576 % of Amount deposited in bank
X = 32 = 100 - (12 + 19 + 21 + 23 + 9) = 100 -
84 = 16%
40. Ans. (D) (X + 3500) × (0.19 + 0.21 - 0.16) =
28 × 44 = 28 × 20 + (28 + X) × (44 - 20 4320
- 8) (X + 3500) × (0.4 - 0.16) = 4320
28 × 24 = (28 + X) × 16 (X + 3500) × 0.24 = 4320
28 + X = 7 × 6 = 42 X + 3500 = 18000
X = 14 X = 14500

41. Ans. (B) 45. Ans. (A)


Initially, Juice = 7x L, Water = 5x L CP of Bag = Rs. 4x
(7x - 24 × 7/12 + 28) /(5x - 24 × 5/12) = CP of Book = Rs. 5x
7/4 4x × 1.2 - 5x × 0.8 = 120
(x - 2 + 4) × 4 = 5x - 10 4.8x - 4x = 120
4x + 8 = 5x - 10 0.8x = 120
x = 18 x = 150
Initial quantity of mixture = 7x + 5x = CP of Book = 5 × 150 = Rs. 750
12 × 18 = 216 L
46. Ans. (A)
42. Ans. (E) 2X - (X - 15) = 40
Time taken = (4w + 12m) × 18/(7w + 2X - X + 15 = 40
15w) X = 25
Cannot be determined

https :
//www. https : https
https : //instagra
//youtube.c
facebo m.com/aas :
om/channe hisharoraso
l/UCYa4_Jr ok.com cial(?) //t.m
Orf8R5Kz2u
/aashis utm_mediu
e/stu

141
OtccXQ m=
haroras copy_link
ocial dified
Downstream Speed = 2X + (X - 15) 51. Ans. (B)
= 3 × 25 - 15 = 75 - 15 = 60 km/hr Numerator = x
Y/60 + Y/40 = 20 Denominator = (4x - 4)
Y/24 = 20 2x/1.5(4x - 4) = 1/2
Y = 480 4x = 6x - 6
Time taken in Downstream 2x = 6
= (480 + 180) /60 = 660/60 = 11 hours x=3
4 time of Fraction = 4x/(4x - 4) = x/(x
47. Ans. (C) - 1) = 3/(3 - 1) = 3/2
Total run scored in remaining
matches 52. Ans. (A)
= 28 × 45 - 15 × 48 - 7 × 41 (x + x + 36) × 9 = (x + 36 - x) × 31.5
= 1260 - 720 - 287 = 1260 - 1007 = (2x + 36) × 9 = 36 × 31.5
253 2x + 36 = 126
Sum of Speed = 126 km/hr
48. Ans. (C)
3840 × X/100 × 5 × (1.3752 - 1) = 4275 53. Ans. (A)
192 × X × (121/64 - 1) = 4275 Smaller number = x
X × 57/64 = 1425/64 Larger number = 3x
X = 25 3x - x = 500
2x = 500
49. Ans. (B) x = 250
Speed of Car A = 798/6 = 133 km/hr
Speed of Car B = 133/1.9 = 70 km/hr 54. Ans. (D)
Distance travelled by B = 70 × (4 + Let Anubhav had initially = x Rs.
24/60) x × 0.55 × 0.27 × 2 + x × 0.45 × (1.22 -
= 70 × 4.4 = 308 km 1) = 2970
0.297x + 0.198x = 2970
50. Ans. (D) 0.495x = 2970
Actual perimeter = 2 × (30 + 24) = 2 × x = Rs. 6000
54 = 108 cm
Changed perimeter = 2 × (30 × 0.85 + 55. Ans. (A)
24 × 1.05) 1.0125 = 1.25 × 0.9 × (1 - x/100)
= 2 × (25.5 + 25.2) = 2 × 50.7 = 101.4 0.81 = 0.9 × (1 - x/100)
cm 1 - x/100 = 0.9
% change in perimeter = (108 - 101.4) x = 10
/108 × 100 = 6.11%

https :
//www. https : https
https : //instagra
//youtube.c
facebo m.com/aas :
om/channe hisharoraso
l/UCYa4_Jr ok.com cial(?) //t.m
Orf8R5Kz2u
/aashis utm_mediu
e/stu

142
OtccXQ m=
haroras copy_link
ocial dified
56. Ans. (B) Radius of Cylinder : Radius of Cone
Faster one complete the work in = x = 12 : 15 = 4 : 5
days
1/x + 1/(x + 4) = 9/80 60. Ans. (B)
(x + x + 4) /x(x + 4) = 9/80 Speed of Stream = 4 km/hr
80 × (2x + 4) = 9x(x + 4) Speed of Boat in still water = 3 × 4 =
160x + 320 = 9x2 + 36x 12 km/hr
9x2 - 124x - 320 = 0 X + 50 = 5 × (12 + 4)
(9x + 20) (x - 16) = 0 X + 50 = 5 × 16 = 80
x = 16 days X = 30
Y + 24 = 5 × (12 - 4)
57. Ans. (D) Y + 24 = 5 × 8 = 40
Initially, Milk = 4x L Y = 16
Water = x L Time taken in Upstream = (4 × 30 +
% of Water = x/5x × 100 = 20% 16) /8
(Y × 0.8 - 20× 0.8) /(Y × 0.2 - 20 × 0.2 = (120 + 16) /8 = 136/8 = 17 hours
+ 12) = 8/5
5 × (0.8Y - 16) = 8 × (0.2Y - 4 + 12) 61. Ans. (A)
4Y - 80 = 1.6Y + 64 P has = 9x mangoes
2.4Y = 144 Q has = 11 mangoes
Y = 60 L (9x - 25) /(11x - 25) = 2/3
27x - 75 = 22x - 50
58. Ans. (B) 5x = 25
At present, Age of Prateek = 34 - 4 = x=5
30 years P has = 9 × 5 = 45 mangoes
Age of Rajat = 30 - 6 = 24 years
Age Deepak = 30 + 24 = 54 years 62. Ans. (A)
4/x = 24/54 X = 22500 × 0.8 × 0.5 × 0.7 = 7000
x=9
63. Ans. (A)
59. Ans. (C) Probability of selling either KKR or
Radius of Cylinder = r cm CSK jersey
π× r2× 10 + 1/3 × π × 152× 10 = 2190π = (4 + 5) /24 = 9/25
r2 + 1/3 × 225 = 219
r2 = 219 - 75 = 144
r = 12

https :
//www. https : https
https : //instagra
//youtube.c
facebo m.com/aas :
om/channe hisharoraso
l/UCYa4_Jr ok.com cial(?) //t.m
Orf8R5Kz2u
/aashis utm_mediu
e/stu

143
OtccXQ m=
haroras copy_link
ocial dified
64. Ans. (C) 67. Ans. (A)
P ➔ 16 15 X = 470/(1.22 - 1.152) = 470/(1.44 -
Q ➔ 20 } 240 {12 1.3225)
R ➔ 24 10 = 470/0.1175 = 4000
(X - 8) × 15 + (X + 5 - X) × 12 + (X + 5)
× 10 = 240 68. Ans. (B)
15X - 120 + 60 + 10X + 50 = 240 200 × 35 + 250 × P = 450 × 20
25X = 360 - 110 = 250 7000 + 250P = 9000
X = 10 250P = 2000
P=8
65. Ans. (B)
Smaller Side of Trapezium = x 69. Ans. (A)
1/2 × (x + 16) × (x - 2) = 104 Average Speed = 500/(20/40 + 140/70
x2 + 14x - 32 = 208 + 340/34)
x2 + 14x - 240 = 0 = 500/(0.5 + 2 + 10) = 500/12.5 = 40
(x + 24) (x - 10) = 0 km/hr
x = 10
Distance between parallel side = 10 - 70. Ans. (B)
2 = 8 cm Profit Ratio, Rajat : Ram : Ritesh = 4
: 0.6 : 1
66. Ans. (C) Difference between profit of Ram
Speed of Stream = x km/hr and Ritesh
54/(8 + x) + 21/(8 - x) = 9 = 5000 × (1 - 0.6) /4 = 5000 × 0.4/4 =
18/(8 + x) + 7/(8 - x) = 3 Rs. 500
144 - 18x + 56 + 7x = 3(64 - x2)
200 - 11x = 192 - 3x2 71. Ans. (C)
3x2 - 11x + 8 = 0 Probability = 1/2 × (4/12 + 5/10) = 1/2
(x - 1) (3x - 8) = 0 × 5/6 = 5/12
x = 1, 8/3
P = 63/(8 + 1) + 63/(8 - 1) 72. Ans. (B)
= 63/9 + 63/7 = 7 + 9 = 16 km Amount spent on Chocolates = 700
× 0.25 = 175
Amount spent given to brother = 700
× 0.75 × (1 - X/100) = 525(1 - X/100)
525(1 - X/100) - 175 = 140

https :
//www. https : https
https : //instagra
//youtube.c
facebo m.com/aas :
om/channe hisharoraso
l/UCYa4_Jr ok.com cial(?) //t.m
Orf8R5Kz2u
/aashis utm_mediu
e/stu

144
OtccXQ m=
haroras copy_link
ocial dified
525(1 - X/100) = 315 6r = 5 × 4p
1 - X/100 = 3/5 p ; r = 3 : 10
X = 40
77. Ans. (A)
73. Ans. (A) Average price of half - sleeve t -
Initial number of boys = x shirts = Rs. x
x× 25.5 - 4 × 23.2 + 5 × 31.66 = (x - 4 (3 + 4 + 5) × A = 3 × (A + 1) + 4 × x + 5
+ 5) × (25.5 + 2) × (A - 1)
25.5x - 92.8 + 158.3 = (x + 1) × 27.5 12A = 3A + 3 + 4x + 5A - 5
25.5x + 65.5 = 27.5x + 27.5 12A = 8A - 2 + 4x
2x = 38 4x = 4A + 2
x = 19 x = (2A + 1) /2

74. Ans. (C) 78. Ans. (A)


P ➔ 30 7 Together they type 220 pages in
Q ➔ 35 } 210 {6 = 220/(34/6 + 45/5)
A ➔ 70 3 = 220/(17/3 + 9) = 220 × 3/44 = 15
Time taken to fill the tank = (210 + 8
× 3) /(7 + 6) 79. Ans. (A)
= (210 + 24) /13 = 234/13 = 18 min Total Amount of R & S = 2300/(5 - 4)
× (7 + 9)
75. Ans. (A) = 2300 × 16 = Rs. 36800
Decreased CP = 350/1.25 = Rs. 280
Actual CP = 280/0.7 = Rs. 400 80. Ans. (C)
MP = 400 × 1.25 = Rs. 500 Breadth of Cuboid = x m
X = 500 × 0.78 = 390 2 × (20 × x + x × 6 + 20 × 6) = 656
20x + 6x + 120 = 328
76. Ans. (C) 26x = 208
Speed of P = p km/hr x=8m
Speed of Q = q km/hr Difference = 20 - 8 = 12 m
Speed of R = r km/hr
4p = q
6r = q + 2 × 2q
6r = 5q

https :
//www. https : https
https : //instagra
//youtube.c
facebo m.com/aas :
om/channe hisharoraso
l/UCYa4_Jr ok.com cial(?) //t.m
Orf8R5Kz2u
/aashis utm_mediu
e/stu

145
OtccXQ m=
haroras copy_link
ocial dified
81. Ans. (C) 120000
Saving of P = (6000 + 6500) × 6 - Investment of B = 16000 × 5 = Rs.
5000 × 12 80000
= 12500 × 6 - 60000 = 75000 - 60000 Investment of C = 20000 × 7 = Rs.
= Rs 15000 140000
Profit Ratio, A : B : C = 120000 :
82. Ans. (B) 80000 : 140000 = 6 : 4 : 7
(150 × X + 7000) × 1.425 = 400 × X × Profit Share of C = 8500 × 7/17 = Rs.
0.95 3500
(150X + 7000) × 3 = 400X × 2
450X + 21000 = 800X 87. Ans. (A)
350X = 21000 Speed of Stream = s km/hr
X = 60 20 × (10 + s) = 30 × (10 - s)
20 + 2s = 30 - 3s
83. Ans. (A) 5s = 10
Number of turn taken by P s=2
= 24 × (4 + 1) /4 = 6 × 5 = 30 X = 20 × (10 + 2) = 20 × 12 = 240
2X - 80 = 2 × 240 - 80 = 480 - 80 = 400
84. Ans. (C)
X = 1440 × 0.9/1.08 = 1440 × 5/6 = 88. Ans. (B)
1200 Weight of Q = x kg
8 × 67 + (x + 26) + x = 10 × (67 - 6)
85. Ans. (B) 536 + 2x + 26 = 610
Weight of additional Type A rice = x 2x = 610 - 562 = 48
kg x = 24 kg
(20 × 3/10 + 21 × 4/7 + x) /(20 × 7/10 +
21 × 3/7) = 21/23 89. Ans. (A)
23 × (6 + 12 + X) = 21 × (14 + 9) Interest received from scheme A =
23 × (18 + X) = 21 × 23 1460 × 0.1 × 5 = Rs. 730
18 + X = 21 Interest received from scheme B =
X=3 (1460 + 730) × 0.1 × T = 219T
Weight of mixture obtained = 20 + 21 219T - 730 = 1022
+ 3 = 44 kg 219T = 1752
T = 8 years
86. Ans. (C)
Investment of A = 10000 × 12 = Rs.

https :
//www. https : https
https : //instagra
//youtube.c
facebo m.com/aas :
om/channe hisharoraso
l/UCYa4_Jr ok.com cial(?) //t.m
Orf8R5Kz2u
/aashis utm_mediu
e/stu

146
OtccXQ m=
haroras copy_link
ocial dified
90. Ans. (C) SP of Charger = (375 - 200) × 0.96
Distance = x m = 175 × 0.96 = Rs. 168
x/18 + x/12 = 30
x × 5/36 = 30 95. Ans. (B)
x = 216 Sum invested = 6250/(2.5 × 1/6) =
Total distance covered by boat = 216 6250 × 12/5 = Rs. 15000
× 2 = 432 m Amount in 2 years = 15000 × (1 + 2 ×
1/6) = 15000 × 4/3 = Rs. 20000
91. Ans. (C)
Efficiency of P : Efficiency of Q = 4 : 96. Ans. (C)
3 Smaller number = x
Efficiency of Q : Efficiency of R = Larger number = (18 - x)
1/20 : 1/30 = 3 : 2 (18 - x) × 1.1 - x × 0.9 = 12
P, Q & R together fill the tank in 19.8 - 1.1x - 0.9x = 12
= (30 × 2) /(4 + 3 - 2) = 60/5 = 12 2x = 7.8
hours x = 3.9
Product = 3.9 × (18 - 3.9) = 3.9 × 14.1
92. Ans. (B) = 54.99 = 55
(2400 × 0.18 × 3) /(X × 0.12 × 4) = 6/5
0.48X = 1296 × 5/6 = 1080 97. Ans. (A)
X = 2250 Average of Bats distributed on
Wednesday and Thursday
93. Ans. (A) = (7 × 53 - 3 × 50 - 85) /2 = (371 - 150
Initially, Alcohol = 2x L - 85) /2
Water = 3x L = (371 - 235) /2 = 136/2 = 68
(2x - 25 × 2/5 + 25) /(3x - 25 × 3/5) =
13/12 98. Ans. (B)
12 × (2x - 10 + 25) = 13 × (3x - 15) Medal won by India = 225 + 180 - 205
24x + 180 = 39x - 195 = 405 - 205 = 200
15x = 375
x = 25 99. Ans. (A)
Initial Quantity ofAlcohol = 2 × 25 = Radius of Circular field = 176/(2 ×
50 L 22/7) = 28 m
Radius including circular path =
94. Ans. (C) 264/(2 × 22/7) = 42 m
CP of Earphone = 275/1.375 = Rs. Area of Road = 22/7 × (422 - 282)
200

https :
//www. https : https
https : //instagra
//youtube.c
facebo m.com/aas :
om/channe hisharoraso
l/UCYa4_Jr ok.com cial(?) //t.m
Orf8R5Kz2u
/aashis utm_mediu
e/stu

147
OtccXQ m=
haroras copy_link
ocial dified
= 22/7 × (1764 - 784) = 22/7 × 980 = = 25/6 : 45/4 : 30/5 = 50 : 135 : 72
3080 cm2 Investment of Om = 12000 × 72/50 =
Rs 17280
100. Ans. (A)
Efficiency of Akhil : Efficiency of 105. Ans. (B)
Vishal = 2 : 3 Maximum marks = 5/(0.9 - 0.88) =
n × (2 + 3) + 15 × 2 = 2 × 30 5/0.02 = 250
n × 5 = 60 - 30 = 30 % of Ranvir = 190/250 × 100 = 76%
n=6
n/s = 6/2 = 3 106. Ans. (C)
Cost of 1st bundle = 100× 1 × 78 = Rs
101. Ans. (C) 7800
8 × 15 + X × 6 = (8 + X) × 10.8 Cost of another bundle = 150 × 3/4 ×
X × (10.8 - 6) = 8 × (15 - 10.8) 78 = Rs. 8775
X × 4.8 = 8 × 4.2 Difference = 8775 - 7800 = Rs. 975
X=7
107. Ans. (A)
102. Ans. (B) Quantity of oil B = x L
1st sum = 8322/(1 + 0.08 × 16) 12/x = (64 - 60) /(60 - 40)
= 8322/(1 + 1.28) = 8322/2.28 = Rs 12/x = 4/20 = 1/5
3650 x = 60 L
2nd sum = 10556//(1 + 12 × 0.11)
= 10556/(1 + 1.32) = 10556/2.32 = Rs 108. Ans. (B)
4550 Stream Speed = x km/hr
Difference = 4550 - 3650 = Rs. 900 Upstream Speed = 2x km/hr
Speed of Boat in still water = 2x + x
103. Ans. (A) = 3x km/hr
At present, Age of Bheem = x years 3x - 2x = 9
Age of Aman = 1.2x years x=9
1.2x - 24 = 6 × (x - 24) Increased Downstream Speed = 9 ×
0.2x - 4 = x - 24 3 + 4 + 9 = 40 km/hr
0.8x = 20 Time taken in Downstream = 324/40
x = 25 years = 8.1 hours

104. Ans. (C) 109. Ans. (A)


Investment Ratio, Mahesh : Naresh :
Om

https :
//www. https : https
https : //instagra
//youtube.c facebo m.com/aas :
om/channe hisharoraso
l/UCYa4_Jr ok.com cial(?) //t.m
Orf8R5Kz2u /aashis utm_mediu
e/stu

148
OtccXQ m=
haroras copy_link
ocial dified
A ➔4 3 114. Ans. (C)
} 12 { Efficiency of A = 5
B ➔6 2 Efficiency of B = 4
Efficiency of leakage = (12/2) /6 = 1 Total Work = 12 × 5 = 60
Tank will be filled in = 12/(3 + 2 - 1) 2 day work = 5 + 4 = 9
= 12/4 = 4 hours 12 day work = 12/2 × 9 = 54
13th day work = 5
110. Ans. (C) Remaining work done by B in = (60 -
Total Share = Rs. x 54 - 1) /4 = 1/4
x × 3/5 × 1,35 + x × 2/5 × 0.75 = x + Time taken to complete the work =
858 12 + 1 + 1/4 = 13(1/4) days
0.81x + 0.3x = x + 858
1.11x - x = 858 115. Ans. (B)
0.11x = 858 Volume of Water : Volume of Liquid
x = Rs. 7800 = (1250 - 1050) : (1050 - 1000)
= 200 : 50 = 4 : 1
111. Ans. (C) Fraction of Water = 4/5
CP = Rs. x Fraction of Liquid = 1/5
MP = 0.9x × 5/3 = Rs. 1.5x
SP at 20% profit = Rs. 1.2x 116. Ans. (C)
Fraction = 1.2x/1.5x = 4/5 Profit Ratio, Varun : Atul
= (3 × 12) : (5 × 9) = 4 : 5
112. Ans. (B) Difference between profit = 1000 × (5
r = (6912 - 5760) /5760 × 100 - 4) /5 = Rs. 200
r = 20%
Principal = 5760/1.22 = Rs. 4000 117. 1Ans. (D)
Monthly salary of Amit = 16940/(0.7 ×
113. Ans. (A) 0.55)
At present, Age of Abhishek = x = 16940/0.385 = Rs. 44000
years Monthly salary of Amar = 44000 × 1.2
(x - 12) × 1.6 = x = Rs. 52800
x - 12 = 0.625x
0.375x = 12 118. Ans. (B)
x = 32 Amount received by C = Rs. 6x
Required years = (32 - 14) /0.45 - 32 Amount received by D = Rs. 7x
= 18/0.45 - 32 = 40 - 32 = 8 years Amount received by B = Rs. (7x -
100)

https :
//www. https : https
https : //instagra
//youtube.c
facebo m.com/aas :
om/channe hisharoraso
l/UCYa4_Jr ok.com cial(?) //t.m
Orf8R5Kz2u
/aashis utm_mediu
e/stu

149
OtccXQ m=
haroras copy_link
ocial dified
Amount received by A = 7x - 100 + 50 121. Ans. (C)
= Rs. (7x - 50) Ratio of Investment period, Mitesh :
(7x - 50) + (7x - 100) + 6x + 7x = 1200 Nitesh
27x = 1200 + 150 = 8000/24000 : 9000/3600 = 4 : 3
27x = 1350 Mitesh invested his amount for = 7 ×
x = 50 4/7 = 4 months
Amount received by D = 7 × 50 = Rs.
350 122. Ans. (B)
Amount received by A = 350 - 50 = Age of Pankaj = 32 × 3 - 64 = 96 - 64
Rs. 300 = 32 years
Average = (350 + 300) /2 = 650/2 = Age of Suresh = 32 × 3 - 76 = 96 - 76
Rs. 325 = 20 years
Age of prakash = 32 × 3 - (32 + 20) =
119. Ans. (A) 96 - 52 = 44 years
A ➔ 24 4 Ratio, Pankaj : Prakash : Suresh =
} 96 { 32 : 44 : 20 = 8 : 11 : 5
B ➔ 32 3
B should be closed after = (96 - 4 × 123. Ans. (A)
18) /3 (X - 10) + (X + 30) = 64 × 5/18 × 18
= (96 - 72) /3 = 24/3 = 8 min 2X + 20 = 320
2X = 300
120. Ans. (C) X = 150
Upstream Speed = 132/26.4 = 5
km/hr 124. Ans. (C)
Speed of Boat in still water = 5 + 3 = Money invested by Prakhar =
8 km/hr 2100/(0.18 × 2 - 1.12 + 1)
Time taken in Upstream = 46.2/(8 + 3) = 2100/(0.36 - 0.21) = 2100/0.15 = Rs.
= 46.2/11 = 4.2 hours 14000
Time taken in Downstream = 86/5 =
17.2 hours 125. Ans. (B)
Difference = 17.2 - 4.2 = 13 hours Age of eldest son = 18 + 3 = 21 years
Age of Father = 21 × 15/7 = 45 years
Age of youngest son = 18 - 3 = 15
years
After 5 years, Age of Father : Age of
youngest Son
= (45 + 5) : (15 + 2) = 50 : 20 = 5 : 2

https :
//www. https : https
https : //instagra
//youtube.c
facebo m.com/aas :
om/channe hisharoraso
l/UCYa4_Jr ok.com cial(?) //t.m
Orf8R5Kz2u
/aashis utm_mediu
e/stu

150
OtccXQ m=
haroras copy_link
ocial dified
126. Ans. (C) 131. Ans. (C)
A ➔ 20 6 Investment of A = 7000 × 14/18 ×
B ➔ 30 } 120 {4 27/14
C ➔ 40 3 = 7000 × 3/2 = Rs. 10500
Amount of milk after 4 min = 4 × 6 =
24 units 132. Ans. (B)
Circumference of Circle = 2 × 22/7 ×
127. Ans. (A) 105 = 660 cm
Probability of choosing 4 Red cards Length of Rectangle = (660/2 + 10) /2
= 26C4/52C4 = 14950/270725 = 46/833 = (330 + 10) /2 = 340/2 = 170 cm
Breadth of Rectangle = 660/2 - 170 =
128. Ans. (B) 330 - 170 = 160 cm
Side of Square paper = √144 = 12 cm Area of Rectangle = 170 × 160 =
Length of Rectangular board = 600/2 27200 cm2
× 3/5 = 180 cm
Breadth of Rectangular board = 133. Ans. (A)
600/2 × 2/5 = 120 cm Amount given to Mohit = 2700/(4 ×
n = (180 × 120) /144 = 15 × 10 = 150 0.05)
= 2700/0.2 = Rs. 13500
129. Ans. (E)
Numbers are = 2a and 5a 134. Ans. (C)
(2a + X) /(5a + X) = 5/11 Time taken on Bike = (8 × 6.75) /45
22a + 11X = 25a + 5X = 54/45 = 1.2 hours = 1.2 × 60 = 72
3a = 6X min
a = 2X
(2a - X) /(5a - X) = 1/3 135. Ans. (B)
6a - 3X = 5a - X Rest of food last for = 400 × (31 - 28)
a = 2X /(400 - 280)
Cannot be determined = 400 × 3/120 = 10 days

130. Ans. (C) 136. Ans. (C)


MP : SP : CP = 30 : 24 : 25 At present, Age of Son = x years
Discount % = (30 - 24) /30 × 100 = Age of Mother = 3x years
20% Age of Father = 32 × 3 - (x + 3x) = (96
- 4x) years
96 - 4x - 6 = 7 × (x - 6)
90 - 4x = 7x - 42

https :
//www. https : https
https : //instagra
//youtube.c
facebo m.com/aas :
om/channe hisharoraso
l/UCYa4_Jr ok.com cial(?) //t.m
Orf8R5Kz2u
/aashis utm_mediu
e/stu

151
OtccXQ m=
haroras copy_link
ocial dified
11x = 132 141. Ans. (C)
x = 12 years Salary of A = 20000 × 8/(8 - 3)
After 8 years, Age of Son = 12 + 8 = = 20000 × 8/5 = Rs. 32000
20 years Saving of A = 32000 × 0.375 = Rs.
12000
137. Ans. (A)
CP of Mixture = 66/1.25 = Rs. 52.8/kg 142. Ans. (B)
Ratio of wheat in mix = (56 - 52.8) : Breadth of Rectangle = x m
(52.8 - 48) 2 × (20 × x + x × 6 + 20 × 6) = 656
= 3.2 : 4.8 = 2 : 3 26x + 120 = 328
26x = 208
138. Ans. (A) x=8
20m × 15 = 15w × 30 Length - Breadth = 20 - 8 = 12 m
m = 1.5w
10 men complete the same work in 143. Ans. (A)
= 15/1.5 = 10 days (7200 × 12) /8400 × (12 - m) } = 12/7
12 - m = 6
139. Ans. (A) m=6
Speed of A = 40 km/hr
Speed of B = 40 × 8/4 = 80 km/hr 144. Ans. (C)
Speed of C = 40 × 3/4 = 30 km/hr Efficiency Ratio, A : B : C = 6 : 3 : 2
Total Distance = x km Time taken by A alone = 6 × (6 + 3 +
(x/4) /40 + (3x/4 × 0.4) /80 + (3x/4 × 2) /6 = 11 days
0.6) /30 = 12
x/160 + 3x/800 + 3x/200 = 12 145. Ans. (B)
x/40 = 12 At present, Age of Sanju = (7x - 8)
x = 480 km years
Age of Manju = (7x - 8) + 5 = (7x - 3)
140. Ans. (C) years
Maximum marks = (93 + 13) /0.424 = Age of Anju = (6x - 8) years
106/0.424 = 250 marks 7x - 3 - 6 = 1.9(6x - 8 - 6)
7x - 9 = 11.4x - 26.6
4.4x = 17.6
x=4
5 years later, Age of Sanju = 7 × 4 - 8
+ 5 = 28 - 3 = 25 years
Age of Manju = 7 × 4 - 3 + 5 = 28 + 2

https :
//www. https : https
https : //instagra
//youtube.c
facebo m.com/aas :
om/channe hisharoraso
l/UCYa4_Jr ok.com cial(?) //t.m
Orf8R5Kz2u
/aashis utm_mediu
e/stu

152
OtccXQ m=
haroras copy_link
ocial dified
= 30 years 150. Ans. (B)
Required % = (30 - 25) /25 × 100 = Investment of Arpan = 6000 × 12 -
20% 1500 × 9 = Rs. 58500
Investment of Darpan = 7500 × 12 -
146. Ans. (C) 2000 × 7 = Rs. 76000
CP of Article = 12.5/(0.2 - 0.15) Profit Ratio, Arpan : Darpan = 58500
= 12.5/0.05 = Rs. 250 : 76000 = 117 : 152
Profit of Arpan = 2280 × 117/152 =
147. Ans. (A) Rs. 1755
Quantity of mixture taken from 1st
container = x L 151. Ans. (B)
{x× 0.75 + (12 - x) × 0.5}/{x 0.25 + (12 At present, Age of Aashi = (8x + 6)
- x) × 0.5) = 5/3 years
3 × (0.75x + 6 - 0.5x) = 5 × (0.25x + 6 - Age of Sara = (11x - 2) years
0.5x) (8x + 6) + (11x - 2) = 40 × 2
3 × 0.25x + 18 = 30 - 5 × 0.25x 19x = 80 - 4 = 76
8 × 0.25x = 12 x=4
2x = 12 Age of Aashi = 8 × 4 + 6 = 38 years
x=6L Age of Sara = 11 × 4 - 2 = 44 - 2 = 42
years
148. Ans. (B) Difference = 42 - 38 = 4 years
Breadth of Rectangle = x m
Length of Rectangle = 1.5x m 152. Ans. (C)
Perimeter of Rectangle = 2 × (1.5x + Efficiency of Mohsin = 1
x) Efficiency of Roshan = 6
= 2 × 2.5x = 5x m Efficiency of Manav = (1 + 6) /2 = 3.5
Perimeter of Square = 5x × 2/5 = 2x Time taken by Manav = (1 + 6) ×
m 18/3.5
Side of Square = 2x/4 = 0.5x m = 7/3.5 × 18 = 36 days
Area of Square : Area of Rectangle
= (0.5x) 2 : (1.5x × x) = 0.25 : 1.5 = 1 : 153. Ans. (D)
6 % of Votes of loser = 100 - (40 + 30) =
30%
149. Ans. (A) Total votes casted = 120/(0.4 - 0.3) =
P% = (1.17 + 0.91 - 2) /2 × 100 1200
= 0.08/2 × 100 = 4%

https :
//www. https : https
https : //instagra
//youtube.c
facebo m.com/aas :
om/channe hisharoraso
l/UCYa4_Jr ok.com cial(?) //t.m
Orf8R5Kz2u
/aashis utm_mediu
e/stu

153
OtccXQ m=
haroras copy_link
ocial dified
154. Ans. (B) 158. Ans. (C)
(4250 × 12) /{(4250 + x) × 8} = 3/5 X = 5460/(1.23 - 1) = 5460/0.728 = Rs
(4250 + x) × 2 = 4250 × 5 7500
(4250 + x) × 2 = 21250 SI = (7500 + 1500) × 0.12 × 2 = 9000 ×
4250 + x = 10625 0.24 = Rs. 2160

155. Ans. (A) 159. Ans. (B)


Actual Income = 100 Radius of Base of Cone = 1914/(22/7
New Income = 85 × 0.7 + 15 × 1.58 × 29) = 21 m
= 59.5 + 23.7 = 83.2 Height of Cylinder = √(292 - 212)
% decrease Income = 100 - 83.2 = = √(841 - 441) = √400 = 20 m
16.8% Volume of Cylindrical container
= 22/7 × 212 × 20 = 27720 m3
156. Ans. (A)
Speed of Stream = 3x km/hr 160. Ans. (D)
Speed of Boat in still water = 8x In Final mixture, Soda : Water
km/hr = (64 × 9/16 + 81) : (64 × 7/16 + 63)
220/(8x - 3x) + 220/(8x + 3x) = 12 = (36 + 81) : (28 + 63) = 117 : 91 = 9 :
220/5x - 220/11x = 12 7
44/x - 20/x = 12
24/x = 12 161. Ans. (B)
x=2 Rohit ➔ 48 5
Speed of Boat in still water = 8 × 2 = } 240 {
16 km/hr Rohit + Manas ➔ 30 8
Time taken by Manas to complete
157. Ans. (C) rest of the work
Length of Train P = 72 × 5/18 × 22 = = (240 - 18 × 8) /(8 - 5)
440 m = (240 - 144) /4 = 96/3 = 32 days
Length of Train Q = 28 × (72 + 63) ×
5/18 - 440 162. Ans. (C)
= 135 × 70/9 - 440 = 1050 - 440 = 610 (540 × 11/18 + X + 130) /(540 × 7/18 +
m X) = 2/1
330 + X + 130 = 2 × (210 + X)
460 + X = 420 + 2X
X = 40

https :
//www. https : https
https : //instagra
//youtube.c
facebo m.com/aas :
om/channe hisharoraso
l/UCYa4_Jr ok.com cial(?) //t.m
Orf8R5Kz2u
/aashis utm_mediu
e/stu

154
OtccXQ m=
haroras copy_link
ocial dified
163. Ans. (D) 168. Ans. (C)
Perimeter of Rectangle = 1728/6 = 17th number = (18 × 68 - 15 × 62) ×
288 m 3/6
Breadth of Rectangle = 288/2(1 + 1.4) = (1224 - 930) /2 = 294/2 = 147
= 144/2.4 = 60 m
Length of Rectangle = 60 × 1.4 = 84 169. Ans. (B)
m Interest in 1 years = (7950 - 7200) /(7
- 4)
164. Ans. (B) = 750/3 = Rs. 250
3 years later, Age of Rishabh = 4 × Sum of Money = 7200 - 250 × 4
10/(10 - 9) - 28 - 3 = 7200 - 1000 = Rs. 6200
= 40 - 31 = 9 years
Age of Aayushi = 9 - 4 = 5 years 170. Ans. (D)
Required % = 4/5 × 100 = 80% Speed of Train = 324/(26.4 - 4.8)
= 324/21.6 = 15 m/s
165. Ans. (C) Length of Train = 4.8 × 15 = 72 m
Monthly Expenditure = (4200 - 600) Time taken by Train Q to cross the
/2 platform
= 3600/2 = Rs. 1800 = (324 + 72/0.5) /(15 × 1.2)
Monthly Saving = 1800 + 600 = Rs. = (324 + 144) /18 = 468/18 = 26
2400 seconds
Increased Monthly Expenditure
= 4200 × 1.6 - 2400 × 1.2 = 6720 - 171. Ans. (B)
2880 = Rs. 3840 8+7+X=8×3
Increase in Monthly Expenditure = X = 24 - 15 = 9
3840 - 1800 = Rs. 2040
172. Ans. (C)
166. Ans. (A) At present, Age of Raghav = 34 + 2 =
(2800 × 12) /{2000 × (12 - X) } = 21/10 36 years
7 × 12 × 10 = 5 × (12 - X) × 21 Age of Samayra = 34 × 14/17 + 2 = 30
12 - X = 2 × 4 = 8 years
X=4 4P : 3Q = (4 × 36) : (3 × 30) = 8 : 5

167. Ans. (C) 173. Ans. (D)


X = 6800 × (1.36 - 1.08) 7500 × (R + 5) /100 × 3 + 9000 × (R -
= 6800 × 0.28 = 1904 4) /100 × 3 = 4995
75 × (R + 5) + 90 × (R - 4) = 1665

https :
//www. https : https
https : //instagra
//youtube.c
facebo m.com/aas :
om/channe hisharoraso
l/UCYa4_Jr ok.com cial(?) //t.m
Orf8R5Kz2u
/aashis utm_mediu
e/stu

155
OtccXQ m=
haroras copy_link
ocial dified
75R + 375 + 90R - 360 = 1665 178. Ans. (C)
165R = 1665 - 15 = 1650 Income of Ravi = Rs. 8x
R = 10 Income of Satish = Rs. 9x
(8x - 8000) /(9x - 10000) = 12/13
174. Ans. (B) 104x - 104000 = 108x - 120000
Speed of Stream = x km/hr 4x = 16000
Speed of Boat R = 1.2x km/hr x = 4000
Speed of Boat S = (1.2x + 3) km/hr Income of Ravi = 8 × 4000 = Rs.
(1.2x + 3) + x = 306/8.5 32000
2.2x + 3 = 36
2.2x = 33 179. Ans. (B)
x = 15 Parth + Rohan ➔ 30 8
Upstream Speed of Boat R = 1.2x - x } 240 {
= 0.2 × 15 = 3 km/hr Parth ➔ 48 5
Efficiency of Rohan = 8 - 5 = 3
175. Ans. (A) Time taken by Sohit = 240/(3 × 4/3) =
Radius of Base = 4x cm 60 days
Height = 3x cm
2 × 22/7 × 4x × 3x = 14784 180. Ans. (D)
528/7 × x2 = 14784 CP of mixture = 70 × 6/7 = 60 Rs./kg
x2 = 196 (2X + 1) /15 = (80 - 60) /(60 - 45)
Area of Base = 2 × 22/7 × (4x) 2 (2X + 1) /15 = 20/15
= 2 × 22/7 × 16 × 196 = 9856 cm2 2X + 1 = 20
2X = 19
176. Ans. (A) X = 9.5
CP of each article = x Rs.
56x - 8x = 3360 181. Ans. (B)
48x = 3360 X = 380 × 0.35/0.2 × 0.8 - 380 × 0.65
x = 70 Rs. = 380 × (1.4 - 0.65) = 380 × 0.75 = 285
SP of 20 articles = 20 × 70 × 1.4 = Rs.
1960 182. Ans. (C)
Investment of Mohini = 7800 × 12 - 7
177. Ans. (C) × 1800
Profit Ratio, Kaira : Saira = (5 × 7) :
(3 × 10) = 7 : 6
Profit Share of Saira = 3315 × 6/13 =
Rs. 1530

https :
//www. https : https
https : //instagra
//youtube.c
facebo m.com/aas :
om/channe hisharoraso
l/UCYa4_Jr ok.com cial(?) //t.m
Orf8R5Kz2u
/aashis utm_mediu
e/stu

156
OtccXQ m=
haroras copy_link
ocial dified
= 93600 - 12600 = Rs. 81000 = 6664/0.952 = Rs. 7000
Investment of Raghav = 7200 × 12 + 188. Ans. (C)
7 × 1800 At present, Age of Nukul = (x + 4)
= 86400 + 12600 = Rs. 99000 years
Profit Ratio, Mohini : Raghav = Age of Meeta = (0.625x + 4) years
81000 : 99000 = 9 : 11 0.625x + 4 + 6 = 0.7 × (x + 4 + 6)
Profit Share of Raghav = 5000 × 0.625x + 10 = 0.7x + 7
11/20 = Rs. 2750 0.075x = 3
x = 40
183.. Ans. (D) Present Age of Meeta = 0.625 × 40 +
Usual Speed = 240/4 = 60 km/hr 4 = 25 + 4 = 29 years
Time taken = 345/(60 × 1.15) + 96/(60
× 0.8) 189. Ans. (B)
= 345/69 + 96/48 = 5 + 2 = 7 hours Efficiency of Vibha = 6
Efficiency of Varun = 5
184. Ans. (B) 30 × x = 6 × (x + y) + 19× x
(1 + R/100) 2 - 1 = 850/3200 30x = 6x + 6y + 19x
(1 + R/100) 2 = 1 + 17/64 = 81/64 30x - 25x = 6y
1 + R/100 = 9/8 5x = 6y
R/100 = 1/8 x/y = 6/5
R = 12.5% Efficiency of Vibha is more than
Varun by
185. Ans. (A) = (6 - 5) /5 × 100 = 20%
Average Weight of P & T = (54 × 5 -
50 × 2) /10 × (5 + 2) /2 190. Ans. (D)
= (270 - 100) /10 × 7/2 = 170/10 × 7/2 Speed of Boat in still water = 360/8 =
= 59.5 kg 45 km/hr
Speed of Stream = 45 × 5/9 = 25
186. Ans. (A) km/hr
100 - (35 + X + 15) = 7200/24000 × X = 840/(45 + 25) = 840/70 = 12 hours
100
100 - (50 + X) = 30
50 + X = 70
X = 20

187. Ans. (C)


CP of Article = 6664/(0.85 × 0.7 × 1.6)

https :
//www. https : https
https : //instagra
//youtube.c
facebo m.com/aas :
om/channe hisharoraso
l/UCYa4_Jr ok.com cial(?) //t.m
Orf8R5Kz2u
/aashis utm_mediu
e/stu

157
OtccXQ m=
haroras copy_link
ocial dified
191. Ans. (B) R/100 = 0.5
40 × 18 = 25 × 10 + (25 + X) × 5 R = 50
(25 + X) × 5 = 720 - 250 = 470
25 + X = 94 197. Ans. (C)
X = 69 150/X - 150/(X + 10) = 4
150(X + 10) - 150X = 4X(X + 10)
192. Ans. (C) 1500 = 4X2 + 40X
1stnumber : 2nd number : 3rd number X2 + 10X - 375 = 0
= 15 : 35 : 56 (X + 25) (X - 15) = 0
Difference between 2nd& 3rd number X = 15
= 424 × (56 - 35) /106 = 4 × 21 = 84 Speed of Tanu : Speed of Rita = (15
+ 10) : 15 = 25 : 15 = 5 : 3
193. Ans. (D)
Investment of Aman = 3000 × 9 = Rs. 198. Ans. (C)
27000 Number of Tiles required = (120 ×
Investment of Piya = 3000/0.8 × 4 = 88) /(2 × 1.5) = 3520
Rs. 15000
Profit Ratio, Aman : Piya = 27000 : 199. Ans. (B)
15000 = 9 : 5 Initially, Soda = a, Water = b
Total Profit = 3600 × (9 + 5) /9 = 400 × (a - a/6) /(b - b/6 + a/6 + b/6) = 1/2
14 = Rs. 5600 2 × (a - a/6) = b + a/6
2 × 5a/6 = b + a/6
194. Ans. (B) 5a/3 - a/6 = b
Saving of Vihan : Saving of Ruhan 3a/2 = b
= (6800 - 5300) : (7200 - 4700) a:b=2:3
= 1500 : 2500 = 3 : 5
200. Ans. (D)
195. Ans. (A) Radius of Circle = r cm
Total Literate people = 10540 + 22/7 × r2 × 90/360 = 154
(10540 - 1560) r2 × 1/4 = 49
= 10540 + 8980 = 19520 r × 1/2 = 7
% of Illiterate people = (30500 - r = 14 cm
19520) /30500 × 100 = 36% Perimeter of Circle = 2 × 22/7 × 14 =
88 cm
196. Ans. (A)
1 + R/100 = 7320/4880
1 + R/100 = 1.5

https :
//www. https : https
https : //instagra
//youtube.c
facebo m.com/aas :
om/channe hisharoraso
l/UCYa4_Jr ok.com cial(?) //t.m
Orf8R5Kz2u
/aashis utm_mediu
e/stu

158
OtccXQ m=
haroras copy_link
ocial dified
201. Ans. (B) 5 × 4 = 2 × (12 - X)
Speed of Cycle = x km/hr 10 = 12 - X
Speed of Car = 2.75x km/hr X=2
132/2.75x + 36/x = 3 + 30/60
48/x + 36/x = 3.5 206. Ans. (A)
84/x = 3.5 At present, Age of Vipul = x years
x = 24 Age of Ravi = 0.9x years
Speed of Car = 2.75 × 24 = 66 km/hr (0.9x + 15) /(x + 15) = 14/15
15 × (0.9x + 15) = 14 × (x + 15)
202. Ans. (C) 13.5x + 225 = 14x + 210
Speed of Boat in still water = x km/hr 0.5x = 15
Downstream Speed = 1.2x km/hr x = 30
Speed of Stream = 1.2x - x = 0.2x 21 years Ago, Age of Vipul = 30 - 21
km/hr = 9 years
x - 0.2x × 0.9 = 246/12 Age of Ravi = 0.9 × 30 - 21 = 27 - 21 =
x - 0.18x = 20.5 6 years
0.82x = 20.5 Ratio = 9 : 6 = 3 : 2
x = 25
Time taken in Downstream = 246/(1.2 207. Ans. (C)
× 25) = 8.2 hours 3 digit number is = 162/(1.4 - 1) =
162/0.4 = 405
203. Ans. (D) Sum of digit = 4 + 0 + 5 = 9
CP of each item = Rs. x
39x - 12x = 7803 208. Ans. (C)
27x = 7803 P + Q ➔ 24 4
x = Rs. 289 } 96 {
P ➔ 32 3
204. Ans. (B) Work will be complete in = x days
Difference between Saving of 96 = (4 - 3) × x + 3 × (x - 60)
Ritesh&Mohak 96 = x + 3x - 180
= 3000 × 0.35 = Rs. 1050 4x = 276
x = 69 days
205. Ans. (A)
(15000 × 12) /{10000 × (12 - X) } =
1440/(2240 - 1440)
(15 × 12) /{10 × (12 - X) } = 1440/800 =
9/5

https :
//www. https : https
https : //instagra
//youtube.c
facebo m.com/aas :
om/channe hisharoraso
l/UCYa4_Jr ok.com cial(?) //t.m
Orf8R5Kz2u
/aashis utm_mediu
e/stu

159
OtccXQ m=
haroras copy_link
ocial dified
209. Ans. (B) 213. Ans. (D)
x2 + (x + 7) 2 = 132 Total employees in company =
x2 + x2 + 14x + 49 = 169 153/(0.85 × 0.3) = 600
2x2 + 14x - 120 = 0
2(x + 12) (x - 5) = 0 214. Ans. (B)
x=5 (X + 200) × (15/14 × 1/0.6 - 1) = 1012
Radius = x = 5 cm (X + 200) × (25/14 - 1) = 1012
Height = x + 7 = 5 = 7 = 12 cm X + 200 = 1012 × 14/11
Volume of Cone = 1/3 × π × 52 × 12 = X = 1288
100π cm3 X = 1088

210. Ans. (D) 215. Ans. (A)


0.5 × (X + 1500) × 2 × 1/7 = 3000 Initially, Income = Rs. 100
X + 1500 = 21000 Expenditure = 100 × 0.72 = Rs. 72
CI = 0.5 × 21000 × (1.12 - 1) = 10500 × Increased Expenditure = 100 × 1.324
0.21 = Rs. 2205 - 100 × 0.28
= 132.4 - 28 = Rs. 104.4
211. Ans. (B) % increase in Expenditure = (104.4 -
Quantity of water need to be added 72) /72 × 100 = 45%
= 1080 × 5/9 × 0.7/0.3 - 1080 × 4/9
= 1080 × (35/27 - 4/9) = 1080 × 23/27 216. Ans. (A)
= 920 L X × 18 = 28 × 60 - 46 × 24
X × 18 = 1680 - 1104 = 576
212. Ans. (C) X = 32
Common difference between age of
girls = x years 217. Ans. (C)
Age of middle girl = 48/3 = 16 years Height of Cone = (3080 × 3 × 7/22 ×
Age of youngest girl = (16 - x) years 1/142)
Age of eldest girl = (16 + x) years = 2940/196 = 15 cm
(16 - x) (16 + x) = 192
256 - x2 = 192 218. Ans. (C)
x2 = 64 Speed of Boat in still water = x km/hr
x=8 210/(x + 3) + 210/(x - 3) = 24
Age of youngest girl = 16 - 8 = 8 35(x - 3 + x + 3) = 4(x2 - 9)
years 35 × 2x = 4x2 - 36
2x2 - 35x - 18 = 0
(2x - 1) (x - 18) = 0

https :
//www. https : https
https : //instagra
//youtube.c
facebo m.com/aas :
om/channe hisharoraso
l/UCYa4_Jr ok.com cial(?) //t.m
Orf8R5Kz2u
/aashis utm_mediu
e/stu

160
OtccXQ m=
haroras copy_link
ocial dified
x = 18 km/hr 224. Ans. (B)
(x + 21) /(8x + 21) = 9/23
219. Ans. (B) 23 × (x + 21) = 9 × (8x + 21)
(x + 800) × (5 × 0.15 - 1.22 + 1) = 5084 23x + 483 = 72x + 189
(x + 800) × (0.75 - 1.44 + 1) = 5084 49x = 294
(x + 800) × 0.31 = 5084 x=6
x + 800 = 16400 8x = 8 × 6 = 48
x = 15600
2x + 700 = 2 × 15600 + 700 = 31200 + 225. Ans. (A)
700 = 31900 P + 2000 = 3500/(1 + 0.1 × 4)
P + 2000 = 3500/1.4 = 2500
220. Ans. (D) SI in 3 years at 15% = 2500 × 3 × 0.15
X + 40 = 144/(1 - 0.4 - 0.2) = Rs. 1125
X + 40 = 144/0.4 = 360
X = 320 226. Ans. (A)
22000 × 1.2 × (1 - X/100) × 0.85 =
221. Ans. (B) 13464
Saving of Ritu = 8x × 0.75 × 0.9 × 0.5 22000 × (1 - X/100) = 13200
= 2.7x 1 - X/100 = 0.6
Saving of Piya = 5x × 0.5184 = 2.592x X = 40%
Saving of Piya is less than that of
Ritu by 227. Ans. (C)
= (2.7x - 2.592x) /2.7x × 100 = 4% Mihir ➔ 50 3
} 150 {
222. Ans. (C) Anil + Mihir ➔ 30 5
In final mixture, Soda : Alcohol = Efficiency of Shubhi = (5 - 3) × 1.25 =
(270 × 11/18 + 35) : (270 × 7/19 + 25) 2 × 1.25 = 2.5
= (165 + 35) : (105 + 25) = 200 : 130 = Shubhi complete the whole work in
20 : 13 = 150/2.5 = 60 days

223. Ans. (D) 228. Ans. (C)


Unit Digit = 9/(1 + 1.25) = 9/2.25 = 4 (X × 5) /{(X + 4000) × 8} = 400/(1360 -
10’s Digit = 4 × 1.25 = 5
Difference = 5 - 4 = 1
h
t
t
p
s
:
/
/
i
n
s
t
a
g
r
a
m
.
c
o
m
/
a
a
s
h
i
https :
//www.
s
h
https
https :
//youtube.c
facebo a
r
:
om/channe
l/UCYa4_Jr ok.com o
r
//t.m
Orf8R5Kz2u
/aashis a e/stu

161
OtccXQ
haroras s
ocial o
c
dified
i
a
l
(
?
)
u
t
m
_
m
e
d
400) 18X = 1152 - 36 = 1116
(X × 5) /{(X + 4000) × 8} = 400/960 = X = 62
5/12
3 × X = 2 × (X + 4000) 234. Ans. (B)
3X = 2X + 8000 MP of Aricle = 10750/0.86 = Rs.
X = 8000 12500
Discount % given to Sonam = (12500
229. Ans. (B) - 8750) /12500 × 100 = 30%
Speed of Rashi = (24 × 4 × 3) /3.2 =
90 m/min 235. Ans. (A)
Time taken to cover circular field = Males in city Q = 1800000 × 0.55 ×
(2 × 22/7 × 7 × 6) /90 0.52 = 514800
= 264/90 = 44/15 min = 44/15 × 60 =
176 sec 236. Ans. (A)
Monthly Income of Ranu =
230. Ans. (D) 70980/(0.845 × 0.7)
Probability = (8 × 12) /29C2 = (8 × 12) = 70980/0.5915 = Rs. 120000
/406 = 48/203
237. Ans. (C)
231. Ans. (B) Distance between city X & city Y =
(896 × 7/16 + X) /(896 × 9/16 + X + 88) 61.44/2 = 30.72 km
= 2/3 Time taken to complete journey =
3 × (392 + X) = 2 × (504 + X + 88) 30.72/24 + 30.72/16
1176 + 3X = 2X + 1184 = 1.28 + 1.92 = 3.2 hours = 3 hours
X=8 12 minutes
X + 88 = 8 + 88 = 96
238. Ans. (C)
232. Ans. (C) 4200 × (R + 6) /100 × 4 + 5400 × (R +
Number of Days taken = x days 9) /100 × 4 = 14856 - (4200 + 5400)
(15 × 10 × 12) /9 = (16 × 7 × X) /14 42 × (R + 6) × 4 + 54 × (R + 9) × 4 =
5 × 10 × 4 = 8 × X 14856 - 9600 = 5256
X = 25 days 42R + 252 + 54R + 486 = 1314
96R = 1314 - 738 = 576
233. Ans. (D) R=6
X × 18 - (66 + 62 + 83) + (85 + 78 + 84)
= 64 × 18 239. Ans. (B)
18X - 211 + 247 = 1152

https :
//www. https : https
https : //instagra
//youtube.c
facebo m.com/aas :
om/channe hisharoraso
l/UCYa4_Jr ok.com cial(?) //t.m
Orf8R5Kz2u
/aashis utm_mediu
e/stu

162
OtccXQ m=
haroras copy_link
ocial dified
Speed of Stream = 2x km/hr Efficiency of Teena = (2 + 3) × 4/5 = 5
Speed of Boat in still water = 5x × 4/5 = 4
km/hr Together they complete the work in
126/(5x + 2x) + 126/(5x - 2x) = 20 = 90/(2 + 3 + 4) = 90/9 = 10 days
126/7x + 126/3x = 20
18/x + 42/x = 20 244. Ans. (B)
60/x = 20 Income of Rashi = Rs. 8x
x=3 Income of Pawan = Rs. 11x
Time taken = 45/3x + 63/7x (11x - 4000) - (8x - 2800) = 3600
= 15/3 + 9/x = 24/3 = 8 hours 3x - 1200 = 3600
3x = 4800
240. Ans. (D) x = 1600
Side of Square = √400 = 20 cm Total Income of both = 8x + 11x = 19
Length of Rectangle = √(292 - 202) × 1600 = Rs. 30400
= √(841 - 400) = √441 = 21 cm
Perimeter of Rectangle = 2 × (20 + 245. Ans. (A)
21) = 2 × 41 = 82 cm At present, Age of A = 5x years
Age of B = 4x years
241. Ans. (B) (5x + 20) /(4x + 20) = 15/13
MP of Article = 2160/1.2 + 600 = 1800 13 × (5x + 20) = 15 × (4x + 20)
+ 600 = Rs. 2400 65x + 260 = 60x + 300
Discount % = (2400 - 2160) /2400 × 5x = 40
100 = 10% x=8
Age of C = 4 × 8 - 6 = 32 - 6 = 26
242. Ans. (C) years
Sonu : Mohit : Mona = 4/3 : 5/2 : 7/3
= 8 : 15 : 14 246. Ans. (A)
Share of Mohit = 7585 × 15/37 = Rs. Net cost of watch for Neeta = 6800 ×
3075 1.15 + 680
= 7820 + 680 = Rs. 8500
243. Ans. (D) X = 11475 - 8500 = 2975
Rita ➔ 45 2
} 90 { 247. Ans. (C)
Mita ➔ 30 3 {P × (1 + r/100) 2 - P}/{P × r/100 × 2} =
4536/4200
{2r/100 + (r/100) 2}/(2r/100) = 1.08
1 + 1/2 × (r/100) = 1.08

https :
//www. https : https
https : //instagra
//youtube.c
facebo m.com/aas :
om/channe hisharoraso
l/UCYa4_Jr ok.com cial(?) //t.m
Orf8R5Kz2u
/aashis utm_mediu
e/stu

163
OtccXQ m=
haroras copy_link
ocial dified
r/100 = (1.08 - 1) × 2 = 0.16 251. Ans. (C)
r = 16% (X + 2000) /(x + 3000) = (48000 -
28000) /28000
248. Ans. (C) (X + 2000) /(X + 3000) = 20000/28000
Y + (Y + 250) = 4750 = 5/7
2Y = 4750 - 250 = 4500 7 × (X + 2000) = 5 × (X + 3000)
Y = 2250 7X + 14000 = 5X + 15000
{(X + 3000) × 12}/{(X + 9000) × 8} = 2X = 1000
2250/(2250 + 250) X = 500
{(X + 3000) × 3}/{(X + 9000) × 2} =
2250/2500 = 9/10 252. Ans. (B)
(X + 3000) /(X + 9000) = 3/5 (1.252 - 1) /(R/100 × 6) = 5/8
5 × (X + 3000) = 3 × (X + 9000) R/100 × 6 = (1.5625 - 1) × 8/5
5X + 15000 = 3X + 27000 R/100 × 6 = 0.5625 × 8/5 = 0.9
2X = 12000 R = 0.9/6 × 100 = 15
X = 6000
Investment of B = 6000 + 9000 = Rs. 253. Ans. (A)
15000 Initial quantity of Sugar in Bag P =
80 × 0.3/0.2 = 120 kg
249. Ans. (B)
{(X + 50) × 4/9}/{(X + 50) × 5/9 + 40} = 254. Ans. (C)
2/3 b + c = 1.5b
3 × (X + 50) × 4/9 = 2 × (X + 50) × 5/9 c = 0.5b
+ 2 × 40 b = 2c
(X + 50) × (4/3 - 10/9) = 80 c = 2a
(X + 50) × 2/9 = 80 a = 0.5c
X + 50 = 360 C will complete the work in
X = 310 = 15 × (a + b) /c = 15 × (0.5c + 2c) /c
= 15 × 2.5 = 37.5 = 37(1/2) days
250. Ans. (D)
Height of Cylinder = 2112/(22/7 × 82) 255. Ans. (B)
= 672/64 = 10.5 cm At present, Age of Chandan = (9x +
6) years
Age of Chaman = (7x + 6) years
(9x + 6) + 6 = (7x + 6) + 10
2x = 4

https :
//www. https : https
https : //instagra
//youtube.c
facebo m.com/aas :
om/channe hisharoraso
l/UCYa4_Jr ok.com cial(?) //t.m
Orf8R5Kz2u
/aashis utm_mediu
e/stu

164
OtccXQ m=
haroras copy_link
ocial dified
x=2 CP of B = 2500 - 1000 = Rs. 1500
After 12 years, Age of Chaman Difference = 1500 - 1000 = 500
= 7 × 2 + 6 + 12 = 14 + 18 = 32 days
261. Ans. (C)
256. Ans. (C) At present, Age of Renu = (30 - 3) × 2
Length of Train = 12 × 15 + 10 + 11 × - 24
1 + 0.8 = 27 × 2 - 24 = 54 - 24 = 30 years
= 180 + 10 + 11 + 0.8 = 201.8 m Age of Priya = 32 × 2 - 30 = 64 - 30 =
Time taken by train to cross 34 years
platform Age of Srishti = (36.5 + 3) × 2 - 34
= (201.8 + 29.2) /70 = 231/70 = 3.3 = 39.5 × 2 - 34 = 79 - 34 = 45 years
sec Age of Srishti : Age of Renu = 45 : 30
=3:2
257. Ans. (A)
Annual Income = 31992/(0.248 × 262. Ans. (B)
43/150) (8000 × 6) /{9500 × (6 - x) } = (10750 -
= 111600/0.248 = 450000 = 4.5 lakh 4750) /4750
(16 × 6) /{19 × (6 - x) } = 6000/4750 =
258. Ans. (B) 24/19
Height of Cylinder = (4/3 × 33) /22 = (4 4=6-x
× 32) /4 = 9 cm x=2

259. Ans. (C) 263. Ans. (A)


Distance between Delhi &Lucknow = Average of Sum received by A & B =
x km Rs. x
x/50 - x/80 = 6 Sum received by A = Rs. 1.15x
3x/400 = 6 Sum received by B = 2x - 1.15x = Rs.
x = 800 km 0.85x
1.15x - 0.85x = 2880
260. Ans. (C) 0.3x = 2880
CP of A = Rs. x x = 9600
CP of B = Rs. (2500 - x) Sum = 2 × 9600 = Rs. 19200
x × 1.15 + (2500 - x) × 1.25 = 3025
1.15x + 3125 - 1.25x = 3025 264. Ans. (C)
0.1x = 100 Maximum marks in exam
x = 1000 = (9 + 3) /(0.44 - 0.36) = 12/0.08 = 150
CP of A = Rs. 1000

https :
//www. https : https
https : //instagra
//youtube.c
facebo m.com/aas :
om/channe hisharoraso
l/UCYa4_Jr ok.com cial(?) //t.m
Orf8R5Kz2u
/aashis utm_mediu
e/stu

165
OtccXQ m=
haroras copy_link
ocial dified
Passing marks = 150 × 0.44 - 3 = 66 - Volume of Cylinder
3 = 63 = 22/7 × 3.52× 10 = 385 cm3

265. Ans. (B) 269. Ans. (A)


CP of mixture = 100/1,25 = Rs. 80 Downstream Speed = 76.8/(4 + 48/60)
Kashmiri Rice : Basmati Rice = 76.8/(4 + 0.8) = 76.8/4.8 = 16 km/hr
= (80 - 75) : (95 - 80) = 5 : 15 = 1 : 3 Upstream Speed = 16 × (7 - 1) /(7 + 1)
= 16 × 6/8 = 12 km/hr
266. Ans. (C)
Speed of Stream = x km/hr 270. Ans. (C)
Speed of Boat in still water = 3x 500 × 2 × 0.36 = x + 2x
km/hr 3x = 360
48/(3x + x) + 16/(3x - x) = 5 x = 120
48/4x + 16/2x = 5
12/x + 8/x = 5 271. Ans. (C)
20/x = 5 Sari sold at 10% profit : Sari sold at
x=4 20% profit
Time taken = 64/4x + 40/2x = (20 - 16) : (16 - 10) = 4 : 6 = 2 : 3
= 16/4 + 20/4 = 4 + 5 = 9 hours Sari sold at 20% profit = 1200 × 3/5 =
720
267. Ans. (A)
Garima ➔ 30 4 272. Ans. (B)
Prashant ➔ 24 } 120 {5 P : Q : R : S = 9 : 12 : 14 : 15
Rinkesh ➔ 20 6 Share of R = 75000 × 14/50 = Rs.
Work will be completed in = x days 21,000
4 × x + 5 × (x - 3) + 6 × (x - 5) = 120
4x + 5x - 15 + 6x - 30 = 120 273. Ans. (A)
15x = 120 + 45 = 165 Initial Quantity of mixture = x L
x = 11 days (0.45x + 75) /0.55x = 3/2
0.9x + 150 = 1.65x
268. Ans. (B) 0.75x = 150
Radius of Cylinder = r cm x = 200
2 × 22/7× r = 22 Initial Quantity of Milk = 200 × 0.45 =
r = 3.5 cm 90 L

274. Ans. (C)

https :
//www. https : https
https : //instagra
//youtube.c
facebo m.com/aas :
om/channe hisharoraso
l/UCYa4_Jr ok.com cial(?) //t.m
Orf8R5Kz2u
/aashis utm_mediu
e/stu

166
OtccXQ m=
haroras copy_link
ocial dified
Principal = 1120/(3 × 0.2 - 1.22 - 1) At present, Age of Jitu = 4x years
= 1120/(0.6 - 0.44) = 1120/0.16 = Rs. Age of Mahesh = 5x years
7000 Age of Tushar = (5x - 8) years
4x + (5x - 8) = 5x + 40 - 8
275. Ans. (B) 9x = 5x + 40
Difference between marks obtained 4x = 40
by A & D x = 10
= 3 × (73 - 67) = 3 × 6 = 18 At present, Age of Tushar
Marks obtained by A = (158 - 18) /2 = = 5 × 10 - 8 = 50 - 8 = 42 years
140/2 = 70
280. Ans. (C)
276. Ans. (C) A+B ➔8 3
Speed of Boat in Still water = x } 24 {
km/hr A ➔ 12 2
3 × (x + 3) - 4 × (x - 3) = 11 B work for = (24 - 4 × 2) /(3 - 2)
3x + 9 - 4x + 12 = 11 = (24 - 8) /1 = 16 years
x = 21 - 11 = 10
Time taken to cover 75 km in still 281. Ans. (C)
water Efficiency of B = 5
= 75/10 = 7.5 hours Efficiency of A = 6
Efficiency of C = (6 × 30) /(9 + 27/37)
277. Ans. (A) - (5 + 6)
l + b + h = 25 = 180 × 37/360 - 11 = 18.5 - 11 = 7.5
l2 + b2 + h2 = 152 C alone complete the work in = (6 ×
(l + b + h) 2 = 252 30) /7.5 = 24 days
l2 + b2 + h2 + 2 × (lb + bh + lh) = 625
152 + 2(lb + bh + lh) = 625 282. Ans. (B)
2(lb + bh + lh) = 625 - 225 = 400 Profit Ratio, Pawan : Shikha :
Total Surface Area of Body = 400 Suresh
cm2 = (7 × 5) : (6 × 3 + 12 × 2) : (8 × 5) =
35 : 42 : 40
278. Ans. (B) Difference between profit of Shikha&
Literate Female = 75000 × 0.8 × 2/5 - Suresh
75000 × 0.4 × 13/50 = 12250 × (42 - 40) /35 = 350 × 2 =
= 24000 - 7800 = 16200 Rs. 700

279. Ans. (A)

https :
//www. https : https
https : //instagra
//youtube.c
facebo m.com/aas :
om/channe hisharoraso
l/UCYa4_Jr ok.com cial(?) //t.m
Orf8R5Kz2u
/aashis utm_mediu
e/stu

167
OtccXQ m=
haroras copy_link
ocial dified
283. Ans. (A) 288. Ans. (B)
CP of Bike = 25000/(1.1 - 0.6) Initially, Milk = x L
= 25000/0.5 = Rs. 50000 Water = (72 - x) L
Actual SP = 50000 × 0.6 = Rs. 30000 x + 3 = 1.2 × (72 - x + 13)
x + 3 = 1.2 × 85 - 1.2x
284. Ans. (C) 2.2x = 102 - 3 = 99
Income of Kusal = 20 x = 45 L
Expenditure of Kusal = 20 × 4/5 = 16 Initially, Milk : Water = 45 : (72 - 45) =
Saving of Kusal = 20 × 1/5 = 4 45 : 27 = 5 : 3
Saving of Priya = 20 × 3/2 - 16 × 4/3 =
26/3 289. Ans. (A)
Saving of Vivek = 20 × 4/2 - 16 × 5/3 At present, Age of P = (10x - 6) years
= 40/3 Age of Q = 7x years
Ratio of Saving, Kusal : Priya L (10x - 6) - 7x = 3
Vivek 3x = 9
= 4 : 26/3 : 40/3 = 6 : 13 : 20 x=3
Age of P = 10 × 3 - 6 = 30 - 6 = 24
285. Ans. (B) years
Total Students = 500 Age of Q = 7 × 3 = 21 years
Passed Boys = 500 × 3/5 × 5/6 = 250 Sum of Age of P & Q = 24 + 21 = 45
Passed Boys : Total Students = 250 : years
500 = 1 : 2
290. Ans. (C)
286. Ans. (C) Rate of Interest at SI = (1 × 100) /(8 ×
Actual Time taken = x min 1) = 12.5%
Time taken after reducing speed = Rate of Interest at CI = r%
5x/2 min (1 + r/100) 2 = 4
5x/2 - x = 24 1 + r/100 = 2
3x/2 = 24 r = 100%
x = 16 min Rate of SI : Rate of CI = 12.5 : 100 = 1
:8
287. Ans. (A)
A invest his amount for = 14/15 ×
(5750 × 12) /6440
= 1150 × 4/460 = 10 months
A was not in the business for = 12 -
10 = 2 months

https :
//www. https : https
https : //instagra
//youtube.c
facebo m.com/aas :
om/channe hisharoraso
l/UCYa4_Jr ok.com cial(?) //t.m
Orf8R5Kz2u
/aashis utm_mediu
e/stu

168
OtccXQ m=
haroras copy_link
ocial dified
291. Ans. (C) 1.5x = 840
Capital Invested by B = Rs. x x = 560
Capital Invested by A = Rs. (9000 - x) SP of each item = 560 × 1.25 - 28 =
(9000 - x) × 1.5 = x 700 - 28 = Rs. 672
13500 - 1.5x = x
2.5x = 13500 296. Ans. (C)
x = Rs. 5400 Sum of Money = Rs. x
SI = x × 7 × 0.2 = Rs. 1.4x
292. Ans. (B) CI = x × (1.152 - 1) = Rs. 0.3225x
Earning after increment in price 1.4x - 0.3225x = 8620
= (450 × 0.18) × (250 × 1.12) 1.0775x = 8620
= 81 × 280 = Rs. 22680 x = Rs. 8000

293. Ans. (A) 297. Ans. (A)


Amount received by B = Rs. 7280 Kabir + Ronak ➔ 16 3
Amount received by A = 7280 × 3/4 = Ronak + Mehta ➔ 24 } 48 { 2
Rs. 5460 Ronak ➔ 48 1
Amount received by C = 5460 + 220 Efficiency of Kabir = 3 - 1 = 2
= Rs. 5680 Efficiency of Mehta = 2 - 1 = 1
Total Amount distributed by Ram Part of work done by Kabir& Mehta
= 7280 + 5460 + 5680 = Rs. 18420 in 12 days
Monthly Income of Ram = 18420/0.6 = 12 × (2 + 1) /48 = 3/4
= Rs. 30700
298. Ans. (B)
294. Ans. (C) Average Marks of 50 students
In final mixture, Chemical L : = (160 × 75 - 50 × 80 - 60 × 40) /50
Chemical Q = (12000 - 4000 - 2400) /50
= (960 × 7/10 + 480 × 13/20) : (960 × = (12000 - 6400) /50 = 5600/50 = 112
3/10 + 480 × 7/20)
= (672 + 312) : (288 + 168) = 984 :
456 = 41 : 19

295. Ans. (B)


CP of each item = x Rs.
30x + 6x = 30 × (x × 1.25 - 28)
36x = 37.5x - 840

https :
//www. https : https
https : //instagra
//youtube.c
facebo m.com/aas :
om/channe hisharoraso
l/UCYa4_Jr ok.com cial(?) //t.m
Orf8R5Kz2u
/aashis utm_mediu
e/stu

169
OtccXQ m=
haroras copy_link
ocial dified
299. Ans. (A)
Downstream Speed = x km/hr
Upstream Speed = y km/hr
165/x + 120/y = 39
132/x + 90/y = 30
x = 11 km/hr, y = 5 km/hr
Time taken = 220/11 + 80/5
= 20 + 16 = 36 hours

300. Ans. (C)


Maximum Marks in exam
= 117/(0.81 - 0.72) = 117/0.09 = 1300
Marks obtained by Nitin = 1300 ×
0.72 = 936 marks

https :
//www. https : https
https : //instagra
//youtube.c
facebo m.com/aas :
om/channe hisharoraso
l/UCYa4_Jr ok.com cial(?) //t.m
Orf8R5Kz2u
/aashis utm_mediu
e/stu

170
OtccXQ m=
haroras copy_link
ocial dified
https :
//instagram.com/aashisharorasocia
l(?) utm_medium = copy_link

https: //t.me/studified
https : //t.me/studified

https :
//youtube.com/channel/UCYa
4_JrOrf8R5Kz2uOtccXQ

https :
//www.facebook.com/aa
shisharorasocial

https :
//www. https : https
https : //instagra
//youtube.c
facebo m.com/aas :
om/channe hisharoraso
l/UCYa4_Jr ok.com cial(?) //t.m
Orf8R5Kz2u
/aashis utm_mediu
e/stu

171
OtccXQ m=
haroras copy_link
ocial dified
(1 - 5) Directions : Study the following passage carefully and answer the
questions given below.
The data given below is regarding monthly income and expenditure of three
friends P, Q and R. The ratio of monthly income of Q to R is 4 : 5 while the
ratio of monthly expenditure of P and Q is 7 : 4 respectively. Monthly income
of R is 28.57% less than monthly income of P. Average monthly savings of P
and Q is Rs. 11000. Monthly savings of R is 37.5% more than monthly savings
of Q. Monthly savings of P is Rs. 3000 more than monthly saving of R.
Monthly expenditure of Q is 14.28% more than monthly saving of P.
ननिे श : ननम्नललखखत गद्यांश का ध्यानपूवकष अध्ययन कीजजए और नीचे दिए गए प्रश्नों के उत्तर
िीजजए।
नीचे दिया गया डेटा तीन िोस्तों P, Q और R की मालसक आय और व्यय के बारे में है । Q से R की
मालसक आय का अनुपात 4 : 5 है जबकक P और Q के मालसक व्यय का अनुपात क्रमशः 7 : 4 है ।
R की मालसक आय P की मालसक आय से 28.57% कम है । P और Q की औसत मालसक बचत
11000 रुपये है । R की मालसक बचत Q की मालसक बचत से 37.5% अचधक है । P की मालसक
बचत, R की मालसक बचत से 3000 रुपये अचधक है । Q का मालसक व्यय P की मालसक बचत से
14.28% अचधक है ।

1. Find the average of monthly savings of P, Q and R.


P, Q और R की मालसक बचत का औसत ज्ञात कीजजए।
(A) Rs. 11000 (B) Rs. 11760 (C) Rs. 12820
(D) Rs. 13790 (E) None of these

2. Monthly expenditure of Q is what percent of sum of monthly expenditure of


P, Q and R?
Q का मालसक व्यय, P, Q और R के मालसक व्यय के योग का ककतना प्रनतशत है ?
(A) 28.89% (B) 36.12% (C) 25.39%
(D) 34.45% (E) None of these

3. Monthly expenditure of R is how much percent more or less than monthly


income of Q?
R का मालसक व्यय Q की मालसक आय से ककतने प्रनतशत अचधक या कम है ?
(A) 26.66% (B) 20.83% (C) 24.20%
(D) 25% (E) None of these

https :
//www. https : https
https : //instagra
//youtube.c
facebo m.com/aas :
om/channe hisharoraso
l/UCYa4_Jr ok.com cial(?) //t.m
Orf8R5Kz2u
/aashis utm_mediu
e/stu

172
OtccXQ m=
haroras copy_link
ocial dified
4. Find the ratio of expenditure of P to sum of expenditure of Q and R
together.
P के व्यय का Q और R के कुल व्यय के योग से अनप
ु ात ज्ञात कीजजए।
(A) 7 : 6 (B) 4 : 5 (C) 6 : 7
(D) 3 : 5 (E) None of these

5. Find the yearly income of Q.


Q की वावर्षक आय ज्ञात कीजजए।
(A) Rs. 228000 (B) Rs. 288000 (C) Rs. 224000
(D) Rs. 230000 (E) None of these

(6 - 10) Directions : Study the following passage carefully and answer the
questions given below.
The data is about number of persons who have Hotstar subscription in city A
to number of persons who have Netflix subscription in city B is 8 : 5. Number
of persons who have Netflix subscription in city A is 160 which is 20 less than
number of persons who have Hotstar subscription in city C. The ratio of
number of persons who have both subscription in city A to city C is 4 : 5.
Number of persons who have Hotstar subscription in city C is 25% less than
number of persons who have Hotstar subscription in city A. The ratio of
number of persons who have Hotstar subscription to number of persons who
have Netflix subscription in city B is 4 : 3 respectively.
ननिे श : ननम्नललखखत गद्यांश का ध्यानपव ू क
ष अध्ययन कीजजए और नीचे दिए गए प्रश्नों के उत्तर
िीजजए।
यह डेटा उन लोगों की संख्या के बारे में है जजनके पास शहर A में Hotstar सब्सकक्रप्शन है , शहर B
में Netflix सब्सकक्रप्शन वाले व्यजक्तयों की संख्या 8 : 5 है । शहर A में Netflix सब्सकक्रप्शन लेने
वाले व्यजक्तयों की संख्या 160 है जो शहर C में Hotstar सब्सकक्रप्शन लेने वाले व्यजक्तयों की
संख्या से 20 कम है । शहर A में िोनों सब्सकक्रप्शन लेने वाले व्यजक्तयों की संख्या का शहर C से
अनुपात 4 : 5 है । शहर C में Hotstar सब्सकक्रप्शन लेने वाले व्यजक्तयों की संख्या, उन व्यजक्तयों
की संख्या से 25% कम है जजनके पास शहर A में Hotstar सब्सकक्रप्शन है । शहर B में Hotstar
सब्सकक्रप्शन लेने वाले व्यजक्तयों की संख्या का Netflix सब्सकक्रप्शन लेने वाले व्यजक्तयों की
संख्या से अनुपात क्रमशः 4 : 3 है ।

6. Find the average of number of persons who have Netflix subscription in all
three cities.
उन व्यजक्तयों की औसत संख्या ज्ञात कीजजए जजनके पास तीनों शहरों में Netflix की सब्सकक्रप्शन
है ।
(A) 210 (B) 320 (C) 190
(D) 440 (E) None of these

https :
//www. https : https
https : //instagra
//youtube.c
facebo m.com/aas :
om/channe hisharoraso
l/UCYa4_Jr ok.com cial(?) //t.m
Orf8R5Kz2u
/aashis utm_mediu
e/stu

173
OtccXQ m=
haroras copy_link
ocial dified
7. Number of persons of city A who have Hotstar subscription is what percent
of total number of persons who have Hotstar subscription?
शहर A में Hotstar सब्सकक्रप्शन लेने वाले व्यजक्तयों की संख्या, Hotstar सब्सकक्रप्शन लेने वाले
व्यजक्तयों की कुल संख्या का ककतना प्रनतशत है ?
(A) 32.89% (B) 36.12% (C) 38.70%
(D) 34.45% (E) None of these

8. Total number of persons of city D who have both subscription is 32% more
than total number of persons of city C and number of persons of city D who
have Hotstar subscription is 20% more than number of persons of city C who
have Hotstar subscription, then find the number of persons of city D who
have Netflix subscription.
शहर D के व्यजक्तयों की कुल संख्या, जजनके पास िोनों सब्सकक्रप्शन हैं, शहर C के व्यजक्तयों की
कुल संख्या से 32% अचधक है और शहर D के व्यजक्तयों की संख्या, जजनके पास Hotstar
सब्सकक्रप्शन है , शहर C के व्यजक्तयों की संख्या से 20% अचधक है , जजनके पास Hotstar
सब्सकक्रप्शन है , तो शहर D के व्यजक्तयों की संख्या ज्ञात कीजजए जजनके पास Netflix
सब्सकक्रप्शन है ।
(A) 412 (B) 444 (C) 428
(D) 436 (E) None of these

9. Total number of persons of city A is how much percent more or less than
number of persons of city C who have Netflix subscription?
शहर A के व्यजक्तयों की कुल संख्या, शहर C के उन व्यजक्तयों की संख्या से ककतने प्रनतशत अचधक
या कम है , जजनके पास Netflix सब्सकक्रप्शन है ?
(A) 29% (B) 27% (C) 23%
(D) 25% (E) None of these

10. The ratio of number of male to female of city A who have Hotstar and
Netflix subscription is respectively 5 : 3 and 9 : 7, then find the number of
males of city A who have Hotstar and Netflix subscription.
शहर A में Hotstar और Netflix सब्सकक्रप्शन वाले परु
ु र्ों का मदहलाओं से अनप ु ात क्रमशः 5 : 3
और 9 : 7 है , तो शहर A में Hotstar और Netflix सब्सकक्रप्शन वाले पुरुर्ों की संख्या ज्ञात
कीजजए।
(A) 220 (B) 240 (C) 260
(D) 280 (E) None of these

(11- 15) Directions : Study the following passage carefully and answer the
questions given below.
Two shopkeepers A and B sold four different colors balls. The average of
number of balls sold of all four colors by B is 155. The ratio of number of Red

https :
//www. https : https
https : //instagra
//youtube.c
facebo m.com/aas :
om/channe hisharoraso
l/UCYa4_Jr ok.com cial(?) //t.m
Orf8R5Kz2u
/aashis utm_mediu
e/stu

174
OtccXQ m=
haroras copy_link
ocial dified
balls sold by A and B is 8 : 11 respectively. Number of Green balls sold by B is
12.5% more than number of Red balls sold by A. Total number of Yellow balls
sold is 290. Number of Black balls sold by B is 33.33% more than number of
Green balls sold. Number of Yellow balls sold by A is 16.66% less than
number of Black balls sold by B. Number of Black balls sold by A is equal to
average of number of Red and Green balls sold by B. Number of Green balls
sold by A is 13.33% more than number of Green balls sold by B.
ननिे श : ननम्नललखखत गद्यांश का ध्यानपव ू क
ष अध्ययन कीजजए और नीचे दिए गए प्रश्नों के उत्तर
िीजजए।
िो िक ु ानिार A और B ने चार अलग-अलग रं गों की गें िें बेचीं। B द्वारा सभी चार रं गों की बेची गई
गें िों की औसत संख्या 155 है । A और B द्वारा बेची गई लाल गें िों की संख्या का अनुपात क्रमशः 8
: 11 है । B द्वारा बेची गई हरी गें िों की संख्या, A द्वारा बेची गई लाल गें िों की संख्या से
12.5% अचधक है । बेची गई पीली गें िों की कुल संख्या 290 है । B द्वारा बेची गई काली गें िों की
संख्या, बेची गई हरी गें िों की संख्या से 33.33% अचधक है । A द्वारा बेची गई पीली गें िों की संख्या,
B द्वारा बेची गई काली गें िों की संख्या से 16.66% कम है । A द्वारा बेची गई काली गें िों की
संख्या, B द्वारा बेची गई लाल और हरी गें िों की औसत संख्या के बराबर है । A द्वारा बेची गई हरी
गें िों की संख्या, B द्वारा बेची गई हरी गें िों की संख्या से 13.33% अचधक है ।

11. Find the average of number of Red, Black and Yellow balls sold by A.
A द्वारा बेची गई लाल, काली और पीली गें िों की संख्या का औसत ज्ञात कीजजए।
(A) 140 (B) 120 (C) 160
(D) 180 (E) None of these

12. Number of Black balls sold by B is what percent of total number of balls
sold by B?
B द्वारा बेची गई काली गें िों की संख्या, B द्वारा बेची गई गें िों की कुल संख्या का ककतना प्रनतशत
है ?
(A) 28.89% (B) 33.12% (C) 29.03%
(D) 31.45% (E) None of these

13. Number of Red balls sold by B is how much percent more or less than
number of Green balls sold by B?
B द्वारा बेची गई लाल गें िों की संख्या, B द्वारा बेची गई हरी गें िों की संख्या से ककतने प्रनतशत
अचधक या कम है ?
(A) 23.56% (B) 22.22% (C) 24.41%
(D) 25.89% (E) None of these

https :
//www. https : https
https : //instagra
//youtube.c
facebo m.com/aas :
om/channe hisharoraso
l/UCYa4_Jr ok.com cial(?) //t.m
Orf8R5Kz2u
/aashis utm_mediu
e/stu

175
OtccXQ m=
haroras copy_link
ocial dified
14. Number of white balls sold by A and B is 10% and 25% more than number
of Yellow balls sold by A and B, then find the number of white balls sold by A
and B.
A और B द्वारा बेची गई सिेि गें िों की संख्या, A और B द्वारा बेची गई पीली गें िों की संख्या से
10% और 25% अचधक है , तो A और B द्वारा बेची गई सिेि गें िों की संख्या ज्ञात कीजजए।
(A) 360 (B) 380 (C) 320
(D) 340 (E) None of these

15.Find the ratio of sum of number of Red and Black balls sold by A to
number of Red and Green balls sold by B.
A द्वारा बेची गई लाल और काली गें िों की संख्या का B द्वारा बेची गई लाल और हरी गें िों की
संख्या से अनुपात ज्ञात कीजजए।
(A) 4 : 17 (B) 9 : 10 (C) 15 : 8
(D) 6 : 11 (E) None of these

(16 - 20) Directions : Study the following passage carefully and answer the
questions given below.
Number of Burgers sold by P is 36 more than number of Pizzas sold by P.
Number of Pizza and Burgers sold by R is 162 which is 25% less than number
of Pizzas and Burgers sold by P. The ratio of number of Pizzas sold by P to Q
is 5 : 6 respectively. The ratio of number of Burgers sold by R to number of
Burgers sold by P is 4 : 7 respectively. Number of Burgers sold by Q is equal
to average number of Pizza sold by P, Q and R. Number of Burgers sold by S
is 16.66% more than number of Burgers sold by R while number of Burgers
sold by S is 12.5% more than number of Burgers sold by Q.
ननिे श : ननम्नललखखत गद्यांश का ध्यानपव ू क
ष अध्ययन कीजजए और नीचे दिए गए प्रश्नों के उत्तर
िीजजए।
P द्वारा बेचे गए बगषर की संख्या P द्वारा बेचे गए वपज्जा की संख्या से 36 अचधक है । R द्वारा बेचे
गए वपज्जा और बगषर की संख्या 162 है जो P द्वारा बेचे गए वपज्जा और बगषर की संख्या से 25%
कम है । P और Q द्वारा बेचे गए वपज़्जा की संख्या का अनुपात क्रमशः 5 : 6 है । R द्वारा बेचे गए
बगषर की संख्या का P द्वारा बेचे गए बगषर की संख्या से क्रमशः 4 : 7 का अनप ु ात है । Q द्वारा बेचे
गए बगषर की संख्या P, Q और R द्वारा बेचे गए वपज्जा की औसत संख्या के बराबर है । S द्वारा
बेचे गए बगषर की संख्या R द्वारा बेचे गए बगषर की संख्या से 16.66% अचधक है जबकक S द्वारा
बेचे गए बगषर की संख्या Q द्वारा बेचे गए बगषर की संख्या से 12.5% अचधक है ।

16. Find the average of number of Burgers sold by P, Q and R.


P, Q और R द्वारा बेचे गए बगषर की औसत संख्या ज्ञात कीजजए।
(A) 98 (B) 76 (C) 82
(D) 65 (E) None of these

https :
//www. https : https
https : //instagra
//youtube.c
facebo m.com/aas :
om/channe hisharoraso
l/UCYa4_Jr ok.com cial(?) //t.m
Orf8R5Kz2u
/aashis utm_mediu
e/stu

176
OtccXQ m=
haroras copy_link
ocial dified
17. Number of Burgers sold by R is what percent of number of Burgers and
Pizzas sold by P?
R द्वारा बेचे गए बगषर की संख्या, P द्वारा बेचे गए बगषर और वपज्जा की संख्या का ककतना
प्रनतशत है ?
(A) 35.89% (B) 36.12% (C) 33.33%
(D) 34.45% (E) None of these

18. Number of Burgers sold by P is how much percent more or less than
number of Burgers sold by R?
P द्वारा बेचे गए बगषर की संख्या, R द्वारा बेचे गए बगषर की संख्या से ककतने प्रनतशत अचधक या
कम है ?
(A) 65% (B) 75% (C) 55%
(D) 45% (E) None of these

19. Find the ratio of number of Burgers sold by S to number of Pizzas sold by
R.
S द्वारा बेचे गए बगषर की संख्या का R द्वारा बेचे गए वपज्जा की संख्या से अनुपात ज्ञात कीजजए।
(A) 18 : 19 (B) 13 : 17 (C) 12 : 11
(D) 14 : 15 (E) None of these

20. Number of Donuts sold by R and S is respectively 16.66% and 11.11%


more than number of Pizzas sold by R and S, then find the sum of number of
Donuts sold by R and S.
R और S द्वारा बेचे गए डोनट्स की संख्या क्रमशः R और S द्वारा बेचे गए वपज्जा की संख्या से
16.66% और 11.11% अचधक है , तो R और S द्वारा बेचे गए डोनट्स की संख्या का योग ज्ञात
कीजजए।
(A) 498 (B) 225 (C) 356
(D) 242 (E) None of these

(21 - 25) Directions : Study the following passage carefully and answer
the questions given below.
The data is about number of persons who like three different juices. Each
of them like at least one juice. Number of persons who like Mango and
Orange juice is 200 which is equal to number of persons who like only
Orange juice. Sum of number of persons who like both Mango juice and
Pineapple juice but do not like Orange juice and number of persons who
like both Orange juice and Pineapple juice but do not like Mango juice is
300. Number of persons who like Orange juice is 580. The ratio of number
of persons who like only Pineapple juice to number of persons who like
only Orange juice is 6 : 5 respectively. Number of persons who like
Mango juice and Pineapple juice is

https :
//www. https : https
https : //instagra
//youtube.c
facebo m.com/aas :
om/channe hisharoraso
l/UCYa4_Jr ok.com cial(?) //t.m
Orf8R5Kz2u
/aashis utm_mediu
e/stu

177
OtccXQ m=
haroras copy_link
ocial dified
190. Number of persons who only Mango juice is 33.33% more than
number of persons who like Mango juice and Pineapple juice but do not
like Orange juice.
जनदेश : जनम्नजलखित गद्ांश का ध्यानपूवतक अध्ययन कीजिए और नीचे ददए गए प्रश्नों के
उत्तर दीजिए।
डेटा उन लोगों की संख्या के बारे में है िो तीन अलग-अलग िूस पसंद करते हैं । उनमें से
प्रत्येक को कम से कम एक रस पसंद है । आम और संतरे का रस पसंद करने वाले व्यक्तियों
की संख्या 200 है िो केवल संतरे का रस पसंद करने वाले व्यक्तियों की संख्या के बराबर है ।
उन व्यक्तियों की संख्या का योग िो आम का रस और अनानास का रस पसंद करते हैं
ले वकन संतरे का रस पसंद नहीं करते हैं और उन व्यक्तियों की संख्या का योग िो संतरे का
रस और अनानास का रस पसंद करते हैं ले वकन आम का रस पसंद नहीं करते हैं । संतरे का
रस पसंद करने वाले व्यक्तियों की संख्या 580 है । केवल अनानास का रस पसंद करने वाले
व्यक्तियों की संख्या का केवल संतरे का रस पसंद करने वाले व्यक्तियों की संख्या से क्रमशः
6 : 5 का अनुपात है । आम का रस और अनानास का रस पसंद करने वाले व्यक्तियों की
संख्या 190 है । केवल आम का रस पसंद करने वाले व्यक्तियों की संख्या, आम का रस और
अनानास का रस पसंद करने वाले ले वकन संतरे का रस पसंद नहीं करने वाले व्यक्तियों की
संख्या से 33.33% अत्रधक है ।

21. Find the total number of persons who like juices.


जूस पसंि करने वाले व्यजक्तयों की कुल संख्या ज्ञात कीजजए।
(A) 1100 (B) 1760 (C) 1820
(D) 1790 (E) None of these

22. Find the number of persons who like exactly two Juices.
िीक िो जूस पसंि करने वाले व्यजक्तयों की संख्या ज्ञात कीजजए।
(A) 455 (B) 395 (C) 430
(D) 345 (E) None of these

23. Number of persons who like all three juices is what percent of number of
persons who like only Pineapple juice?
तीनों रस पसंि करने वाले व्यजक्तयों की संख्या, केवल अनानास का रस पसंि करने वाले व्यजक्तयों
की संख्या का ककतना प्रनतशत है ?
(A) 30.40% (B) 29.16% (C) 25%
(D) 20.40% (E) None of these

24. Find the ratio of sum of number of persons who like only Mango juice and
number of persons who like both Mango and Pineapple juice but do not like
Orange juice to number of persons who like all three juices.
केवल आम का रस पसंि करने वाले व्यजक्तयों की संख्या और आम और अनानास िोनों का रस
पसंि करने वाले लेककन संतरे का रस पसंि नहीं करने वाले व्यजक्तयों की संख्या का तीनों रस पसंि
करने वाले व्यजक्तयों की संख्या से अनप
ु ात ज्ञात कीजजए।

https :
//www. https : https
https : //instagra
//youtube.c
facebo m.com/aas :
om/channe hisharoraso
l/UCYa4_Jr ok.com cial(?) //t.m
Orf8R5Kz2u
/aashis utm_mediu
e/stu

178
OtccXQ m=
haroras copy_link
ocial dified
(A) 7 : 5 (B) 7 : 6 (C) 5 : 3
(D) 4 : 1 (E) None of these

25. Number of persons who like Pineapple and Orange juice but do not like
Mango juice is how much percent more or less than number of persons who
like Mango and Orange juice but do not like Pineapple juice?
अनानास और संतरे का रस पसंि करने वाले लेककन आम का रस पसंि नहीं करने वाले व्यजक्तयों
की संख्या, आम और संतरे का रस पसंि करने वाले लेककन अनानास का रस पसंि नहीं करने वाले
व्यजक्तयों की संख्या से ककतने प्रनतशत अचधक या कम है ?
(A) 42.85% (B) 38.46% (C) 51.20%
(D) 33.50% (E) None of these

(26 - 30) Direction : Study the following passage carefully and answer the
questions given below.
The total number of mobile phones sold by a store in June and July is 1600
and all number of mobile sold in June is half more than that of in July. The
store sold mobile phones of just four brands for example Samsung, Phillips,
LG and Sony. 16% of total phones are sold by store Phillips and total number
of Phillips phones sold by store in the month of July is 40% less than that of
Phillips phones sold by store in the month of June. Number of Sony phones
sold by store in June is 20% more than that of Phillips phones sold by store
in the month of June. A total number of LG phones sold by store in the month
of June is 10% less than total Samsung phones in the same month and
number of Samsung phones sold by store in July is 10%more than total
Samsung phones sold by store in June. The proportion of Total LG and Sony
phones in the month of July is 5 : 3.

ननिे श : ननम्नललखखत गद्यांश का ध्यानपव ू कष अध्ययन कीजजए और नीचे दिए गए प्रश्नों के उत्तर
िीजजए।
जून और जुलाई में एक स्टोर द्वारा बेचे गए मोबाइल िोन की कुल संख्या 1600 है और जून में बेचे
गए मोबाइलों की संख्या जल ु ाई की तल
ु ना में आधी अचधक है । । स्टोर ने Samsung, Phillips,
LG और Sony जैसे लसिष चार ब्ांडों के मोबाइल िोन बेच।े कुल िोन का 16% Phillips स्टोर
द्वारा बेचा जाता है और जल ु ाई के महीने में स्टोर द्वारा बेचे गए Phillips िोन की कुल संख्या जन ू
के महीने में स्टोर द्वारा बेचे गए Phillips िोन की तुलना में 40% कम है । जून में स्टोर द्वारा बेचे
गए Sony िोन की संख्या जून के महीने में स्टोर द्वारा बेचे गए Phillips िोन की तुलना में 20%
अचधक है । जन ू के महीने में स्टोर द्वारा बेचे गए LG िोन की कुल संख्या उसी महीने में
Samsung के कुल िोन से 10% कम है और जुलाई में स्टोर द्वारा बेचे गए Samsung िोन की
संख्या जन ू में स्टोर द्वारा बेचे गए कुल Samsung िोन से 10% अचधक है । जल ु ाई के महीने में
कुल LG और Sony िोन का अनुपात 5 : 3 है ।

https :
//www. https : https
https : //instagra
//youtube.c
facebo m.com/aas :
om/channe hisharoraso
l/UCYa4_Jr ok.com cial(?) //t.m
Orf8R5Kz2u
/aashis utm_mediu
e/stu

179
OtccXQ m=
haroras copy_link
ocial dified
26. Total LG phones sold by store in June is what percent more than total
Sony phones sold by store in July?
जून में स्टोर द्वारा बेचे गए कुल LG फोन, जुलाई में स्टोर द्वारा बेचे गए कुल Sony फोन से
ककतने प्रनतशत अचधक हैं?
(A) 210% (B) 230% (C) 270%
(D) 300% (E) None of these

27. What percentage of total LG phones sold by store in June is less than
total Samsung phones sold by store in July?
जून में स्टोर द्वारा बेचे गए कुल LG िोन का ककतना प्रनतशत जुलाई में स्टोर द्वारा बेचे गए कुल
Samsung िोन से कम है ?
(A) 200/11% (B) 275/7% (C) 25%
(D) 150/7% (E) None of these

28. Find the ratio of total Sony phones sold by store in June to total Phillips
phones sold by store in July.
जून में स्टोर द्वारा बेचे गए कुल Sony िोन का जुलाई में स्टोर द्वारा बेचे गए कुल Phillips िोन
से अनुपात ज्ञात कीजजए।
(A) 3 : 1 (B) 2 : 3 (C) 2 : 1
(D) 4 : 1 (E) None of these

29. What was the average number of Phillips, LG, and Sony phones sold by
store in July?
जलु ाई में स्टोर द्वारा बेचे गए Phillips, LG और Sony िोन की औसत संख्या क्या थी?
(A) 92 (B) 100 (C) 102
(D) 96 (E) None of these

30. What is the difference between the total number of Samsung phones sold
by store in both months and the total number of LG phones sold by store in
both the months?
िोनों महीनों में स्टोर द्वारा बेचे गए Samsung िोन की कुल संख्या और िोनों महीनों में स्टोर
द्वारा बेचे गए LG िोन की कुल संख्या के बीच का अंतर ककतना है ?
(A) 253 (B) 251 (C) 255
(D) 264 (E) None of these

(31-35) Directions : Study the following passage carefully and answer the
questions given below.
A college has three sections in it and students enrolled according to their
preferences. These sections are Arts, Business and Cardio. Total
students in college are 460. 5% of the total students are enrolled for all
the three sections.

https :
//www. https : https
https : //instagra
//youtube.c
facebo m.com/aas :
om/channe hisharoraso
l/UCYa4_Jr ok.com cial(?) //t.m
Orf8R5Kz2u
/aashis utm_mediu
e/stu

180
OtccXQ m=
haroras copy_link
ocial dified
69 enrolled for both Business and Cardio but not enrolled for Arts.
Number of students enrolled for both Arts and Business but not enrolled
for Cardio is same as the number of students enrolled for both Arts and
Cardio but not enrolled for Business. Total students enrolled for Cardio is
50% of total students in college. Number of students enrolled for Arts
only is 5/3 times the number of students enrolled for both Business and
Cardio but not enrolled for Arts. 46 students enrolled for both Arts and
Business but not enrolled for Cardio.
जनदेश : जनम्नजलखित गद्ांश का ध्यानपूवतक अध्ययन कीजिए और नीचे ददए गए प्रश्नों के
उत्तर दीजिए।
एक कॉले ि में तीन िंड होते हैं और छािों को उनकी पसंद के अनुसार नामांवकत वकया
िाता है । ये सेक्शन हैं कला , व्यवसाय और कारडि यो। कॉले ि में कुल छाि 460 हैं । कुल
छािों में से 5% तीनों वगों के जलए नामांवकत हैं । 69 ने व्यवसाय और कारडि यो दोनों के जलए
नामांकन वकया ले वकन कला के जलए नामांवकत नहीं वकया। कला और व्यवसाय दोनों के
जलए नामांवकत ले वकन कारडि यो के जलए नामांवकत नहीं होने वाले छािों की संख्या कला
और कारडि यो दोनों के जलए नामांवकत ले वकन व्यवसाय के जलए नामांवकत नहीं होने वाले
छािों की संख्या के समान है । कारडि यो के जलए नामांवकत कुल छाि कॉले ि में कुल छािों
का 50% हैं । केवल कला के जलए नामांवकत छािों की संख्या व्यवसाय और कारडि यो दोनों के
जलए नामांवकत ले वकन कला के जलए नामांवकत नहीं होने वाले छािों की संख्या का 5/3
गुना है । 46 छािों ने कला और व्यवसाय दोनों के जलए नामांकन वकया ले वकन कारडि यो
नामांवकत नहीं वकया।

31. What is the total number of students enrolled for Business in college?
कॉलेज में व्यवसाय के ललए नामांककत छात्रों की कुल संख्या ककतनी है ?
(A) 208 (B) 230 (C) 207
(D) 240 (E) None of these

32. Sum of the number of students enrolled for Arts only and that enrolled for
all three sections is what percentage of more/less than the number of
students enrolled for Business only?
केवल कला के ललए नामांककत छात्रों की संख्या और सभी तीन वगों के ललए नामांककत छात्रों की
संख्या का योग केवल व्यवसाय के ललए नामांककत छात्रों की संख्या से ककतने प्रनतशत अचधक/कम
है ?
(A) 100% more (B) 100% less (C) 50% more
(D) 50% less (E) None of these

33. The enrollment fee of Arts student is Rs. 75 per student. Find the total
revenue generated from Arts section.
कला के छात्र का नामांकन शल्
ु क 75 रुपये प्रनत छात्र है । कला अनभ
ु ाग से उत्पन्न कुल राजस्व ज्ञात
कीजजए।

https :
//www. https : https
https : //instagra
//youtube.c
facebo m.com/aas :
om/channe hisharoraso
l/UCYa4_Jr ok.com cial(?) //t.m
Orf8R5Kz2u
/aashis utm_mediu
e/stu

181
OtccXQ m=
haroras copy_link
ocial dified
(A) Rs. 17255 (B) Rs. 17250 (C) Rs. 15000
(D) Rs. 15250 (E) None of these

34. What is the ratio of total students enrolled for Business to total students
enrolled for Arts?
व्यवसाय के ललए नामांककत कुल छात्रों का कला के ललए नामांककत कुल छात्रों से अनुपात ककतना
है ?
A) 8 : 9 (B) 4 : 7 (C) 9 : 10
(D) 13 : 11 (E) None of these

35. What is the average of total number of students enrolled for both Cardio
and Business but not enrolled for Arts, total number of students enrolled for
Arts only and total number of students enrolled for Cardio only?
काडडषयो और व्यवसाय िोनों के ललए नामांककत छात्रों की कुल संख्या, लेककन कला के ललए
नामांककत नहीं, केवल कला के ललए नामांककत छात्रों की कुल संख्या और केवल काडडषयो के ललए
नामांककत छात्रों की कुल संख्या का औसत क्या है ?
(A) 42 (B) 56 (C) 89
(D) 92 (E) None of these

(36 - 40) Directions : Study the following passage carefully and answer the
questions given below.
Aman distributes 40% of his monthly income among his three daughters A, B
and C and saves remaining Rs. 36000. A receives 1/3th of the amount received
by B and C together and B received Rs. 250 more than C. B spends 40% of her
share and saves the remaining amount. Expenditure of A and C are in the ratio
5 : 6 respectively. Expenditure of B is Rs. 650 more than the expenditure of A.
ननिे श : ननम्नललखखत गद्यांश का ध्यानपूवक ष अध्ययन कीजजए और नीचे दिए गए प्रश्नों के उत्तर
िीजजए।
अमन अपनी मालसक आय का 40% अपनी तीन बेदटयों A, B और C के बीच ववतररत करता है और
शेर् 36000 रुपये बचाता है । A को B और C द्वारा प्राप्त रालश का 1/3 भाग प्राप्त होता है और B
को C से 250 रुपये अचधक प्राप्त होते हैं। B अपने दहस्से का 40% खचष करती है और शेर् रालश बचा
लेती है । A और C का व्यय क्रमशः 5 : 6 के अनुपात में है । B का व्यय A के व्यय से 650 रुपये
अचधक है ।

36. What is the respective ratio of savings of B to savings of C?


B की बचत का C की बचत से संबंचधत अनुपात क्या है ?
(A) 208 : 315 (B) 230 : 211 (C) 219 : 211
(D) 240 : 255 (E) None of these

37. What is the average of expenditure of A, B and C?

https :
//www. https : https
https : //instagra
//youtube.c
facebo m.com/aas :
om/channe hisharoraso
l/UCYa4_Jr ok.com cial(?) //t.m
Orf8R5Kz2u
/aashis utm_mediu
e/stu

182
OtccXQ m=
haroras copy_link
ocial dified
A, B और C के व्यय का औसत ककतना है ?
(A) Rs. 3416.67 (B) Rs. 6500.32 (C) Rs. 3500
(D) Rs. 6012 (E) None of these

38. What is the difference between savings of A and B taken together and
share of C?
A और B की कुल बचत और C के रहस्से के बीच वकतना अंतर है ?
(A) Rs. 1700 (B) Rs. 400 (C) Rs. 1500
(D) Rs. 1600 (E) None of these

39. If C gives 2/5 of her savings into charity and A gives 3/5th of her savings
into charity, then find the remaining amount saved by A and C taken together.
यदि C अपनी बचत का 2/5 िान में िे ती है और A अपनी बचत का 3/5 भाग िान में िे ती है , तो A
और C द्वारा बचाई गई शेर् रालश ज्ञात कीजजए।
(A) Rs. 4175 (B) Rs. 4300 (C) Rs. 4365
(D) Rs. 4500 (E) None of these

40. Total saving of A, B and C taken together is approximately what percent of


monthly income of Aman?
A, B और C की लमलाकर कुल बचत, अमन की मालसक आय का लगभग ककतना प्रनतशत है ?
(A) 42% (B) 50% (C) 23%
(D) 23% (E) None of these

(41 - 45 ) Direction : Study the following passage carefully and answer the
questions given below.
There are three Villages : Village P, Village Q and Village R in a small city. Total
population of all the villages is 7200. All the residents of the villages are either
a Government employee or a Private employee. Out of the total residents 30%
are from village Q. Ratio of the Government employee to the Private employee
in village Q is 5 : 3. Number of Private employees in village P is 20% more
than the number of Private employees in village Q. Number of Government
employees in village P is 30% less than the number of Government employees
in village Q. Number of Government employees in village R is 1800.
ननिे श : ननम्नललखखत गद्यांश का ध्यानपव ू क
ष अध्ययन कीजजए और नीचे दिए गए प्रश्नों के उत्तर
िीजजए।
एक छोटे से शहर में तीन गाँव हैं : गाँव P, गाँव Q और गाँव R। सभी गांवों की कुल जनसंख्या 7200
है । सभी गांवों के ननवासी या तो सरकारी कमषचारी हैं या ननजी कमषचारी हैं। कुल ननवालसयों में से
30% गाँव Q से हैं। गाँव Q में सरकारी कमषचारी का ननजी कमषचारी से अनुपात 5 : 3 है । गाँव P में
ननजी कमषचाररयों की संख्या गाँव Q में ननजी कमषचाररयों की संख्या से 20% अचधक है । गाँव P में
सरकारी कमषचाररयों की संख्या गाँव Q में सरकारी कमषचाररयों की संख्या से 30% कम है । गाँव R में
सरकारी कमषचाररयों की संख्या 1800 है ।

https :
//www. https : https
https : //instagra
//youtube.c
facebo m.com/aas :
om/channe hisharoraso
l/UCYa4_Jr ok.com cial(?) //t.m
Orf8R5Kz2u
/aashis utm_mediu
e/stu

183
OtccXQ m=
haroras copy_link
ocial dified
41. Find the difference between the number of Government employees in
village P and number of Private employees in village Q.
गाँव P में सरकारी कमषचाररयों की संख्या और गाँव Q में ननजी कमषचाररयों की संख्या के बीच अंतर
ज्ञात कीजजए।
(A) 210 (B) 230 (C) 135
(D) 155 (E) None of these

42. Find the total population of village R.


गाँव R की कुल जनसंख्या ज्ञात कीजजए।
(A) 3123 (B) 2100 (C) 3215
(D) 2252 (E) None of these

43. Number of Government employees in village Q is what percent of the


number of Government employees in village R?
गाँव Q में सरकारी कमषचाररयों की संख्या, गाँव R में सरकारी कमषचाररयों की संख्या का ककतना
प्रनतशत है ?
(A) 70% (B) 75% (C) 60%
(D) 50% (E) None of these

44. Find the respective ratio between the number of Private employees in
Village P and in village R.
गाँव P और गाँव R में ननजी कमषचाररयों की संख्या के बीच संबंचधत अनुपात ज्ञात कीजजए।
(A) 35 : 29 (B) 45 : 31 (C) 36 : 49
(D) 49 : 36 (E) None of these

45. Total population in village P is what percent of total population of village


Q?
गाँव P की कुल जनसंख्या गाँव Q की कुल जनसंख्या का ककतना प्रनतशत है ?
(A) 54% (B) 33.33% (C) 14.28%
(D) 88.75% (E) None of these

(46-50) Direction : Study the following passage carefully and answer the
questions given below.
There are three mobile manufacturing companies namely Samsung, Apple
and MI. The sale of mobile phones in terms of units sold is given for
December 2022 and January 2023. The number of mobile phones sold by
Samsung in December 2022 is 25000 which is 9000 less than the number of
mobile phones sold by Apple in January 2023. The ratio between the number
of mobile phones sold by Apple and Samsung in December 2022 is 7 : 5. The
total number of mobile phones sold in December 2022 is 98000. MI sold 13500

https :
//www. https : https
https : //instagra
//youtube.c
facebo m.com/aas :
om/channe hisharoraso
l/UCYa4_Jr ok.com cial(?) //t.m
Orf8R5Kz2u
/aashis utm_mediu
e/stu

184
OtccXQ m=
haroras copy_link
ocial dified
more mobile phones than Samsung sold in January 2023. The total number of
mobile phones sold in January 2023 is 91500.

ननिे श : ननम्नललखखत गद्यांश का ध्यानपूवकष अध्ययन कीजजए और नीचे दिए गए प्रश्नों के उत्तर
िीजजए।
Samsung , Apple और MI नाम से तीन मोबाइल ननमाषण कंपननयां हैं। बेची गई इकाइयों के
संिभष में मोबाइल िोन की बबक्री दिसंबर 2022 और जनवरी 2023 के ललए िी गई है । दिसंबर 2022
में Samsung द्वारा बेचे गए मोबाइल िोन की संख्या 25000 है जो जनवरी 2023 में Apple
द्वारा बेचे गए मोबाइल िोन की संख्या से 9000 कम है । दिसंबर 2022 में Apple और
Samsung द्वारा बेचे गए मोबाइल िोन की संख्या के बीच का अनुपात 7 : 5 है । दिसंबर 2022 में
बेचे गए मोबाइल िोन की कुल संख्या 98000 है । MI ने जनवरी 2023 में Samsung की तल ु ना में
13500 अचधक मोबाइल िोन बेच।े जनवरी 2023 में बेचे गए मोबाइल िोन की कुल संख्या 91500
है ।

46. Find the difference between the number of mobile phones sold by MI in
January 2023 and number of mobile phones sold by Apple in December 2022.
जनवरी 2023 में MI द्वारा बेचे गए मोबाइल िोन की संख्या और दिसंबर 2022 में Apple द्वारा
बेचे गए मोबाइल िोन की संख्या के बीच अंतर ज्ञात कीजजए।
(A) 210 (B) 230 (C) 500
(D) 550 (E) None of these

47. By how much percent the sale of Apple phone has reduced from
December 2022 to January 2023?
दिसंबर 2022 से जनवरी 2023 तक Apple िोन की बबक्री ककतने प्रनतशत कम हुई है ?
(A) 2.85% (B) 2.6% (C) 3%
(D) 20% (E) None of these

48. Another mobile phone company is OnePlus which sold 20% more phones
than Samsung in December 2022, what is the number of mobile phones sold
by OnePlus in the same month and year?
एक अन्य मोबाइल िोन कंपनी OnePlus है जो दिसंबर 2022 में Samsung की तल ु ना में 20%
अचधक िोन बेचती है , उसी महीने और वर्ष में OnePlus द्वारा बेचे गए मोबाइल िोन की संख्या
ककतनी है ?
(A) 20,222 (B) 30,000 (C) 40,000
(D) 10,000 (E) None of these

49. The sale of phones produced by MI increased by 90% in December 2022 as


compared to the same month of the previous year, then what is the number of
mobile phones sold by MI in December 2021?

https :
//www. https : https
https : //instagra
//youtube.c
facebo m.com/aas :
om/channe hisharoraso
l/UCYa4_Jr ok.com cial(?) //t.m
Orf8R5Kz2u
/aashis utm_mediu
e/stu

185
OtccXQ m=
haroras copy_link
ocial dified
MI द्वारा उत्पादित िोन की बबक्री दिसंबर 2022 में वपछले वर्ष की तुलना में इसी महीने में 90%
बढ गई है , तो दिसंबर 2021 में MI द्वारा बेचे गए मोबाइल िोन की संख्या ककतनी है ?
(A) 10,000 (B) 30,000 (C) 20,000
(D) 45,000 (E) None of these

50. What are the average number of mobile phones sold by Apple in both
December 2022 and January 2023?
दिसंबर 2022 और जनवरी 2023 िोनों में Apple द्वारा बेचे गए मोबाइल िोन की औसत संख्या
ककतनी है ?
(A) 25300 (B) 25100 (C) 25500
(D) 34500 (E) None of these

(51 - 55) Directions : Study the following passage carefully and answer the
questions given below.
The data given is regarding the number of magazines in three different
languages sold by two shopkeepers A and B. Total number of magazines sold
by both the shopkeepers is 3000 out of which 28% is Telugu magazines sold
by both the shopkeepers. The number of Telugu magazines sold by
shopkeeper A is 40 less than that of shopkeeper B and the number of Hindi
magazines sold by shopkeeper A is 66(2/3)% of the number of English
magazines sold by shopkeeper A. The number of Telugu magazines sold by
shopkeeper B is 20 more than the half of the number of Hindi magazines sold
by B and total Hindi magazines sold by both the shopkeepers is 160 more
than total English magazines by shopkeepers.
ननिे श : ननम्नललखखत गद्यांश का ध्यानपूवक ष अध्ययन कीजजए और नीचे दिए गए प्रश्नों के उत्तर
िीजजए।
दिया गया डेटा िो िक ु ानिारों A और B द्वारा बेची गई तीन अलग - अलग भार्ाओं में पबत्रकाओं की
संख्या के संबंध में है । िोनों िकु ानिारों द्वारा बेची गई पबत्रकाओं की कुल संख्या 3000 है , जजसमें से
28% तेलुगु पबत्रकाएं िोनों िक ु ानिारों द्वारा बेची जाती हैं। िक
ु ानिार A द्वारा बेची गई तेलुगु
पबत्रकाओं की संख्या िक ु ानिार B की तुलना में 40 कम है और िक ु ानिार A द्वारा बेची गई दहंिी
पबत्रकाओं की संख्या, िक ु ानिार A द्वारा बेची गई अंग्रेजी पबत्रकाओं की संख्या का 66(2/3)% है ।
तेलुगु की संख्या िक ु ानिार B द्वारा बेची गई पबत्रकाएँ B द्वारा बेची गई दहंिी पबत्रकाओं की संख्या
के आधे से 20 अचधक हैं और िोनों िक ु ानिारों द्वारा बेची गई कुल दहंिी पबत्रकाएँ िकु ानिारों द्वारा
बेची गई कुल अंग्रेजी पबत्रकाओं से 160 अचधक हैं।

51. What is the average number of magazines in all the three languages sold
by shopkeeper A?
िक
ु ानिार A द्वारा बेची गई तीनों भार्ाओं की पबत्रकाओं की औसत संख्या ककतनी है ?

https :
//www. https : https
https : //instagra
//youtube.c
facebo m.com/aas :
om/channe hisharoraso
l/UCYa4_Jr ok.com cial(?) //t.m
Orf8R5Kz2u
/aashis utm_mediu
e/stu

186
OtccXQ m=
haroras copy_link
ocial dified
(A) 210 (B) 230 (C) 400
(D) 155 (E) None of these

52. The number of Telugu magazine sold by shopkeeper B is what percent


more or less than the number of English magazines sold by shopkeeper A?
िक
ु ानिार B द्वारा बेची गई तेलग
ु ू पबत्रका की संख्या, िक
ु ानिार A द्वारा बेची गई अंग्रेजी पबत्रकाओं
की संख्या से ककतने प्रनतशत अचधक या कम है ?
(A) 8.33% (B) 12.5% (C) 11.11%
(D) 22.52% (E) None of these

53. What is the ratio of the number of Telugu and English magazines together
sold by shopkeeper B to the number of English and Hindi magazines together
sold by shopkeeper A?
िकु ानिार B द्वारा बेची गई तेलुगू और अंग्रेजी पबत्रकाओं की कुल संख्या का िक
ु ानिार A द्वारा
बेची गई अंग्रेजी और दहंिी पबत्रकाओं की कुल संख्या से अनुपात ककतना है ?
(A) 5 : 6 (B) 6 : 5 (C) 1 : 2
(D) 5 : 1 (E) None of these

54. The number of German magazines is 30% more than total number of
English magazines sold by A and B. Find the difference between total number
of German magazines and total number of Telugu magazines sold by
shopkeeper A and B both.
जमषन पबत्रकाओं की संख्या A और B द्वारा बेची गई अंग्रेजी पबत्रकाओं की कुल संख्या से 30%
अचधक है । जमषन पबत्रकाओं की कुल संख्या और िक ु ानिार A और B िोनों द्वारा बेची गई तेलुगू
पबत्रकाओं की कुल संख्या के बीच अंतर ज्ञात कीजजए।
(A) 350 (B) 450 (C) 460
(D) 490 (E) None of these

55. Find the difference between the total number of magazines sold by
shopkeeper A and that of total magazines sold by shopkeeper B.
िक
ु ानिार A द्वारा बेची गई पबत्रकाओं की कुल संख्या और िक
ु ानिार B द्वारा बेची गई कुल
पबत्रकाओं के बीच का अंतर ज्ञात कीजजए।
(A) 500 (B) 330 (C) 140
(D) 600 (E) None of these

(56 - 60) Directions : Study the following passage carefully and answer the
questions given below.
Following is the data regarding the number of males and females and the
number of literates and illiterates in 3 Villages : A, B and C.
Gender ratio = Number of Males : Number of Females

https :
//www. https : https
https : //instagra
//youtube.c
facebo m.com/aas :
om/channe hisharoraso
l/UCYa4_Jr ok.com cial(?) //t.m
Orf8R5Kz2u
/aashis utm_mediu
e/stu

187
OtccXQ m=
haroras copy_link
ocial dified
The are 600 people in village A where the gender ratio is 3 : 2 and the
percentage literacy is 66(2/3)%. The number of males in village A is same as
that in village B and the ratio of the number of literates to illiterates in village
B is 5 : 3. Out of all females, 45% are from village C. Number of females in
village B is 160 less than in village C. The total number of people in village C
is 40 more than double the number of literates in village A. There are 620
illiterate people in all 3 villages combined.
ननिे श : ननम्नललखखत गद्यांश का ध्यानपव ू क
ष अध्ययन कीजजए और नीचे दिए गए प्रश्नों के उत्तर
िीजजए।
3 गांवों A, B और C में पुरुर्ों और मदहलाओं की संख्या और साक्षरों और ननरक्षरों की संख्या के बारे
में आंकडे ननम्नललखखत हैं।
ललंगानुपात = पुरुर्ों की संख्या : मदहलाओं की संख्या
गाँव A में 600 लोग हैं जहाँ ललंगानप ु ात 3 : 2 है और प्रनतशत साक्षरता 66(2/3)% है । गाँव A में
पुरुर्ों की संख्या गाँव B के समान है और गाँव B में साक्षरों और ननरक्षरों की संख्या का अनुपात 5 :
3 है । सभी मदहलाओं में से 45% गाँव C से हैं। गाँव B में मदहलाओं की संख्या गाँव C से 160 कम
है । गाँव C में लोगों की कुल संख्या गाँव A में साक्षरों की संख्या के िोगन
ु े से 40 अचधक है । सभी 3
गाँवों में संयुक्त रूप से 620 ननरक्षर लोग हैं।

56. What is the over all gender ratio in all the villages combined?
सभी गांवों को लमलाकर समग्र ललंगानुपात ककतना है ?
(A) 4 : 5 (B) 2 : 3 (C) 3 : 2
(D) 4 : 3 (E) None of these

57. What is the over all percentage literacy in all the villages combined?
सभी गांवों में कुल लमलाकर कुल साक्षरता का प्रनतशत ककतना है ?
(A) 69% (B) 100% (C) 50%
(D) 75% (E) None of these

58. The sum of the number of males in village A and B is what percent greater
than the sum of the number of females in villages A and C?
गाँव A और B में परु
ु र्ों की संख्या का योग, गाँव A और C में मदहलाओं की संख्या के योग से
ककतना प्रनतशत अचधक है ?
(A) 60% (B) 20% (C) 50%
(D) 15% (E) None of these

59. What is the difference between the average number of literates in village B
and C and the average number of illiterates in village A and B?
गाँव B और C में साक्षरों की औसत संख्या और गाँव A और B में ननरक्षरों की औसत संख्या के बीच
ककतना अंतर है ?

https :
//www. https : https
https : //instagra
//youtube.c
facebo m.com/aas :
om/channe hisharoraso
l/UCYa4_Jr ok.com cial(?) //t.m
Orf8R5Kz2u
/aashis utm_mediu
e/stu

188
OtccXQ m=
haroras copy_link
ocial dified
(A) 289 (B) 470 (C) 285
(D) 570 (E) None of these

60. What is the ratio of the difference between the number of literates and
number of females in village A to the difference between the number of males
and illiterates in village C?
गाँव A में साक्षरों की संख्या और मदहलाओं की संख्या के बीच के अंतर का गाँव C में पुरुर्ों और
ननरक्षरों की संख्या के बीच के अंतर से अनपु ात ककतना है ?
(A) 12 : 17 (B) 16 : 27 (C) 19 : 17
(D) 116 : 17 (E) None of these

(6 1- 65) Directions : Study the following passage carefully and answer the
questions given below.
In an exhibition of paintings, there were paintings of three most popular
painters A, B and C. It was found that total 800 customers have visited
exhibition. 40% customers purchased painting of A, 60% customers
purchased painting of B and 50% customers purchased painting of C. 40
customers purchased paintings of all three painters. 120 customers
purchased painting of only A and B but not C, 80 customers purchased
paintings of only A and C But not B and 140 customers purchased paintings
from only B and C but not A.
ननिे श : ननम्नललखखत गद्यांश का ध्यानपूवक ष अध्ययन कीजजए और नीचे दिए गए प्रश्नों के उत्तर
िीजजए।
चचत्रों की एक प्रिशषनी में , तीन सबसे लोकवप्रय चचत्रकारों A, B और C के चचत्र थे। यह पाया गया कक
कुल 800 ग्राहकों ने प्रिशषनी का िौरा ककया। 40% ग्राहकों ने A की पें दटंग खरीिी, 60% ग्राहकों ने B
की पें दटंग खरीिी और 50% ग्राहकों ने C की पें दटंग खरीिी। 40 ग्राहकों ने तीनों पें टरों की पें दटंग
खरीिीं। 120 ग्राहकों ने केवल A और B की पें दटंग खरीिी लेककन C की नहीं, 80 ग्राहकों ने केवल A
और C की पें दटंग खरीिी लेककन B की नहीं और 140 ग्राहकों ने केवल B और C की पें दटंग खरीिी
लेककन A की नहीं।

61. What is the total number of customers who purchased paintings of only A,
only B and only C?
केवल A, केवल B और केवल C की पें दटंग खरीिने वाले ग्राहकों की कुल संख्या ककतनी है ?
(A) 200 (B) 300 (C) 400
(D) 240 (E) None of these

62. How many customers did not purchase any of the three painter's
paintings?
ककतने ग्राहकों ने तीनों चचत्रकारों की कोई भी पें दटंग नहीं खरीिी?

https :
//www. https : https
https : //instagra
//youtube.c
facebo m.com/aas :
om/channe hisharoraso
l/UCYa4_Jr ok.com cial(?) //t.m
Orf8R5Kz2u
/aashis utm_mediu
e/stu

189
OtccXQ m=
haroras copy_link
ocial dified
(A) 20 (B) 30 (C) 40
(D) 50 (E) None of these

63. How many of them purchased two paintings of at least two painters?
उनमें से ककतने लोगों ने कम से कम िो चचत्रकारों की िो पें दटंग खरीिीं?
(A) 350 (B) 450 (C) 450
(D) 420 (E) None of these

64. Total number of customers who purchased paintings of minimum two


painters are approximately what percent of customers who purchased
paintings of maximum two painters?
न्यूनतम िो चचत्रकारों की पें दटंग खरीिने वाले ग्राहकों की कुल संख्या, अचधकतम िो चचत्रकारों की
पें दटंग खरीिने वाले ग्राहकों की संख्या का लगभग ककतना प्रनतशत है ?
(A) 60% (B) 40% (C) 51%
(D) 75% (E) None of these

65. Total number of customers who purchased A's paintings are


approximately what percent more or less than customers who purchased
paintings of only B?
A की पें दटंग खरीिने वाले ग्राहकों की कुल संख्या, केवल B की पें दटंग खरीिने वाले ग्राहकों की तुलना
में लगभग ककतने प्रनतशत अचधक या कम है ?
(A) 42% (B) 56% (C) 89%
(D) 78% (E) None of these

(66 - 70) Directions : Study the following passage carefully and answer the
questions given below.
Following is the information about number of boys and number of girls in
each of the 5 (A, B, C, D and E) classes. Ratio of total number of boys to the
total number of girls in class A is 9 : 7 and total number of students in class A
is 80. Total number of girls in class B is 20% less than the total number of
boys in class B and total number of students in class B is 28 more than total
number of students in class A. Number of boys and girls in class C are 50 and
60 respectively. Number of boys and girls in class D are 14 less and 4 more
than the number of boys and girls in class C respectively. The ratio of number
of boys to the girls in class E is 2 : 3 and total number of students in class E
is 10 more than the total number of students in class D.
ननिे श : ननम्नललखखत गद्यांश का ध्यानपूवक ष अध्ययन कीजजए और नीचे दिए गए प्रश्नों के उत्तर
िीजजए।
ननम्नललखखत जानकारी 5 कक्षाओं (A, B, C, D और E) में से प्रत्येक में लडकों की संख्या और
लडककयों की संख्या के बारे में है । कक्षा A में लडकों की कुल संख्या का लडककयों की कुल संख्या से

https :
//www. https : https
https : //instagra
//youtube.c
facebo m.com/aas :
om/channe hisharoraso
l/UCYa4_Jr ok.com cial(?) //t.m
Orf8R5Kz2u
/aashis utm_mediu
e/stu

190
OtccXQ m=
haroras copy_link
ocial dified
अनुपात 9 : 7 है और कक्षा A में छात्रों की कुल संख्या 80 है । कक्षा B में लडककयों की कुल संख्या,
कक्षा B में लडकों की कुल संख्या से 20% कम है और कक्षा B में छात्रों की कुल संख्या कक्षा A में
छात्रों की कुल संख्या से 28 अचधक है । कक्षा C में लडकों और लडककयों की संख्या क्रमशः 50 और
60 है । कक्षा D में लडकों और लडककयों की संख्या कक्षा C में लडकों और लडककयों की संख्या से
क्रमशः 14 कम और 4 अचधक है । कक्षा E में लडकों की संख्या का लडककयों से अनप ु ात 2 : 3 है और
कक्षा E में छात्रों की कुल संख्या कक्षा D में छात्रों की कुल संख्या से 10 अचधक है ।

66. What is the ratio of the total number of boys in classes B and C together
to the total number of girls in these two classes together?
कक्षा B और C में लमलाकर लडकों की कुल संख्या का इन िो कक्षाओं में लमलाकर लडककयों की कुल
संख्या से अनुपात ककतना है ?
(A) 21 : 51 (B) 23 : 55 (C) 56 : 54
(D) 55 : 54 (E) None of these

67. What is the average number of girls in all five classes together?
सभी पांच कक्षाओं में लमलाकर लडककयों की औसत संख्या ककतनी है ?
(A) 54.6 (B) 21.56 (C) 32.15
(D) 22 (E) None of these

68. What is the difference between the number of boys in class C and average
number of boys in all five classes together?
कक्षा C में लडकों की संख्या और सभी पाँच कक्षाओं में लडकों की औसत संख्या के बीच ककतना
अंतर है ?
(A) 2 (B) 3 (C) 10
(D) 5 (E) None of these

69. In which of the class number of girls are more than average number of
girls in all five classes together?
ककस कक्षा में लडककयों की संख्या सभी पाँच कक्षाओं में लडककयों की औसत संख्या से अचधक है ?
(A) Only A (B) Only B (C) All C,D,E
(D) Only D (E) None of these

70. Total number of students in class B is what percentage more than total
number of students in class D?
कक्षा B में ववद्याचथषयों की कुल संख्या, कक्षा D में ववद्याचथषयों की कुल संख्या से ककतने प्रनतशत
अचधक है ?
(A) 10% (B) 5% (C) 6%
(D) 8% (E) None of these

https :
//www. https : https
https : //instagra
//youtube.c
facebo m.com/aas :
om/channe hisharoraso
l/UCYa4_Jr ok.com cial(?) //t.m
Orf8R5Kz2u
/aashis utm_mediu
e/stu

191
OtccXQ m=
haroras copy_link
ocial dified
(71-75) Directions : Study the following passage carefully and answer the
questions given below.
A flower shop owner sells three types of flowers : Rose, Lilly and Jasmine.
Total number of Rose flowers in the shop is twice the number of Lilly flowers
in the shop. Total number of Jasmine flowers in the shop is half the number of
Lilly flowers in the shop. 2/3rd of the total Rose flowers were sold and the rest
were unsold. Total number of Lily flowers that were sold are 150 less than the
flowers sold of Rose and rest of the Lilly flowers were unsold. Only 10 flowers
of Jasmine were unsold and rest were sold. Total number of Rose flowers
present in the shop were 600.
ननिे श : ननम्नललखखत गद्यांश का ध्यानपूवक ष अध्ययन कीजजए और नीचे दिए गए प्रश्नों के उत्तर
िीजजए।
एक िूल की िक ु ान का माललक तीन प्रकार के िूल बेचता है : गुलाब, ललली और चमेली। िक ु ान में
गुलाब के िूलों की कुल संख्या िक ु ान में ललली के िूलों की संख्या की िोगुनी है । िक
ु ान में चमेली के
िूलों की कुल संख्या िकु ान में ललली के िूलों की संख्या की आधी है । कुल गुलाब के िूलों का 2/3
भाग बेच दिया गया और शेर् बबना बबके रह गए। बेचे गए ललली के िूलों की कुल संख्या, गुलाब के
बेचे गए िूलों की संख्या से 150 कम है और शेर् ललली के िूल बबना बबके थे। चमेली के केवल 10
िूल नहीं बबके और बाकी बबक गए। िक ु ान में मौजूि गुलाब के िूलों की कुल संख्या 600 थी।

71. Total Jasmine flowers present in the shop were approximately what
percent of the sold flowers of Rose and sold flowers of Lilly?
िक
ु ान में मौजि
ू कुल चमेली के िूल गल
ु ाब के बेचे गए िूलों और ललली के बेचे गए िूलों का लगभग
ककतना प्रनतशत है ?
(A) 42% (B) 30% (C) 23%
(D) 15% (E) None of these

72. Find the total number of unsold flowers of all three types.
तीनों प्रकार के बबना बबके िूलों की कुल संख्या ज्ञात कीजजए।
(A) 260 (B) 450 (C) 400
(D) 150 (E) None of these

73. Total number of Lilly flowers sold are what percent more than unsold Rose
flowers?
बेचे गए ललली के िूलों की कुल संख्या बबना बबके गुलाब के िूलों की संख्या से ककतने प्रनतशत
अचधक है ?
(A) 70% (B) 25% (C) 60%
(D) 50% (E) None of these

https :
//www. https : https
https : //instagra
//youtube.c
facebo m.com/aas :
om/channe hisharoraso
l/UCYa4_Jr ok.com cial(?) //t.m
Orf8R5Kz2u
/aashis utm_mediu
e/stu

192
OtccXQ m=
haroras copy_link
ocial dified
74. Find the ratio of total number of Jasmine flowers present to the total
number of unsold Rose flowers and unsold Lilly flowers.
मौजिू चमेली के िूलों की कुल संख्या का बबना बबके गल
ु ाब के िूलों और बबना बबके ललली के िूलों
की कुल संख्या से अनुपात ज्ञात कीजजए।
(A) 35 : 29 (B) 4 : 3 (C) 3 : 5
(D) 49 : 36 (E) None of these

75. If some client of the shop wants to decorate his home with Rose flowers
and buys the unsold Rose flowers of the shop for Rs. 10,000. Find the price of
each unsold Rose flower bought by client.
अगर िक ु ान का कोई ग्राहक अपने घर को गल ु ाब के िूलों से सजाना चाहता है और िक ु ान के बबना
बबके गुलाब के िूलों को 10,000 रुपये में खरीिता है । ग्राहक द्वारा खरीिे गए प्रत्येक बबना बबके
गलु ाब के िूल की कीमत ज्ञात कीजजए।
(A) 60 (B) 20 (C) 100
(D) 50 (E) None of these

(76 - 80) Directions : Study the following passage carefully and answer the
questions given below.
The data is about number of persons who like Coffee, Tea and Juice. Each
persons like at least one drink. Number of persons who like only Coffee is
120. The ratio of number of persons who like both Coffee and Tea to number
of persons who like only Coffee is 6 : 5 respectively. Number of persons who
like Coffee and Tea but do not like Juice is 25% less than number of persons
who like Tea and Juice but do not like Coffee. Number of persons who like
Coffee is 336. Number of persons who like only Juice is 100% more than
number of persons who like both Coffee and Juice but do not like Tea, which
is 12.5% more than number of persons who like both Tea and Juice but do not
like Coffee. Number of persons who like only Tea is 33.33% more than number
of persons who like only Juice.
ननिे श : ननम्नललखखत गद्यांश का ध्यानपूवक ष अध्ययन कीजजए और नीचे दिए गए प्रश्नों के उत्तर
िीजजए।
डेटा कॉिी, चाय और जूस पसंि करने वाले व्यजक्तयों की संख्या के बारे में है । प्रत्येक व्यजक्त कम
से कम एक पेय पसंि करता है । केवल कॉफी पसंि करने वाले व्यजक्तयों की संख्या 120 है । कॉफी
और चाय िोनों पसंि करने वाले व्यजक्तयों की संख्या का केवल कॉफी पसंि करने वाले व्यजक्तयों
की संख्या से अनुपात क्रमशः 6 : 5 है । कॉिी और चाय पसंि करने वाले लेककन जूस पसंि नहीं
करने वाले व्यजक्तयों की संख्या चाय और जस ू पसंि करने वाले लेककन कॉिी पसंि नहीं करने वाले
व्यजक्तयों की संख्या से 25% कम है । कॉिी पसंि करने वाले व्यजक्तयों की संख्या 336 है । केवल
जस ू पसंि करने वाले व्यजक्तयों की संख्या कॉिी और जस ू िोनों पसंि करने वाले लेककन चाय पसंि
नहीं करने वाले व्यजक्तयों की संख्या से 100% अचधक है , जो चाय और जूस िोनों पसंि करने वाले
लेककन कॉिी पसंि नहीं करने वाले व्यजक्तयों की संख्या से 12.5% अचधक है । केवल चाय पसंि

https :
//www. https : https
https : //instagra
//youtube.c
facebo m.com/aas :
om/channe hisharoraso
l/UCYa4_Jr ok.com cial(?) //t.m
Orf8R5Kz2u
/aashis utm_mediu
e/stu

193
OtccXQ m=
haroras copy_link
ocial dified
करने वाले व्यजक्तयों की संख्या, केवल जस
ू पसंि करने वाले व्यजक्तयों की संख्या से 33.33%
अचधक है ।

76. Find the number of persons who like two drinks.


िो पेय पसंि करने वाले व्यजक्तयों की संख्या ज्ञात कीजजए।
(A) 280 (B) 320 (C) 190
(D) 440 (E) None of these

77. Number of persons who like both Tea and Juice but do not like Coffee is
what percent of number of persons who like Tea?
चाय और जस ू िोनों पसंि करने वाले लेककन कॉिी पसंि नहीं करने वाले व्यजक्तयों की संख्या, चाय
पसंि करने वाले व्यजक्तयों की संख्या का ककतना प्रनतशत है ?
(A) 14% (B) 12% (C) 16%
(D) 18% (E) None of these

78. Number of persons who like both Juice and Coffee but do not like Tea is
how much percent more or less than number of persons who like only Tea?
जसू और कॉिी िोनों पसंि करने वाले लेककन चाय पसंि नहीं करने वाले व्यजक्तयों की संख्या,
केवल चाय पसंि करने वाले व्यजक्तयों की संख्या से ककतने प्रनतशत अचधक या कम है ?
(A) 63.8% (B) 62.5% (C) 64.7%
(D) 61.6% (E) None of these

79. Find the total number of persons who like drinks.


पेय पसंि करने वाले व्यजक्तयों की कुल संख्या ज्ञात कीजजए।
(A) 752 (B) 748 (C) 724
(D) 736 (E) None of these

80. The ratio of number of males to number of females who like only Juice is 7
: 5 respectively, then find the number of females who like only Juice.
केवल जूस पसंि करने वाली मदहलाओं की संख्या से पुरुर्ों की संख्या का अनुपात क्रमशः 7 : 5 है ,
तो केवल जस
ू पसंि करने वाली मदहलाओं की संख्या ज्ञात कीजजए।
(A) 50 (B) 60 (C) 40
(D) 70 (E) None of these

(81-85) Directions : Study the following passage carefully and answer the
questions given below.
The data is about number of Teachers, number of Doctors, number of
Engineers and number of Lawyers. The ratio of number of Female Teachers to
number of Male Doctors is 2 : 3 respectively. Number of Female Doctors are
27.27% more than number of Female Lawyers. Number of Male to Female

https :
//www. https : https
https : //instagra
//youtube.c
facebo m.com/aas :
om/channe hisharoraso
l/UCYa4_Jr ok.com cial(?) //t.m
Orf8R5Kz2u
/aashis utm_mediu
e/stu

194
OtccXQ m=
haroras copy_link
ocial dified
Lawyers are in the ratio of 8 : 7 respectively. Total number of Lawyers are 330
which is 10 less than total number of Teachers. The ratio of number of Male
Teachers to Male Engineers is 6 : 7 respectively. Total number of Engineers
are 460 and number of Male Engineers are 40 less than number of Female
Engineers.
ननिे श : ननम्नललखखत गद्यांश का ध्यानपव ू क
ष अध्ययन कीजजए और नीचे दिए गए प्रश्नों के उत्तर
िीजजए।
डेटा लशक्षकों की संख्या, डॉक्टरों की संख्या, इंजीननयरों की संख्या और वकीलों की संख्या के बारे में
है । मदहला लशक्षकों की संख्या का पुरुर् डॉक्टरों की संख्या से अनुपात क्रमशः 2 : 3 है । मदहला
वकीलों की संख्या की तुलना में मदहला डॉक्टरों की संख्या 27.27% अचधक है । पुरुर् से मदहला
वकीलों की संख्या क्रमशः 8 : 7 के अनप ु ात में है । वकीलों की कुल संख्या 330 है जो लशक्षकों की
कुल संख्या से 10 कम है । पुरुर् लशक्षकों की संख्या का पुरुर् इंजीननयरों से अनुपात क्रमशः 6 : 7 है ।
इंजीननयरों की कुल संख्या 460 है और परु ु र् इंजीननयरों की संख्या मदहला इंजीननयरों की संख्या से
40 कम है ।

81. Find the ratio of sum of number of Male Doctors and Male Engineers to
sum of number of Female Doctors and Female Lawyers.
परु
ु र् डॉक्टरों और परु
ु र् इंजीननयरों की संख्या के योग का मदहला डॉक्टरों और मदहला वकीलों की
संख्या के योग से अनुपात ज्ञात कीजजए।
(A) 9 : 7 (B) 4 : 3 (C) 8 : 5
(D) 1 : 2 (E) None of these

82. Number of Female Teachers is what percent of total number of Female


professionals?
मदहला लशक्षकों की संख्या मदहला पेशव
े रों की कुल संख्या का ककतना प्रनतशत है ?
(A) 22.47% (B) 20.89% (C) 21.05%
(D) 23.18% (E) None of these

83. Number of Female Engineers is how much percent more or less than
number of Male Teachers?
मदहला इंजीननयरों की संख्या परु
ु र् लशक्षकों की संख्या से ककतने प्रनतशत अचधक या कम है ?
(A) 36.12% (B) 38.88% (C) 34.89%
(D) 32.14% (E) None of these

84. Find the average of number of Females in all four professions.


सभी चार व्यवसायों में मदहलाओं की संख्या का औसत ज्ञात कीजजए।
(A) 160 (B) 180 (C) 170
(D) 190 (E) None of these

https :
//www. https : https
https : //instagra
//youtube.c
facebo m.com/aas :
om/channe hisharoraso
l/UCYa4_Jr ok.com cial(?) //t.m
Orf8R5Kz2u
/aashis utm_mediu
e/stu

195
OtccXQ m=
haroras copy_link
ocial dified
85. Number of Male and Female shopkeepers is respectively 60% and 40%
more than number of Male and Female Teachers, then find the difference
between number of Male and Female shopkeepers.
पुरुर् और मदहला िक
ु ानिारों की संख्या क्रमशः पुरुर् और मदहला लशक्षकों की संख्या से 60% और
40% अचधक है , तो परु
ु र् और मदहला िक ु ानिारों की संख्या के बीच का अंतर ज्ञात कीजजए।
(A) 66 (B) 64 (C) 62
(D) 68 (E) None of these

(86 - 90) Directions : Study the following passage carefully and answer the
questions given below.
A survey is conducted in a school to know about number of students who like
three different games. 150 students does not like any one games and 60
students like all three games. The ratio of number of students who like only
Free Fire to who like only PUBG is 4 : 5 respectively. Number of students who
like only Candy Crush is equal to average of number of students who like only
Free Fire and only PUBG. Number of students who like all three games is
16.66% of number of students who like only Free Fire. Number of students
who like Free Fire and PUBG is 170 which is 30 less than number of students
who like Free Fire and Candy Crush. The ratio of number of students who like
only PUBG to number of students who like both PUBG and Candy Crush but
do not like Free Fire is 3 : 1 respectively.
ननिे श : ननम्नललखखत गद्यांश का ध्यानपव ू क
ष अध्ययन कीजजए और नीचे दिए गए प्रश्नों के उत्तर
िीजजए।
तीन अलग-अलग खेलों को पसंि करने वाले छात्रों की संख्या के बारे में जानने के ललए एक स्कूल में
एक सवेक्षण ककया जाता है । 150 ववद्याचथषयों को कोई एक खेल पसंि नहीं है और 60 ववद्याचथषयों
को तीनों खेल पसंि हैं। केवल Free Fire पसंि करने वाले छात्रों की संख्या का केवल PUBG पसंि
करने वाले छात्रों की संख्या से क्रमशः 4 : 5 का अनप ु ात है । केवल Candy Crush पसंि करने वाले
छात्रों की संख्या, केवल Free Fire और केवल PUBG पसंि करने वाले छात्रों की औसत संख्या के
बराबर है । तीनों गेम पसंि करने वाले छात्रों की संख्या केवल Free Fire पसंि करने वाले छात्रों की
संख्या का 16.66% है । Free Fire और PUBG पसंि करने वाले छात्रों की संख्या 170 है जो Free
Fire और Candy Crush पसंि करने वाले छात्रों की संख्या से 30 कम है । केवल PUBG पसंि
करने वाले छात्रों की संख्या का PUBG और Candy Crush िोनों पसंि करने वाले लेककन Free
Fire पसंि नहीं करने वाले छात्रों की संख्या का अनुपात क्रमशः 3 : 1 है ।

86. Find the number of students who like exactly two games.
िीक िो खेल पसंि करने वाले ववद्याचथषयों की संख्या ज्ञात कीजजए।
(A) 400 (B) 300 (C) 200
(D) 500 (E) None of these

https :
//www. https : https
https : //instagra
//youtube.c
facebo m.com/aas :
om/channe hisharoraso
l/UCYa4_Jr ok.com cial(?) //t.m
Orf8R5Kz2u
/aashis utm_mediu
e/stu

196
OtccXQ m=
haroras copy_link
ocial dified
87. The ratio of number of boys to number of girls who like Candy Crush is 2 :
3, then find the number of boys who like Candy Crush.
Candy Crush पसंि करने वाले लडकों की संख्या का लडककयों की संख्या से अनुपात 2 : 3 है , तो
Candy Crush पसंि करने वाले लडकों की संख्या ज्ञात कीजजए।
(A) 308 (B) 306 (C) 302
(D) 304 (E) None of these

88. Find the total number of students in school.


स्कूल में छात्रों की कुल संख्या ज्ञात कीजजए।
(A) 1818 (B) 1825 (C) 1809
(D) 1833 (E) None of these

89. Number of students who like Candy Crush and PUBG but do not like Free
Fire is what percent of total number of students who like PUBG?
Candy Crush और PUBG पसंि करने वाले लेककन Free Fire पसंि नहीं करने वाले छात्रों की
संख्या, PUBG पसंि करने वाले छात्रों की कुल संख्या का ककतना प्रनतशत है ?
(A) 18.02% (B) 27.33% (C) 23.12%
(D) 19.48% (E) None of these

90. Number of number of students who like Candy Crush and PUBG but do
not like Free Fire is how much percent or less than number of students who
like Free Fire and PUBG but do not like Candy Crush?
Candy Crush और PUBG पसंि करने वाले लेककन Free Fire पसंि नहीं करने वाले छात्रों की
संख्या Free Fire और PUBG पसंि करने वाले लेककन Candy Crush पसंि नहीं करने वाले
छात्रों की संख्या से ककतने प्रनतशत या कम है ?
A) 34.89% (B) 36.36% (C) 37.14%
(D) 35.27% (E) None of these

(91-95) Directions : Study the following passage carefully and answer the
questions given below.
The data is regarding number of boys and number of girls of 10th, 11th and
12th class. The ratio of number of boys in 10th class to that 11th class is 5 : 4.
Number of girls of 11th class is 16.66% less than number of girls of 10th
class. Total number of students in 11th class is 171. The ratio of number of
boys of 10th class to number of boys of 12th class is 5 : 6. Total number of
students in 10th class is 210 and total number of students of 12th class is
28.56% more than number of students of 10th class.
ननिे श : ननम्नललखखत गद्यांश का ध्यानपूवक
ष अध्ययन कीजजए और नीचे दिए गए प्रश्नों के उत्तर
िीजजए।

https :
//www. https : https
https : //instagra
//youtube.c
facebo m.com/aas :
om/channe hisharoraso
l/UCYa4_Jr ok.com cial(?) //t.m
Orf8R5Kz2u
/aashis utm_mediu
e/stu

197
OtccXQ m=
haroras copy_link
ocial dified
डेटा 10वीं, 11वीं और 12वीं कक्षा के लडकों और लडककयों की संख्या के बारे में है । 10वीं कक्षा में
लडकों की संख्या का 11वीं कक्षा से अनुपात 5 : 4 है । 11वीं कक्षा की लडककयों की संख्या 10वीं
कक्षा की लडककयों की संख्या से 16.66% कम है । 11वीं कक्षा में छात्रों की कुल संख्या 171 है । 10वीं
कक्षा के लडकों की संख्या का 12वीं कक्षा के लडकों की संख्या से अनुपात 5 : 6 है । 10वीं कक्षा में
छात्रों की कुल संख्या 210 है और 12वीं कक्षा के छात्रों की कुल संख्या 10वीं कक्षा के छात्रों की संख्या
से 28.56% अचधक है ।

91. Find the average of number of boys in all three classes.


तीनों कक्षाओं में लडकों की संख्या का औसत ज्ञात कीजजए।
(A) 120 (B) 130 (C) 140
(D) 150 (E) None of these

92. Number of girls of 11th class is what percent of total number of students
of 12th class?
11वीं कक्षा की लडककयों की संख्या, 12वीं कक्षा के कुल छात्रों की संख्या का ककतना प्रनतशत है ?
(A) 25.89% (B) 26.12% (C) 27.77%
(D) 24.45% (E) None of these

93. Number of boys of 12th class is how much percent more or less than
number of girls of 10th class?
12वीं कक्षा के लडकों की संख्या, 10वीं कक्षा की लडककयों की संख्या से ककतने प्रनतशत अचधक या
कम है ?
(A) 40% (B) 60% (C) 80%
(D) 20% (E) None of these

94. Find the ratio of number of number of boys to number of girls of 10th
class.
10वीं कक्षा में लडकों की संख्या का लडककयों की संख्या से अनप
ु ात ज्ञात कीजजए।
(A) 4 : 3 (B) 5 : 6 (C) 7 : 9
(D) 8 : 5 (E) None of these

95. Number of girls and number of boys in 9th class is respectively 10% and
20% less than number of girls and number of boys of 10th class, then find the
sum of number of girls and number of boys of 9th class.
9वीं कक्षा में लडककयों की संख्या और लडकों की संख्या 10वीं कक्षा की लडककयों और लडकों की
संख्या से क्रमशः 10% और 20% कम है , तो 9वीं कक्षा की लडककयों और लडकों की संख्या का योग
ज्ञात कीजजए।
(A) 240 (B) 180 (C) 177
(D) 243 (E) None of these

https :
//www. https : https
https : //instagra
//youtube.c
facebo m.com/aas :
om/channe hisharoraso
l/UCYa4_Jr ok.com cial(?) //t.m
Orf8R5Kz2u
/aashis utm_mediu
e/stu

198
OtccXQ m=
haroras copy_link
ocial dified
(96-100) Directions : Study the following passage carefully and answer the
questions given below.
Data given is about number of Pen, Pencil and Eraser sold by A, B and C. The
ratio of number of Pen sold by A to number of Pencil sold by B is 5 : 6
respectively. Total number of Eraser sold by all three shopkeepers is 162.
Number of Pencil sold by A is 33.33% more than number of Erasers sold by A.
Number of Erasers sold by C is 54 which 9 more than number of Erasers sold
by A. Total number of items sold by A is 185. The ratio of number of Erasers
sold by B to number of Pens sold by C is 7 : 8 respectively. Number of Pens
sold by B is 13.33% more than number of Pencils sold by C. Number of Pens
sold by B is 5 more than number of Pens sold by A.
ननिे श : ननम्नललखखत गद्यांश का ध्यानपूवक ष अध्ययन कीजजए और नीचे दिए गए प्रश्नों के उत्तर
िीजजए।
दिया गया डेटा A, B और C द्वारा बेचे गए पेन, पें लसल और रबड की संख्या के बारे में है । A द्वारा
बेचे गए पेन की संख्या का B द्वारा बेची गई पें लसल की संख्या से अनुपात क्रमशः 5 : 6 है । तीनों
िक ु ानिारों द्वारा बेची गई रबड की कुल संख्या 162 है । A द्वारा बेची गई पें लसल की संख्या, A
द्वारा बेची गई रबड की संख्या से 33.33% अचधक है । C द्वारा बेची गई रबड की संख्या 54 है जो
A द्वारा बेची गई रबड की संख्या से 9 अचधक है । कुल A द्वारा बेची गई वस्तुओं की संख्या 185 है ।
B द्वारा बेची गई रबड की संख्या का C द्वारा बेची गई पेन की संख्या से क्रमशः 7 : 8 का अनुपात
है । B द्वारा बेची गई पेन की संख्या C द्वारा बेची गई पें लसल की संख्या से 13.33% अचधक है । B
द्वारा बेची गई पेन की संख्या A द्वारा बेची गई पेन की संख्या से 5 अचधक है ।

96. Find the average of number of items sold by C.


C द्वारा बेची गई वस्तुओं की औसत संख्या ज्ञात कीजजए।
(A) 67 (B) 53 (C) 75
(D) 49 (E) None of these

97. Number of Pens sold by B is what percent of total number of Pens sold by
all three shopkeepers?
B द्वारा बेचे गए पेनों की संख्या, तीनों िक
ु ानिारों द्वारा बेचे गए कुल पेनों की संख्या का ककतना
प्रनतशत है ?
(A) 32.89% (B) 36.12% (C) 35.86%
(D) 334.45% (E) None of these

98. Number of Erasers sold by C is how much percent more or less than
number of Pencils sold by B?
C द्वारा बेची गई रबड की संख्या, B द्वारा बेची गई पें लसल की संख्या से ककतने प्रनतशत अचधक या
कम है ?
(A) 41.56% (B) 43.75% (C) 44.89%
(D) 44.89% (E) None of these

https :
//www. https : https
https : //instagra
//youtube.c
facebo m.com/aas :
om/channe hisharoraso
l/UCYa4_Jr ok.com cial(?) //t.m
Orf8R5Kz2u
/aashis utm_mediu
e/stu

199
OtccXQ m=
haroras copy_link
ocial dified
99. Number of Pens and Pencils sold by D is respectively 12.5% and 20%
more than number of Pens and Pencils sold by C, then find the total number
of Pens and Pencils sold by D.
D द्वारा बेचे गए पेन और पें लसल की संख्या C द्वारा बेचे गए पेन और पें लसल की संख्या से क्रमशः
12.5% और 20% अचधक है , तो D द्वारा बेचे गए पेन और पें लसल की कुल संख्या ज्ञात कीजजए।
(A) 175 (B) 173 (C) 177
(D) 171 (E) None of these

100. If the price of each Pen, each Pencil and each Eraser is respectively Rs.
5, Rs. 8 and Rs. 7. Find the amount received by A from sell of all three items.
यदि प्रत्येक पेन, प्रत्येक पें लसल और प्रत्येक रबड की कीमत क्रमशः 5 रुपये, 8 रुपये और 7 रुपये है ,
तो तीनों वस्तओु ं को बेचने से A को प्राप्त रालश ज्ञात कीजजए।
(A) Rs. 1245 (B) Rs. 1195 (C) Rs. 1363
(D) Rs. 1457 (E) None of these

(101 - 105) Directions : Study the following passage carefully and answer the
questions given below.
The data is regarding number of laptops of sold by Dell, HP and Asus in April,
May and June respectively. In April, the ratio of number of laptops sold by Dell
to number of laptops sold by Asus is 4 : 5 respectively. Number of laptops
sold by Asus in June is respectively 10% less and 20 more than number of
laptops sold by Asus in April and May respectively. The ratio of number of
laptops sold by HP to number of laptops sold by Asus in June is respectively
4 : 3. Number of laptops sold by HP in May is 14.28% more than number of
laptops sold by Dell in same month. Total number of laptops sold in May is
850 which is 30 more than total number of laptops sold by Asus in all three
months. Number of laptops sold by HP in all three months is 3 times of
number of laptops sold by Asus in April. The ratio of number of laptops sold
by HP in April to number of laptops sold by Dell in June is 2 : 3 respectively.
ननिे श : ननम्नललखखत गद्यांश का ध्यानपूवक ष अध्ययन कीजजए और नीचे दिए गए प्रश्नों के उत्तर
िीजजए।
डेटा क्रमशः अप्रैल, मई और जन ू में Dell , HP और Asus द्वारा बेचे गए लैपटॉप की संख्या से
संबंचधत है । अप्रैल में , Dell द्वारा बेचे गए लैपटॉप की संख्या का Asus द्वारा बेचे गए लैपटॉप की
संख्या से अनप ु ात क्रमशः 4 : 5 है । जन ू में Asus द्वारा बेचे गए लैपटॉप की संख्या क्रमशः अप्रैल
और मई में Asus द्वारा बेचे गए लैपटॉप की संख्या से क्रमशः 10% कम और 20 अचधक है । HP
द्वारा बेचे गए लैपटॉप की संख्या का Asus द्वारा जून में बेचे गए लैपटॉप की संख्या से क्रमशः 4 :
3 का अनप ु ात है । मई में HP द्वारा बेचे गए लैपटॉप की संख्या उसी महीने में Dell द्वारा बेचे गए
लैपटॉप की संख्या से 14.28% अचधक है । मई में बेचे गए लैपटॉप की कुल संख्या 850 है जो Asus
द्वारा सभी तीन महीनों में बेचे गए लैपटॉप की कुल संख्या से 30 अचधक है ।

https :
//www. https : https
https : //instagra
//youtube.c
facebo m.com/aas :
om/channe hisharoraso
l/UCYa4_Jr ok.com cial(?) //t.m
Orf8R5Kz2u
/aashis utm_mediu
e/stu

200
OtccXQ m=
haroras copy_link
ocial dified
सभी तीन महीनों में HP द्वारा बेचे गए लैपटॉप की संख्या, Asus द्वारा अप्रैल में बेचे गए लैपटॉप
की संख्या का 3 गुना है । अप्रैल में HP द्वारा बेचे गए लैपटॉप की संख्या का जून में Dell द्वारा बेचे
गए लैपटॉप की संख्या से क्रमशः 2 : 3 का अनुपात है ।

101. Find the average of number of laptops sold by all three companies in
June.
जून में तीनों कंपननयों द्वारा बेचे गए लैपटॉप की औसत संख्या ज्ञात कीजजए।
(A) 320 (B) 340 (C) 310
(D) 330 (E) None of these

102. Number of laptops sold by HP in May is what percent of total number of


laptops sold in May by all three companies?
मई में HP द्वारा बेचे गए लैपटॉप की संख्या तीनों कंपननयों द्वारा मई में बेचे गए लैपटॉप की कुल
संख्या का ककतना प्रनतशत है ?
(A) 35.89% (B) 36.12% (C) 37.64%
(D) 34.45% (E) None of these

103. The number of laptops sold by HP in all the 3 months is what percent of
the number of laptops sold by all the three companies in all the 3 months
together?
सभी 3 महीनों में HP द्वारा बेचे गए लैपटॉप की संख्या सभी तीन कंपननयों द्वारा सभी 3 महीनों में
बेचे गए लैपटॉप की संख्या का ककतना प्रनतशत है ?
(A) 34.10% (B) 35.02% (C) 36.78%
(D) 37.69% (E) None of these

104. Number of laptops sold by Dell and HP in July is respectively 9.09% and
11.11% more than previous month, then find the sum of number of laptops
sold by Dell and HP in July.
जलु ाई में Dell और HP द्वारा बेचे गए लैपटॉप की संख्या वपछले महीने की तल
ु ना में क्रमशः
9.09% और 11.11% अचधक है , तो जुलाई में Dell और HP द्वारा बेचे गए लैपटॉप की संख्या का
योग ज्ञात कीजजए।
(A) 730 (B) 740 (C) 750
(D) 760 (E) None of these

105. What is ratio of sum of number of laptops sold by Dell and Asus in April
to number of laptops sold by Asus in June?
अप्रैल में Dell और Asus द्वारा बेचे गए लैपटॉप की संख्या का जून में Asus द्वारा बेचे गए
लैपटॉप की संख्या से ककतना अनुपात है ?
(A) 3 : 4 (B) 2 : 1 (C) 7 : 8
(D) 7 : 8 (E) None of these

https :
//www. https : https
https : //instagra
//youtube.c
facebo m.com/aas :
om/channe hisharoraso
l/UCYa4_Jr ok.com cial(?) //t.m
Orf8R5Kz2u
/aashis utm_mediu
e/stu

201
OtccXQ m=
haroras copy_link
ocial dified
(106-110) Directions : Study the following passage carefully and answer the
questions given below.
The data is about number of Milk chocolate and number of White chocolate
sold by A, B, C and D. Total number of chocolate sold by C is 590. Number of
Milk chocolate sold by B is 20% less than number of Milk chocolate sold by C.
The ratio of number of White chocolate sold by B to number of White
chocolate sold by C is 5 : 6 and the ratio of number of Milk chocolate sold by
C to number of White chocolate sold by D is 7 : 6 respectively. Total number
of chocolate sold by D is 570 which is 90 more than total number of chocolate
sold by B. The ratio of number of Milk chocolate to number of White
chocolate sold by A is 5 : 6 and number of White chocolate sold by A is equal
number of White chocolate sold by D.
ननिे श : ननम्नललखखत गद्यांश का ध्यानपव ू क
ष अध्ययन कीजजए और नीचे दिए गए प्रश्नों के उत्तर
िीजजए।
डेटा A, B, C और D द्वारा बेची गई लमल्क चॉकलेट की संख्या और व्हाइट चॉकलेट की संख्या के
बारे में है । C द्वारा बेची गई चॉकलेट की कुल संख्या 590 है । B द्वारा बेची गई लमल्क चॉकलेट की
संख्या, C द्वारा बेची गई लमल्क चॉकलेट की संख्या से 20% कम है । B द्वारा बेची गई व्हाइट
चॉकलेट की संख्या का C द्वारा बेची गई व्हाइट चॉकलेट की संख्या से अनुपात 5 : 6 है और C
द्वारा बेची गई लमल्क चॉकलेट की संख्या का D द्वारा बेची गई व्हाइट चॉकलेट की संख्या से
अनुपात क्रमशः 7 : 6 है । D द्वारा बेची गई चॉकलेट की कुल संख्या 570 है जो B द्वारा बेची गई
चॉकलेट की कुल संख्या से 90 अचधक है । A द्वारा बेची गई लमल्क चॉकलेट की संख्या का व्हाइट
चॉकलेट की संख्या से अनुपात 5 : 6 है और A द्वारा बेची गई व्हाइट चॉकलेट की संख्या, D द्वारा
बेची गई व्हाइट चॉकलेट की संख्या के बराबर है ।

106. Find the average of number of White chocolate sold by all four
shopkeepers.
सभी चार िक
ु ानिारों द्वारा बेची गई व्हाइट चॉकलेट की औसत संख्या ज्ञात कीजजए।
(A) 260 (B) 320 (C) 190
(D) 440 (E) None of these

107. Number of Milk chocolate sold by B is what percent of total number of


Milk chocolate sold by all four shopkeepers?
B द्वारा बेची गई लमल्क चॉकलेट की संख्या, चारों िक
ु ानिारों द्वारा बेची गई लमल्क चॉकलेट की
कुल संख्या का ककतना प्रनतशत है ?
(A) 22.89% (B) 26.12% (C) 24.34%
(D) 28.45% (E) None of these

108. Number of Milk chocolate and White chocolate sold by E is respectively


11.11% and 10% more than number of Milk chocolate and White chocolate
sold by D, then find the sum of number of Milk chocolate and White chocolate

https :
//www. https : https
https : //instagra
//youtube.c
facebo m.com/aas :
om/channe hisharoraso
l/UCYa4_Jr ok.com cial(?) //t.m
Orf8R5Kz2u
/aashis utm_mediu
e/stu

202
OtccXQ m=
haroras copy_link
ocial dified
sold by E.
E द्वारा बेची गई लमल्क चॉकलेट और व्हाइट चॉकलेट की संख्या, D द्वारा बेची गई लमल्क
चॉकलेट और व्हाइट चॉकलेट की संख्या से क्रमशः 11.11% और 10% अचधक है , तो E द्वारा बेची
गई लमल्क चॉकलेट और व्हाइट चॉकलेट की संख्या का योग ज्ञात कीजजए।
(A) 410 (B) 630 (C) 520
(D) 380 (E) None of these

109. Number of Dark chocolate sold by A and B is respectively 20% and


28.56% more than number of Milk chocolate sold by A and B, then find the
total number of Dark chocolate sold by A and B together.
A और B द्वारा बेची गई डाकष चॉकलेट की संख्या, A और B द्वारा बेची गई लमल्क चॉकलेट की
संख्या से क्रमशः 20% और 28.56% अचधक है , तो A और B द्वारा लमलाकर बेची गई डाकष
चॉकलेट की कुल संख्या ज्ञात कीजजए।
(A) 630 (B) 650 (C) 640
(D) 660 (E) None of these

110. Number of Milk chocolate sold by C is how much percent more or less
than number of White chocolate sold by B?
C द्वारा बेची गई लमल्क चॉकलेट की संख्या, B द्वारा बेची गई व्हाइट चॉकलेट की संख्या से
ककतने प्रनतशत अचधक या कम है ?
(A) 74% (B) 75% (C) 73%
(D) 72% (E) None of these

(111 - 115) Directions : Study the following passage carefully and answer the
questions given below.
Total number of students in a school is 870 out of them each students like at
least one game among Cricket, Chess and Football. Total number of students
who like Cricket is 630 which is 7 times of number of student who like only
Chess. The ratio of number of students who like Cricket and Chess to number
of student who like only Chess is 4 : 1 respectively and the ratio of number of
students who like all three games to number of students who like both
Football and Cricket but don't like Chess is 3 : 2. Number of students who like
only Football is 70 which is 10 less than number of students who like both
Football and Chess but don't like Cricket. Number of students who like both
Chess and Football but don't like Cricket is 33.33% of number of students
who like all three games.
ननिे श : ननम्नललखखत गद्यांश का ध्यानपूवक ष अध्ययन कीजजए और नीचे दिए गए प्रश्नों के उत्तर
िीजजए।
एक स्कूल में छात्रों की कुल संख्या 870 है उनमें से प्रत्येक छात्र कक्रकेट, शतरं ज और िुटबॉल के
बीच कम से कम एक खेल पसंि करता है । कक्रकेट पसंि करने वाले छात्रों की कुल संख्या 630 है जो
केवल शतरं ज पसंि करने वाले छात्रों की संख्या का 7 गुना है । कक्रकेट और शतरं ज

https :
//www. https : https
https : //instagra
//youtube.c
facebo m.com/aas :
om/channe hisharoraso
l/UCYa4_Jr ok.com cial(?) //t.m
Orf8R5Kz2u
/aashis utm_mediu
e/stu

203
OtccXQ m=
haroras copy_link
ocial dified
पसंि करने वाले छात्रों की संख्या का केवल शतरं ज पसंि करने वाले छात्रों की संख्या से अनुपात
क्रमशः 4 : 1 है और तीनों खेल पसंि करने वाले छात्रों की संख्या का िुटबॉल और कक्रकेट िोनों पसंि
करने वाले लेककन शतरं ज पसंि नहीं करने वाले छात्रों की संख्या का अनुपात 3 : 2 है । केवल
िुटबॉल पसंि करने वाले छात्रों की संख्या 70 है जो िुटबॉल और शतरं ज िोनों पसंि करने वाले
लेककन कक्रकेट पसंि नहीं करने वाले छात्रों की संख्या से 10 कम है । शतरं ज और िुटबॉल िोनों पसंि
करने वाले लेककन कक्रकेट पसंि नहीं करने वाले छात्रों की संख्या तीनों खेलों को पसंि करने वाले
छात्रों की संख्या का 33.33% है ।

111. Find the number of students who like Football.


िुटबॉल पसंि करने वाले छात्रों की संख्या ज्ञात कीजजए।
(A) 550 (B) 520 (C) 530
(D) 540 (E) None of these

112. Find the number of students who like only one game.
केवल एक खेल पसंि करने वाले छात्रों की संख्या ज्ञात कीजजए।
(A) 240 (B) 250 (C) 270
(D) 260 (E) None of these

113. Number of students who like both Cricket and Football but don’t like
Chess is how much percent more/less than number of students who like only
Chess?
कक्रकेट और िुटबॉल िोनों को पसंि करने वाले लेककन शतरं ज को पसंि नहीं करने वाले छात्रों की
संख्या केवल शतरं ज को पसंि करने वाले छात्रों की संख्या से ककतने प्रनतशत अचधक/कम है ?
(A) 75.12% (B) 77.77% (C) 73.69%
(D) 79.52% (E) None of these

114. The ratio of the number of boys to the number of girls who like only
Cricket is 5 : 6 and the ratio of the number of boys to the number of girls who
like only Chess is 5 : 4. Find the difference between number of boys who like
only Cricket and number of girls who like only Chess.
केवल कक्रकेट पसंि करने वाले लडकों की संख्या का लडककयों की संख्या से अनुपात 5 : 6 है और
केवल शतरं ज पसंि करने वाले लडकों की संख्या का लडककयों की संख्या से अनुपात 5 : 4 है । केवल
कक्रकेट पसंि करने वाले लडकों की संख्या और केवल शतरं ज पसंि करने वाली लडककयों की संख्या
के बीच अंतर ज्ञात कीजजए।
(A) 30 (B) 40 (C) 20
(D) 10 (E) None of these

https :
//www. https : https
https : //instagra
//youtube.c
facebo m.com/aas :
om/channe hisharoraso
l/UCYa4_Jr ok.com cial(?) //t.m
Orf8R5Kz2u
/aashis utm_mediu
e/stu

204
OtccXQ m=
haroras copy_link
ocial dified
115. What is the percentage of students who like Cricket out of the total
students who like Cricket?
कक्रकेट पसंि करने वाले कुल छात्रों में से कक्रकेट पसंि करने वाले छात्रों का प्रनतशत ककतना है ?
(A) 18.59% (B) 18.46% (C) 16.22%
(D) 19.67 % (E) None of these

(116 - 120) Directions : Study the following passage carefully and answer the
questions given below.
The data is about number of male and female in four different localities. Total
number of males in all four locality is 900 which is 5 times of number of
females of locality B. The ratio of number of males in locality B to number of
females in locality C is 3 : 2. Number of males in locality D is equal to number
of female in locality A. The ratio of number of males in locality A to C is 9 : 8
respectively. Total number of persons in locality B is 420 which 16.66% more
than total number of persons in locality C. The ratio of number of males in
locality C to number of females in locality D is 5 : 3 respectively.
ननिे श : ननम्नललखखत गद्यांश का ध्यानपव ू क
ष अध्ययन कीजजए और नीचे दिए गए प्रश्नों के उत्तर
िीजजए।
डेटा चार अलग - अलग इलाकों में पुरुर्ों और मदहलाओं की संख्या के बारे में है । सभी चार इलाकों में
परु
ु र्ों की कुल संख्या 900 है जो इलाके B की मदहलाओं की संख्या का 5 गुना है । इलाके B में परु ु र्ों
की संख्या का इलाके C में मदहलाओं की संख्या से अनुपात 3 : 2 है । इलाके D में पुरुर्ों की संख्या
इलाके A में मदहलाओं की संख्या के बराबर है । इलाके A में परुु र्ों की संख्या का C से अनपु ात
क्रमशः 9 : 8 है । इलाके B में व्यजक्तयों की कुल संख्या 420 है , जो इलाके C में व्यजक्तयों की कुल
संख्या से 16.66% अचधक है । इलाके C में पुरुर्ों की संख्या का इलाके D में मदहलाओं की संख्या से
अनप ु ात क्रमशः 5 : 3 है ।

116. Find the average of number of males in locality A, locality C and locality
D.
इलाके A, इलाके C और इलाके D में पुरुर्ों की औसत संख्या ज्ञात कीजजए।
(A) 220 (B) 152 (C) 137
(D) 163 (E) None of these

117. Number of males in locality B is what percent of total number of males in


all four localities?
इलाके B में परु
ु र्ों की संख्या सभी चार इलाकों में परु
ु र्ों की कुल संख्या का ककतना प्रनतशत है ?
(A) 22.89% (B) 28.12% (C) 26.66%
(D) 24.45% (E) None of these

https :
//www. https : https
https : //instagra
//youtube.c
facebo m.com/aas :
om/channe hisharoraso
l/UCYa4_Jr ok.com cial(?) //t.m
Orf8R5Kz2u
/aashis utm_mediu
e/stu

205
OtccXQ m=
haroras copy_link
ocial dified
118. Number of males in locality A is how much percent more or less than
number of females in locality D?
इलाके A में परु
ु र्ों की संख्या, इलाके D में मदहलाओं की संख्या से ककतने प्रनतशत अचधक या कम
है ?
(A) 85.6% (B) 87.5% (C) 83.1%
(D) 81.9% (E) None of these

119. Find the ratio of sum of number of males in locality B and locality C to
sum of number of females in locality B and locality C.
इलाके B और इलाके C में परु ु र्ों की संख्या के योग का इलाके B और इलाके C में मदहलाओं की
संख्या के योग से अनुपात ज्ञात कीजजए।
(A) 24 : 19 (B) 25 : 16 (C) 23 : 18
(D) 22 : 17 (E) None of these

120. Number of males and females in locality E is respectively 20% and 6.66%
more than number of males and females in locality D, then find the total
number of persons in locality E.
इलाके E में परु
ु र्ों और मदहलाओं की संख्या क्रमशः 20% और इलाके D में पुरुर्ों और मदहलाओं की
संख्या से 6.66% अचधक है , तो इलाके E में व्यजक्तयों की कुल संख्या ज्ञात कीजजए।
(A) 462 (B) 478 (C) 456
(D) 410 (E) None of these

(121 - 125) Directions : Study the following passage carefully and answer the
questions given below.
The data is about number of students of a city who are preparing for banking
exams have three different mocks. Each students have at least one mock.
Number of students who have all three mocks is 280. The ratio of number of
students who have only Oliveboard mocks to number of students who have
only Guidely mocks is 4 : 3. Number of students who have only Oliveboard
mocks is equal to number of students who have both Guidely and Testzone
mocks. The ratio of number of students who have all three mocks to number
of students who have both Oliveboard and Testzone mock but doesn't have
Guidely mock is 4 : 3 respectively. Number of students who have only
Testzone mocks is 400 which is 16.66% less than number of students who
have only Oliveboard mocks. Number of students who have Oliveboard and
Guidely mocks but doesn't have Testzone mocks is 33.33% of number of
students who have only Oliveboard mocks.
ननिे श : ननम्नललखखत गद्यांश का ध्यानपव
ू क
ष अध्ययन कीजजए और नीचे दिए गए प्रश्नों के उत्तर
िीजजए।

https :
//www. https : https
https : //instagra
//youtube.c
facebo m.com/aas :
om/channe hisharoraso
l/UCYa4_Jr ok.com cial(?) //t.m
Orf8R5Kz2u
/aashis utm_mediu
e/stu

206
OtccXQ m=
haroras copy_link
ocial dified
डेटा एक शहर के छात्रों की संख्या के बारे में है , जो बैंककंग परीक्षा की तैयारी कर रहे हैं, उनके पास
तीन अलग - अलग मॉक हैं। प्रत्येक छात्र के पास कम से कम एक मॉक है । सभी तीन मॉक वाले
छात्रों की संख्या 280 है । केवल Oliveboard मॉक वाले छात्रों की संख्या का केवल Guidely मॉक
वाले छात्रों की संख्या से अनुपात 4 : 3 है । केवल Oliveboard मॉक वाले छात्रों की संख्या उन छात्रों
की संख्या के बराबर है जजनके पास Guidely और Testzone िोनों मॉक हैं। उन छात्रों की संख्या
का जजनके पास तीनों मॉक हैं, उन छात्रों की संख्या से जजनके पास Oliveboard और Testzone
िोनों मॉक हैं, लेककन Guidely मॉक नहीं है , अनप ु ात क्रमशः 4 : 3 है । केवल Testzone मॉक वाले
छात्रों की संख्या 400 है जो केवल Oliveboard मॉक वाले छात्रों की संख्या से 16.66% कम है । उन
छात्रों की संख्या जजनके पास Oliveboard और Guidely मॉक हैं, लेककन Testzone मॉक नहीं हैं,
केवल Oliveboard मॉक वाले छात्रों की संख्या का 33.33% है ।

121. Number of students who have only Guidely mocks is what percent of
number of students who have Guidely mocks?
केवल Guidely मॉक वाले छात्रों की संख्या, Guidely मॉक वाले छात्रों की संख्या का ककतना
प्रनतशत है ?
(A) 36% (B) 32% (C) 34%
(D) 38% (E) None of these

122. The ratio of number of students who have 6 months plan to 1 year plan of
only Oliveboard and only Guidely is respectively 5 : 7 and 4 : 5, then find the
sum of number of students who have 6 months plan of only Oliveboard and
only Guidely.
केवल Oliveboard और केवल गाइड की 6 महीने की योजना का 1 वर्ष की योजना वाले छात्रों की
संख्या का अनुपात क्रमशः 5 : 7 और 4 : 5 है , तो उन छात्रों की संख्या का योग ज्ञात कीजजए जजनके
पास केवल Oliveboard और केवल गाइड की 6 महीने की योजना है ।
(A) 320 (B) 340 (C) 360
(D) 380 (E) None of these

123. Find the number of students who have at least two mocks.
कम से कम िो मॉक वाले छात्रों की संख्या ज्ञात कीजजए।
(A) 810 (B) 850 (C) 870
(D) 830 (E) None of these

124. Find the number of students who doesn't have Oliveboard mocks.
उन ववद्याचथषयों की संख्या ज्ञात कीजजए जजनके पास Oliveboard मॉक नहीं है ।
(A) 930 (B) 950 (C) 970
(D) 960 (E) None of these

https :
//www. https : https
https : //instagra
//youtube.c
facebo m.com/aas :
om/channe hisharoraso
l/UCYa4_Jr ok.com cial(?) //t.m
Orf8R5Kz2u
/aashis utm_mediu
e/stu

207
OtccXQ m=
haroras copy_link
ocial dified
125. Number of students who have both Oliveboard mock and Testzone mock
but doesn't have Guidely mock is how much percent more or less than
number of students who have only Oliveboard mocks?
उन छात्रों की संख्या जजनके पास Oliveboard मॉक और Testzone मॉक िोनों हैं, लेककन
Guidely मॉक नहीं है , केवल Oliveboard मॉक वाले छात्रों की संख्या से ककतने प्रनतशत अचधक
या कम है ?
(A) 54.12% (B) 56.25% (C) 58.96%
(D) 52.13% (E) None of these

(126 - 130) Directions : Study the following passage carefully and answer the
questions given below.
Below data is given regarding number of students who attended webinar for
career council on three courses - Agricultural course, Engineering course and
Medical course. Total number of students who attended the webinar is 1000
and each student attended webinar on either one course or more than one
course. 14.8%, 13.6% and 14.4% of total students attended webinar on only
Agricultural course, on only Engineering course and on only Medical course
respectively. The number of students who attended webinar on both
Agricultural and Medical courses but do not attended webinar on Engineering
course is 20 more than the number of students who attended webinar on only
Agriculture course. The number of students who attended webinar on all
three courses together is 10 more than the number of students who attended
webinar on both Agricultural and Engineering courses but do not attended
webinar on Medical course. 13% of total students attend webinar on all three
courses together.
ननिे श : ननम्नललखखत गद्यांश का ध्यानपव ू कष अध्ययन कीजजए और नीचे दिए गए प्रश्नों के उत्तर
िीजजए।
नीचे डेटा तीन पाठ्यक्रमों - करवर् पाठ्यक्रम, इंजीननयररंग पाठ्यक्रम और चचककत्सा पाठ्यक्रम पर
कैररयर पररर्ि के ललए वेबबनार में भाग लेने वाले छात्रों की संख्या के संबंध में दिया गया है ।
वेबबनार में भाग लेने वाले छात्रों की कुल संख्या 1000 है और प्रत्येक छात्र ने एक कोसष या एक से
अचधक कोसष पर वेबबनार में भाग ललया। कुल छात्रों के 14.8%, 13.6% और 14.4% ने क्रमशः
केवल करवर् पाठ्यक्रम, केवल इंजीननयररंग पाठ्यक्रम और केवल चचककत्सा पाठ्यक्रम पर वेबबनार
में भाग ललया। करवर् और चचककत्सा िोनों पाठ्यक्रमों पर वेबबनार में भाग लेने वाले छात्रों की संख्या,
लेककन इंजीननयररंग पाठ्यक्रम पर वेबबनार में भाग नहीं लेने वाले छात्रों की संख्या केवल करवर्
पाठ्यक्रम पर वेबबनार में भाग लेने वाले छात्रों की संख्या से 20 अचधक है । तीनों पाठ्यक्रमों में एक
साथ वेबबनार में भाग लेने वाले छात्रों की संख्या उन छात्रों की संख्या से 10 अचधक है , जजन्होंने करवर्
और इंजीननयररंग िोनों पाठ्यक्रमों पर वेबबनार में भाग ललया, लेककन मेडडकल पाठ्यक्रम पर
वेबबनार में भाग नहीं ललया। कुल छात्रों में से 13% तीनों पाठ्यक्रमों में एक साथ वेबबनार में भाग
लेते हैं।

https :
//www. https : https
https : //instagra
//youtube.c
facebo m.com/aas :
om/channe hisharoraso
l/UCYa4_Jr ok.com cial(?) //t.m
Orf8R5Kz2u
/aashis utm_mediu
e/stu

208
OtccXQ m=
haroras copy_link
ocial dified
126. What is the ratio of number of students who attended webinar on
Agricultural course to the number of students who attended webinar on
Engineering course?
करवर् पाठ्यक्रम पर वेबबनार में भाग लेने वाले छात्रों की संख्या का इंजीननयररंग पाठ्यक्रम में
वेबबनार में भाग लेने वाले छात्रों की संख्या से अनप
ु ात ककतना है ?
(A) 299 : 277 (B) 297 : 261 (C) 283 : 270
(D) 281 : 277 (E) None of these

127. How many students attended webinar on exactly two courses?


वास्तव में िो पाठ्यक्रमों में ककतने छात्रों ने वेबबनार में भाग ललया?
(A) 442 (B) 450 (C) 400
(D) 150 (E) None of these

128. The number of students who attended webinar on both Engineering and
Medical courses is approximately what percentage of the number of students
who attended webinar on both Agricultural and Medical courses?
इंजीननयररंग और मेडडकल िोनों पाठ्यक्रमों में वेबबनार में भाग लेने वाले छात्रों की संख्या, करवर् और
चचककत्सा िोनों पाठ्यक्रमों में वेबबनार में भाग लेने वाले छात्रों की संख्या का लगभग ककतना
प्रनतशत है ?
(A) 70% (B) 95% (C) 60%
(D) 50% (E) None of these

129. What is the difference between the number of students who attended
webinar on only Medical course and the number of students who attended
webinar on only Engineering course?
केवल मेडडकल पाठ्यक्रम पर वेबबनार में भाग लेने वाले छात्रों की संख्या और केवल इंजीननयररंग
पाठ्यक्रम पर वेबबनार में भाग लेने वाले छात्रों की संख्या के बीच ककतना अंतर है ?
(A) 20 (B) 14 (C) 8
(D) 15 (E) None of these

130. How many students attended webinar on exactly one course?


िीक एक पाठ्यक्रम में ककतने छात्रों ने वेबबनार में भाग ललया?
(A) 398 (B) 460 (C) 436
(D) 428 (E) None of these

(131-135) Directions : Study the following passage carefully and answer the
questions given below.
There is a coaching institute in which student study for various exams SSC,
Bank, Railway and UPSC in two different batches Morning and Evening.

https :
//www. https : https
https : //instagra
//youtube.c
facebo m.com/aas :
om/channe hisharoraso
l/UCYa4_Jr ok.com cial(?) //t.m
Orf8R5Kz2u
/aashis utm_mediu
e/stu

209
OtccXQ m=
haroras copy_link
ocial dified
Morning : Total students in Railways batch is double of the total students in
Bank batch which is 40 less than the total students in SSC batch. Total
students in UPSC batch is one - third of the total students in SSC batch.
Evening : Number of students in SSC batch is same as the number of
students in Railway batch in Morning shift. Number of students in Bank batch
and UPSC batch is 200 and 50 respectively. Average of students in all batches
is 125 which is 35 more than the total students in Railway batch.
ननिे श : ननम्नललखखत गद्यांश का ध्यानपव ू क
ष अध्ययन कीजजए और नीचे दिए गए प्रश्नों के उत्तर
िीजजए।
एक कोचचंग संस्थान है जजसमें छात्र SSC, बैंक, रे लवे और UPSC की ववलभन्न परीक्षाओं के ललए
सब ु ह और शाम िो अलग-अलग बैचों में अध्ययन करते हैं।
सुबह : रे लवे बैच में कुल छात्र बैंक बैच में कुल छात्रों से िोगुना है जो SSC बैच में कुल छात्रों की
तल ु ना में 40 कम है । UPSC बैच में कुल छात्र SSC बैच में कुल छात्रों का एक-नतहाई है ।
शाम : SSC बैच में छात्रों की संख्या सुबह की पाली में रे लवे बैच में छात्रों की संख्या के बराबर है ।
बैंक बैच और UPSC बैच में छात्रों की संख्या क्रमशः 200 और 50 है । सभी बैचों में छात्रों का औसत
125 है जो रे लवे बैच के कुल छात्रों से 35 अचधक है ।

131. Ratio of boys to girls in SSC Evening batch is 21 : 19, then what is the
difference between boys and girls in SSC Evening batch?
SSC शाम की पाली में लडकों का लडककयों से अनुपात 21 : 19 है , तो SSC शाम की पाली में
लडकों और लडककयों का अंतर ककतना है ?
A) 5 (B) 7 (C) 8
(D) 2 (E) None of these

132. What is the ratio of total students in all the four batches together in
Morning shift to that in Evening shift?
सुबह की पाली में एक साथ सभी चार बैचों में कुल छात्रों की संख्या का शाम की पाली से ककतना
अनपु ात है ?
(A) 4 : 5 (B) 2 : 3 (C) 5 : 1
(D) 8 : 7 (E) None of these

133. Total students in a Bank batch in both the shifts together is what percent
more than the total students in Railway batch in both the shifts together?
िोनों पाललयों में एक साथ बैंक बैच में कुल छात्र िोनों पाललयों में एक साथ रे लवे बैच में कुल छात्रों
की तुलना में ककतने प्रनतशत अचधक है ?
(A) 10% (B) 12% (C) 14%
(D) 15% (E) None of these

134. If total number of boys in UPSC batch in both the shifts together are 54,
then what is the ratio of boys to the girls in UPSC batch in both the shifts

https :
//www. https : https
https : //instagra
//youtube.c
facebo m.com/aas :
om/channe hisharoraso
l/UCYa4_Jr ok.com cial(?) //t.m
Orf8R5Kz2u
/aashis utm_mediu
e/stu

210
OtccXQ m=
haroras copy_link
ocial dified
together?
यदि िोनों पाललयों में लमलाकर UPSC बैच में लडकों की कुल संख्या 54 है , तो UPSC बैच में िोनों
पाललयों में लडकों का लडककयों से अनपु ात ककतना है ?
(A) 35 : 29 (B) 3 : 2 (C) 3 : 5
(D) 49 : 36 (E) None of these

135. If 75% and 80% of Railway batch in Morning and Evening shift
respectively cleared the exam, then how many students didn't clear the
Railway exam from both the shifts together?
यदि सुबह और शाम की पाली में रे लवे बैच के क्रमशः 75% और 80% परीक्षा उत्तीणष करते हैं, तो
ककतने छात्रों ने िोनों पाललयों से एक साथ रे लवे परीक्षा उत्तीणष नहीं की?
(A) 60 (B) 20 (C) 55
(D) 58 (E) None of these

(136 - 140) Directions : Study the following passage carefully and answer the
questions given below.
Below data is given regarding number of Mobiles and Laptops repaired by a
shop in three months July, August and September. The ratio of number of
Mobiles to Laptops repaired by the shop in three months together is 1 : 1 and
number of Laptops repaired in September is 83(1/3)% of total Laptop and
Mobiles repaired by the shop in September. The number of Laptops repaired
in July and August are thrice and twice of the number of Mobiles repaired in
September respectively and number of Mobile repaired in July is equal to
number of Laptops repaired in September. The number of Mobiles repaired in
July is 10 more than that of in August.
ननिे श : ननम्नललखखत गद्यांश का ध्यानपूवक ष अध्ययन कीजजए और नीचे दिए गए प्रश्नों के उत्तर
िीजजए।
नीचे तीन महीनों जलु ाई, अगस्त और लसतंबर में एक िक ु ान द्वारा मरम्मत ककए गए मोबाइल और
लैपटॉप की संख्या के संबंध में डेटा दिया गया है । तीन महीनों में एक साथ िक ु ान द्वारा मरम्मत
ककए गए मोबाइलों से लैपटॉप की संख्या का अनुपात 1 : 1 है और लसतंबर में मरम्मत ककए गए
लैपटॉप की संख्या लसतंबर में िक ु ान द्वारा मरम्मत ककए गए कुल लैपटॉप और मोबाइल का
83(1/3)% है । जुलाई और अगस्त में मरम्मत ककए गए लैपटॉप की संख्या लसतंबर में मरम्मत ककए
गए मोबाइलों की संख्या से क्रमशः तीगन ु ी और िोगन ु ी है और जल ु ाई में मरम्मत ककए गए मोबाइल
की संख्या लसतंबर में मरम्मत ककए गए लैपटॉप की संख्या के बराबर है । जुलाई में मरम्मत ककए
गए मोबाइलों की संख्या अगस्त की तुलना में 10 अचधक है ।

136. What is the ratio of number of Mobiles repaired in July to the number of
Laptops repaired in August?
जुलाई में मरम्मत ककए गए मोबाइलों की संख्या का अगस्त में मरम्मत ककए गए लैपटॉपों की
संख्या से अनुपात ककतना है ?

https :
//www. https : https
https : //instagra
//youtube.c
facebo m.com/aas :
om/channe hisharoraso
l/UCYa4_Jr ok.com cial(?) //t.m
Orf8R5Kz2u
/aashis utm_mediu
e/stu

211
OtccXQ m=
haroras copy_link
ocial dified
(A) 5 : 3 (B) 1 : 1 (C) 5 : 2
(D) 5 : 4 (E) None of these

137. The number of Laptops repaired in July and August together is what
percentage of the number of Mobiles repaired in August and September
together?
जुलाई और अगस्त में एकसाथ मरम्मत ककए गए लैपटॉप की संख्या अगस्त और लसतंबर में
एकसाथ मरम्मत ककए गए मोबाइल की संख्या का ककतना प्रनतशत है ?
(A) 100% (B) 150% (C) 50%
(D) 80% (E) None of these

138. What is the average number of Laptops repaired in July and September
together?
जुलाई और लसतंबर में लमलाकर मरम्मत ककए गए लैपटॉप की औसत संख्या ककतनी है ?
(A) 35 (B) 40 (C) 45
(D) 30 (E) None of these

139. What is the difference between the number of Mobile and Laptops
together repaired in August and that in July?
अगस्त में और जुलाई में मरम्मत ककए गए मोबाइल और लैपटॉप की कुल संख्या के बीच ककतना
अंतर है ?
(A) 30 (B) 10 (C) 20
(D) 40 (E) None of these

140.. The number of Laptops repaired in September is what percentage more


or less than the number of Mobiles repaired in August?
लसतंबर में मरम्मत ककए गए लैपटॉप की संख्या अगस्त में मरम्मत ककए गए मोबाइल की संख्या से
ककतने प्रनतशत अचधक या कम है ?
(A) 25% (B) 56% (C) 30%
(D) 20% (E) None of these

(141-145) Directions : Study the following passage carefully and answer the
questions given below.
Following is the information about number of students in 5 centers of two
classes A and B. Number of students in Bangalore center of classes A and B
are 1200 and 1500 respectively. Number of students in Pune center of classes
A and B are 30 and 850 less than the Bangalore center of respective classes.
Number of students in Mumbai center of classes A and B are 1320 and 1180
respectively. Number of students in Delhi center of class A is 100 less than
the number of students in Mumbai center of class B whereas number of

https :
//www. https : https
https : //instagra
//youtube.c
facebo m.com/aas :
om/channe hisharoraso
l/UCYa4_Jr ok.com cial(?) //t.m
Orf8R5Kz2u
/aashis utm_mediu
e/stu

212
OtccXQ m=
haroras copy_link
ocial dified
students in Delhi center of class B is 120 less than the number of students in
Mumbai center of class A. Total number of students in Hyderabad center of
classes A and B together is 3000 and the ratio of the number students in
Hyderabad center of classes A and B is 13 : 12.
ननिे श : ननम्नललखखत गद्यांश का ध्यानपूवक ष अध्ययन कीजजए और नीचे दिए गए प्रश्नों के उत्तर
िीजजए।
िो कक्षाओं A और B के 5 केंद्रों में छात्रों की संख्या के बारे में जानकारी ननम्नललखखत है । कक्षा A
और B के बैंगलोर केंद्र में छात्रों की संख्या क्रमशः 1200 और 1500 है । कक्षा A और B के पण ु े केंद्र
में छात्रों की संख्या संबंचधत कक्षाओं के बैंगलोर केंद्र से क्रमशः 30 और 850 कम है । कक्षा A और B
के मुंबई केंद्र में छात्रों की संख्या क्रमशः 1320 और 1180 है । कक्षा A के दिल्ली केंद्र में छात्रों की
संख्या कक्षा B के मंब ु ई केंद्र में छात्रों की संख्या से 100 कम है जबकक कक्षा B के दिल्ली केंद्र में
छात्रों की संख्या कक्षा A के मुंबई केंद्र में छात्रों की संख्या से 120 कम है । कक्षा A और B के
है िराबाि केंद्र में एक साथ छात्रों की कुल संख्या 3000 है और कक्षा A और B के है िराबाि केंद्र में
छात्रों की संख्या का अनुपात 13 : 12 है ।

141. What is the ratio of the difference between the number of students in
classes A and B in Pune center to the that of in Mumbai center?
पण
ु े केंद्र में कक्षा A और B में छात्रों की संख्या का मंब
ु ई केंद्र में छात्रों की संख्या से अंतर का अनप
ु ात
ककतना है ?
(A) 2 : 7 (B) 13 : 7 (C) 26 : 7
(D) 13 : 18 (E) None of these

142. Total number of students in Bangalore center of classes A and B together


is what percent more than the total number of students in Mumbai center of
classes A and B together?
कक्षा A और B के बैंगलोर केंद्र में छात्रों की कुल संख्या कक्षा A और B के मंब
ु ई केंद्र में छात्रों की
कुल संख्या से ककतने प्रनतशत अचधक है ?
(A) 8% (B) 10% (C) 15%
(D) 6% (E) None of these

143. What is the average number of students in all 5 centers together of class
B?
कक्षा B के सभी 5 केंद्रों में छात्रों की औसत संख्या ककतनी है ?
(A) 1100 (B) 1194 (C) 1200
(D) 1545 (E) None of these

144. What is the ratio of the total number of students in Bangalore and Delhi
centers of class A to the total number of students in Bangalore and Delhi
centers of class B?

https :
//www. https : https
https : //instagra
//youtube.c
facebo m.com/aas :
om/channe hisharoraso
l/UCYa4_Jr ok.com cial(?) //t.m
Orf8R5Kz2u
/aashis utm_mediu
e/stu

213
OtccXQ m=
haroras copy_link
ocial dified
कक्षा A के बैंगलोर और दिल्ली केंद्रों में छात्रों की कुल संख्या का कक्षा B के बैंगलोर और दिल्ली केंद्रों
में छात्रों की कुल संख्या से ककतना अनुपात है ?
(A) 35 : 43 (B) 39 : 43 (C) 38 : 45
(D) 33 : 43 (E) None of these

145. What is the difference between the total number of students in classes A
and B together in the center with maximum number of students and the total
number of students in classes A and B together in the center with minimum
number of students?
अचधकतम छात्रों की संख्या वाले केंद्र में कक्षा A और B में कुल छात्रों की कुल संख्या और न्यन ू तम
छात्रों की संख्या वाले केंद्र में कक्षा A और B में कुल छात्रों की संख्या के बीच ककतना अंतर है ?
(A) 1120 (B) 1150 (C) 1800
(D) 1180 (E) None of these

(146 - 150) Directions : Study the following passage carefully and answer the
questions given below.
Following information gives the data regarding number of teachers in Private
and Government schools in three different areas A, B and C. Total number of
teachers in Private schools are 470 and the ratio of number of Private and
Government teachers in area C is 5 : 3 respectively. 40% teachers in
Government schools in area A are males and remaining 108 are females.
Number of teachers in Government schools in area B are 50 less than number
of teachers in Private schools in same area. Number of teachers in
Government and Private schools in area B are in the ratio 5 : 7 respectively.
There are total 325 teachers in area A.
ननिे श : ननम्नललखखत गद्यांश का ध्यानपूवक ष अध्ययन कीजजए और नीचे दिए गए प्रश्नों के उत्तर
िीजजए।
ननम्नललखखत जानकारी तीन अलग-अलग क्षेत्रों A, B और C में ननजी और सरकारी स्कूलों में
लशक्षकों की संख्या के बारे में डेटा िे ती है । ननजी स्कूलों में लशक्षकों की कुल संख्या 470 है और क्षेत्र
C में ननजी और सरकारी लशक्षकों की संख्या का अनप ु ात क्रमशः 5 : 3 है । क्षेत्र A के सरकारी स्कूलों
में 40% लशक्षक पुरुर् हैं और शेर् 108 मदहलाएँ हैं। क्षेत्र B में सरकारी ववद्यालयों में लशक्षकों की
संख्या समान क्षेत्र में ननजी ववद्यालयों में लशक्षकों की संख्या से 50 कम है । क्षेत्र B में सरकारी और
ननजी स्कूलों में लशक्षकों की संख्या क्रमशः 5 : 7 के अनप ु ात में है । क्षेत्र A में कुल 325 लशक्षक हैं।

146. What is the difference between number of teachers in Government


schools in area C and number of teachers in Private schools in area B?
क्षेत्रC में सरकारी स्कूलों में लशक्षकों की संख्या और क्षेत्र B में ननजी स्कूलों में लशक्षकों की संख्या के
बीच ककतना अंतर है ?

https :
//www. https : https
https : //instagra
//youtube.c
facebo m.com/aas :
om/channe hisharoraso
l/UCYa4_Jr ok.com cial(?) //t.m
Orf8R5Kz2u
/aashis utm_mediu
e/stu

214
OtccXQ m=
haroras copy_link
ocial dified
(A) 95 (B) 90 (C) 85
(D) 80 (E) None of these

147. Approximately what percent of teachers in area A are in Private schools?


क्षेत्र A में लगभग ककतने प्रनतशत लशक्षक ननजी ववद्यालयों में हैं?
(A) 45% (B) 21% (C) 32.15%
(D) 22% (E) None of these

148. What is the respective ratio of number of teachers in Government school


in area A and C?
क्षेत्र A और C में सरकारी स्कूल में लशक्षकों की संख्या का क्रमशः अनपु ात ककतना है ?
(A) 2 : 3 (B) 2 : 1 (C) 10 : 9
(D) 5 : 1 (E) None of these

149. In area C, the ratio of male to female teachers in Private schools is 1 : 2


respectively, then number of female teachers in Private schools in area C are
approximately what percent more/less than total number of teachers in
Government schools in area A?
क्षेत्र C में , ननजी ववद्यालयों में पुरुर् और मदहला लशक्षकों का अनुपात क्रमशः 1 : 2 है , तो क्षेत्र C में
ननजी ववद्यालयों में मदहला लशक्षकों की संख्या, क्षेत्र A में सरकारी ववद्यालयों में लशक्षकों की कुल
संख्या से लगभग ककतने प्रनतशत अचधक/कम है ?
(A) 62% (B) 70% (C) 56%
(D) 45% (E) None of these

150. How many teachers are there in the Government schools in all given
areas taken together?
दिए गए सभी क्षेत्रों में सरकारी स्कूलों में कुल लमलाकर ककतने लशक्षक हैं?
(A) 375 (B) 400 (C) 257
(D) 395 (E) None of these

https :
//www. https : https
https : //instagra
//youtube.c
facebo m.com/aas :
om/channe hisharoraso
l/UCYa4_Jr ok.com cial(?) //t.m
Orf8R5Kz2u
/aashis utm_mediu
e/stu

215
OtccXQ m=
haroras copy_link
ocial dified
(1 - 5)
Friend ➔ P Q R Total
Income 42000 24000 30000 96000
Expenditure 28000 16000 19000 63000
Saving 14000 8000 11000 33000

1. Ans. (A)
Average of monthly saving of P, Q & R = 33000/3 = Rs. 11000

2. Ans. (C)
% of monthly expenditure of Q = 16000/63000 × 100 = 25.39%

3. Ans. (B)
Required % = (24000 - 19000) /24000 × 100 = 20.83%

4. Ans. (B)
Ratio = 28000 : (16000 : 19000) = 28000 : 35000 = 4 : 5

5. Ans. (B)
Yearly Income of Q = 24000 × 12 = Rs. 288000

(6 - 10)
City ➔ A B C Total
Hotstar 240 200 180 620
Netflix 160 150 320 630
Total 400 350 500 1250

06. Ans. (A)


Average of persons who have Netflix subscription = 630/3 = 210

07. Ans. (C)


% of persons who have Hotstar subscription in city A = 240/620 × 100 =
38.70%

08. Ans. (B)

https :
//www. https : https
https : //instagra
//youtube.c
facebo m.com/aas :
om/channe hisharoraso
l/UCYa4_Jr ok.com cial(?) //t.m
Orf8R5Kz2u
/aashis utm_mediu
e/stu

216
OtccXQ m=
haroras copy_link
ocial dified
Person who have Netflix subscription in city D = 500 × 1.32 - 180 × 1.2 = 660 -
216 = 444

09. Ans. (D)


Required % = (400 - 320) /320 × 100 = 25%

10. Ans. (B)


Number of Males who have subscription = 240 × 5/8 + 160 × 9/16 = 150 + 90 =
240

(11 - 15)
Colour➔ Red Green Black Yellow Total
Shopkeeper A 120 153 150 150 573
Shopkeeper B 165 135 180 140 620
Total 285 288 330 290 1193
11. Ans. (A)
Average = (120 + 150 + 150) /3 = 420/3 = 140

12. Ans. (C)


% of Black balls sold by B = 180/620 × 100 = 29.03%

13. Ans. (B)


Required % = (165 - 135) /135× 100 = 22.22%

14. Ans. (D)


White balls sold by A & B = 150 × 1.1 + 140 × 1.25 = 165 + 175 = 340

15. Ans. (B)


Ratio = (120 + 150) : (165 + 135) = 270 : 300 = 9 : 10

(16 - 20)

https :
//www. https : https
https : //instagra
//youtube.c
facebo m.com/aas :
om/channe hisharoraso
l/UCYa4_Jr ok.com cial(?) //t.m
Orf8R5Kz2u
/aashis utm_mediu
e/stu

217
OtccXQ m=
haroras copy_link
ocial dified
Shopkeeper ➔ P Q R S Total
Pizzas 90 108 90 108 396
Burgers 126 96 72 84 378
Total 216 204 162 192 774

16. Ans. (A)


Average = (126 + 96 + 72) /3 = 294/3 = 98

17. Ans. (C)


Required % = 72/216× 100 = 33.33%

18. Ans. (B)


Required % = (126 - 72) /72 × 100 = 75%

19. Ans. (D)


Ratio = 84 : 90 = 14 : 15

20. Ans. (B)


Donuts sold by R & S = 90 × 7/6 + 108 × 10/9 = 105 + 120 = 225

(21 - 25)

21. Ans. (A)


Total person who like Juice = 480 + 200 + 180 + 240 = 1100

https :
//www. https : https
https : //instagra
//youtube.c
facebo m.com/aas :
om/channe hisharoraso
l/UCYa4_Jr ok.com cial(?) //t.m
Orf8R5Kz2u
/aashis utm_mediu
e/stu

218
OtccXQ m=
haroras copy_link
ocial dified
22. Ans. (C)
Person who like exactly 2 juice = 130 + 120 + 180 = 430

23. Ans. (B)


Required % = 70/240× 100 = 29.16%

24. Ans. (D)


Ratio = (160 + 120) : 70 = 280 : 70 = 4 : 1

25. Ans. (B)


Required % = (180 - 130) /130 × 100 = 38.46%

(26 - 30)

Brand ➔ Samsung Phillips LG Sony Total


June 320 160 288 192 960
July 352 96 120 72 640
Total 672 256 408 264 1600
26. Ans. (E)
Required % = (288 - 72) /72 × 100 = 300%

27. Ans. (A)


Required % = (352 - 288) /352 × 100 = 200/11%

28. Ans. (C)


Ratio = 192 : 96 = 2 : 1

29. Ans. (D)


Average = (96 + 120 + 72) /3 = 288/3 = 96

30. Ans. (D)


Difference = 672 - 408 = 264

(31 - 35)

https :
//www. https : https
https : //instagra
//youtube.c
facebo m.com/aas :
om/channe hisharoraso
l/UCYa4_Jr ok.com cial(?) //t.m
Orf8R5Kz2u
/aashis utm_mediu
e/stu

219
OtccXQ m=
haroras copy_link
ocial dified
31. Ans. (C)
Students enrolled for Business = 207

32. Ans. (A)


Sum of students enrolled for Arts only and for all 3 sections = 115 + 23 = 138
Required % = (138 - 69) /69 × 100 = 100% more

33. Ans. (B)


Revenue from Arts section = 230 × 75 = Rs 17250

34. Ans. (C)


Ratio = 207 : 230 = 9 : 10

35. Ans. (D)


Average = (69 + 115 + 92) /3 = 276/3 = 92

(36 - 40)
Aman has = 36000/0.6 = Rs. 60000
Amount distributed among daughters = 60000 × 0.4 = Rs. 24000
A received = 24000 × 1/4 = Rs. 6000
B received = (24000 × 3/4 + 250) /2 = (18000 + 250) /2 = 18250/2 = Rs. 9125
C received = 9125 - 250 = Rs. 8875
Expenditure of B = 9125 × 0.4 = Rs. 3650, Saving of B = 9125 × 0.6 = Rs. 5475
Expenditure of A = 3650 - 650 = Rs. 3000,

https :
//www. https : https
https : //instagra
//youtube.c
facebo m.com/aas :
om/channe hisharoraso
l/UCYa4_Jr ok.com cial(?) //t.m
Orf8R5Kz2u
/aashis utm_mediu
e/stu

220
OtccXQ m=
haroras copy_link
ocial dified
Saving of A = 6000 - 3000 = Rs. 3000
Expenditure of C = 3000 × 6/5 = Rs. 3600,
Saving of C = 8875 - 3600 = Rs. 5275

36. Ans. (C)


Ratio = 5475 : 5275 = 219 : 211

37. Ans. (A)


Average = (3000 + 3650 + 3600) /3 = 10250/3 = Rs. 3416.66

38. Ans. (B)


Difference = 8875 - (3000 + 5475) = 8875 - 8475 = Rs. 400

39. Ans. (C)


Remaining amount of A & C = 5275 × 3/5 + 3000 × 2/5 = 3165 + 1200 = Rs.
4365

40. Ans. (D)


Required % = (3000 + 5475 + 5275) /60000 × 100 = 13750/60000 × 100 =
22.91% = 23%

(41 - 45)
Village➔ P Q R Total
Government 945 1350 1800 4095
Private 972 810 1323 3105
Total 1917 2160 3123 7200

41. Ans. (C)


Difference = 945 - 810 = 135

42. Ans. (A)


Total population of village R = 3123

43. Ans. (B)


Required % = 1350/1800 × 100 = 75%

https :
//www. https : https
https : //instagra
//youtube.c
facebo m.com/aas :
om/channe hisharoraso
l/UCYa4_Jr ok.com cial(?) //t.m
Orf8R5Kz2u
/aashis utm_mediu
e/stu

221
OtccXQ m=
haroras copy_link
ocial dified
44. Ans. (C)
Ratio = 972 : 1323 = 36 : 49

45. Ans. (D)


Required % = 1917/2160 × 100 = 88.75%

(46 - 50)
Company➔ Samsung Apple MI Total
December 2022 25000 35000 38000 98000
January 2023 22000 34000 35500 91500
Total 47000 69000 73500 189500

46. Ans. (C)


Difference = 35500 - 35000 = 500

47. Ans. (A)


% reduction in Apple phones from December 2022 to January 2023
= (35000 - 34000) /35000 × 100 = 2.85%

48. Ans. (B)


OnePlus phones sold in December 2022 = 25000 × 1.2 = 30000

49. Ans. (C)


MI phones sold in December 2021 = 38000/1,9 = 20000

50. Ans. (D)


Average of Apple phone sold = 69000/2 = 34500

(51 - 55)
Language ➔ Hindi English Telugu Total
A 320 480 400 1200
B 840 520 440 1800
Total 1160 1000 840 3000

51. Ans. (C)

https :
//www. https : https
https : //instagra
//youtube.c
facebo m.com/aas :
om/channe hisharoraso
l/UCYa4_Jr ok.com cial(?) //t.m
Orf8R5Kz2u
/aashis utm_mediu
e/stu

222
OtccXQ m=
haroras copy_link
ocial dified
Average of magazines sold by Shopkeeper A = 1200/3 = 400

52. Ans. (A)


Required % = (480 - 440) /480 × 100 = 8.33%

53. Ans. (B)


Ratio = (440 + 520) : (480 + 320) = 960 : 800 = 6 : 5

54. Ans. (C)


German magazine sold = 1000 × 1.3 = 1300
Difference = 1300 - 840 = 460

55. Ans. (D)


Difference = 1800 - 1200 = 600

(56 - 60)
Village ➔ A B C Total
Male 360 360 480 1200
Female 240 200 360 800
Literate 400 350 630 1380
Illiterate 200 210 210 620
Total 600 560 840 2000
56. Ans. (C)
Gender Ratio = 1200 : 800 = 3 : 2

57. Ans. (A)


% of Literacy in all villages = 1380/2000 × 100 = 69%

58. Ans. (B)


Males in village A & B = 360 + 360 = 720
Females in village A & C = 240 + 360 = 600
Required % = (720 - 600) /600 × 100 = 20%

59. Ans. (C)


Average of Literates in village B & C = (350 + 630) /2 = 980/2 = 490

https :
//www. https : https
https : //instagra
//youtube.c
facebo m.com/aas :
om/channe hisharoraso
l/UCYa4_Jr ok.com cial(?) //t.m
Orf8R5Kz2u
/aashis utm_mediu
e/stu

223
OtccXQ m=
haroras copy_link
ocial dified
Average of Illiterates in village A & B = (200 + 210) /2 = 410/2 = 205
Difference = 490 - 205 = 285

60. Ans. (B)


Ratio = (400 - 240) : (480 - 210) = 160 : 270 = 16 : 27

(61 - 65)

61. Ans. (C)


Customers who purchased painting of only A, only B and only C = 80 + 180
+ 140 = 400

62. Ans. (A)


Total customers who purchase painting = 480 + 80 + 80 + 140 = 780
Customers who did not purchase painting = 800 - 780 = 20

63. Ans. (B)


Customers who purchased painting of at least 2 painters = 120 + 80 + 140 =
340

64. Ans. (C)


Customers who purchased painting of minimum 2 painters = 120 + 80 + 140
+ 40 = 380
Customers who purchased painting of maximum 2 painters
= 120 + 80 + 140 + 80 + 180 + 140 = 740

https :
//www. https : https
https : //instagra
//youtube.c
facebo m.com/aas :
om/channe hisharoraso
l/UCYa4_Jr ok.com cial(?) //t.m
Orf8R5Kz2u
/aashis utm_mediu
e/stu

224
OtccXQ m=
haroras copy_link
ocial dified
Required % = 380/740 × 100 = 51.35% = 51%

65. Ans. (D)


Required % = (320 - 180) /180 × 100 = 77.77% = 78%

(66 - 70)
Class ➔ A B C D E Total
Boys 45 60 50 36 44 235
Girls 35 48 60 64 66 273
Total 80 108 110 100 110 508

66. Ans. (D)


Ratio = (60 + 50) : (48 + 60) = 110 : 108 = 55 : 54

67. Ans. (A)


Average number of Girls = 273/5 = 54.6

68. Ans. (B)


Average number of Boys = 235/5 = 47
Difference = 50 - 47 = 3

69. Ans. (C)


Number of Girls more than average number of Girls is in = All C, D & E

70. Ans. (D)


Required % = (108 - 100) /100 × 100 = 8%

(71 - 75)
Flower ➔ Rose Lilly Jasmine Total
Sold 400 250 140 790
Unsold 200 50 10 260
Total 600 300 150 1050

71. Ans. (C)


Required % = 150/(400 + 250) × 100 = 150/650 × 100 = 23%

https :
//www. https : https
https : //instagra
//youtube.c
facebo m.com/aas :
om/channe hisharoraso
l/UCYa4_Jr ok.com cial(?) //t.m
Orf8R5Kz2u
/aashis utm_mediu
e/stu

225
OtccXQ m=
haroras copy_link
ocial dified
72. Ans. (A)
Total unsold flowers = 260

73. Ans. (B)


Required % = (250 - 200) /200 × 100 = 25%

74. Ans. (C)


Ratio = 150 : (200 + 50) = 150 : 250 = 3 : 5

75. Ans. (D)


Price of each unsold Rose flower = 10000/200 = 50

(76 - 80)

76. Ans. (A)


Persons who like 2 drinks = 48 + 64 + 72 + 96 = 280

77. Ans. (C)


Required % = 64/400 × 100 = 16%

78. Ans. (B)


Required % = (192 - 72) /192 × 100 = 62.5%

79. Ans. (D)


Total person who like drink = 336 + 192 + 64 + 144 = 736

https :
//www. https : https
https : //instagra
//youtube.c
facebo m.com/aas :
om/channe hisharoraso
l/UCYa4_Jr ok.com cial(?) //t.m
Orf8R5Kz2u
/aashis utm_mediu
e/stu

226
OtccXQ m=
haroras copy_link
ocial dified
80. Ans. (B)
Female who like only Juice = 144 × 5/12 = 60

(81 - 85)
Profession➔ Teacher Doctor Engineer Lawyer Chennai
Male 180 240 210 176 806
Female 160 196 250 154 760
Total 340 436 460 330 1566

81. Ans. (A)


Ratio = (240 + 210) : (196 + 154) = 450 : 350 = 9 : 7

82. Ans. (C)


% of Female Teachers = 160/760 × 100 = 21.05%

83. Ans. (B)


Required % = (250 - 180) /180× 100 = 38.88%

84. Ans. (D)


Average of Female professionals = 760/4 = 190

85. Ans. (B)


Male Shopkeepers = 180 × 1.6 = 288
Female Shopkeepers = 160 × 1.4 = 224
Difference = 288 - 224 = 64

(86 - 90)

https :
//www. https : https
https : //instagra
//youtube.c
facebo m.com/aas :
om/channe hisharoraso
l/UCYa4_Jr ok.com cial(?) //t.m
Orf8R5Kz2u
/aashis utm_mediu
e/stu

227
OtccXQ m=
haroras copy_link
ocial dified
86. Ans. (A)
Students who like exactly 2 games = 140 + 110 + 150 = 400

87. Ans. (C)


Boys who like Candy Crush = 755 × 2/5 = 302

88. Ans. (B)


Total Students in school = 670 + 405 + 150 + 450 + 150 = 1825

89. Ans. (D)


Required % = 150/770 × 100 = 19.48%

90. Ans. (B)


Required % = (150 - 110) /110 × 100 = 36.36%

(91 - 95)
Class ➔ 10th 11th 12th Total
Boys 120 96 144 360
Girls 90 75 126 291
Total 210 171 270 651

91. Ans. (A)


Average number of Boys = 360/3 = 120

https :
//www. https : https
https : //instagra
//youtube.c
facebo m.com/aas :
om/channe hisharoraso
l/UCYa4_Jr ok.com cial(?) //t.m
Orf8R5Kz2u
/aashis utm_mediu
e/stu

228
OtccXQ m=
haroras copy_link
ocial dified
92. Ans. (C)
Required % = 75/270 × 100 = 27.77%

93. Ans. (B)


Required % = (144 - 90) /90× 100 = 60%

94. Ans. (A)


Ratio = 120 : 90 = 4 : 3

95. Ans. (C)


Boys & Girls in 9th class = 120 × 0.8 + 90 × 0.9 = 96 + 81 = 177

(96 - 100)
Shopkeeper➔ A B C Total
Pens 80 85 72 237
Pencils 60 96 75 231
Erasers 45 63 54 162
Total 185 244 201 630
96. Ans. (A)
Average number of items sold by C = 201/3 = 67

97. Ans. (C)


% of Pens sold by B = 85/237× 100 = 35.86%

98. Ans. (B)


Required % = (96 - 54) /96× 100 = 43.75%

99. Ans. (D)


Pen & Pencil sold by D = 72 × 1.125 + 75 × 1.2 = 81 + 90 = 171

100. Ans. (B)


Amount received by A = 80 × 5 + 60 × 8 + 45 × 7 = 400 + 480 + 315 = Rs. 1195

(101 - 105)

https :
//www. https : https
https : //instagra
//youtube.c
facebo m.com/aas :
om/channe hisharoraso
l/UCYa4_Jr ok.com cial(?) //t.m
Orf8R5Kz2u
/aashis utm_mediu
e/stu

229
OtccXQ m=
haroras copy_link
ocial dified
Month➔ April May June Total
Dell 240 280 330 850
HP 220 320 360 900
Asus 300 250 270 820
Total 760 850 960 2570
101. Ans. (A)
Average of laptops sold in June = 960/3 = 320

102. Ans. (C)


In May, % of HP laptop sold = 320/850× 100 = 37.64%

103. Ans. (B)


% of HP laptops sold = 900/2570 × 100 = 35.02%

104. Ans. (D)


Dell & HP laptop sold in July = 330 × 12/11 + 360 × 10/9 = 360 + 400 = 760

105. Ans. (B)


Ratio = (240 + 300) : 270 = 540 : 270 = 2 : 1

(106 - 110)
Shopkeeper ➔ A B C D Total
Milk Chocolate 250 280 350 270 1150
White Chocolate 300 200 240 300 1040
Total 550 480 590 570 2190

106. Ans. (A)


Average of White Chocolates sold = 1040/4 = 260

107. Ans. (C)


% of Milk chocolates sold by B = 280/1150× 100 = 24.34%

108. Ans. (B)


Chocolates sold by E = 270 × 10/9 + 300 × 1.3 = 300 + 330 = 630

https :
//www. https : https
https : //instagra
//youtube.c
facebo m.com/aas :
om/channe hisharoraso
l/UCYa4_Jr ok.com cial(?) //t.m
Orf8R5Kz2u
/aashis utm_mediu
e/stu

230
OtccXQ m=
haroras copy_link
ocial dified
109. Ans. (D)
Dark Chocolates sold by A & B = 250 × 1.2 + 280 × 9/7 = 300 + 360 = 660

110. Ans. (B)


Required % = (350 - 200) /200 × 100 = 75%

(111 - 115)

111. Ans. (A)


Students who like Football = 550

112. Ans. (C)


Students who like only 1 game = 110 + 90 + 70 = 270

113. Ans. (B)


Required % = (160 - 90) /90 × 100 = 77.77%

114. Ans. (D)


Boys who like only Cricket = 110 × 5/11 = 50
Girls who like only Chess = 90 × 4/9 = 40
Difference = 50 - 40 = 10

115. Ans. (B)


Required % = 110/630 × 100 = 17.46%

https :
//www. https : https
https : //instagra
//youtube.c
facebo m.com/aas :
om/channe hisharoraso
l/UCYa4_Jr ok.com cial(?) //t.m
Orf8R5Kz2u
/aashis utm_mediu
e/stu

231
OtccXQ m=
haroras copy_link
ocial dified
(116-120)

Locality ➔ A B C D Total
Males 225 240 200 235 900
Females 235 180 160 120 695
Total 460 420 360 355 1595

116. Ans. (A)


Average = (225 + 200 + 235) /3 = 660/3 = 220

117. Ans. (C)


% of males in locality B = 240/900 ×100 = 26.66%

118. Ans. (B)


Required % = (225 - 120) /120 × 100 = 87.5%

119. Ans. (D)


Required % = (240 + 200) : (180 + 160) = 440 : 340 = 22 : 17

120. Ans. (D)


Persons in locality E = 235 × 1.2 + 120 × 16/15 = 282 + 128 = 410

(121 - 125)

https :
//www. https : https
https : //instagra
//youtube.c
facebo m.com/aas :
om/channe hisharoraso
l/UCYa4_Jr ok.com cial(?) //t.m
Orf8R5Kz2u
/aashis utm_mediu
e/stu

232
OtccXQ m=
haroras copy_link
ocial dified
121. Ans. (A)
Required % = 360/1000 × 100 = 36%

122. Ans. (C)


Students who have 6 months plan of only Oliveboard& only Guidely
= 480 × 5/12 + 360 × 4/9 = 200 + 160 = 360

123. Ans. (B)


Students who have at least 2 mocks = 160 + 210 + 200 + 280 = 850

124. Ans. (D)


Students who doesn’t have Oliveboard mocks = 360 + 200 + 400 = 960

125. Ans. (B)


Required % = (480 - 210) /480 × 100 = 56.25%

(126 - 130)

126. Ans. (C)


Ratio = 566 : 540 = 283 : 270

127. Ans. (A)


Students attended webinar on exactly 2 courses = 120 + 154 + 168 = 442

128. Ans. (B)

https :
//www. https : https
https : //instagra
//youtube.c
facebo m.com/aas :
om/channe hisharoraso
l/UCYa4_Jr ok.com cial(?) //t.m
Orf8R5Kz2u
/aashis utm_mediu
e/stu

233
OtccXQ m=
haroras copy_link
ocial dified
Required % = (130 + 154) /(130 + 168) × 100 = 284/298 × 100 = 95.30% = 95%

129. Ans. (C)


Difference = 144 - 136 = 8

130. Ans. (D)


Students attended webinar on exactly one course = 148 + 136 + 144 = 428

(131 - 135)
Exam➔ SSC Bank Railway UPSC Total
Morning 120 80 160 40 400
Evening 160 200 90 50 500
Total 280 280 250 90 900

131. Ans. (C)


Difference between Boys and Girls in SSC Evening Batch = 160 × (21 - 19)
/40 = 4 × 2 = 8

132. Ans. (A)


Ratio = 400 : 500 = 4 : 5

133. Ans. (B)


Required % = (280 - 250) /250 × 100 = 12%

134. Ans. (B)


Ratio = 54 : (90 - 54) = 54 : 36 = 3 : 2

135. Ans. (D)


Students who didn’t clear the Railway exam = 160 × 0.25 + 90 × 0.2 = 40 + 18
= 58

(136 - 140)

https :
//www. https : https
https : //instagra
//youtube.c
facebo m.com/aas :
om/channe hisharoraso
l/UCYa4_Jr ok.com cial(?) //t.m
Orf8R5Kz2u
/aashis utm_mediu
e/stu

234
OtccXQ m=
haroras copy_link
ocial dified
Month➔ July August September Total
Mobile 50 40 10 100
Laptop 20 30 50 100
Total 70 70 60 200

136. Ans. (A)


Ratio = 50 : 30 = 5 : 3

137. Ans. (A)


Required % = (20 + 30) /(40 + 10) × 100 = 50/50 × 100 = 100%

138. Ans. (A)


Average = (20 + 50) /2 = 70/2 = 35

139. Ans. (D)


Difference = (50 + 40) - (20 + 30) = 90 ; 50 = 40

140.Ans. (A)
Required % = (50 - 40) /40 × 100 = 25%

(141-145)
Area ➔ A B C Total
Government School 180 125 90 395
Private School 145 175 150 470
Total 325 300 240 865

141. Ans. (C)


Ratio = (1170 - 650) : (1320 - 1180) = 520 : 140 = 26 : 7

142. Ans. (A)


Required % = (2700 - 2500) /2500 × 100 = 8%

143. Ans. (B)


Average Students in Class B = 5970/5 = 1194

https :
//www. https : https
https : //instagra
//youtube.c
facebo m.com/aas :
om/channe hisharoraso
l/UCYa4_Jr ok.com cial(?) //t.m
Orf8R5Kz2u
/aashis utm_mediu
e/stu

235
OtccXQ m=
haroras copy_link
ocial dified
144. Ans. (C)
Ratio = (1200 + 1080) : (1500 + 1200) = 2280 : 2700 = 38 : 45

145. Ans. (D)


Difference = 3000 - 1820 = 1180

(146 - 150)
Teachers in Government School in Area A = 108/0.6 = 180

Center ➔ Bangalore Pune Mumbai Hyderabad Delhi Total


Class A 1200 1170 1320 1560 1080 6330
Class B 1500 650 1180 1440 1200 5970
Total 2700 1820 2500 3000 2280 12300
146. Ans. (C)
Difference = 175 - 90 = 85

147. Ans. (A)


% of Private Teachers in Area A = 145/325 × 100 = 44.61% = 45%

148. Ans. (B)


Ratio = 180 : 90 = 2 : 1

149. Ans. (C)


Female Teachers in Private school in Area C = 150 × 2/3 = 100
Required % = 100/180 × 100 = 55.55% = 56%

150. Ans. (D)


Teachers in Government Schools = 395

https :
//www. https : https
https : //instagra
//youtube.c
facebo m.com/aas :
om/channe hisharoraso
l/UCYa4_Jr ok.com cial(?) //t.m
Orf8R5Kz2u
/aashis utm_mediu
e/stu

236
OtccXQ m=
haroras copy_link
ocial dified
https :
//instagram.com/aashisharorasocia
l(?) utm_medium = copy_link

https: //t.me/studified
https : //t.me/studified

https :
//youtube.com/channel/UCYa
4_JrOrf8R5Kz2uOtccXQ

https :
//www.facebook.com/aa
shisharorasocial

https :
//www. https : https
https : //instagra
//youtube.c
facebo m.com/aas :
om/channe hisharoraso
l/UCYa4_Jr ok.com cial(?) //t.m
Orf8R5Kz2u
/aashis utm_mediu
e/stu

237
OtccXQ m=
haroras copy_link
ocial dified
(1 - 5) Directions : Study the following chart carefully and answer the
questions given below.
The line graph given below shows the number of food orders delivered by
Zomato and Swiggy on five different days.
ननिे श : ननम्नललखखत चाटष का ध्यानपूवक
ष अध्ययन कीजजए और नीचे दिए गए प्रश्नों के उत्तर
िीजजए।
नीचे दिया गया लाइन ग्राि Zomato और Swiggy द्वारा पांच अलग - अलग दिनों में डडलीवर
ककए गए खाने के ऑडषर की संख्या को िशाषता है ।

1. Find the average of number of food orders delivered by Zomato on all five
days together.
सभी पांच दिनों में एक साथ Zomato द्वारा डडलीवर ककए गए खाने के ऑडषर की औसत संख्या
ज्ञात कीजजए।
(A) 344 (B) 354 (C) 390
(D) 322 (E) None of these

2. Number of food orders delivered by Swiggy on Thursday is what percent of


total number of food orders delivered by Swiggy on all five days?
गरु
ु वार को Swiggy द्वारा डडलीवर ककए गए िूड ऑडषर की संख्या, Swiggy द्वारा सभी पांच
दिनों में डडलीवर ककए गए िूड ऑडषर की कुल संख्या का ककतना प्रनतशत है ?
(A) 16.75% (B) 19.14% (C) 14.25%
(D) 13.60% (E) None of these

3. Number of food orders delivered by Zomato on Tuesday is how much

https :
//www. https : https
https : //instagra
//youtube.c
facebo m.com/aas :
om/channe hisharoraso
l/UCYa4_Jr ok.com cial(?) //t.m
Orf8R5Kz2u
/aashis utm_mediu
e/stu

238
OtccXQ m=
haroras copy_link
ocial dified
percent more or less than number of food orders delivered by Zomato on
Monday?
मंगलवार को Zomato द्वारा डडलीवर ककए गए फूड ऑडषर की संख्या, Zomato द्वारा सोमवार
को डडलीवर ककए गए फूड ऑडषर की संख्या से ककतने प्रनतशत अचधक या कम है ?
(A) 77.77% (B) 72.50% (C) 67.50%
(D) 60.50% (E) None of these

4. Number of food orders delivered by Zomato and Swiggy on Saturday is


respectively 14.28% and 9.09% more than number of food orders delivered
by Zomato and Swiggy on Friday, then find the sum of number of food
orders delivered by Zomato and Swiggy on Saturday.
शननवार को Zomato और Swiggy द्वारा डडलीवर ककए गए िूड ऑडषर की संख्या शुक्रवार को
Zomato और Swiggy द्वारा डडलीवर ककए गए िूड ऑडषर की संख्या से क्रमशः 14.28% और
9.09% अचधक है , तो शननवार को Zomato और Swiggy द्वारा डडलीवर ककए गए िूड ऑडषर
की संख्या का योग ज्ञात कीजजए।
(A) 880 (B) 850 (C) 810
(D) 830 (E) None of these

5. Number of food orders delivered by Uber eats on Wednesday is 16.66%


less than average of number of food orders delivered by Swiggy and Zomato
on Wednesday and number of food orders delivered by Uber eats on
Tuesday is 20% less than number of food orders delivered by Swiggy on
Tuesday. Find the sum of number of food orders delivered by Uber eats on
Wednesday and Tuesday.
बधु वार को Uber eats द्वारा डडलीवर ककए गए िूड ऑडषर की संख्या बधु वार को Swiggy और
Zomato द्वारा डडलीवर ककए गए िूड ऑडषर की संख्या के औसत से 16.66% कम है और
मंगलवार को Uber eats द्वारा डडलीवर ककए गए िूड ऑडषर की संख्या मंगलवार को Swiggy
द्वारा डडलीवर ककए गए िूड ऑडषर की संख्या से 20% कम है । बुधवार और मंगलवार को Uber
Eats द्वारा डडलीवर ककए गए खाने के ऑडषर की संख्या का योग ज्ञात कीजजए।
(A) 654 (B) 680 (C) 642
(D) 606 (E) None of these

(6 - 10) Directions : Study the following chart carefully and answer the
questions given below.
The pie chart given below shows the percentage distribution of Keyboards
sold by HP in five different months. Total number of Keyboard sold is 2400.
ननिे श : ननम्नललखखत चाटष का ध्यानपूवक ष अध्ययन कीजजए और नीचे दिए गए प्रश्नों के उत्तर
िीजजए।
नीचे दिया गया पाई चाटष पांच अलग-अलग महीनों में HP द्वारा बेचे गए कीबोडष का प्रनतशत
ववतरण िशाषता है । बेचे गए कीबोडष की कुल संख्या 2400 है ।

https :
//www. https : https
https : //instagra
//youtube.c
facebo m.com/aas :
om/channe hisharoraso
l/UCYa4_Jr ok.com cial(?) //t.m
Orf8R5Kz2u
/aashis utm_mediu
e/stu

239
OtccXQ m=
haroras copy_link
ocial dified
6. Number of Keyboards sold in February is how much percent more or less
than number of Keyboards sold in January?
िरवरी में बेचे गए कीबोडष की संख्या जनवरी में बेचे गए कीबोडष की संख्या से ककतने प्रनतशत
अचधक या कम है ?
(A) 26.50% (B) 32.50% (C) 28.50%
(D) 27.77% (E) None of these

7. The ratio of number of Wired to Wireless Keyboard sold in April and May
is respectively 7 : 5 and 5 : 3, then find the difference between number of
Wired Keyboards sold in April and May.
अप्रैल और मई में बेचे गए वायडष और वायरलेस कीबोडष की संख्या का अनप ु ात क्रमशः 7 : 5 और
5 : 3 है , तो अप्रैल और मई में बेचे गए वायडष कीबोडष की संख्या के बीच का अंतर ज्ञात कीजजए।
(A) 105 (B) 96 (C) 80
(D) 120 (E) None of these

8. Number of Keyboards sold in June and July is respectively 12.5% and


25% more than number of Keyboards sold in March and February. Find the
total number of Keyboards sold in June and July.
जून और जुलाई में बेचे गए कीबोडष की संख्या माचष और िरवरी में बेचे गए कीबोडष की संख्या से
क्रमशः 12.5% और 25% अचधक है । जून और जुलाई में बेचे गए कीबोडष की कुल संख्या ज्ञात
कीजजए।
(A) 1172 (B) 1203 (C) 1150
(D) 1250 (E) None of these

9. Find the average of number of Keyboards sold in January, February and


March.

https :
//www. https : https
https : //instagra
//youtube.c
facebo m.com/aas :
om/channe hisharoraso
l/UCYa4_Jr ok.com cial(?) //t.m
Orf8R5Kz2u
/aashis utm_mediu
e/stu

240
OtccXQ m=
haroras copy_link
ocial dified
जनवरी, िरवरी और माचष में बेचे गए कीबोडष की औसत संख्या ज्ञात कीजजए।
(A) 480 (B) 345 (C) 364
(D) 336 (E) None of these

10. Number of Keyboards sold by Dell and Asus in January is respectively


25% and 8.33% more than number of Keyboards sold in January by HP, then
find the sum of number of Keyboards sold by Dell and Asus in January.
जनवरी में Dell और Asus द्वारा बेचे गए कीबोडष की संख्या, HP द्वारा जनवरी में बेचे गए
कीबोडष की संख्या से क्रमशः 25% और 8.33% अचधक है , तो जनवरी में Dell और Asus द्वारा
बेचे गए कीबोडष की संख्या का योग ज्ञात कीजजए।
(A) 1080 (B) 1222 (C) 1436
(D) 1008 (E) None of these

(11 - 15) Directions : Study the following chart carefully and answer the
questions given below.
The line chart given below shows the number of shirts sold by five different
shopkeepers in three different months.
ननिे श : ननम्नललखखत चाटष का ध्यानपूवक ष अध्ययन कीजजए और नीचे दिए गए प्रश्नों के उत्तर
िीजजए।
नीचे दिया गया लाइन चाटष तीन अलग - अलग महीनों में पांच अलग - अलग िक ु ानिारों द्वारा
बेची गई कमीजों की संख्या को िशाषता है ।

11. Number of shirts sold in January by N is what percent of total number of


shirts sold in January?
N द्वारा जनवरी में बेची गई कमीजों की संख्या जनवरी में बेची गई कमीजों की कुल संख्या का
ककतना प्रनतशत है ?

https :
//www. https : https
https : //instagra
//youtube.c
facebo m.com/aas :
om/channe hisharoraso
l/UCYa4_Jr ok.com cial(?) //t.m
Orf8R5Kz2u
/aashis utm_mediu
e/stu

241
OtccXQ m=
haroras copy_link
ocial dified
(A) 16.42% (B) 15.23% (C) 18.29%
(D) 17.34% (E) None of these

12. The ratio of number of Cotton shirts sold to number of Rayon shirts sold
in January and March by M is 5 : 4 and 3 : 4 respectively, then find the sum
of number of Cotton shirts sold in January and number of Rayon shirts sold
in March.
M द्वारा जनवरी और माचष में बेची गई कॉटन कमीजों की संख्या का बेची गई रे यॉन कमीजों की
संख्या से अनुपात क्रमशः 5 : 4 और 3 : 4 है , तो जनवरी में बेची गई कॉटन कमीजों की संख्या
और माचष में बेची गई रे यॉन कमीजों की संख्या का योग ज्ञात कीजजए।
(A) 360 (B) 370 (C) 350
(D) 340 (E) None of these

13. Find the ratio of sum of number of shirts sold in February by N and P to
sum of number of shirts sold in March by M and N.
N और P द्वारा िरवरी में बेची गई कमीजों की संख्या के योग का M और N द्वारा माचष में बेची
गई कमीजों की संख्या से अनुपात ज्ञात कीजजए।
(A) 67 : 68 (B) 61 : 62 (C) 63 : 64
(D) 65 : 66 (E) None of these

14. Number of shirts sold in April by M and N is respectively 22.22% and 20%
more than number of shirts sold in March, then find the number of shirts
sold in April by M and N.
M और N द्वारा अप्रैल में बेची गई कमीजों की संख्या माचष में बेची गई कमीजों की संख्या से
क्रमशः 22.22% और 20% अचधक है , तो M और N द्वारा अप्रैल में बेची गई कमीजों की संख्या
ज्ञात कीजजए।
(A) 823 (B) 815 (C) 836
(D) 841 (E) None of these

15. Number of shirts sold by P in March is how much percent more or less
than number of shirts sold by N in January?
माचष में P द्वारा बेची गई कमीजों की संख्या जनवरी में N द्वारा बेची गई कमीजों की संख्या से
ककतने प्रनतशत अचधक या कम है ?
(A) 14.23% (B) 12.89% (C) 16.45%
(D) 18.86% (E) None of these

(16-20) Directions : Study the following chart carefully and answer the
questions given below.
The bar graph given below shows the sum of number of Jio and Airtel Sim
sold and difference between number of Jio and Airtel Sim sold on five
different days.

https :
//www. https : https
https : //instagra
//youtube.c
facebo m.com/aas :
om/channe hisharoraso
l/UCYa4_Jr ok.com cial(?) //t.m
Orf8R5Kz2u
/aashis utm_mediu
e/stu

242
OtccXQ m=
haroras copy_link
ocial dified
जनदेश : जनम्नजलखित चाटत का ध्यानपूवतक अध्ययन कीजिए और नीचे ददए गए प्रश्नों के
उत्तर दीजिए।
नीचे ददया गया बार ग्राफ पांच अलग - अलग ददनों में बेचे गए Jio और Airtel जसम की
संख्या और Jio और Airtel जसम की संख्या के बीच अंतर को दशातता है ।

16. Find the average of number of Airtel Sim sold on Tuesday, Wednesday
and Thursday.
मंगलवार, बध
ु वार और गरु
ु वार को बेची गई Airtel लसम की औसत संख्या ज्ञात कीजजए।
(A) 196 (B) 184 (C) 172
(D) 168 (E) None of these

17. Number of Jio Sim sold on Wednesday is what percent of sum of number
of Jio Sim sold on Monday, Tuesday and Wednesday?
बुधवार को बेचे गए Jio लसम की संख्या सोमवार, मंगलवार और बुधवार को बेचे गए Jio लसम के
योग का ककतना प्रनतशत है ?
(A) 31.56% (B) 30.87% (C) 32.89%
(D) 33.41% (E) None of these

18. If number of Jio and Airtel Sim sold on Saturday is respectively 30% and
10% more than number of Jio and Airtel Sim sold on Thursday, then find the
difference between number of Jio and Airtel Sim sold on Saturday.
यदि शननवार को बेचे गए Jio और Airtel लसम की संख्या गुरुवार को बेचे गए Jio और Airtel
लसम की संख्या से क्रमशः 30% और 10% अचधक है , तो शननवार को बेचे गए Jio और Airtel
लसम की संख्या के बीच का अंतर ज्ञात कीजजए।
(A) 65 (B) 75 (C) 55
(D) 85 (E) None of these

19. Number of Jio Sim sold on Monday is how much percent more or less
than number of Airtel Sim sold on Thursday?

https :
//www. https : https
https : //instagra
//youtube.c
facebo m.com/aas :
om/channe hisharoraso
l/UCYa4_Jr ok.com cial(?) //t.m
Orf8R5Kz2u
/aashis utm_mediu
e/stu

243
OtccXQ m=
haroras copy_link
ocial dified
सोमवार को बेची गई Jio लसम की संख्या गुरुवार को बेची गई Airtel लसम की संख्या से ककतने
प्रनतशत अचधक या कम है ?
(A) 22.22% (B) 28.96% (C) 26.45%
(D) 24.10% (E) None of these

20. Number of Vodafone Sim sold on Thursday and Friday is respectively


40% and 28.56% more than number of Airtel Sim sold on Thursday and
Friday, then find the sum of number of Vodafone Sim sold on Thursday and
Friday.
गरु
ु वार और शक्र
ु वार को बेची गई Vodafone लसम की संख्या गरु
ु वार और शक्र
ु वार को बेची गई
Airtel लसम की संख्या से क्रमशः 40% और 28.56% अचधक है , तो गुरुवार और शुक्रवार को बेची
गई Vodafone लसम की संख्या का योग ज्ञात कीजजए।
(A) 458 (B) 442 (C) 463
(D) 477 (E) None of these

(21 - 25) Directions : Study the following chart carefully and answer the
questions given below.
The table given below shows the number of persons who visits four
different cities in four different months.
ननिे श : ननम्नललखखत चाटष का ध्यानपव
ू क
ष अध्ययन कीजजए और नीचे दिए गए प्रश्नों के उत्तर
िीजजए।
नीचे िी गई ताललका चार अलग-अलग महीनों में चार अलग-अलग शहरों में जाने वाले व्यजक्तयों
की संख्या िशाषती है ।

City January February March April Total

M 280 325 215 195 1015

N 185 287 320 240 1032

O 305 164 198 245 91 2

P 210 224 227 220 881

21. Find the average of number of persons who visits all four cities in
February.
िरवरी में सभी चार शहरों का िौरा करने वाले व्यजक्तयों की औसत संख्या ज्ञात कीजजए।
(A) 270 (B) 230 (C) 250
(D) 210 (E) None of these

https :
//www. https : https
https : //instagra
//youtube.c
facebo m.com/aas :
om/channe hisharoraso
l/UCYa4_Jr ok.com cial(?) //t.m
Orf8R5Kz2u
/aashis utm_mediu
e/stu

244
OtccXQ m=
haroras copy_link
ocial dified
22. Number of persons who visits city N in January is what percent of total
number of persons who visits city N in all four months?
जनवरी में शहर N का िौरा करने वाले व्यजक्तयों की संख्या सभी चार महीनों में शहर N का िौरा
करने वाले व्यजक्तयों की कुल संख्या का ककतना प्रनतशत है ?
(A) 18.45% (B) 17.92% (C) 19.56%
(D) 16.23% (E) None of these

23. Number of persons who visits city O in April is how much percent more
or less than number of persons who visits city N in January?
अप्रैल में शहर O का िौरा करने वाले व्यजक्तयों की संख्या, जनवरी में शहर N का िौरा करने वाले
व्यजक्तयों की संख्या से ककतने प्रनतशत अचधक या कम है ?
(A) 32.43% (B) 38.56% (C) 36.78%
(D) 34.89% (E) None of these

24. Number of persons who visits city M and N in May is respectively 20%
and 37.5% more than number of persons who visits city M and N in April,
then find the sum of number of persons who visits city M and N in May.
मई में शहर M और N का िौरा करने वाले व्यजक्तयों की संख्या अप्रैल में शहर M और N का िौरा
करने वाले व्यजक्तयों की संख्या से क्रमशः 20% और 37.5% अचधक है , तो मई में शहर M और N
का िौरा करने वाले व्यजक्तयों की संख्या का योग ज्ञात कीजजए।
(A) 564 (B) 312 (C) 678
(D) 429 (E) None of these

25. The ratio of male to female who visits city O and P in April is respectively
4 : 3 and 6 : 5, then find the number of males who visits city O and P in April.
अप्रैल में शहर O और P का िौरा करने वाले पुरुर्ों का मदहलाओं से अनुपात क्रमशः 4 : 3 और 6 :
5 है , तो अप्रैल में शहर O और P का िौरा करने वाले पुरुर्ों की संख्या ज्ञात कीजजए।
(A) 240 (B) 230 (C) 260
(D) 260 (E) None of these

(26-30) Directions : Study the following chart carefully and answer the
questions given below.
The pie chart given below shows the percentage distribution of number of
chairs manufactured in five different months. Total number of Chairs
manufacture = 2800.
ननिे श : ननम्नललखखत चाटष का ध्यानपूवक ष अध्ययन कीजजए और नीचे दिए गए प्रश्नों के उत्तर
िीजजए।
नीचे दिया गया पाई चाटष पांच अलग-अलग महीनों में ननलमषत कुलसषयों की संख्या का प्रनतशत
ववतरण िशाषता है । ननलमषत कुलसषयों की कुल संख्या = 2800।

https :
//www. https : https
https : //instagra
//youtube.c
facebo m.com/aas :
om/channe hisharoraso
l/UCYa4_Jr ok.com cial(?) //t.m
Orf8R5Kz2u
/aashis utm_mediu
e/stu

245
OtccXQ m=
haroras copy_link
ocial dified
26. Number of chairs manufactured in March is how much percent more or
less than number of chairs manufactured in February?
माचष में ननलमषत कुलसषयों की संख्या िरवरी में ननलमषत कुलसषयों की संख्या से ककतने प्रनतशत अचधक
या कम है ?
(A) 29.89% (B) 29.89% (C) 27.02%
(D) 26.31% (E) None of these

27. In January, 71.42% chairs sold and rest are in next month. Out of total
stock of chairs 75% are sold in February. Find the sum of number of chairs
sold in February.
जनवरी में , 71.42% कुलसषयाँ बबकीं और शेर् अगले महीने में । िरवरी में कुलसषयों के कुल स्टॉक में
से 75% की बबक्री हो जाती है । िरवरी में बेची गई कुलसषयों की संख्या का योग ज्ञात कीजजए।
(A) 537 (B) 997 (C) 745
(D) 682 (E) None of these

28. The ratio of number of wooden chairs to number of plastic chairs


manufactured in March and May is 5 : 3 and 4 : 3 respectively, then find the
difference between number of wooden chairs manufactured in March and
May together.
माचष और मई में ननलमषत लकडी की कुलसषयों की संख्या का प्लाजस्टक कुलसषयों की संख्या से अनुपात
क्रमशः 5 : 3 और 4 : 3 है , तो माचष और मई में एकसाथ ननलमषत लकडी की कुलसषयों की संख्या के
बीच का अंतर ज्ञात कीजजए।
(A) 172 (B) 164 (C) 188
(D) 196 (E) None of these

https :
//www. https : https
https : //instagra
//youtube.c
facebo m.com/aas :
om/channe hisharoraso
l/UCYa4_Jr ok.com cial(?) //t.m
Orf8R5Kz2u
/aashis utm_mediu
e/stu

246
OtccXQ m=
haroras copy_link
ocial dified
29. What is the ratio of number of chairs manufactured in March to sum of
number of number of chairs manufactured in April and May together?
माचष में ननलमषत कुलसषयों की संख्या का अप्रैल और मई में लमलाकर ननलमषत कुलसषयों की संख्या के
योग से अनुपात ककतना है ?
(A) 12 : !7 (B) 13 : 14 (C) 15 : 18
(D) 16 : 19 (E) None of these

30. Number of chairs manufactured in June and July is 25% and 10% more
than previous month, then find the sum of number of chairs manufactured in
June and July together.
जून और जुलाई में ननलमषत कुलसषयों की संख्या वपछले महीने की तुलना में 25% और 10% अचधक
है , तो जून और जुलाई में लमलाकर ननलमषत कुलसषयों की संख्या का योग ज्ञात कीजजए।
(A) 218 (B) 1422 (C) 1336
(D) 1176 (E) None of these

(31 - 35) The line graph given below shows the number of students who
preparing for NEET and number of students who preparing for JEE in five
different cities.
ननिे श : ननम्नललखखत चाटष का ध्यानपूवक
ष अध्ययन कीजजए और नीचे दिए गए प्रश्नों के उत्तर
िीजजए।
नीचे दिया गया लाइन ग्राि पांच अलग-अलग शहरों में NEET की तैयारी करने वाले छात्रों की
संख्या और JEE की तैयारी करने वाले छात्रों की संख्या को िशाषता है ।

31. The ratio of number of boys to number of girls who are preparing for
NEET in city B and City C is respectively 7 : 5 and 7 : 6, then find the sum of
number of boys who are preparing for NEET in city B to number of girls who
are preparing for NEET in city C.

https :
//www. https : https
https : //instagra
//youtube.c
facebo m.com/aas :
om/channe hisharoraso
l/UCYa4_Jr ok.com cial(?) //t.m
Orf8R5Kz2u
/aashis utm_mediu
e/stu

247
OtccXQ m=
haroras copy_link
ocial dified
शहर B और शहर C में NEET की तैयारी करने वाले लडकों की संख्या का लडककयों की संख्या से
क्रमशः 7 : 5 और 7 : 6 का अनुपात है , तो शहर B में NEET की तैयारी करने वाले लडकों की
संख्या का शहर C में NEET की तैयारी करने वाली लडककयों की संख्या का योग ज्ञात कीजजए।
(A) 640 (B) 780 (C) 850
(D) 890 (E) None of these

32. Number of students who are preparing for NEET and JEE in city F is
respectively 23% and 10% more than number of students who are preparing
for NEET and JEE in city E, then find the sum of number of students who are
preparing for NEET and JEE in city F.
शहर F में NEET और JEE की तैयारी करने वाले छात्रों की संख्या, शहर E में NEET और JEE
की तैयारी करने वाले छात्रों की संख्या से क्रमशः 23% और 10% अचधक है , तो शहर F में NEET
और JEE की तैयारी कर रहे छात्रों की संख्या का योग ज्ञात कीजजए।
(A) 2105 (B) 2009 (C) 2269
(D) 2378 (E) None of these

33. Find the average of number of students who are preparing for NEET in all
five cities together.
सभी पांच शहरों में लमलाकर NEET की तैयारी कर रहे छात्रों की औसत संख्या ज्ञात कीजजए।
(A) 792 (B) 884 (C) 632
(D) 545 (E) None of these

34. Number of students who are preparing for JEE in city A is what percent
of number of students who are preparing for JEE in all five cities together?
शहर A में JEE की तैयारी करने वाले छात्रों की संख्या सभी पांच शहरों में लमलाकर JEE की
तैयारी करने वाले छात्रों की संख्या का ककतना प्रनतशत है ?
(A) 24.86% (B) 21.56% (C) 23.45%
(D) 22.10% (E) None of these

35. Find the ratio of sum of number of students who are preparing for NEET
in city A and B to sum of number of students who are preparing for JEE in
city B and C.
शहर A और B में NEET की तैयारी करने वाले छात्रों की संख्या का शहर B और C में JEE की
तैयारी करने वाले छात्रों की संख्या के योग से अनुपात ज्ञात कीजजए।
(A) 82 : 19 (B) 61 : 74 (C) 56 : 83
(D) 71 : 63 (E) None of these

(36-40) Directions : Study the following chart carefully and answer the
questions given below.

https :
//www. https : https
https : //instagra
//youtube.c
facebo m.com/aas :
om/channe hisharoraso
l/UCYa4_Jr ok.com cial(?) //t.m
Orf8R5Kz2u
/aashis utm_mediu
e/stu

248
OtccXQ m=
haroras copy_link
ocial dified
The bar graph given below shows the number of wired earphones and
wireless earphones sold by Samsung on five different days.
ननिे श : ननम्नललखखत चाटष का ध्यानपूवक
ष अध्ययन कीजजए और नीचे दिए गए प्रश्नों के उत्तर
िीजजए।
नीचे दिया गया बार ग्राि Samsung द्वारा पांच अलग-अलग दिनों में बेचे गए वायडष ईयरिोन
और वायरलेस ईयरिोन की संख्या को िशाषता है ।

36. If number of wired earphones and number of wireless earphones sold on


Saturday is respectively 20% more and 20% less than number of wired
earphones and number of wireless earphones sold on Thursday, then find
the difference between number of wired earphones and number of wireless
earphones sold on Saturday.
यदि शननवार को बेचे गए वायडष ईयरिोन की संख्या और वायरलेस ईयरिोन की संख्या गरु
ु वार
को बेचे गए वायडष ईयरिोन की संख्या और वायरलेस ईयरिोन की संख्या से क्रमशः 20% अचधक
और 20% कम है , तो शननवार को बेचे गए वायडष ईयरिोन की संख्या और वायरलेस ईयरिोन की
संख्या के बीच अंतर ज्ञात कीजजए।
(A) 53 (B) 31 (C) 28
(D) 48 (E) None of these

37. Total number of earphones sold by Vivo on Tuesday and Wednesday is


respectively 28.56% and 22.22% more than total number of earphones sold
by Samsung on Tuesday and Wednesday respectively, then find the sum of
total number of earphones sold by Vivo on Tuesday and Wednesday.
मंगलवार और बुधवार को Vivo द्वारा बेचे गए ईयरफोन की कुल संख्या क्रमशः Samsung
द्वारा मंगलवार और बुधवार को बेचे गए ईयरफोन की कुल संख्या से क्रमशः 28.56% और
22.22% अचधक है , तो मंगलवार और बुधवार को Vivo द्वारा बेचे गए ईयरफोन की कुल संख्या
का योग ज्ञात कीजजए।
(A) 300 (B) 400 (C) 100
(D) 200 (E) None of these

https :
//www. https : https
https : //instagra
//youtube.c
facebo m.com/aas :
om/channe hisharoraso
l/UCYa4_Jr ok.com cial(?) //t.m
Orf8R5Kz2u
/aashis utm_mediu
e/stu

249
OtccXQ m=
haroras copy_link
ocial dified
38. Find the average of number of wired earphones sold on Monday,
Tuesday, Thursday and Friday.
सोमवार, मंगलवार, गुरुवार और शुक्रवार को बेचे गए वायडष ईयरफोन की औसत संख्या ज्ञात
कीजजए।
(A) 70 (B) 80 (C) 60
(D) 50 (E) None of these

39. Total number of earphones sold on Thursday is how much percent more
or less than total number of earphones sold on Monday?
गरु
ु वार को बेचे गए ईयरफोन की कुल संख्या, सोमवार को बेचे गए ईयरफोन की कुल संख्या से
ककतने प्रनतशत अचधक या कम है ?
(A) 32% (B) 34% (C) 36%
(D) 33% (E) None of these

40. Number of wireless earphones sold on Tuesday is what percent of total


number of wireless earphones sold on all five days?
मंगलवार को बेचे गए वायरलेस ईयरिोन की संख्या सभी पांच दिनों में बेचे गए वायरलेस
ईयरिोन की कुल संख्या का ककतना प्रनतशत है ?
(A) 14.75% (B) 12.89% (C) 16.45%
(D) 18.07% (E) None of these

(41 - 45) Directions : Study the following chart carefully and answer the
questions given below.
The bar graph given below shows the quantity of Wheat flour and quantity of
Rice flour produced in five different months. (Quantity in kg)
ननिे श : ननम्नललखखत चाटष का ध्यानपूवक ष अध्ययन कीजजए और नीचे दिए गए प्रश्नों के उत्तर
िीजजए।
नीचे दिया गया बार ग्राि पांच अलग-अलग महीनों में उत्पादित गेहूं के आटे की मात्रा और चावल
के आटे की मात्रा को िशाषता है । (ककग्रा में मात्रा)

https :
//www. https : https
https : //instagra
//youtube.c
facebo m.com/aas :
om/channe hisharoraso
l/UCYa4_Jr ok.com cial(?) //t.m
Orf8R5Kz2u
/aashis utm_mediu
e/stu

250
OtccXQ m=
haroras copy_link
ocial dified
41. Find the average of quantity of Rice flour produced in all five months.
सभी पांच महीनों में उत्पादित चावल के आटे की औसत मात्रा ज्ञात कीजजए।
(A) 234 (B) 254 (C) 242
(D) 222 (E) None of these

42. Quantity of Wheat flour produced in January is how much percent more
or less than total quantity of Rice and Wheat flour produced in January?
जनवरी में उत्पादित गेहूं के आटे की मात्रा जनवरी में उत्पादित चावल और गेहूं के आटे की कुल
मात्रा से ककतने प्रनतशत अचधक या कम है ?
(A) 37.49% (B) 51.28% (C) 42.12%
(D) 28.61% (E) None of these

43. Find the ratio between sum of quantity of Wheat flour produced in March
and April to quantity of Rice flour produced in March and April.
माचष और अप्रैल में उत्पादित गेहूं के आटे की मात्रा का माचष और अप्रैल में उत्पादित चावल के आटे
की मात्रा से अनुपात ज्ञात कीजजए।
(A) 27 : 23 (B) 21 : 22 (C) 28 : 29
(D) 26 : 25 (E) None of these

44. Quantity of Wheat flour and quantity of Rice flour produced in June is
respectively 25% and 20% more than quantity of Wheat flour and quantity of
Rice flour produced in February, then find the sum of quantity of Wheat flour
and quantity of Rice flour produced in June.
जून में उत्पादित गेहूं के आटे की मात्रा और चावल के आटे की मात्रा िरवरी में उत्पादित गेहूं के
आटे की मात्रा और चावल के आटे की मात्रा से क्रमशः 25% और 20% अचधक है , तो जन ू में
उत्पादित गेहूं के आटे की मात्रा और चावल के आटे की मात्रा का योग ज्ञात कीजजए।
(A) 589 (B) 412 (C) 356
(D) 648 (E) None of these

45. If the price of Wheat flour and Rice flour is Rs. 32/kg and Rs. 45/kg, then
find the total revenue generated in May from sell of total quantity of Wheat
flour and Rice flour produced in same month.
यदि गेहूं के आटे और चावल के आटे की कीमत 32 रुपये/ककग्रा और 45 रुपये/ककग्रा है , तो उसी
महीने में उत्पादित गेहूं के आटे और चावल के आटे की कुल मात्रा की बबक्री से मई में अजजषत कुल
राजस्व ज्ञात कीजजए।
(A) Rs. 23145 (B) Rs. 22473 (C) Rs. 21368
(D) Rs. 20981 (E) None of these

(46 - 50) Directions : Study the following chart carefully and answer the
questions given below.

https :
//www. https : https
https : //instagra
//youtube.c
facebo m.com/aas :
om/channe hisharoraso
l/UCYa4_Jr ok.com cial(?) //t.m
Orf8R5Kz2u
/aashis utm_mediu
e/stu

251
OtccXQ m=
haroras copy_link
ocial dified
The pie chart given below shows the percentage distribution of total number
of students in five different schools. The line graph given below shows the
ratio of number of boys to number of girls in five different schools.
ननिे श : ननम्नललखखत चाटष का ध्यानपूवक ष अध्ययन कीजजए और नीचे दिए गए प्रश्नों के उत्तर
िीजजए।
नीचे दिया गया पाई चाटष पांच अलग-अलग स्कूलों में छात्रों की कुल संख्या के प्रनतशत ववतरण को
िशाषता है । नीचे दिया गया लाइन ग्राि पांच अलग-अलग स्कूलों में लडकों की संख्या से लडककयों
की संख्या के अनुपात को िशाषता है ।

46. Find the average of total number of students of NPS, JPS and KPS.
NPS, JPS और KPS के छात्रों की कुल संख्या का औसत ज्ञात कीजजए।
(A) 578 (B) 316 (C) 246
(D) 488 (E) None of these

47. Number of boys in NPS is what percent of total number of students of


KPS?
NPS में लडकों की संख्या, KPS के कुल छात्रों की संख्या का ककतना प्रनतशत है ?
(A) 35.56% (B) 37.5% (C) 36.89%
(D) 34.42% (E) None of these

48. Number of boys of RPS is how much percent more or less than number
of boys of KPS?
RPS के लडकों की संख्या, KPS के लडकों की संख्या से ककतने प्रनतशत अचधक या कम है ?
(A) 24.30% (B) 26.66% (C) 25.50%
(D) 23.40% (E) None of these

49. Number of boys and girls who participate in games to who does not
participate in games are in the ratio of 5 : 4 and 5 : 7 respectively, then find
the sum of number of boys who participate in games and number of girls
who does not participate in games in HPS.

https :
//www. https : https
https : //instagra
//youtube.c
facebo m.com/aas :
om/channe hisharoraso
l/UCYa4_Jr ok.com cial(?) //t.m
Orf8R5Kz2u
/aashis utm_mediu
e/stu

252
OtccXQ m=
haroras copy_link
ocial dified
खेलों में भाग लेने वाले लडकों और लडककयों की संख्या का खेलों में भाग नहीं लेने वालों की संख्या
का अनुपात क्रमशः 5 : 4 और 5 : 7 है , तो HPS में खेलों में भाग लेने वाले लडकों की संख्या और
खेलों में भाग नहीं लेने वाली लडककयों की संख्या का योग ज्ञात कीजजए।
(A) 285 (B) 245 (C) 264
(D) 236 (E) None of these

50. Number of boys and number of girls of LBS is respectively 11.11% and
8.33% more than number of boys and number of girls of KPS, then find the
total number of students of LBS.
LBS के लडकों और लडककयों की संख्या क्रमशः KPS के लडकों और लडककयों की संख्या से
11.11% और 8.33% अचधक है , तो LBS के छात्रों की कुल संख्या ज्ञात कीजजए।
(A) 618 (B) 522 (C) 536
(D) 634 (E) None of these

(51 - 55) The line graph given below shows the Percentage of total number of
cars and percentage of cars sold in the four given showrooms A, B, C and D.
Total number of cars in all the showrooms together is 3600.
ननिे श : ननम्नललखखत चाटष का ध्यानपूवकष अध्ययन कीजजए और नीचे दिए गए प्रश्नों के उत्तर
िीजजए।
नीचे दिया गया लाइन ग्राि चार दिए गए शोरूम A, B, C और D में कारों की कुल संख्या का
प्रनतशत और बेची गई कारों का प्रनतशत िशाषता है ।
सभी शोरूम में कारों की कुल संख्या 3600 है ।

51. What is the ratio of the total number of unsold cars in A and D together
to the total number of sold cars in C?
A और D में एकसाथ बेची गई कारों की कुल संख्या का C में बेची गई कारों की कुल संख्या से
अनुपात क्या है ?
(A) 5 : 2 (B) 5 : 3 (C) 5 : 1
(D) 4 : 1 (E) None of these

https :
//www. https : https
https : //instagra
//youtube.c
facebo m.com/aas :
om/channe hisharoraso
l/UCYa4_Jr ok.com cial(?) //t.m
Orf8R5Kz2u
/aashis utm_mediu
e/stu

253
OtccXQ m=
haroras copy_link
ocial dified
52. Ratio of the number of cars sold in B and E is 4 : 9. If 27% of total
number of cars sold in E, then what is the difference between the total
number of cars in E and C?
B और E में बेची गई कारों की संख्या का अनुपात 4 : 9 है । यदि E में बेची गई कारों की कुल
संख्या का 27% है , तो E और C में कारों की कुल संख्या के बीच ककतना अंतर है ?
(A) 310 (B) 320 (C) 330
(D) 380 (E) None of these

53. Find the total number of unsold cars in B, C and D.


B, C और D में बेची गई कारों की कुल संख्या ज्ञात कीजजए।
(A) 1200 (B) 1500 (C) 1800
(D) 1400 (E) None of these

54. The number of cars sold in B is what percent of the number of cars sold
in A and D together?
B में बेची गई कारों की संख्या, A और D में लमलाकर बेची गई कारों की संख्या का ककतना
प्रनतशत है ?
(A) 70% (B) 60% (C) 80%
(D) 50% (E) None of these

55. Ratio of the number cars in F and the number of unsold cars in D is 2 : 1.
If the number of cars sold in F is 30%, then find the difference between the
number of unsold cars in A and F.
F में कारों की संख्या और D में न बबकने वाली कारों की संख्या का अनुपात 2 : 1 है । यदि F में
बेची गई कारों की संख्या 30% है , तो A और F में बेची गई कारों की संख्या के बीच का अंतर ज्ञात
कीजजए।
(A) 36 (B) 38 (C) 40
(D) 42 (E) 45

(56 - 60) Directions : Study the following chart carefully and answer the
questions given below.
The Table shows the cost price and profit percent of two shops P and Q for 4
articles - A, B, C and D.
ननिे श : ननम्नललखखत चाटष का ध्यानपूवक
ष अध्ययन कीजजए और नीचे दिए गए प्रश्नों के उत्तर
िीजजए।
ताललका 4 वस्तुओं - A, B, C और D के ललए िो िक
ु ानों P और Q का लागत मूल्य और लाभ
प्रनतशत िशाषती है ।

https :
//www. https : https
https : //instagra
//youtube.c
facebo m.com/aas :
om/channe hisharoraso
l/UCYa4_Jr ok.com cial(?) //t.m
Orf8R5Kz2u
/aashis utm_mediu
e/stu

254
OtccXQ m=
haroras copy_link
ocial dified
Cost Price Profit % in shop
Article Profit % in shop P
(In Rupees) Q

A 800 12.5% 10%

B 1000 5% 7.5%

C 1250 8% 10%

D 750 15% 12%

56. If the marked prices of article A at shop P and article B at shop Q are Rs.
960 and Rs. 1125 respectively, then find total discount offered (in rupees) on
article A at shop P and article B at shop Q.
यदि िक ु ान P पर वस्तु A और िक ु ान Q पर वस्तु B का अंककत मूल्य क्रमशः 960 रुपये और
1125 रुपये है , तो िक
ु ान P पर वस्त ु A और िक
ु ान Q पर वस्तु B पर िी गई कुल छूट (रुपये में )
ज्ञात कीजजए।
(A) Rs. 110 (B) Rs. 150 (C) Rs. 160
(D) Rs. 180 (E) None of these

57. If successive discounts offered on marked price of article D at shop Q


are 20% and 16%, then find the marked price of article D at shop Q.
यदि िक ु ान Q पर वस्तु D के अंककत मूल्य पर क्रलमक छूट 20% और 16% है , तो िक
ु ान Q पर
वस्तु D का अंककत मूल्य ज्ञात कीजजए।
(A) Rs. 1200 (B) Rs. 1000 (C) Rs. 1500
(D) Rs. 1250 (E) None of these

58. Profit earned on another E at shop Q is Rs. 60 more than the profit
earned at article C at shop P. If the cost price of article E is 20% less than the
cost price of article B, then find the profit percent of article E at shop Q.
िक ु ान Q पर अन्य E पर अजजषत लाभ, िक ु ान P पर वस्तु C पर अजजषत लाभ से 60 रुपये अचधक
है । यदि वस्तु E का लागत मूल्य वस्तु B के लागत मूल्य से 20% कम है , तो िक
ु ान Q पर वस्तु
E का लाभ प्रनतशत ज्ञात कीजजए।
(A) 10% (B) 15% (C) 20%
(D) 25% (E) None of these

59. What is the difference between the average selling price of article A and
article B at shop Q and the average selling price of article C and article D at
shop P?

https :
//www. https : https
https : //instagra
//youtube.c
facebo m.com/aas :
om/channe hisharoraso
l/UCYa4_Jr ok.com cial(?) //t.m
Orf8R5Kz2u
/aashis utm_mediu
e/stu

255
OtccXQ m=
haroras copy_link
ocial dified
िकु ान Q पर वस्तु A और वस्तु B के औसत ववक्रय मूल्य और िक
ु ान P पर वस्तु C और वस्तु D
के औसत ववक्रय मूल्य के बीच ककतना अंतर है ?
(A) Rs. 132.5 (B) Rs. 128.75 (C) Rs. 131.5
(D) Rs. 141.5 (E) None of these

60. How much total profit is earned by shop P by the sale of articles A, B and
C?
वस्तु A, B और C की बबक्री से िक
ु ान P द्वारा कुल ककतना लाभ अजजषत ककया जाता है ?
(A) Rs. 754 (B) Rs. 250 (C) Rs. 400
(D) Rs. 500 (E) None of these

(61-65) Directions : Study the following chart carefully and answer the
questions given below.
The line graph given below shows the initial and final word count of five
different documents. It also shows the number of deleted words of these five
documents.
Note : Number of deleted words is equal to difference between the initial and
final word count of that document.
ननिे श : ननम्नललखखत चाटष का ध्यानपूवक ष अध्ययन कीजजए और नीचे दिए गए प्रश्नों के उत्तर
िीजजए।
नीचे दिया गया लाइन ग्राि पांच अलग-अलग िस्तावेजों की प्रारं लभक और अंनतम शब्ि गणना
दिखाता है । यह इन पांच िस्तावेजों के हटाए गए शब्िों की संख्या भी दिखाता है ।
नोट : हटाए गए शब्िों की संख्या उस िस्तावेज की प्रारं लभक और अंनतम शब्ि गणना के बीच के
अंतर के बराबर है ।

https :
//www. https : https
https : //instagra
//youtube.c
facebo m.com/aas :
om/channe hisharoraso
l/UCYa4_Jr ok.com cial(?) //t.m
Orf8R5Kz2u
/aashis utm_mediu
e/stu

256
OtccXQ m=
haroras copy_link
ocial dified
61. Find the average number of deleted words in document A and document
E.
िस्तावेज A और िस्तावेज E में हटाए गए शब्िों की औसत संख्या ज्ञात कीजजए।
(A) 200 (B) 210 (C) 220
(D) 240 (E) None of these

62. Initial word count of document D is what percent more than the number
of deleted words of document C?
िस्तावेज D की आरं लभक शब्ि गणना, िस्तावेज C के हटाए गए शब्िों की संख्या से ककतने
प्रनतशत अचधक है ?
(A) 440% (B) 480% (C) 460%
(D) 420% (E) None of these

63. If the sum of the final word count of documents D and F is 1428. Find the
final word count of document F.
यदि िस्तावेज D और F की अंनतम शब्ि गणना का योग 1428 है । िस्तावेज F की अंनतम शब्ि
गणना ज्ञात कीजजए।
(A) 686% (B) 668% (C) 862%
(D) 866% (E) None of these

64. Find the ratio of the initial word count of document B to the final word
count of document C.
िस्तावेज B की प्रारं लभक शब्ि गणना का िस्तावेज C की अंनतम शब्ि गणना से अनुपात ज्ञात
कीजजए।
(A) 5 : 6 (B) 4 : 5 (C) 6 : 5
(D) 5 : 7 (E) None of these

65. If 45% and 25% deleted words of documents A and E respectively were
meaningless, find the total number of meaningless words that were deleted
from documents A and E together.
यदि िस्तावेज A और E के क्रमशः 45% और 25% हटाए गए शब्ि अथषहीन थे, तो िस्तावेज A
और E से एक साथ हटाए गए अथषहीन शब्िों की कुल संख्या ज्ञात कीजजए।
(A) 156 (B) 164 (C) 126
(D) 166 (E) None of these

(66 - 70) Directions : Study the following chart carefully and answer the
questions given below.
The table given below shows the total number of students appeared in an
exam to the number of candidates who qualified the exam and also the
number of candidates who did not qualify the exam in different years.

https :
//www. https : https
https : //instagra
//youtube.c
facebo m.com/aas :
om/channe hisharoraso
l/UCYa4_Jr ok.com cial(?) //t.m
Orf8R5Kz2u
/aashis utm_mediu
e/stu

257
OtccXQ m=
haroras copy_link
ocial dified
ननिे श : ननम्नललखखत चाटष का ध्यानपूवकष अध्ययन कीजजए और नीचे दिए गए प्रश्नों के उत्तर
िीजजए।
नीचे िी गई ताललका परीक्षा में उपजस्थत होने वाले छात्रों की कुल संख्या, परीक्षा उत्तीणष करने वाले
उम्मीिवारों की संख्या और ववलभन्न वर्ों में परीक्षा उत्तीणष नहीं करने वाले उम्मीिवारों की संख्या
को िशाषती है ।

Students who did


Year Students Appeared : Students Qualified
not qualify

2010 18 : 13 20000

2011 9:8 4500

2012 12 : 7 40000

2013 5:3 25000

2014 13 : 10 10800

66. Approximately what percent of number of students qualified in 2011 and


2013 together out of total number of students appeared in the same years?
वर्ष 2011 और 2013 में एकसाथ उत्तीणष होने वाले छात्रों की संख्या समान वर्ों में उपजस्थत छात्रों
की कुल संख्या का लगभग ककतना प्रनतशत है ?
(A) 69% (B) 86% (C) 86%
(D) 91% (E) None of these

67. What is the average number of qualified students in 2010, 2012 and
2014?
2010, 2012 और 2014 में योग्य छात्रों की औसत संख्या ककतनी है ?
(A) 45000 (B) 48000 (C) 25000
(D) 50000 (E) None of these

68. What is the respective ratio of appeared candidates in 2010 to qualified


candidates in 2014?
2010 में उपजस्थत हुए उम्मीिवारों का 2014 में योग्य उम्मीिवारों से संबंचधत अनुपात ककतना
है ?
(A) 2 : 1 (B) 3 : 2 (C) 7 : 8
(D) 9 : 4 (E) None of these

https :
//www. https : https
https : //instagra
//youtube.c
facebo m.com/aas :
om/channe hisharoraso
l/UCYa4_Jr ok.com cial(?) //t.m
Orf8R5Kz2u
/aashis utm_mediu
e/stu

258
OtccXQ m=
haroras copy_link
ocial dified
69. In 2013, If each student who qualifies gets Rs. 120 as cash prize then,
what is the total amount given to the students who qualified? (in lakhs)
2013 में , यदि उत्तीणष होने वाले प्रत्येक छात्र को नकि पुरस्कार के रूप में 120 रुपये लमलते हैं, तो
अहषता प्राप्त करने वाले छात्रों को िी गई कुल रालश ककतनी है ? (लाखों में )
(A) 55 lakh (B) 48 lakh (C) 45 lakh
(D) 36 lakh (E) None of these

70. How many number of more or less students appeared in 2012 as


compared to that in previous year?
वपछले वर्ष की तल
ु ना में वर्ष 2012 में ककतने अचधक या कम छात्र उपजस्थत हुए?
(A) 21800 (B) 55500 (C) 23600
(D) 20900 (E) None of these

(71 - 75) Directions : Study the following chart carefully and answer the
questions given below.
Below line graph shows the number of jeans of two brands (Lee and Rapido)
sold by a shopkeeper in five years (2011, 2012, 2013, 2014 and 2015) :
Note : The ratio of number of Rapido jeans sold to the number of Pepe jeans
sold in each year is 4 : 5.
ननिे श : ननम्नललखखत चाटष का ध्यानपव ू क
ष अध्ययन कीजजए और नीचे दिए गए प्रश्नों के उत्तर
िीजजए।
नीचे दिया गया रे खा ग्राि पांच वर्ों (2011, 2012, 2013, 2014 और 2015) में एक िक ु ानिार
द्वारा बेची गई िो ब्ांडों (Lee और Rapido) की जींस की संख्या को िशाषता है :
नोट : प्रत्येक वर्ष बेची गई Rapido जींस की संख्या का Pepe जींस की संख्या से अनुपात 4 : 5
है ।

https :
//www. https : https
https : //instagra
//youtube.c
facebo m.com/aas :
om/channe hisharoraso
l/UCYa4_Jr ok.com cial(?) //t.m
Orf8R5Kz2u
/aashis utm_mediu
e/stu

259
OtccXQ m=
haroras copy_link
ocial dified
71. If the number of jeans of Pepe brand sold in 2016 is 11/16 times of the
number of jeans of Pepe brand sold in the previous year, then the number of
jeans of Pepe brand sold in 2016 is how many more or less than the number
of jeans of Lee brand sold in 2014?
यदि 2016 में बेची गई Pepe ब्ांड की जींस की संख्या वपछले वर्ष में बेची गई Pepe ब्ांड की जींस
की संख्या का 11/16 गुना है , तो 2016 में बेची गई Pepe ब्ांड की जींस की संख्या 2014 में Lee
ब्ांड की बेची गई जींस की संख्या से ककतनी अचधक या कम है ?
(A) 460 (B) 650 (C) 350
(D) 840 (E) None of these

72. The number of jeans of Lee brand sold in 2016 is 50% of the number of
jeans of Rapido brand sold in 2012, then what is the number of jeans of Lee
brand sold in 2016?
2016 में बेची गई Lee ब्ांड की जींस की संख्या 2012 में बेची गई Rapido ब्ांड की जींस की
संख्या का 50% है , तो 2016 में Lee ब्ांड की बेची गई जींस की संख्या ककतनी है ?
(A) 2400 (B) 2600 (C) 2100
(D) 2200 (E) None of these

73. What is the ratio of number of jeans of Lee brand sold in 2011 to the
number of jeans of Pepe brand sold in 2014?
2011 में बेची गई Lee ब्ांड की जींस की संख्या का 2014 में बेची गई Pepe ब्ांड की जींस की
संख्या से अनुपात ककतना है ?
(A) 11 : 15 (B) 24 : 25 (C) 15 : 14
(D) 16 : 15 (E) None of these

74. What is the average number of jeans of Pepe brand sold in 2011 and
2013, together?
2011 और 2013 में एक साथ बेची गई Pepe ब्ांड की जींस की औसत संख्या ककतनी है ?
(A) 4750 (B) 5250 (C) 4750
(D) 6450 (E) None of these

75. If the number of Lee jeans, Rapido jeans and Pepe jeans sold in 2017 is
20%, 50% and 40% more than that of sold in 2014, respectively, then what is
the total number of jeans sol by the shopkeeper in 2017?
यदि 2017 में बेची गई Lee जींस, Rapido जींस और Pepe जींस की संख्या क्रमशः 2014 में
बेची गई जींस की संख्या से 20%, 50% और 40% अचधक है , तो 2017 में िक
ु ानिार द्वारा बेची
गई जींस की कुल संख्या ककतनी है ?
(A) 16250 (B) 15750 (C) 16650
(D) 15880 (E) None of these

https :
//www. https : https
https : //instagra
//youtube.c
facebo m.com/aas :
om/channe hisharoraso
l/UCYa4_Jr ok.com cial(?) //t.m
Orf8R5Kz2u
/aashis utm_mediu
e/stu

260
OtccXQ m=
haroras copy_link
ocial dified
(76-80) Directions : Study the following chart carefully and answer the
questions given below.
The table given below shows the number of kits manufactured by four
companies and percentage of kits distributed among different cricket
academies among the students of these academies.
Note : Total number of kits manufactured by any academy = Number of kits
distributed + Number of kits rejected.
Each student gets equal number of kits of that academy.
ननिे श : ननम्नललखखत चाटष का ध्यानपूवक ष अध्ययन कीजजए और नीचे दिए गए प्रश्नों के उत्तर
िीजजए।
नीचे िी गई ताललका चार कंपननयों द्वारा ननलमषत ककटों की संख्या और ववलभन्न कक्रकेट
अकािलमयों में अकािलमयों के छात्रों के बीच ववतररत ककटों का प्रनतशत िशाषती है ।
नोट : ककसी भी अकािमी द्वारा ननलमषत ककटों की कुल संख्या = ववतररत ककटों की संख्या +
अस्वीकरत ककटों की संख्या।

Kits % of kits Number of


Academy
manufactured distributed students

A 6600 90% 22

B 4800 75% 18

C 2500 80% 8

D 4200 85% 17

E 5400 70% 21

76. What is the average number of kits manufactured by A, B and E?


A, B और E द्वारा ननलमषत ककटों की औसत संख्या ककतनी है ?
(A) 6900 (B) 7600 (C) 5600
(D) 9100 (E) None of these

77. The number of kits distributed by cricket academy D is what percent


more or less than the number of kits distributed by academy E?
कक्रकेट अकािमी D द्वारा ववतररत ककट की संख्या अकािमी E द्वारा ववतररत ककट की संख्या से
ककतने प्रनतशत अचधक या कम है ?

https :
//www. https : https
https : //instagra
//youtube.c
facebo m.com/aas :
om/channe hisharoraso
l/UCYa4_Jr ok.com cial(?) //t.m
Orf8R5Kz2u
/aashis utm_mediu
e/stu

261
OtccXQ m=
haroras copy_link
ocial dified
(A) 11/9% (B) 50/9% (C) 16/9%
(D) 22.63% (E) None of these

78. What is the respective ratio between the number of kits received by each
student of academy A and number of kits received by each student of
academy D?
अकािमी A के प्रत्येक छात्र द्वारा प्राप्त ककटों की संख्या और अकािमी D के प्रत्येक छात्र द्वारा
प्राप्त ककटों की संख्या के बीच संबंचधत अनुपात क्या है ?
(A) 9 : 7 (B) 3 : 2 (C) 7 : 8
(D) 9 : 4 (E) None of these

79. Find the total number of kits rejected by all the five academies together.
सभी पाँच अकािलमयों द्वारा एक साथ अस्वीकार की गई ककटों की कुल संख्या ज्ञात कीजजए।
(A) 5830 (B) 4510 (C) 4610
(D) 3610 (E) None of these

80. If academy D has 4 more students and redistribute the number of kits in
such a way that each of its student get equal number of kits, then find the
difference between the number of kits received by each student of academy
D and number of kits received by each student of academy C.
यदि अकािमी D में 4 और छात्र हैं और ककट की संख्या को इस तरह पुनववषतररत करते हैं कक उसके
प्रत्येक छात्र को समान संख्या में ककट लमलें , तो अकािमी D के प्रत्येक छात्र द्वारा प्राप्त ककटों की
संख्या और अकािमी C के प्रत्येक छात्र द्वारा प्राप्त ककटों की संख्या के बीच अंतर ज्ञात कीजजए।
(A) 28 (B) 80 (C) 90
(D) 80 (E) None of these

(81- 85) Directions : Study the following chart carefully and answer the
questions given below.
The pie chart given below shows the percentage distribution of the matches
played by different teams participating in IPL. Total number of matches =
6000
ननिे श : ननम्नललखखत चाटष का ध्यानपूवक ष अध्ययन कीजजए और नीचे दिए गए प्रश्नों के उत्तर
िीजजए।
नीचे दिया गया पाई चाटष आईपीएल में भाग लेने वाली ववलभन्न टीमों द्वारा खेले गए मैचों के
प्रनतशत ववतरण को िशाषता है । मैचों की कुल संख्या = 6000

https :
//www. https : https
https : //instagra
//youtube.c
facebo m.com/aas :
om/channe hisharoraso
l/UCYa4_Jr ok.com cial(?) //t.m
Orf8R5Kz2u
/aashis utm_mediu
e/stu

262
OtccXQ m=
haroras copy_link
ocial dified
81. If 20% of the matches played by KKR and 40% of matches played by SRH
are declared as draw, then what is number of matches played by both teams
which are not declared as draw?
यदि KKR द्वारा खेले गए 20% मैच और SRH द्वारा खेले गए 40% मैच ड्रॉ घोवर्त ककए जाते
हैं, तो िोनों टीमों द्वारा खेले गए मैचों की संख्या ककतनी है , जजन्हें ड्रॉ घोवर्त नहीं ककया गया है ?
(A) 1800 (B) 1250 (C) 1380
(D) 1340 (E) None of these

82. If the ratio of win and lost matches played by RR is 2 : 3. What is the
difference between the won and lost matches that are played by RR?
यदि RR द्वारा खेले गए जीत और हारे हुए मैचों का अनुपात 2 : 3 है । RR द्वारा खेले गए जीते
और हारे हुए मैचों में ककतना अंतर है ?
(A) 240 (B) 530 (C) 430
(D) 340 (E) None of these

83. What is the difference between the sum of matches played by MI and
SRH and the sum of matches played by DC and RR?
MI और SRH द्वारा खेले गए मैचों के योग और DC और RR द्वारा खेले गए मैचों के योग के
बीच का अंतर ककतना है ?
(A) 1100 (B) 1200 (C) 1000
(D) 600 (E) None of these

84. If 75% of matches played by DC are on home ground out of which 80% of
them are won by DC, then find the number of matches lost by DC on home
ground.

https :
//www. https : https
https : //instagra
//youtube.c
facebo m.com/aas :
om/channe hisharoraso
l/UCYa4_Jr ok.com cial(?) //t.m
Orf8R5Kz2u
/aashis utm_mediu
e/stu

263
OtccXQ m=
haroras copy_link
ocial dified
यदि DC द्वारा खेले गए 75% मैच घरे लू मैिान पर होते हैं, जजनमें से 80% DC द्वारा जीते जाते
हैं, तो घरे लू मैिान पर DC द्वारा हारे गए मैचों की संख्या ज्ञात कीजजए।
(A) 104 (B) 102 (C) 135
(D) 180 (E) None of these

85. What is the average number of matches played DC, RR and MI?
DC, RR और MI खेले गए मैचों की औसत संख्या ककतनी है ?
(A) 1580 (B) 1720 (C) 640
(D) 1300 (E) None of these

(86 - 90) Directions : Study the following chart carefully and answer the
questions given below.
Following table shows the number of participants in 6 tournaments of
Baseball, Football and Cricket. (Some of the data is missing)
ननिे श : ननम्नललखखत चाटष का ध्यानपव
ू क
ष अध्ययन कीजजए और नीचे दिए गए प्रश्नों के उत्तर
िीजजए।
ननम्नललखखत ताललका बेसबॉल, िुटबॉल और कक्रकेट के 6 टूनाषमेंटों में प्रनतभाचगयों की संख्या
िशाषती है । (कुछ डेटा गायब है )

Tournament Cricket Football Baseball

1 15 18 ______

2 25 _____ 30

3 _______ 30 40

4 30 _____ 16

5 24 18 _____

6 30 _______ 30

86. If sum of number of participants in 1st tournament of all 3 games was 53


and sum of number of participants in 5th tournament of 3 games was 70,
then what is the average number of participants in first 5 tournaments of
Baseball?
यदि सभी 3 खेलों के पहले टूनाषमेंट में प्रनतभाचगयों की संख्या का योग 53 था और 3 खेलों के 5वें
टूनाषमेंट में प्रनतभाचगयों की संख्या का योग 70 था, तो बेसबॉल के पहले 5 टूनाषमेंट में प्रनतभाचगयों
की औसत संख्या क्या है ?

https :
//www. https : https
https : //instagra
//youtube.c
facebo m.com/aas :
om/channe hisharoraso
l/UCYa4_Jr ok.com cial(?) //t.m
Orf8R5Kz2u
/aashis utm_mediu
e/stu

264
OtccXQ m=
haroras copy_link
ocial dified
(A) 11.76% (B) 10.48% (C) 26.8%
(D) 36.8% (E) None of these

87. If sum of number of participants in all 6 tournaments of Football is 145


and number of participants in 2nd and 4th Football tournaments were 75%
and 20% more than the number of participants in 6th Football tournament
respectively, then find the sum of number of participants in all games of 2nd
tournament.
यदि िुटबॉल के सभी 6 टूनाषमेंट में भाग लेने वालों की संख्या का योग 145 है और िस ू रे और चौथे
िुटबॉल टूनाषमेंट में भाग लेने वालों की संख्या क्रमशः 75% और 20% छिे िुटबॉल टूनाषमेंट में
भाग लेने वालों की संख्या से अचधक है , तो िस ू रे टूनाषमेंट के सभी खेलों में प्रनतभाचगयों की संख्या
का योग ज्ञात कीजजए।
(A) 80 (B) 90 (C) 60
(D) 50 (E) None of these

88. If sum of number of participants in all three games of 3rd tournament


was 90, then what is the average number of participants in all 6 tournaments
of Cricket?
यदि तीसरे टूनाषमेंट के तीनों खेलों में प्रनतभाचगयों की संख्या का योग 90 था, तो कक्रकेट के सभी 6
टूनाषमेंटों में प्रनतभाचगयों की औसत संख्या क्या है ?
(A) 24 (B) 25 (C) 63
(D) 19 (E) None of these

89. Sum of the number of participants in 3rd and 5th tournament of Football
is what percentage of sum of number of participants in 4th and 6th
tournament of Cricket?
िुटबॉल के तीसरे और पांचवें टूनाषमेंट में भाग लेने वालों की संख्या का योग, कक्रकेट के चौथे और
छिे टूनाषमेंट में भाग लेने वालों की संख्या का ककतना प्रनतशत है ?
(A) 60% (B) 50% (C) 80%
(D) 40% (E) None of these

90. What is the ratio of sum of number of participants in Baseball in only 4


tournaments given in the table to the sum of number of participants in
Cricket in only given 5 tournaments of given table?
ताललका में दिए गए केवल 4 टूनाषमेंट में बेसबॉल में भाग लेने वालों की संख्या के योग का िी गई
ताललका के केवल 5 टूनाषमेंट में कक्रकेट में भाग लेने वालों की संख्या के योग से अनुपात क्या है ?
(A) 17 : 54 (B) 16 : 23 (C) 29 : 31
(D) 19 : 14 (E) None of these

https :
//www. https : https
https : //instagra
//youtube.c
facebo m.com/aas :
om/channe hisharoraso
l/UCYa4_Jr ok.com cial(?) //t.m
Orf8R5Kz2u
/aashis utm_mediu
e/stu

265
OtccXQ m=
haroras copy_link
ocial dified
(91-95) Directions : Study the following chart carefully and answer the
questions given below.
Below pie chart shows the percentage distribution of people who learn to
play six different instruments in an institute.
Total number of people who learn to play six different instruments = 24000
And below table shows the percentage of male who learn to play these six
instruments in the institute.
ननिे श : ननम्नललखखत चाटष का ध्यानपूवकष अध्ययन कीजजए और नीचे दिए गए प्रश्नों के उत्तर
िीजजए।
नीचे पाई चाटष उन लोगों के प्रनतशत ववतरण को िशाषता है जो एक संस्थान में छह अलग-अलग
वाद्ययंत्र बजाना सीखते हैं।
छह अलग-अलग वाद्य यंत्र बजाना सीखने वालों की कुल संख्या = 24000
और नीचे िी गई ताललका संस्थान में इन छह उपकरणों को बजाना सीखने वाले पुरुर्ों का प्रनतशत
िशाषती है ।

Instrument % of Male Students

Guitar 60%

Drum 55%

Flute 70%

Piano 40%

Sitar 45%

Mandolin 65%

https :
//www. https : https
https : //instagra
//youtube.c
facebo m.com/aas :
om/channe hisharoraso
l/UCYa4_Jr ok.com cial(?) //t.m
Orf8R5Kz2u
/aashis utm_mediu
e/stu

266
OtccXQ m=
haroras copy_link
ocial dified
91. What is the average number of females who learn to play Flute, Piano
and Sitar together?
बांसुरी, वपयानो और लसतार बजाना सीखने वाली मदहलाओं की औसत संख्या ककतनी है ?
(A) 1890 (B) 1530 (C) 1880
(D) 1780 (E) None of these

92. The number of males who learn to play Guitar and Flute together is
approximately what percentage of the number of females who learn to play
Drum and Mandolin together?
चगटार और बांसरु ी एक साथ बजाना सीखने वाले परु
ु र्ों की संख्या, ड्रम और मैंडोललन एक साथ
बजाना सीखने वाली मदहलाओं की संख्या का लगभग ककतना प्रनतशत है ?
(A) 118% (B) 187% (C) 112%
(D) 54% (E) None of these

93. What is the ratio of the number of females who learn to play Guitar and
Piano to the number of males who learn to play Drum and Sitar together?
चगटार और वपयानो बजाना सीखने वाली मदहलाओं की संख्या का ड्रम और लसतार बजाना सीखने
वाले पुरुर्ों की संख्या से अनुपात ककतना है ?
(A) 10 : 11 (B) 24 : 17 (C) 17 : 24
(D) 17 : 21 (E) None of these

94. What is the difference between the number of males who learn to play
Mandolin and Piano together and the number of females who learn to play
Drum and Sitar together?
मैंडोललन और वपयानो एक साथ बजाना सीखने वाले परुु र्ों की संख्या और एक साथ ड्रम और
लसतार बजाना सीखने वाली मदहलाओं की संख्या के बीच ककतना अंतर है ?
(A) 3021 (B) 3052 (C) 3072
(D) 3931 (E) None of these

95. The number of males who learn to play Flute and Mandolin together is
approximately what percentage more or less than the number of females
who learn to play Sitar and Guitar together?
बांसुरी और मैंडोललन एक साथ बजाना सीखने वाले पुरुर्ों की संख्या, लसतार और चगटार एक साथ
बजाना सीखने वाली मदहलाओं की संख्या से लगभग ककतने प्रनतशत अचधक या कम है ?
(A) 13% (B) 133% (C) 64%
(D) 166% (E) None of these

(96 - 100) Directions : Study the following chart carefully and answer the
questions given below.
Following table shows the marks scored by 5 students un four subjects.

https :
//www. https : https
https : //instagra
//youtube.c
facebo m.com/aas :
om/channe hisharoraso
l/UCYa4_Jr ok.com cial(?) //t.m
Orf8R5Kz2u
/aashis utm_mediu
e/stu

267
OtccXQ m=
haroras copy_link
ocial dified
(Some of the data are missing in the table). Maximum marks in Math and
Science are 150 each whereas maximum marks in English and Hindi are 100
each.
ननिे श : ननम्नललखखत चाटष का ध्यानपूवक ष अध्ययन कीजजए और नीचे दिए गए प्रश्नों के उत्तर
िीजजए।
ननम्नललखखत ताललका 5 छात्रों द्वारा चार ववर्यों में प्राप्त अंकों को िशाषती है । (ताललका में कुछ
डेटा गायब हैं) । गखणत और ववज्ञान में अचधकतम अंक 150 प्रत्येक हैं जबकक अंग्रेजी और दहंिी में
अचधकतम अंक 100 प्रत्येक हैं।

Student Hindi English Math Science

Aditi 75 _____ 135 ____

Bhanu ______ 70 140 145

Chitra 80 64 _______ ____

Diksha ______ 80 135 115

Eshan 40 45 _____ 60

96. If percentage of marks scored by Bhanu in combing all four subjects


together is 83%, then how many marks Bhanu scored in Hindi?
यदि भानू द्वारा सभी चार ववर्यों को लमलाकर प्राप्त ककए गए अंकों का प्रनतशत 83% है , तो भानु
ने दहंिी में ककतने अंक प्राप्त ककए?
(A) 65% (B) 50% (C) 60%
(D) 40% (E) None of these

97. If out of total marks Aditi scored 80% marks in Science which is 50%
more than the marks scored by her in English, then calculate the marks
scored by her in English.
यदि कुल अंकों में से अदिनत ने ववज्ञान में 80% अंक प्राप्त ककए हैं जो उसके द्वारा अंग्रेजी में
प्राप्त अंकों से 50% अचधक है , तो उसके द्वारा अंग्रेजी में प्राप्त अंकों की गणना कीजजए।
(A) 50 (B) 80 (C) 85
(D) 90 (E) None of these

98. If total marks scored by five students in Hindi is 305 and ratio of the
marks scored by Bhanu in Hindi to the marks scored by Diksha in Hindi is 6
: 5, then how many marks did Diksha score in Hindi?

https :
//www. https : https
https : //instagra
//youtube.c
facebo m.com/aas :
om/channe hisharoraso
l/UCYa4_Jr ok.com cial(?) //t.m
Orf8R5Kz2u
/aashis utm_mediu
e/stu

268
OtccXQ m=
haroras copy_link
ocial dified
यदि पांच छात्रों द्वारा दहंिी में प्राप्त कुल अंक 305 हैं और भानु द्वारा दहंिी में प्राप्त अंकों का
िीक्षा द्वारा दहंिी में प्राप्त अंकों से अनुपात 6 : 5 है , तो िीक्षा ने दहंिी में ककतने अंक प्राप्त ककए?
(A) 50 (B) 60 (C) 100
(D) 85 (E) None of these

99.What is the difference between the total marks scored by Bhanu in


Science and Math together and the marks scored by Diksha in Math and
English together?
भानु द्वारा ववज्ञान और गखणत में लमलाकर प्राप्त ककए गए कुल अंकों और िीक्षा द्वारा गखणत
और अंग्रेजी में लमलाकर प्राप्त ककए गए अंकों के बीच का अंतर ककतना है ?
(A) 50 (B) 60 (C) 70
(D) 75 (E) None of these

100. If marks scored by Chitra in Math is 43 more than marks scored by


Eshan in Math and sum of marks scored by five persons in Math is 563, then
determine the marks scored by Eshan in Math.
यदि गखणत में चचत्रा द्वारा प्राप्त अंक, ईशान द्वारा गखणत में प्राप्त अंकों से 43 अचधक है और
गखणत में पांच व्यजक्तयों द्वारा प्राप्त अंकों का योग 563 है , तो गखणत में ईशान द्वारा प्राप्त
अंकों का ननधाषरण कीजजए।
(A) 45 (B) 55 (C) 65
(D) 50 (E) None of these

(101- 105) Directions : Study the following chart carefully and answer the
questions given below.
There are 5 bags each of which contains coins of two different
denominations Rs. 5 and Rs. 10.
Note : Total worth of Rs. 10 denomination coins in each bag is more than
that of Rs. 5 denomination coins in the bag.
ननिे श : ननम्नललखखत चाटष का ध्यानपूवक ष अध्ययन कीजजए और नीचे दिए गए प्रश्नों के उत्तर
िीजजए।
5 बैग हैं जजनमें से प्रत्येक में 5 रुपये और 10 रुपये के िो अलग - अलग मूल्यवगष के लसक्के हैं।
नोट : प्रत्येक बैग में 10 रुपये मूल्यवगष के लसक्कों का कुल मूल्य बैग में 5 रुपये मूल्यवगष के
लसक्कों से अचधक है ।

https :
//www. https : https
https : //instagra
//youtube.c
facebo m.com/aas :
om/channe hisharoraso
l/UCYa4_Jr ok.com cial(?) //t.m
Orf8R5Kz2u
/aashis utm_mediu
e/stu

269
OtccXQ m=
haroras copy_link
ocial dified
101. What is the ratio between the total worth of both denomination coins in
Bag C to that in Bag E?
बैग C और बैग E में िोनों मूल्यवगष के लसक्कों के कुल मूल्य के बीच का अनुपात क्या है ?
(A) 4 : 7 (B) 12 : 5 (C) 9 : 7
(D) 13 : 7 (E) None of these

102. Total worth of Rs. 10 denomination coins in bag D is what percentage


more than the total worth of Rs. 5 denomination coins in bag A?
बैग D में 10 रुपये मूल्यवगष के लसक्कों का कुल मूल्य, बैग A में 5 रुपये मूल्यवगष के लसक्कों के
कुल मूल्य से ककतने प्रनतशत अचधक है ?
(A) 62.5% (B) 43.2% (C) 34%
(D) 34% (E) None of these

103. What is the average number of Rs. 10 denomination coins in the bags
B, C and E?
बैग B, C और E में 10 रुपये मल्
ू यवगष के लसक्कों की औसत संख्या ककतनी है ?
(A) 15 (B) 16 (C) 10
(D) 6 (E) None of these

104. In how many bags, the total number of Rs. 5 denomination coins is in
between 30 to 40?
ककतने थैलों में , 5 रुपये के लसक्कों की कुल संख्या 30 से 40 के बीच है ?
(A) 4 (B) 1 (C) 2
(D) 3 (E) None of these

https :
//www. https : https
https : //instagra
//youtube.c
facebo m.com/aas :
om/channe hisharoraso
l/UCYa4_Jr ok.com cial(?) //t.m
Orf8R5Kz2u
/aashis utm_mediu
e/stu

270
OtccXQ m=
haroras copy_link
ocial dified
105. What is the difference between the total number of Rs. 5 denomination
coins and that of Rs. 10 denomination coins in the bag A, C and E taken
together?
बैग A, C और E को लमलाकर 5 रुपये मूल्यवगष के लसक्कों और 10 रुपये मूल्यवगष के लसक्कों की
कुल संख्या के बीच ककतना अंतर है ?
(A) 15 (B) 17 (C) 64
(D) 32 (E) None of these

(106-110) Directions : Study the following chart carefully and answer the
questions given below.
Following is the data regarding the average number of runs scored in a
match, number of matches played, number of sixes and fours hit by 4
Batsmen. Sixes and fours are collectively called as boundaries.
ननिे श : ननम्नललखखत चाटष का ध्यानपूवक ष अध्ययन कीजजए और नीचे दिए गए प्रश्नों के उत्तर
िीजजए।
एक मैच में बनाए गए रनों की औसत संख्या, खेले गए मैचों की संख्या, 4 बल्लेबाजों द्वारा छक्कों
और चौकों की संख्या के बारे में आंकडे ननम्नललखखत हैं। छक्के और चौके को सामूदहक रूप से
सीमा कहा जाता है ।

Average runs Number of Number of Number of


Players
in a match matches played Sixes hit Fours hit

Virat 48 60 64 120

Shikhar 40 54 36 94

Rohit 38 80 70 108

Rahul 44 40 38 48

Pant 32 22 30 51

106. For all the players combined, what is difference between the runs from
boundaries and runs other than boundaries?
संयुक्त रूप से सभी खखलाडडयों के ललए, बाउं ड्री से रन और बाउं ड्री के अलावा अन्य रन के बीच क्या
अंतर है ?
(A) 1176 (B) 1048 (C) 4320
(D) 3680 (E) None of these

https :
//www. https : https
https : //instagra
//youtube.c
facebo m.com/aas :
om/channe hisharoraso
l/UCYa4_Jr ok.com cial(?) //t.m
Orf8R5Kz2u
/aashis utm_mediu
e/stu

271
OtccXQ m=
haroras copy_link
ocial dified
107. The sum of the runs other than boundaries by Virat and Shikhar is how
much greater than the sum of the runs from boundaries by Rohit, Rahul and
Pant?
ववराट और लशखर द्वारा बाउं ड्री के अलावा रन का योग, रोदहत, राहुल और पंत द्वारा बाउं ड्री से रन
के योग से ककतना अचधक है ?
(A) 1280 (B) 1928 (C) 1640
(D) 5000 (E) None of these

108. For all the Batsmen combined what is the difference between runs from
sixes and runs from fours?
सभी बल्लेबाजों को लमलाकर छक्के से रन और चौके से रन में ककतना अंतर है ?
(A) 256 (B) 250 (C) 635
(D) 198 (E) None of these

109. What percentage of runs scored by Pant, come from boundaries?


पंत ने ककतने प्रनतशत रन बाउं ड्री से बनाए हैं?
(A) 60(9/11) (B) 50(7/11)% (C) 54(6/11)%
(D) 40% (E) None of these

110. What is the difference between the total runs scored from sixes and
fours by Rahul?
राहुल द्वारा छक्कों और चौकों से बनाए गए कुल रनों के बीच का अंतर ककतना है ?
(A) 17 (B) 36 (C) 29
(D) 19 (E) None of these

(111 - 115) Directions : Study the following chart carefully and answer the
questions given below.
A survey is conducting on the candidates preparing for government exams.
Pie chart given below shows the percentage distribution of number of
students following different coaching institute for their preparation. Total
Students preparing for government exam = 1250.
ननिे श : ननम्नललखखत चाटष का ध्यानपूवक ष अध्ययन कीजजए और नीचे दिए गए प्रश्नों के उत्तर
िीजजए।
सरकारी परीक्षा की तैयारी कर रहे अभ्यचथषयों पर सवे कराया जा रहा है । नीचे दिया गया पाई चाटष
उनकी तैयारी के ललए ववलभन्न कोचचंग संस्थानों में जाने वाले छात्रों की संख्या के प्रनतशत ववतरण
को िशाषता है । सरकारी परीक्षा की तैयारी करने वाले कुल छात्र = 1250।

https :
//www. https : https
https : //instagra
//youtube.c
facebo m.com/aas :
om/channe hisharoraso
l/UCYa4_Jr ok.com cial(?) //t.m
Orf8R5Kz2u
/aashis utm_mediu
e/stu

272
OtccXQ m=
haroras copy_link
ocial dified
111. What is the average number of students preparing for the exams from
coaching institute P, Q and S together?
कोचचंग संस्थान P, Q और S से लमलाकर परीक्षा की तैयारी करने वाले छात्रों की औसत संख्या
ककतनी है ?
(A) 200 (B) 210 (C) 250
(D) 240 (E) None of these

112. If out of total number of students preparing from coaching institute R


ratio of the boys to girls is 5 : 3, then what is the total number of girl
students preparing from coaching institute R?
यदि कोचचंग संस्थान से तैयारी करने वाले छात्रों की कुल संख्या में से लडकों का लडककयों से R का
अनुपात 5 : 3 है , तो कोचचंग संस्थान R से तैयारी करने वाली लडककयों की कुल संख्या ककतनी
है ?
(A) 150 (B) 180 (C) 210
(D) 200 (E) None of these

113. If students from coaching institute F are also added, then students
preparing from coaching institute Q becomes 12.5% of total students
preparing from all the six coaching institutes, then how many students are
preparing from coaching institute F?
यदि कोचचंग संस्थान F के छात्रों को भी जोडा जाता है , तो कोचचंग संस्थान Q से तैयारी करने वाले
छात्रों की संख्या सभी छह कोचचंग संस्थानों से तैयारी करने वाले कुल छात्रों का 12.5% हो जाती है ,
तो कोचचंग संस्थान F से तैयारी करने वाले छात्रों की संख्या ककतनी है ?
(A) 650 (B) 350 (C) 620
(D) 660 (E) None of these

https :
//www. https : https
https : //instagra
//youtube.c
facebo m.com/aas :
om/channe hisharoraso
l/UCYa4_Jr ok.com cial(?) //t.m
Orf8R5Kz2u
/aashis utm_mediu
e/stu

273
OtccXQ m=
haroras copy_link
ocial dified
114. If number of boys preparing is 150 each from coaching institute Q and
S, then what is the ratio of girls preparing from coaching institute Q that
from coaching institute S?
यदि कोचचंग संस्थान Q और S से तैयारी करने वाले लडकों की संख्या 150 प्रत्येक है , तो कोचचंग
संस्थान Q से तैयारी करने वाली लडककयों का कोचचंग संस्थान S से तैयारी करने वाली लडककयों
का अनुपात ककतना है ?
(A) 5 : 6 (B) 4 : 5 (C) 1 : 2
(D) 5 : 6 (E) None of these

115. If number of students preparing from coaching institute F is 250, then


number of students preparing from coaching institute P becomes what
percent of students of all six coaching institutes together?
यदि कोचचंग संस्थान F से तैयारी करने वाले छात्रों की संख्या 250 है , तो कोचचंग संस्थान P से
तैयारी करने वाले छात्रों की संख्या सभी छह कोचचंग संस्थानों के लमलाकर छात्रों का ककतना
प्रनतशत है ?
(A) 15% (B) 25% (C) 30%
(D) 20% (E) None of these

(116 - 120) Directions : Study the following chart carefully and answer the
questions given below.
The table given below shows the number of notebooks manufactured by
seven companies and percentage of notebooks sold by these five
companies.
ननिे श : ननम्नललखखत चाटष का ध्यानपूवक
ष अध्ययन कीजजए और नीचे दिए गए प्रश्नों के उत्तर
िीजजए।
नीचे िी गई ताललका सात कंपननयों द्वारा ननलमषत नोटबुक की संख्या और इन पांच कंपननयों
द्वारा बेची गई नोटबुक का प्रनतशत िशाषती है ।

Notebooks % of Notebooks
Company
manufactured sold

A 15300 80%
B 12600 _____
C ______ 90%
D 16200 _____
E ______ 75%
F _______ 80%
G 13600 95%

https :
//www. https : https
https : //instagra
//youtube.c
facebo m.com/aas :
om/channe hisharoraso
l/UCYa4_Jr ok.com cial(?) //t.m
Orf8R5Kz2u
/aashis utm_mediu
e/stu

274
OtccXQ m=
haroras copy_link
ocial dified
116. If the number of notebooks manufactured by company C and E is 25%
and 35% more than that of company D and company G respectively, then
what is the number of unsold notebooks of company C and E together?
यदि कंपनी C और E द्वारा ननलमषत नोटबुक्स की संख्या क्रमशः कंपनी D और कंपनी G की
तल
ु ना में 25% और 35% अचधक है , तो कंपनी C और E की लमलाकर न बबकी नोटबक ु की संख्या
ककतनी है ?
(A) 6155 (B) 8500 (C) 6615
(D) 6691 (E) None of these

117. What is the ratio of the number of notebooks sold by company A to the
number of notebooks sold by company B, If 75% of the number of
notebooks manufactured is sold by company B?
कंपनी A द्वारा बेची गई नोटबुक की संख्या का कंपनी B द्वारा बेची गई नोटबुक की संख्या से
अनुपात ककतना है , यदि ननलमषत नोटबुक की संख्या का 75% कंपनी B द्वारा बेची जाती है ?
(A) 45 : 49 (B) 136 : 105 (C) 250 : 363
(D) 503 : 105 (E) None of these

118. If the number of notebooks manufactured by company E and F together


is twice the number of notebooks manufactured by company B and the ratio
of number of notebooks manufactured by company E and F is 3 : 4, then
what is the difference between the number of unsold notebooks of company
E and F?
यदि कंपनी E और F द्वारा लमलकर ननलमषत नोटबुक की संख्या कंपनी B द्वारा ननलमषत नोटबुक
की संख्या से िोगुनी है और कंपनी E और F द्वारा ननलमषत नोटबुक की संख्या का अनुपात 3 : 4
है , तो कंपनी E और F की न बबकी नोटबकु की संख्या के बीच ककतना अंतर है ?
(A) 180 (B) 540 (C) 360
(D) 300 (E) None of these

119. If the ratio of the number of notebooks manufactured by company C, D


and E is 8 : 9 : 7 and 25% of notebooks are unsold in company D, then what
is the average number of notebooks sold by C, D and E together?
यदि कंपनी C, D और E द्वारा ननलमषत नोटबुक की संख्या का अनुपात 8 : 9 : 7 है और कंपनी D
में 25% नोटबुक नहीं बबकती हैं, तो C, D और E द्वारा लमलाकर बेची गई नोटबुक की औसत
संख्या ककतनी है ?
(A) 21800 (B) 55500 (C) 11520
(D) 20900 (E) None of these

120. If the 50% of number of notebooks manufactured is sold by company B


and the number of notebooks manufactured by F is 12400. The number of
notebooks sold by A and B together is what percent less or more than the

https :
//www. https : https
https : //instagra
//youtube.c
facebo m.com/aas :
om/channe hisharoraso
l/UCYa4_Jr ok.com cial(?) //t.m
Orf8R5Kz2u
/aashis utm_mediu
e/stu

275
OtccXQ m=
haroras copy_link
ocial dified
number of notebooks sold by F and G together?
यदि ननलमषत नोटबुक की संख्या का 50% कंपनी B द्वारा बेची जाती है और F द्वारा ननलमषत
नोटबुक की संख्या 12400 है । A और B द्वारा लमलाकर बेची गई नोटबुक की संख्या, F और G
द्वारा लमलाकर बेची गई नोटबुक की संख्या से ककतने प्रनतशत कम या अचधक है ?
(A) 15.55% (B) 18.82% (C) 12.44%
(D) 11.11% (E) None of these

(121 - 125) Directions : Study the following chart carefully and answer the
questions given below.
The line graph given below shows that percentage by which the number of
Samsung phones sold are more than the Nokia phones sold and difference
between Samsung and Nokia phones (Samsung - Nokia) sold by the five
different shops A, B, C, D and E. Note : All the shops sold only these two
types of phones.
ननिे श : ननम्नललखखत चाटष का ध्यानपव ू क
ष अध्ययन कीजजए और नीचे दिए गए प्रश्नों के उत्तर
िीजजए।
नीचे दिया गया लाइन ग्राि पांच अलग - अलग िक ु ानों A, B, C, D और E में उस प्रनतशत को
जजसके द्वारा बेचे गए Samsung फोनों की संख्या बेचे गए Nokia फोनों की संख्या से अचधक है
और Samsung और Nokia िोन (Samsung - Nokia) के बीच अंतर को िशाषता है । नोट :
सभी िक ु ानों पर केवल यही िो प्रकार के फोन बेचे गए।

121. Total number of phones sold by shop B is what percent more than the
total number of phones sold by shop E?
िक
ु ान B द्वारा बेचे गए फोनों की कुल संख्या, िक
ु ान E द्वारा बेचे गए फोनों की कुल संख्या से
ककतने प्रनतशत अचधक है ?
(A) 20% (B) 30% (C) 50%
(D) 60% (E) None of these

122. Total Samsung phones sold by shops A, C and E is how much more

https :
//www. https : https
https : //instagra
//youtube.c
facebo m.com/aas :
om/channe hisharoraso
l/UCYa4_Jr ok.com cial(?) //t.m
Orf8R5Kz2u
/aashis utm_mediu
e/stu

276
OtccXQ m=
haroras copy_link
ocial dified
than the total number of Nokia phones sold by those shops together?
िकु ानों A, C और E द्वारा बेचे गए कुल Samsung िोन उन िकु ानों द्वारा बेचे गए Nokia
िोनों की कुल संख्या से ककतने अचधक हैं?
(A) 45 (B) 55 (C) 60
(D) 60 (E) None of these

123. What is the ratio of total phones sold by shop C to the total phones sold
by shop D?
िक
ु ान C द्वारा बेचे गए कुल फोनों का िक
ु ान D द्वारा बेचे गए कुल फोनों से अनुपात ककतना है ?
(A) 11 : 10 (B) 13 : 10 (C) 12 : 13
(D) 10 : 13 (E) None of these

124. What is the difference between the average number of Samsung phones
sold by shops A, B and D and average number of Nokia phones sold by
shops B, D and E?
िकु ान A, B और D द्वारा बेचे गए Samsung िोन की औसत संख्या और िक
ु ान B, D और E
द्वारा बेचे गए Nokia िोन की औसत संख्या के बीच ककतना अंतर है ?
(A) 10 (B) 20 (C) 15
(D) 25 (E) None of these

125. Total number of Samsung phones sold by shops A, B and C together is


how much more than the total number of Nokia phones sold by shops C, D
and E together?
िक
ु ानों A, B और C द्वारा लमलाकर बेचे गए Samsung फोनों की कुल संख्या िक
ु ानों C, D और
E द्वारा लमलाकर बेचे गए Nokia फोनों की कुल संख्या से ककतनी अचधक है ?
(A) 56 (B) 64 (C) 126
(D) 90 (E) None of these

(126 - 130) Directions : Study the following chart carefully and answer the
questions given below.
Three candidates P, Q and R participated in an election. Table given below
shows the total votes received by P, Q and R from three wards.
Note : There are only these three candidates and a voter can vote for any on
candidate.
ननिे श : ननम्नललखखत चाटष का ध्यानपव ू क
ष अध्ययन कीजजए और नीचे दिए गए प्रश्नों के उत्तर
िीजजए।
एक चन ु ाव में तीन उम्मीिवारों P, Q और R ने भाग ललया। नीचे िी गई ताललका तीन वाडों से P,
Q और R द्वारा प्राप्त कुल मतों को िशाषती है ।
नोट : केवल ये तीन उम्मीिवार हैं और एक मतिाता ककसी भी उम्मीिवार को वोट िे सकता है ।

https :
//www. https : https
https : //instagra
//youtube.c
facebo m.com/aas :
om/channe hisharoraso
l/UCYa4_Jr ok.com cial(?) //t.m
Orf8R5Kz2u
/aashis utm_mediu
e/stu

277
OtccXQ m=
haroras copy_link
ocial dified
Votes Average of votes Average of votes
Wards
received by P received by P and Q received by Q and R

A 3600 4000 3000

B 4500 3150 2500

C 4200 4500 3200

126. What is the average number of votes received by candidate P from all
the three wards together?
सभी तीन वाडों से एक साथ उम्मीिवार P द्वारा प्राप्त मतों की औसत संख्या क्या है ?
(A) 3800 (B) 3500 (C) 4100
(D) 4500 (E) None of these

127. If all the voters from ward C casted their votes, then how many total
voters are there in ward C?
यदि वाडष C के सभी मतिाताओं ने मतिान ककया, तो वाडष C में कुल ककतने मतिाता हैं?
(A) 10500 (B) 10600 (C) 15000
(D) 12000 (E) None of these

128. Find the ratio of total votes received by candidate R from ward A to that
from ward C.
वाडष A से उम्मीिवार R द्वारा वाडष C से प्राप्त कुल मतों का अनुपात ज्ञात कीजजए।
(A) 1 : 1 (B) 3 : 2 (C) 7 : 8
(D) 9 : 4 (E) None of these

129. What is the total number of votes received by the candidate from ward
B who has received the maximum number of votes in that ward?
वाडष B से उस उम्मीिवार को कुल ककतने वोट लमले हैं, जजसने उस वाडष में सबसे अचधक वोट प्राप्त
ककए हैं?
(A) 5500 (B) 2500 (C) 4500
(D) 5000 (E) None of these

130. In which ward candidate Q received maximum number of votes?


ककस वाडष में उम्मीिवार Q को सबसे अचधक मत प्राप्त हुए?

https :
//www. https : https
https : //instagra
//youtube.c
facebo m.com/aas :
om/channe hisharoraso
l/UCYa4_Jr ok.com cial(?) //t.m
Orf8R5Kz2u
/aashis utm_mediu
e/stu

278
OtccXQ m=
haroras copy_link
ocial dified
(A) Only A (B) Only B (C) Both A and C
(D) Both A and B (E) None of these

(131 - 135) Directions : Study the following chart carefully and answer the
questions given below.
The pie chart given below shows the percentage distribution of the number
of each types of unit manufactured in a factory. Given table shows the
percentage of sold units of each type. Total number of units manufactured =
4800
ननिे श : ननम्नललखखत चाटष का ध्यानपव ू क
ष अध्ययन कीजजए और नीचे दिए गए प्रश्नों के उत्तर
िीजजए।
नीचे दिया गया पाई चाटष एक कारखाने में ननलमषत प्रत्येक प्रकार की इकाई की संख्या का प्रनतशत
ववतरण िशाषता है । िी गई ताललका प्रत्येक प्रकार की बेची गई इकाइयों का प्रनतशत िशाषती है ।
ननलमषत इकाइयों की कुल संख्या = 4800

Type Percentage of sold units

P 45%

Q 55%

R 25%

S 75%

T 85%

https :
//www. https : https
https : //instagra
//youtube.c
facebo m.com/aas :
om/channe hisharoraso
l/UCYa4_Jr ok.com cial(?) //t.m
Orf8R5Kz2u
/aashis utm_mediu
e/stu

279
OtccXQ m=
haroras copy_link
ocial dified
131. What is the ratio of the number of units sold of type P to the number of
unsold units of type Q?
P प्रकार की बेची गई इकाइयों की संख्या का Q प्रकार की न बबकी इकाइयों की संख्या से अनुपात
क्या है ?
(A) 2 : 3 (B) 4 : 5 (C) 3 : 2
(D) 4 : 1 (E) None of these

132. Find the difference between the number of unsold units of type P and
the number of sold units of type Q.
P प्रकार की न बबकी इकाइयों की संख्या और Q प्रकार की बेची गई इकाइयों की संख्या के बीच
अंतर ज्ञात कीजजए।
(A) 132 (B) 541 (C) 189
(D) 150 (E) None of these

133. The sum of the number of unsold units of type S and type T is what
percent greater than the number of sold units of type R?
S प्रकार और T प्रकार की न बबकी इकाइयों की संख्या का योग R प्रकार की बेची गई इकाइयों की
संख्या से ककतने प्रनतशत अचधक है ?
(A) 110% (B) 115% (C) 100%
(D) 600% (E) None of these

134. The number of unsold units of type R is what percent of the number of
sold units of type S?
R प्रकार की न बबकी इकाइयों की संख्या S प्रकार की बेची गई इकाइयों की संख्या का ककतना
प्रनतशत है ?
(A) 10% (B) 70% (C) 80%
(D) 18% (E) None of these

135. What is the difference between the number of sold units of type T and
type P?
T प्रकार और P प्रकार की बेची गई इकाइयों की संख्या के बीच का अंतर ककतना है ?
(A) 800 (B) 500 (C) 400
(D) 900 (E) None of these

(136 - 140) Directions : Study the following chart carefully and answer the
questions given below.
Following table shows the number of colleges who offer three courses BCA,
MCA and DCA.
ननिे श : ननम्नललखखत चाटष का ध्यानपूवक
ष अध्ययन कीजजए और नीचे दिए गए प्रश्नों के उत्तर
िीजजए।

https :
//www. https : https
https : //instagra
//youtube.c
facebo m.com/aas :
om/channe hisharoraso
l/UCYa4_Jr ok.com cial(?) //t.m
Orf8R5Kz2u
/aashis utm_mediu
e/stu

280
OtccXQ m=
haroras copy_link
ocial dified
ननम्न ताललका उन कॉलेजों की संख्या िशाषती है जो तीन पाठ्यक्रम BCA, MCA और DCA
प्रिान करते हैं।

% of students
% of students Number of students
College pursuing
pursuing BCA pursuing MCA
DCA

P 16.67% 33.33% 243

Q 30% 45% 960

R 25% 25% 294

S 35% 55% 500

T 40% 30% 300

136. Total number of students studying in college S is what percent of total


number of students studying in college T?
कॉलेज S में पढने वाले छात्रों की कुल संख्या, कॉलेज T में पढने वाले छात्रों की कुल संख्या का
ककतना प्रनतशत है ?
(A) 200% (B) 300% (C) 500%
(D) 100% (E) None of these

137. Find the difference between the total number of college students
studying BCA from college R and S and total number of college students
studying MCA from college P and R.
कॉलेज R और S से BCA पढने वाले कॉलेज के छात्रों की कुल संख्या और कॉलेज P और R से
MCA पढने वाले कॉलेज के छात्रों की कुल संख्या के बीच अंतर ज्ञात कीजजए।
(A) 1830 (B) 1360 (C) 1500
(D) 1200 (E) None of these

138. Total number of students studying DCA from college S is approximately


what percent more than students studying MCA from college Q?
कॉलेज S से DCA पढने वाले छात्रों की कुल संख्या, कॉलेज Q से MCA पढने वाले छात्रों की
संख्या से लगभग ककतने प्रनतशत अचधक है ?
(A) 187% (B) 150% (C) 125%
(D) 50% (E) None of these

https :
//www. https : https
https : //instagra
//youtube.c
facebo m.com/aas :
om/channe hisharoraso
l/UCYa4_Jr ok.com cial(?) //t.m
Orf8R5Kz2u
/aashis utm_mediu
e/stu

281
OtccXQ m=
haroras copy_link
ocial dified
139. Total number of girls studying DCA in college Q are 988 and rest are
boys. In college P, ratio of girls to boys studying DCA is 3 : 6. Find the total
number of boys studying DCA from college P and college Q.
कॉलेज Q में DCA पढने वाली लडककयों की कुल संख्या 988 है और बाकी लडके हैं। कॉलेज P में ,
DCA पढने वाली लडककयों का लडकों से अनप ु ात 3 : 6 है । कॉलेज P और कॉलेज Q से DCA
पढने वाले लडकों की कुल संख्या ज्ञात कीजजए।
(A) 600 (B) 454 (C) 848
(D) 245 (E) None of these

140. Find the average number of students studying BCA from all the
colleges together.
सभी कॉलेजों से लमलाकर BCA पढने वाले छात्रों की औसत संख्या ज्ञात कीजजए।
(A) 606 (B) 706 (C) 500
(D) 400 (E) None of these

(141 - 145) Directions : Study the following chart carefully and answer the
questions given below.
Following line graph shows the revenue and expenditure of Company A
through 5 years.
Profit = Revenue - Expenditure
Percentage profit = Profit/Expenditure × 100
ननिे श : ननम्नललखखत चाटष का ध्यानपूवकष अध्ययन कीजजए और नीचे दिए गए प्रश्नों के उत्तर
िीजजए।
ननम्नललखखत लाइन ग्राि 5 वर्ों के माध्यम से कंपनी A के राजस्व और व्यय को िशाषता है ।
लाभ = आगम - व्यय
प्रनतशत लाभ = लाभ/व्यय × 100

https :
//www. https : https
https : //instagra
//youtube.c
facebo m.com/aas :
om/channe hisharoraso
l/UCYa4_Jr ok.com cial(?) //t.m
Orf8R5Kz2u
/aashis utm_mediu
e/stu

282
OtccXQ m=
haroras copy_link
ocial dified
141. In which year the percentage profit is the greatest?
ककस वर्ष में लाभ का प्रनतशत सबसे अचधक है ?
(A) 2014 (B) 2016 (C) 2015
(D) 2017 (E) None of these

142. In which year the percentage profit is the least?


ककस वर्ष में लाभ प्रनतशत सबसे कम है ?
(A) 2018 (B) 2014 (C) 2016
(D) 2018 (E) None of these

143. What is the ratio of the profit in 2015 to that in 2017?


2015 में लाभ का 2017 में लाभ से अनुपात क्या है ?
(A) 2 : 3 (B) 3 : 2 (C) 1 : 2
(D) 2 : 1 (E) None of these

144. The profit in 2016 is what percentage of the sum of the profit in years
2014 and 2018?
2016 में लाभ वर्ष 2014 और 2018 में लाभ के योग का ककतना प्रनतशत है ?
(A) 60% (B) 50% (C) 75%
(D) 65% (E) None of these

145. If in 2019, the revenue increases by 45% and expenditure increases by


55% with respect to 2018, then what is the percentage decrease in the profit
from 2018 to 2019?
यदि 2019 में , राजस्व में 45% की वद्
र चध होती है और 2018 के संबंध में व्यय में 55% की वद्
र चध
होती है , तो 2018 से 2019 तक लाभ में प्रनतशत की कमी ककतनी है ?
(A) 50% (B) 60% (C) 30%
(D) 55% (E) None of these

(146 - 150) Directions : Study the following chart carefully and answer the
questions given below.
In a town the houses are painted Blue, Yellow, Brown or Pink.
ननिे श : ननम्नललखखत चाटष का ध्यानपूवक ष अध्ययन कीजजए और नीचे दिए गए प्रश्नों के उत्तर
िीजजए।
एक कस्बे में घरों को नीले, पीले, भरू े या गलु ाबी रं ग से रं गा जाता है ।

https :
//www. https : https
https : //instagra
//youtube.c
facebo m.com/aas :
om/channe hisharoraso
l/UCYa4_Jr ok.com cial(?) //t.m
Orf8R5Kz2u
/aashis utm_mediu
e/stu

283
OtccXQ m=
haroras copy_link
ocial dified
Blue Yellow houses Ratio of Houses
Localities Pink houses
houses and Pink houses Blue : Brown
M 41 67 41 : 29 30

N 15 52 3:5 12

P 36 81 9:8 54

Q 28 54 7 : 13 33

R 35 47 5:6 16

146. What percentage (approx.) of the total number of houses in locality Q


are painted Brown?
इलाके Q में घरों की कुल संख्या का लगभग ककतना प्रनतशत (लगभग) भूरे रं ग से रं गा गया है ?
(A) 30% (B) 45% (C) 39%
(D) 25% (E) None of these

147. What is the ratio of the total number of houses in locality N which are
Blue and Yellow to the total number of houses in locality R which are
painted Brown or Pink?
इलाके N में नीले और पीले घरों की कुल संख्या का इलाके R में भूरे या गुलाबी रं ग से रं गे घरों की
कुल संख्या से अनुपात क्या है ?
(A) 55 : 58 (B) 54 : 57 (C) 53 : 59
(D) 52 : 55 (E) None of these

148. What is the difference between the total number of Brown houses in
localities M and P and the total number of houses in locality Q which are
painted Yellow and Blue?
इलाके M और P में भूरे घरों की कुल संख्या और इलाके Q में पीले और नीले रं ग से रं गे घरों की
कुल संख्या के बीच ककतना अंतर है ?
(A) 12 (B) 25 (C) 63
(D) 19 (E) None of these

https :
//www. https : https
https : //instagra
//youtube.c
facebo m.com/aas :
om/channe hisharoraso
l/UCYa4_Jr ok.com cial(?) //t.m
Orf8R5Kz2u
/aashis utm_mediu
e/stu

284
OtccXQ m=
haroras copy_link
ocial dified
149. What is the total number of Brown houses in localities N and R?
इलाके N और R में भूरे घरों की कुल संख्या ककतनी है ?
(A) 60 (B) 54 (C) 67
(D) 70 (E) None of these

150. Total number of Yellow houses in localities M and Q is _____ than the
total number of houses in locality P which are not Yellow or Pink. Fill in the
blank.
इलाके M और Q में पीले घरों की कुल संख्या, इलाके P में घरों की कुल संख्या की तुलना में
_____ है , जो पीले या गल
ु ाबी नहीं हैं। ररक्त स्थान को भरें ।
(A) 10 more (B) 10 less (C) 15 more
(D) 20 less (E) None of these

(151 - 155) Directions : Study the following chart carefully and answer the
questions given below.
The line graph given below shows the total number of persons and
percentage of number of literate persons live in five different cities.
ननिे श : ननम्नललखखत चाटष का ध्यानपूवक ष अध्ययन कीजजए और नीचे दिए गए प्रश्नों के उत्तर
िीजजए।
नीचे दिया गया लाइन ग्राि पांच अलग - अलग शहरों में रहने वाले व्यजक्तयों की कुल संख्या और
साक्षर व्यजक्तयों की संख्या का प्रनतशत िशाषता है ।

151. Number of literate and number of illiterate persons in city F is


respectively 20% more and 20% less than number of literate and number of
illiterate persons in city E, then find the difference between number of
literate and number of illiterate persons in city F.
शहर F में साक्षर और ननरक्षर व्यजक्तयों की संख्या शहर E में साक्षर और ननरक्षर व्यजक्तयों की
संख्या से क्रमशः 20% अचधक और 20% कम है , तो शहर F में साक्षर और ननरक्षर व्यजक्तयों की
संख्या के बीच का अंतर ज्ञात कीजजए।

https :
//www. https : https
https : //instagra
//youtube.c
facebo m.com/aas :
om/channe hisharoraso
l/UCYa4_Jr ok.com cial(?) //t.m
Orf8R5Kz2u
/aashis utm_mediu
e/stu

285
OtccXQ m=
haroras copy_link
ocial dified
(A) 1280 (B) 1240 (C) 1260
(D) 1220 (E) None of these

152. The ratio of number of male to female literate and illiterate persons in
city B is respectively 5 : 4 and 7 : 5, then find the difference between number
of literate males and number of illiterate females in city B.
शहर B में पुरुर्ों का साक्षर और ननरक्षर मदहलाओं से अनुपात क्रमशः 5 : 4 और 7 : 5 है , तो शहर
B में साक्षर पुरुर्ों की संख्या और ननरक्षर मदहलाओं की संख्या के बीच का अंतर ज्ञात कीजजए।
(A) 330 (B) 320 (C) 350
(D) 340 (E) None of these

153. Find the average of number of illiterate persons of city B, C and E.


शहर B, C और E के ननरक्षर व्यजक्तयों की संख्या का औसत ज्ञात कीजजए।
(A) 1584 (B) 1578 (C) 1596
(D) 1562 (E) None of these

154. Number of illiterate persons of city A is what percent of total number of


illiterate persons in all five cities?
शहर A के ननरक्षर व्यजक्तयों की संख्या सभी पांच शहरों में ननरक्षर व्यजक्तयों की कुल संख्या का
ककतना प्रनतशत है ?
(A) 21.03% (B) 23.56% (C) 27.14%
(D) 25.87% (E) None of these

155. Number of literate persons of city C is how much percent more or less
than number of literate persons of city B?
शहर C के साक्षर व्यजक्तयों की संख्या शहर B के साक्षर व्यजक्तयों की संख्या से ककतने प्रनतशत
अचधक या कम है ?
(A) 44.23% (B) 42.89% (C) 43.45%
(D) 41.34% (E) None of these

(156-160) Directions : Study the following chart carefully and answer the
questions given below.
The bar graph given below shows the number of dupattas, number of
comforters and number of quilts sold by five different shopkeepers.
जनदेश : जनम्नजलखित चाटत का ध्यानपूवतक अध्ययन कीजिए और नीचे ददए गए प्रश्नों के
उत्तर दीजिए।
नीचे ददया गया बार ग्राफ पांच अलग - अलग दुकानदारों द्वारा बेचे गए दुपट्टों की संख्या,
कम्फटत सत की संख्या और रिाई की संख्या को दशातता है ।

https :
//www. https : https
https : //instagra
//youtube.c
facebo m.com/aas :
om/channe hisharoraso
l/UCYa4_Jr ok.com cial(?) //t.m
Orf8R5Kz2u
/aashis utm_mediu
e/stu

286
OtccXQ m=
haroras copy_link
ocial dified
156. Find the average of number of dupattas sold by all five shopkeepers.
सभी पांच िक
ु ानिारों द्वारा बेची गई िप
ु ट्टों की औसत संख्या ज्ञात कीजजए।
(A) 1150 (B) 1180 (C) 1170
(D) 1160 (E) None of these

157. Number of comforters sold by P is what percent of sum of number of


comforters sold by R, S and T together?
P द्वारा बेचे गए कम्िटष सष की संख्या, R, S और T द्वारा लमलाकर बेचे गए कम्िटष सष की संख्या
के योग का ककतना प्रनतशत है ?
(A) 36.56% (B) 37.93% (C) 38.89%
(D) 39.52% (E) None of these

158. The ratio of number of silk to woolen quilts sold by R and S is


respectively 8 : 5 and 3 : 2, then find the difference between number of silk
quilts sold by R and number of woolen quilts sold by S.
R और S द्वारा बेची गई रे शम की ऊनी रजाई की संख्या का अनप ु ात क्रमशः 8 : 5 और 3 : 2 है ,
तो R द्वारा बेची गई रे शम की रजाई की संख्या और S द्वारा बेची गई ऊनी रजाई की संख्या के
बीच का अंतर ज्ञात कीजजए।
(A) 62 (B) 88 (C) 56
(D) 74 (E) None of these

159. Find the ratio between sum of number of comforters sold by R and T
together to sum of number of quilts sold by Q and T together.
R और T द्वारा लमलाकर बेचे गए कम्िटष सष की संख्या का Q और T द्वारा लमलाकर बेची गई
रजाईयों की संख्या के योग से अनुपात ज्ञात कीजजए।
(A) 1 : 1 (B) 2 : 3 (C) 4 : 5
(D) 6 : 7 (E) None of these

https :
//www. https : https
https : //instagra
//youtube.c
facebo m.com/aas :
om/channe hisharoraso
l/UCYa4_Jr ok.com cial(?) //t.m
Orf8R5Kz2u
/aashis utm_mediu
e/stu

287
OtccXQ m=
haroras copy_link
ocial dified
160. Number of dupattas, number of comforters and number of quilts sold
by M is respectively 10% more, 15% more and 10% less than number of
dupattas, number of comforters and number of quilts sold by S, then find
the sum of number of dupattas, number of comforters and number of quilts
sold by M.
M द्वारा बेचे गए िप ु ट्टों की संख्या, कम्िटष सष की संख्या और रजाईयों की संख्या S द्वारा बेचे
गए िपु ट्टों की संख्या, कम्िटष सष की संख्या और रजाईयों की संख्या से क्रमशः 10% अचधक,
15% अचधक और 10% कम है , तो M द्वारा बेचे गए िप ु ट्टों की संख्या, कम्िटष सष की संख्या और
रजाईयों की संख्या का योग ज्ञात कीजजए।
(A) 3964 (B) 3962 (C) 3963
(D) 3966 (E) None of these

(161 - 165) Directions : Study the following chart carefully and answer the
questions given below.
The line chart given below shows the number of persons who visits Pune,
Delhi, Mumbai, Agra and Chennai in April, May and June.
ननिे श : ननम्नललखखत चाटष का ध्यानपूवक ष अध्ययन कीजजए और नीचे दिए गए प्रश्नों के उत्तर
िीजजए।
नीचे दिया गया लाइन चाटष अप्रैल, मई और जून में पुणे, दिल्ली, मुंबई, आगरा और चेन्नई जाने
वाले व्यजक्तयों की संख्या िशाषता है ।

161. Find the average of number of persons who visits Delhi in all three
months.
सभी तीन महीनों में दिल्ली का िौरा करने वाले व्यजक्तयों की औसत संख्या ज्ञात कीजजए।
(A) 760 (B) 720 (C) 700
(D) 740 (E) None of these

https :
//www. https : https
https : //instagra
//youtube.c
facebo m.com/aas :
om/channe hisharoraso
l/UCYa4_Jr ok.com cial(?) //t.m
Orf8R5Kz2u
/aashis utm_mediu
e/stu

288
OtccXQ m=
haroras copy_link
ocial dified
162. Number of persons who visited Chennai in May is what percent of total
number of persons who visits all five cities in May?
मई में चेन्नई जाने वाले व्यजक्तयों की संख्या मई में सभी पांच शहरों का िौरा करने वाले
व्यजक्तयों की कुल संख्या का ककतना प्रनतशत है ?
(A) 16.41% (B) 15.08% (C) 17.56%
(D) 18.31% (E) None of these

163. Number of persons who visits Mumbai in May is how much percent
more or less than number of persons who visits Agra in May?
मई में मंब
ु ई जाने वाले व्यजक्तयों की संख्या मई में आगरा जाने वाले व्यजक्तयों की संख्या से
ककतने प्रनतशत अचधक या कम है ?
(A) 32.43% (B) 35.89% (C) 33.12%
(D) 34.64% (E) None of these

164. Number of persons who visits Mumbai and Agra in July is respectively
20% and 15.38% more than previous month, then find the sum of number of
persons who visits Mumbai and Agra in July.
जुलाई में मुंबई और आगरा जाने वाले व्यजक्तयों की संख्या वपछले महीने की तुलना में क्रमशः
20% और 15.38% अचधक है , तो जुलाई में मुंबई और आगरा जाने वाले व्यजक्तयों की संख्या का
योग ज्ञात कीजजए।
(A) 1866 (B) 1852 (C) 1836
(D) 1841 (E) None of these

165. The ratio of number of Males to Females who visits Mumbai in April and
May is respectively 9 : 8 and 5 : 9, then find the sum of number of Males who
visits Mumbai in April and number of Females who visits Mumbai in May.
अप्रैल और मई में मुंबई जाने वाले पुरुर्ों की संख्या का मदहलाओं से अनुपात क्रमशः 9 : 8 और 5
: 9 है , तो अप्रैल में मुंबई जाने वाले पुरुर्ों की संख्या और मई में मुंबई जाने वाली मदहलाओं की
संख्या का योग ज्ञात कीजजए।
(A) 1341 (B) 1185 (C) 1219
(D) 1080 (E) None of these

(166 - 170) Directions : Study the following chart carefully and answer the
questions given below.
The pie chart given below shows the percentage distribution of data cables
sold by a Varun on five different days. Total number of data cables sold =
400.
ननिे श : ननम्नललखखत चाटष का ध्यानपूवक ष अध्ययन कीजजए और नीचे दिए गए प्रश्नों के उत्तर
िीजजए।
नीचे दिया गया पाई चाटष वरुण द्वारा पांच अलग - अलग दिनों में बेचे गए डेटा केबलों के प्रनतशत
ववतरण को िशाषता है । बेचे गए डेटा केबलों की कुल संख्या = 400।

https :
//www. https : https
https : //instagra
//youtube.c
facebo m.com/aas :
om/channe hisharoraso
l/UCYa4_Jr ok.com cial(?) //t.m
Orf8R5Kz2u
/aashis utm_mediu
e/stu

289
OtccXQ m=
haroras copy_link
ocial dified
166. Find the average of number of data cables sold on Monday, Tuesday,
Wednesday and Thursday.
सोमवार, मंगलवार, बध
ु वार और गरु
ु वार को बेची गई डेटा केबलों की औसत संख्या ज्ञात कीजजए।
(A) 96 (B) 68 (C) 72
(D) 84 (E) None of these

167. The ratio of number of C - Type data cables to number of USB data
cables sold on Thursday and Friday is respectively 8 : 5 and 5 : 3, then find
the sum of number of C - Type data cables sold on Thursday and Friday.
गुरुवार और शुक्रवार को बेचे गए C - प्रकार के डेटा केबलों की संख्या का USB डेटा केबलों की
संख्या से क्रमशः 8 : 5 और 5 : 3 का अनुपात है , तो गुरुवार और शुक्रवार को बेचे गए C - प्रकार
के डेटा केबलों की संख्या का योग ज्ञात कीजजए।
(A) 105 (B) 104 (C) 106
(D) 103 (E) None of these

168. Number of data cables sold by another shopkeeper Tarun on Monday


and Tuesday is respectively 18.18% and 25% more than number of data
cables sold by Varun on same days, then find the sum of number of data
cables sold on Monday and Tuesday by Tarun.
अन्य िक ु ानिार तरुण द्वारा सोमवार और मंगलवार को बेची गई डेटा केबलों की संख्या वरुण
द्वारा समान दिनों में बेची गई डेटा केबलों की संख्या से क्रमशः 18.18% और 25% अचधक है , तो
तरुण द्वारा सोमवार और मंगलवार को बेची गई डेटा केबलों की संख्या का योग ज्ञात कीजजए।
(A) 172 (B) 189 (C) 168
(D) 193 (E) None of these

169. Number of data cables sold on Thursday is how much percent more or
less than number of data cables sold on Wednesday?

https :
//www. https : https
https : //instagra
//youtube.c
facebo m.com/aas :
om/channe hisharoraso
l/UCYa4_Jr ok.com cial(?) //t.m
Orf8R5Kz2u
/aashis utm_mediu
e/stu

290
OtccXQ m=
haroras copy_link
ocial dified
गुरुवार को बेची गई डेटा केबल की संख्या, बुधवार को बेची गई डेटा केबल की संख्या से ककतने
प्रनतशत अचधक या कम है ?
(A) 36.84% (B) 38.96% (C) 32.10%
(D) 32.10% (E) None of these

170. Number of data cables sold on Friday is what percent of sum of number
of data cables sold on Wednesday and Thursday?
शुक्रवार को बेची गई डेटा केबल की संख्या, बुधवार और गुरुवार को बेची गई डेटा केबल की संख्या
का ककतना प्रनतशत है ?
(A) 32.78% (B) 34.12% (C) 33.66%
(D) 35.55% (E) None of these

(171 - 175) Directions : Study the following chart carefully and answer
the questions given below.
The bar graph given below shows the number of Comic books and
number of Story books sold in January, February, March, April and May
in two different years.
जनदेश : जनम्नजलखित चाटत का ध्यानपूवतक अध्ययन कीजिए और नीचे ददए गए प्रश्नों के
उत्तर दीजिए।
नीचे ददया गया बार ग्राफ दो अलग - अलग वर्षों में िनवरी, फरवरी, माचत, अप्रैल और मई
में बेची गई कॉजमक वकताबों की संख्या और कहानी की वकताबों की संख्या को दशातता है ।

171. The ratio of English Comic books to Hindi Comic books sold in
February in 2021 and 2022 is respectively 5 : 3 and 5 : 4, then find the
sum of number of English Comic books sold in February 2021 and
number of Hindi Comic books sold in February 2022.
फरवरी 2021 और 2022 में बेची गई अंग्रेिी कॉजमक पुस्तकों से रहिं दी कॉजमक पुस्तकों का
अनुपात क्रमशः 5 : 3 और 5 : 4 है , तो फरवरी 2021 में बेची गई अंग्रेिी कॉजमक पुस्तकों
की संख्या और फरवरी 2022 में बेची गई रहिं दी कॉजमक पुस्तकों की संख्या का योग ज्ञात

https :
//www. https : https
https : //instagra
//youtube.c
facebo m.com/aas :
om/channe hisharoraso
l/UCYa4_Jr ok.com cial(?) //t.m
Orf8R5Kz2u
/aashis utm_mediu
e/stu

291
OtccXQ m=
haroras copy_link
ocial dified
कीजिए।
(A) 168 (B) 182 (C) 197
(D) 173 (E) None of these

172. What is ratio of sum of number of Comic books sold in February and
May 2021 to sum of number of Story books sold in May in 2021 and 2022?
िरवरी और मई 2021 में बेची गई कॉलमक ककताबों की संख्या का मई में 2021 और 2022 में
बेची गई कहानी की ककताबों की संख्या से अनुपात ककतना है ?
(A) 9 : 8 (B) 4 : 5 (C) 6 : 7
(D) 3 : 2 (E) None of these

173. If price of each Comic book and each Story book is Rs. 35 and Rs. 23
respectively, then find the amount received by shopkeeper from sell of
Comic books in May 2021 and Story books in May 2021.
यदि प्रत्येक कॉलमक बक
ु और प्रत्येक स्टोरी बक
ु की कीमत क्रमशः 35 रुपये और 23 रुपये है , तो
मई 2021 में कॉलमक बुक और मई 2021 में स्टोरी बुक की बबक्री से िक
ु ानिार को प्राप्त रालश
ज्ञात कीजजए।
(A) Rs. 10515 (B) Rs. 10263 (C) Rs. 10487
(D) Rs. 10349 (E) None of these

174. Find the average of number of Story books sold in all five months in
2021.
2021 में सभी पांच महीनों में बेची गई कहानी की ककताबों की औसत संख्या ज्ञात कीजजए।
(A) 202 (B) 205 (C) 203
(D) 204 (E) None of these

175. Number of Story books sold in March 2022 is what percent of total
number of Story books sold in all five months in 2022?
माचष 2022 में बेची गई कहानी की ककताबों की संख्या 2022 में सभी पांच महीनों में बेची गई
कहानी की ककताबों की कुल संख्या का ककतना प्रनतशत है ?
(A) 14.23% (B) 12.89% (C) 16.45%
(D) 19.84% (E) None of these

(176 - 180) Directions : Study the following chart carefully and answer the
questions given below.
The table given below shows the percentage of number males, number of
children and total number of persons in five different cities.
Total number of persons = Male + Female + Children
ननिे श : ननम्नललखखत चाटष का ध्यानपूवक
ष अध्ययन कीजजए और नीचे दिए गए प्रश्नों के उत्तर
िीजजए।
नीचे िी गई ताललका पांच अलग - अलग शहरों में पुरुर्ों की संख्या, बच्चों की संख्या और

https :
//www. https : https
https : //instagra
//youtube.c
facebo m.com/aas :
om/channe hisharoraso
l/UCYa4_Jr ok.com cial(?) //t.m
Orf8R5Kz2u
/aashis utm_mediu
e/stu

292
OtccXQ m=
haroras copy_link
ocial dified
व्यजक्तयों की कुल संख्या का प्रनतशत िशाषती है ।
व्यजक्तयों की कुल संख्या = पुरुर् + मदहला + बच्चे

176. The ratio of number of children going to school and not going to school
in city C and City D is respectively 4 : 7 and 11 : 7, then find the sum of
number of children going to school in city C and number of children not
going to school in city D.
शहर C और शहर D में स्कूल जाने वाले और स्कूल नहीं जाने वाले बच्चों की संख्या का अनुपात
क्रमशः 4 : 7 और 11 : 7 है , तो शहर C में स्कूल जाने वाले बच्चों की संख्या और शहर D में
स्कूल न जाने वाले बच्चों की संख्या का योग ज्ञात कीजजए।
(A) 830 (B) 850 (C) 870
(D) 810 (E) None of these

177. Number of males and females in city F and G is respectively 20% and
30% of number of males and females in city C and A, then find the sum of
number of males and female in city F and city G.
शहर F और G में पुरुर्ों और मदहलाओं की संख्या क्रमशः शहर C और A में पुरुर्ों और मदहलाओं
की संख्या का 20% और 30% है , तो शहर F और शहर G में पुरुर्ों और मदहलाओं की संख्या का
योग ज्ञात कीजजए।
(A) 1053 (B) 1057 (C) 1055
(D) 1051 (E) None of these

178. Find the average of number of females in all five cities.


सभी पांच शहरों में मदहलाओं की औसत संख्या ज्ञात कीजजए।
(A) 965 (B) 963 (C) 967
(D) 969 (E) None of these

179. Number of males in city D is how much percent more or less than
number of males in city C?
शहर D में परु
ु र्ों की संख्या शहर C में परु
ु र्ों की संख्या से ककतने प्रनतशत अचधक या कम है ?

https :
//www. https : https
https : //instagra
//youtube.c
facebo m.com/aas :
om/channe hisharoraso
l/UCYa4_Jr ok.com cial(?) //t.m
Orf8R5Kz2u
/aashis utm_mediu
e/stu

293
OtccXQ m=
haroras copy_link
ocial dified
(A) 32.22% (B) 38.96% (C) 36.45%
(D) 34.10% (E) None of these

180. Number of children in city A is what percent of total number of children


in all five cities?
शहर A में बच्चों की संख्या सभी पाँच शहरों में बच्चों की कुल संख्या का ककतना प्रनतशत है ?
(A) 27.04% (B) 23.09% (C) 25.06%
(D) 21.05% (E) None of these

(181 - 185) Directions : Study the following chart carefully and answer the
questions given below.
The bar graph given below shows the percentage of cars sold in 2022 and
number of cars sold in 2021 by five different companies.
ननिे श : ननम्नललखखत चाटष का ध्यानपूवक ष अध्ययन कीजजए और नीचे दिए गए प्रश्नों के उत्तर
िीजजए।
नीचे दिया गया बार ग्राि 2022 में बेची गई कारों का प्रनतशत और 2021 में पांच अलग-अलग
कंपननयों द्वारा बेची गई कारों की संख्या को िशाषता है ।

181. Number of cars sold by Kia in 2021 and 2022 respectively 30% and 18%
more than number of cars sold by FORCE in 2021 and 2022, then find the
total number of cars sold by Kia in 2021 and 2022.
2021 और 2022 में Kia द्वारा बेची गई कारों की संख्या क्रमशः 2021 और 2022 में FORCE
द्वारा बेची गई कारों की संख्या से 30% और 18% अचधक है , तो 2021 और 2022 में Kia द्वारा
बेची गई कारों की कुल संख्या ज्ञात कीजजए।
(A) 1648 (B) 1524 (C) 1332
(D) 1462 (E) None of these

https :
//www. https : https
https : //instagra
//youtube.c
facebo m.com/aas :
om/channe hisharoraso
l/UCYa4_Jr ok.com cial(?) //t.m
Orf8R5Kz2u
/aashis utm_mediu
e/stu

294
OtccXQ m=
haroras copy_link
ocial dified
182. Number of cars sold by FORCE and FORD in 2020 is respectively
16.66% and 18.18% less than number of cars sold by FORCE and FORD in
2021, then find the total number of cars sold by FORCE and FORD in 2020.
2020 में FORCE और FORD द्वारा बेची गई कारों की संख्या 2021 में FORCE और FORD
द्वारा बेची गई कारों की संख्या से क्रमशः 16.66% और 18.18% कम है , तो 2020 में FORCE
और FORD द्वारा बेची गई कारों की कुल संख्या ज्ञात कीजजए।
(A) 820 (B) 904 (C) 767
(D) 612 (E) None of these

183. Number of cars sold by HONDA in 2021 is what percent of total number
of cars sold by FORD in both years?
2021 में होंडा द्वारा बेची गई कारों की संख्या िोनों वर्ों में िोडष द्वारा बेची गई कारों की कुल
संख्या का ककतना प्रनतशत है ?
(A) 48.96% (B) 42.15% (C) 44.36%
(D) 46.58% (E) None of these

184. Number of cars sold by FORD in 2022 is how much percent more or
less than the number of cars sold by HONDA in 2022?
2022 में FORD द्वारा बेची गई कारों की संख्या 2022 में HONDA द्वारा बेची गई कारों की
संख्या से ककतने प्रनतशत अचधक या कम है ?
(A) 69.23% (B) 67.45% (C) 68.12%
(D) 66.52% (E) None of these

185. Find the ratio of number of cars sold by FORD in 2021 to number of
cars sold by TOYOTA in 2022.
2021 में FORD द्वारा बेची गई कारों की संख्या का 2022 में TOYOTA द्वारा बेची गई कारों की
संख्या से अनुपात ज्ञात कीजजए।
(A) 6 : 5 (B) 8 : 3 (C) 4 : 5
(D) 7 : 9 (E) None of these

(186 - 190) Directions : Study the following chart carefully and answer the
questions given below.
The line graph given below shows the quantity of Peanut oil and quantity of
Soyabean oil produced in five different months.
ननिे श : ननम्नललखखत चाटष का ध्यानपव ू क
ष अध्ययन कीजजए और नीचे दिए गए प्रश्नों के उत्तर
िीजजए।
नीचे दिया गया लाइन ग्राि मूंगिली के तेल की मात्रा और पांच अलग-अलग महीनों में उत्पादित
सोयाबीन तेल की मात्रा को िशाषता है ।

https :
//www. https : https
https : //instagra
//youtube.c
facebo m.com/aas :
om/channe hisharoraso
l/UCYa4_Jr ok.com cial(?) //t.m
Orf8R5Kz2u
/aashis utm_mediu
e/stu

295
OtccXQ m=
haroras copy_link
ocial dified
186. Find the average quantity of Peanut oil produced in all the 5 months.
सभी 5 महीनों में उत्पादित मूंगिली के तेल की औसत मात्रा ज्ञात कीजजए।
(A) 774 (B) 756 (C) 742
(D) 768 (E) None of these

187. Quantity of Soyabean oil produced in May is what percent of total


quantity of Soyabean oil produced in all 5 months?
मई में उत्पादित सोयाबीन तेल की मात्रा सभी 5 महीनों में उत्पादित सोयाबीन तेल की कुल मात्रा
का ककतना प्रनतशत है ?
(A) 18.56% (B) 19.48% (C) 17.89%
(D) 16.41% (E) None of these

188. Quantity of Peanut oil and quantity of Soyabean oil produced in June is
respectively 7.69% and 25% more than quantity of Peanut oil and quantity of
Soyabean oil produced in May, then find the sum of quantity of Peanut oil
and quantity of Soyabean oil produced in June.
मूंगिली के तेल की मात्रा और जून में उत्पादित सोयाबीन तेल की मात्रा मई में उत्पादित मूंगिली
के तेल और सोयाबीन के तेल की मात्रा से क्रमशः 7.69% और 25% अचधक है , तो जून में
उत्पादित मूंगिली के तेल और सोयाबीन के तेल की मात्रा का योग ज्ञात कीजजए।
(A) 1870 (B) 1930 (C) 1750
(D) 1680 (E) None of these

189. Quantity of Coconut oil produced in January and February is


respectively 25% and 20% less than quantity of Peanut oil produced in
January and February, then find the sum of quantity of Coconut oil produced
in January and February.
जनवरी और िरवरी में उत्पादित नाररयल तेल की मात्रा जनवरी और िरवरी में उत्पादित
मूंगिली के तेल की मात्रा से क्रमशः 25% और 20% कम है , तो जनवरी और िरवरी में उत्पादित
नाररयल तेल की मात्रा का योग ज्ञात कीजजए।
(A) 1110 (B) 1280 (C) 1090
(D) 1320 (E) None of these

https :
//www. https : https
https : //instagra
//youtube.c
facebo m.com/aas :
om/channe hisharoraso
l/UCYa4_Jr ok.com cial(?) //t.m
Orf8R5Kz2u
/aashis utm_mediu
e/stu

296
OtccXQ m=
haroras copy_link
ocial dified
190. Quantity of Peanut oil produced in May is how much percent more or
less than quantity of Soyabean oil produced in February?
मई में उत्पादित मूंगिली के तेल की मात्रा िरवरी में उत्पादित सोयाबीन तेल की मात्रा से ककतने
प्रनतशत अचधक या कम है ?
(A) 28.13% (B) 22.78% (C) 24.15%
(D) 26.38% (E) None of these

(191 - 195) Directions : Study the following chart carefully and answer the
questions given below.
The table given below shows the number of LED TV sold by three different
shopkeepers of five different companies.
ननिे श : ननम्नललखखत चाटष का ध्यानपूवक
ष अध्ययन कीजजए और नीचे दिए गए प्रश्नों के उत्तर
िीजजए।
नीचे िी गई ताललका पांच अलग - अलग कंपननयों के तीन अलग - अलग िक ु ानिारों द्वारा बेचे
गए LED TV की संख्या िशाषती है ।

Total % of TV
Company TV sold by A : TV sold by B
TV sold by C
LG 380 4:3 30%

Sony 400 5:3 36%

Panasonic 350 3:4 42%

Samsung 360 5:4 35%

Haier 420 7:5 40%

191. Number of LED TV sold by A and B of MI is respectively 28.56% and


57.14% more than number of LED TV sold by A and B of Haier, then find the
total number of MI LED TV sold by A and B together.
MI के A और B द्वारा बेचे गए LED TV की संख्या क्रमशः 28.56% और 57.14% Haier के A
और B द्वारा बेचे गए LED TV की संख्या से अचधक है , तो A और B द्वारा लमलाकर बेचे गए MI
LED TV की कुल संख्या ज्ञात कीजजए।
(A) 346 (B) 332 (C) 354
(D) 361 (E) None of these

https :
//www. https : https
https : //instagra
//youtube.c
facebo m.com/aas :
om/channe hisharoraso
l/UCYa4_Jr ok.com cial(?) //t.m
Orf8R5Kz2u
/aashis utm_mediu
e/stu

297
OtccXQ m=
haroras copy_link
ocial dified
192. Number of LED TV of LG and Sony sold by D is respectively 16.66% and
12.5% more than number of LED TV of LG and Sony sold by C, then find the
sum of number of LED TV of LG and Sony sold by D.
D द्वारा बेचे गए LG और Sony के LED TV की संख्या, C द्वारा बेचे गए LG और Sony के
LED TV की संख्या से क्रमशः 16.66% और 12.5% अचधक है , तो D द्वारा बेचे गए LG और
Sony के LED TV की संख्या का योग ज्ञात कीजजए।
(A) 275 (B) 295 (C) 285
(D) 265 (E) None of these

193. Number of Panasonic LED TV sold by C is how much percent more or


less than number of Sony LED TV sold by B?
C द्वारा बेचे गए Panasonic LED TV की संख्या, B द्वारा बेचे गए Sony LED TV की
संख्या से ककतने प्रनतशत अचधक या कम है ?
(A) 53.12% (B) 51.08% (C) 54.16%
(D) 52.89% (E) None of these

194. Number of Samsung LED TV sold by C is what percent of total number


of LED TV sold by C of all five companies?
C द्वारा बेचे गए Samsung LED TV की संख्या, सभी पांच कंपननयों के C द्वारा बेचे गए
LED TV की कुल संख्या का ककतना प्रनतशत है ?
(A) 18.02% (B) 16.45% (C) 17.23%
(D) 15.37% (E) None of these

195. Find the average of number of LED TV sold by B of all five companies.
सभी पांच कंपननयों के B द्वारा बेचे गए LED TV की औसत संख्या ज्ञात कीजजए।
(A) 105 (B) 106 (C) 108
(D) 107 (E) None of these

(196 - 200) Directions : Study the following chart carefully and answer the
questions given below.
The bar chart given below shows the number of food orders delivered by
Zomato Within 3 km and above 3 km.
ननिे श : ननम्नललखखत चाटष का ध्यानपूवक
ष अध्ययन कीजजए और नीचे दिए गए प्रश्नों के उत्तर
िीजजए।
नीचे दिया गया बार चाटष Zomato द्वारा 3 ककमी के भीतर और 3 ककमी से अचधक के िूड ऑडषर
की संख्या को िशाषता है ।

https :
//www. https : https
https : //instagra
//youtube.c
facebo m.com/aas :
om/channe hisharoraso
l/UCYa4_Jr ok.com cial(?) //t.m
Orf8R5Kz2u
/aashis utm_mediu
e/stu

298
OtccXQ m=
haroras copy_link
ocial dified
196. Total number of food orders delivered by Swiggy on Thursday and
Friday is 16.66% and 14.28% more than total number of food orders
delivered by Zomato on Thursday and Friday, then find the total number of
food orders delivered by Swiggy on Thursday and Friday.
गुरुवार और शुक्रवार को जस्वगी द्वारा डडलीवर ककए गए िूड ऑडषर की कुल संख्या 16.66% और
Zomato द्वारा गुरुवार और शुक्रवार को डडलीवर ककए गए िूड ऑडषर की कुल संख्या से 14.28%
अचधक है , तो गरु
ु वार और शक्र
ु वार को जस्वगी द्वारा डडलीवर ककए गए िूड ऑडषर की कुल संख्या
ज्ञात कीजजए।
(A) 642 (B) 992 (C) 822
(D) 712 (E) None of these

197. Number of food orders delivered within 3 km and above 3 km on


Saturday is respectively 12.5% and 8.33% more than number of food orders
delivered within 3 km and above 3 km on Friday, then find the total number
of food orders delivered on Saturday.
शननवार को 3 ककमी के भीतर और 3 ककमी से अचधक के भीतर डडलीवर ककए गए िूड ऑडषर की
संख्या क्रमशः 12.5% और 8.33% अचधक है , जो शक्र
ु वार को 3 ककमी और 3 ककमी से अचधक के
भीतर डडलीवर ककए गए िूड ऑडषर की संख्या से अचधक है , तो शननवार को डडलीवर ककए गए िूड
ऑडषर की कुल संख्या ज्ञात कीजजए।
(A) 310 (B) 309 (C) 312
(D) 313 (E) None of these

198. Number of food orders delivered above 3 km on Thursday is how much


percent more or less than number of food orders delivered within 3 km on
Monday?
गुरुवार को 3 ककमी से ऊपर डडलीवर ककए गए िूड ऑडषर की संख्या, सोमवार को 3 ककमी के भीतर
डडलीवर ककए गए िूड ऑडषर की संख्या से ककतने प्रनतशत अचधक या कम है ?

https :
//www. https : https
https : //instagra
//youtube.c
facebo m.com/aas :
om/channe hisharoraso
l/UCYa4_Jr ok.com cial(?) //t.m
Orf8R5Kz2u
/aashis utm_mediu
e/stu

299
OtccXQ m=
haroras copy_link
ocial dified
(A) 73.45% (B) 72.22% (C) 74.89%
(D) 71.37% (E) None of these

199. Number of food orders delivered within 3 km on Thursday is what


percent of total number of food orders delivered within 3 km on all five
days?
गुरुवार को 3 ककमी के भीतर डडलीवर ककए गए िूड ऑडषर की संख्या सभी पांच दिनों में 3 ककमी के
भीतर डडलीवर ककए गए िूड ऑडषर की कुल संख्या का ककतना प्रनतशत है ?
(A) 21.06% (B) 28.96% (C) 26.45%
(D) 24.10% (E) None of these

200. Find the average of number of food orders delivered above 3 km on all
five days.
सभी पांच दिनों में 3 ककमी से ऊपर डडलीवर ककए गए खाने के ऑडषर की संख्या का औसत ज्ञात
कीजजए।
(A) 154 (B) 156 (C) 152
(D) 158 (E) None of these

(201 - 205) Directions : Study the following chart carefully and answer the
questions given below.
The line chart given below shows the marks obtained by Priya, Jiya and
Divya in five different subject. Maximum marks in each subject is 150.
ननिे श : ननम्नललखखत चाटष का ध्यानपूवक ष अध्ययन कीजजए और नीचे दिए गए प्रश्नों के उत्तर
िीजजए।
नीचे दिया गया लाइन चाटष पांच अलग-अलग ववर्यों में वप्रया, जजया और दिव्या द्वारा प्राप्त अंकों
को िशाषता है । प्रत्येक ववर्य में अचधकतम अंक 150 हैं।

https :
//www. https : https
https : //instagra
//youtube.c
facebo m.com/aas :
om/channe hisharoraso
l/UCYa4_Jr ok.com cial(?) //t.m
Orf8R5Kz2u
/aashis utm_mediu
e/stu

300
OtccXQ m=
haroras copy_link
ocial dified
201. What percentage of total marks did Priya score in all the subjects
together?
वप्रया ने सभी ववर्यों में लमलाकर कुल ककतने प्रनतशत अंक प्राप्त ककए?
(A) 76.2% (B) 75.3% (C) 78.8%
(D) 77.4% (E) None of these

202. Who scored the highest and the lowest marks in all the five subjects
together?
सभी पांच ववर्यों में लमलाकर सबसे अचधक और सबसे कम अंक ककसने प्राप्त ककए?
(A) Priya & Diya (B) Jiya & Diya (C) Diya & Priya
(D) Priya & Jiya (E) None of these

203. The marks scored by Jiya in English is what percent more or less than
the marks scored by Jiya in Math?
जजया द्वारा अंग्रेजी में प्राप्त अंक, जजया द्वारा गखणत में प्राप्त अंकों से ककतने प्रनतशत अचधक
या कम है ?
(A) 15% (B) 18% (C) 16%
(D) 17% (E) None of these

204. Find the average of marks obtained by Diya in all five subjects.
सभी पांच ववर्यों में िीया द्वारा प्राप्त अंकों का औसत ज्ञात कीजजए।
(A) 114 (B) 112 (C) 116
(D) 118 (E) None of these

205. If the marks obtained by Priya, Jiya and Diya in GK are respectively 10%
more, 10% less and 11.11% more than the marks obtained in Math
respectively, then what is the total marks obtained by all three in GK?
यदि GK में वप्रया, जजया और िीया द्वारा प्राप्त अंक क्रमशः गखणत में प्राप्त अंकों से क्रमशः
10% अचधक, 10% कम और 11.11% अचधक हैं, तो GK में तीनों द्वारा प्राप्त कुल अंक क्या हैं?
(A) 578 (B) 422 (C) 256
(D) 349 (E) None of these

(206 - 210) Directions : Study the following chart carefully and answer the
questions given below.
The Table given below shows the number of Table fans and number of
Ceiling fans sold by five different shopkeepers.
ननिे श : ननम्नललखखत चाटष का ध्यानपूवक
ष अध्ययन कीजजए और नीचे दिए गए प्रश्नों के उत्तर
िीजजए।
नीचे िी गई ताललका में पांच अलग-अलग िक ु ानिारों द्वारा बेचे गए टे बल पंखों और छत के पंखों
की संख्या को िशाषया गया है ।

https :
//www. https : https
https : //instagra
//youtube.c
facebo m.com/aas :
om/channe hisharoraso
l/UCYa4_Jr ok.com cial(?) //t.m
Orf8R5Kz2u
/aashis utm_mediu
e/stu

301
OtccXQ m=
haroras copy_link
ocial dified
Shopkeeper Table Fan : Ceiling Fan Total

P 5:3 576

Q 7:6 650

R 7:5 600

S 5:4 720

T 9:8 680

206. Find the average of number of Table fans sold by all five shopkeepers.
सभी पांच िक
ु ानिारों द्वारा बेचे गए टे बल पंखों की औसत संख्या ज्ञात कीजजए।
(A) 196 (B) 412 (C) 225
(D) 364 (E) None of these

207. Number of Table fans sold and Ceiling fans sold by M is


respectively 22.22% and 12.5% more than number of Table fan and
number of Ceiling fans sold by T, then find the number of Table fans
and number of Ceiling fans sold by M.
M द्वारा बेचे गए टे बल पंिों की संख्या और बेचे गए छत के पंिों की संख्या, T द्वारा बेचे गए
टे बल पंिों और छत के पंिों की संख्या से क्रमशः 22.22% और 12.5% अत्रधक है , तो M
द्वारा बेच गए टे बल पंिों की संख्या और छत के पंिों की संख्या ज्ञात कीजिए।
A) 700 (B) 800 (C) 900
(D) 600 (E) None of these

208. The ratio of number of Wall fans to number of Ceiling fans sold by Q
and R is respectively 7 : 6 and 6 : 5. Find the total number of Wall fans sold
by Q and R together.
Q और R द्वारा बेचे गए िीवार के पंखों की संख्या का छत के पंखों की संख्या से क्रमशः 7 : 6
और 6 : 5 का अनुपात है । Q और R द्वारा लमलाकर बेचे गए िीवार के पंखों की कुल संख्या ज्ञात
कीजजए।
(A) 610 (B) 650 (C) 630
(D) 670 (E) None of these

https :
//www. https : https
https : //instagra
//youtube.c
facebo m.com/aas :
om/channe hisharoraso
l/UCYa4_Jr ok.com cial(?) //t.m
Orf8R5Kz2u
/aashis utm_mediu
e/stu

302
OtccXQ m=
haroras copy_link
ocial dified
209. Number of Ceiling fans sold by P is how much percent more or less
than number of Table fans sold by S?
P द्वारा बेचे गए छत के पंखों की संख्या, S द्वारा बेचे गए टे बल पंखों की संख्या से ककतने
प्रनतशत अचधक या कम है ?
(A) 46% (B) 44% (C) 45%
(D) 43% (E) None of these

210. What is ratio of sum of number of Table fans sold by R and S to sum of
number of Ceiling fans sold by Q and R?
R और S द्वारा बेचे गए टे बल पंखों की संख्या के योग का Q और R द्वारा बेचे गए छत के पंखों
की संख्या के योग से अनुपात ककतना है ?
(A) 19 : 16 (B) 12 : 17 (C) 14 : 13
(D) 15 : 11 (E) None of these

(211 - 215) Directions : Study the following chart carefully and answer the
questions given below.
The line graph given below shows the number of PhonePe and Google Pay
users in five different cities.
ननिे श : ननम्नललखखत चाटष का ध्यानपूवक
ष अध्ययन कीजजए और नीचे दिए गए प्रश्नों के उत्तर
िीजजए।
नीचे दिया गया लाइन ग्राफ पाँच अलग - अलग शहरों में PhonePe और Google Pay
उपयोगकताषओं की संख्या दिखाता है ।

211. Number of PhonePe users and Google Pay users in city X is


respectively 40% and 12.5% more than number of PhonePe users and
Google Pay users in city P, than find the sum of number of Google Pay and
PhonePe users in city X.

https :
//www. https : https
https : //instagra
//youtube.c
facebo m.com/aas :
om/channe hisharoraso
l/UCYa4_Jr ok.com cial(?) //t.m
Orf8R5Kz2u
/aashis utm_mediu
e/stu

303
OtccXQ m=
haroras copy_link
ocial dified
शहर X में PhonePe उपयोगकताषओं और Google Pay उपयोगकताषओं की संख्या क्रमशः शहर
P में PhonePe उपयोगकताषओं और Google Pay उपयोगकताषओं की संख्या से 40% और
12.5% अचधक है , शहर X में Google Pay और PhonePe उपयोगकताषओं की संख्या का योग
ज्ञात कीजजए।
(A) 661 (B) 665 (C) 669
(D) 663 (E) None of these

212. Number of PhonePe users in city Q is what percent of total number of


users in all five cities together?
Q शहर में PhonePe उपयोगकताषओं की संख्या सभी पांच शहरों में लमलाकर उपयोगकताषओं की
कुल संख्या का ककतना प्रनतशत है ?
(A) 16.23% (B) 18.15% (C) 14.78%
(D) 12.04% (E) None of these

213. The ratio of male to female Google Pay users in city R and city S is
respectively 4 : 3 and 5 : 4, then find the sum of number male Google Pay
users in city R and city S.
शहर R और शहर S में Google Pay उपयोगकताषओं का पुरुर् से मदहला उपयोगकताषओं का
अनुपात क्रमशः 4 : 3 और 5 : 4 है , तो शहर R और शहर S में पुरुर् Google Pay
उपयोगकताषओं की संख्या का योग ज्ञात कीजजए।
(A) 294 (B) 298 (C) 296
(D) 292 (E) None of these

214. Number of Paytm users in city Q and T is respectively 18.18% and


15.38% more than number of PhonePe users in city Q and T, then find the
sum of number of Paytm users in city Q and T.
शहर Q और T में Paytm उपयोगकताषओं की संख्या शहर Q और T में PhonePe
उपयोगकताषओं की संख्या से क्रमशः 18.18% और 15.38% अचधक है , तो शहर Q और T में
Paytm उपयोगकताषओं की संख्या का योग ज्ञात कीजजए।
(A) 560 (B) 470 (C) 380
(D) 610 (E) None of these

215. Number of PhonePe users in city P is how much percent more or less
than number of PhonePe users in city R?
शहर P में PhonePe उपयोगकताषओं की संख्या, शहर R में PhonePe उपयोगकताषओं की
संख्या से ककतने प्रनतशत अचधक या कम है ?
(A) 34.23% (B) 38.89% (C) 36.45%
(D) 32.55% (E) None of these

https :
//www. https : https
https : //instagra
//youtube.c
facebo m.com/aas :
om/channe hisharoraso
l/UCYa4_Jr ok.com cial(?) //t.m
Orf8R5Kz2u
/aashis utm_mediu
e/stu

304
OtccXQ m=
haroras copy_link
ocial dified
(216 - 220) Directions : Study the following chart carefully and answer the
questions given below.
The bar graph given below shows the sum of number of bedsheets sold by A
and B and difference between number of bedsheets sold by A and B in five
different months.
ननिे श : ननम्नललखखत चाटष का ध्यानपूवक
ष अध्ययन कीजजए और नीचे दिए गए प्रश्नों के उत्तर
िीजजए।
नीचे दिया गया बार ग्राि पांच अलग - अलग महीनों में A और B द्वारा बेची गई चािरों की
संख्या और A और B द्वारा बेची गई चािरों की संख्या के बीच के अंतर को िशाषता है ।

216. Find the average of number of bedsheets sold by A in all five months
सभी पांच महीनों में A द्वारा बेची गई चािरों की औसत संख्या ज्ञात कीजजए।
(A) 493 (B) 461 (C) 478
(D) 486 (E) None of these

217. Number of bedsheets sold by B in March is what percent of total


number of bedsheets sold by B in all five months?
माचष में B द्वारा बेची गई चािरों की संख्या, सभी पाँच महीनों में B द्वारा बेची गई चािरों की कुल
संख्या का ककतना प्रनतशत है ?
(A) 22% (B) 21% (C) 23%
(D) 24% (E) None of these

218. The ratio of cotton to silk bedsheets sold by A and B in February is


respectively 5 : 3 and 6 : 5, then find the difference between number of
cotton bedsheets sold by A to number of cotton bedsheets sold by B.

https :
//www. https : https
https : //instagra
//youtube.c
facebo m.com/aas :
om/channe hisharoraso
l/UCYa4_Jr ok.com cial(?) //t.m
Orf8R5Kz2u
/aashis utm_mediu
e/stu

305
OtccXQ m=
haroras copy_link
ocial dified
िरवरी में A और B द्वारा बेची गई कपास की और रे शम की चािरों का अनुपात क्रमशः 5 : 3
और 6 : 5 है , तो A द्वारा बेची गई कपास की चािरों की संख्या का B द्वारा बेची गई कपास की
चािरों की संख्या से अंतर ज्ञात कीजजए।
(A) 110% (B) 120% (C) 120%
(D) 140% (E) None of these

219. Number of bedsheets sold by A and B in June is respectively 20% and


14.28% more than number of bedsheets sold by A and B in May, then find
the total number of bedsheets sold by A and B in June.
जनू में A और B द्वारा बेची गई चािरों की संख्या मई में A और B द्वारा बेची गई चािरों की
संख्या से क्रमशः 20% और 14.28% अचधक है , तो जून में A और B द्वारा बेची गई चािरों की
कुल संख्या ज्ञात कीजजए।
(A) 916 (B) 922 (C) 934
(D) 908 (E) None of these

220. Number of bedsheets sold by B in May is how much percent more or


less than total number of bedsheets sold by A and B in April?
मई में B द्वारा बेची गई चािरों की संख्या, अप्रैल में A और B द्वारा बेची गई चािरों की कुल
संख्या से ककतने प्रनतशत अचधक या कम है ?
(A) 52.13% (B) 54.78% (C) 56.14%
(D) 58.82% (E) None of these

(221 - 225) The data is regarding number of Bats and number of Balls sold
by five different shopkeepers.
ननिे श : ननम्नललखखत चाटष का ध्यानपव ू क
ष अध्ययन कीजजए और नीचे दिए गए प्रश्नों के उत्तर
िीजजए।
डेटा पांच अलग-अलग िक ु ानिारों द्वारा बेचे गए बल्लो और गें िों की संख्या के संबंध में है ।

https :
//www. https : https
https : //instagra
//youtube.c
facebo m.com/aas :
om/channe hisharoraso
l/UCYa4_Jr ok.com cial(?) //t.m
Orf8R5Kz2u
/aashis utm_mediu
e/stu

306
OtccXQ m=
haroras copy_link
ocial dified
221. The number of Bats and Balls sold by F is respectively 8.33% and
15.38% more than number of Bats and Balls sold by E, then find the total
number of items sold by F.
F द्वारा बेचे गए बल्लो और गें िों की संख्या क्रमशः E द्वारा बेचे गए बल्लो और गें िों की संख्या
से 8.33% और 15.38% अचधक है , तो F द्वारा बेची गई वस्तओ ु ं की कुल संख्या ज्ञात कीजजए।
(A) 763 (B) 772 (C) 784
(D) 758 (E) None of these

222. The ratio of number of Rubber Balls to Plastic Balls sold by C and A is
respectively 6 : 7 and 4 : 5, then find sum of number of Rubber Balls sold by
C and number of Plastic Balls sold by A.
C और A द्वारा बेची गई रबड की गें िों की संख्या का प्लाजस्टक की गें िों से अनुपात क्रमशः 6 : 7
और 4 : 5 है , तो C द्वारा बेची गई रबड की गें िों की संख्या और A द्वारा बेची गई प्लाजस्टक की
गें िों की संख्या का योग ज्ञात कीजजए।
(A) 460 (B) 420 (C) 450
(D) 440 (E) None of these

223. Find the average of number of Balls sold by A, B and C together.


A, B और C द्वारा लमलाकर बेची गई गें िों की औसत संख्या ज्ञात कीजजए।
(A) 388 (B) 372 (C) 391
(D) 364 (E) None of these

224. Number of Bats sold by C is what percent of total number of Bats sold
by all five shopkeepers?
C द्वारा बेचे गए बल्लो की संख्या सभी पांच िक
ु ानिारों द्वारा बेचे गए बल्लो की कुल संख्या का
ककतना प्रनतशत है ?
(A) 17% (B) 18% (C) 19%
(D) 16% (E) None of these

225. Number of Bats sold by C is what percent of total number of items sold
by D?
C द्वारा बेचे गए बल्लो की संख्या, D द्वारा बेची गई वस्तुओं की कुल संख्या का ककतना प्रनतशत
है ?
(A) 40% (B) 10% (C) 20%
(D) 33% (E) None of these

(226 - 230) Directions : Study the following chart carefully and answer the
questions given below.
The pie chart given below shows the number of dairy booths in five different
cities.

https :
//www. https : https
https : //instagra
//youtube.c
facebo m.com/aas :
om/channe hisharoraso
l/UCYa4_Jr ok.com cial(?) //t.m
Orf8R5Kz2u
/aashis utm_mediu
e/stu

307
OtccXQ m=
haroras copy_link
ocial dified
Total number of Dairy booths in 5 cities = 1600.
ननिे श : ननम्नललखखत चाटष का ध्यानपूवक ष अध्ययन कीजजए और नीचे दिए गए प्रश्नों के उत्तर
िीजजए।
नीचे दिया गया पाई चाटष पांच अलग-अलग शहरों में डेयरी बूथों की संख्या िशाषता है ।
5 शहरों में डेयरी बथ
ू ों की कुल संख्या = 1600।

226. Number of dairy booths in city A and B is respectively 8.33% and 12.5%
more than number of dairy booths in city S and T, then find the sum of
number of dairy booths in city A and B.
शहर A और B में डेयरी बथ
ू ों की संख्या शहर S और T में डेयरी बथ
ू ों की संख्या से क्रमशः 8.33%
और 12.5% अचधक है , तो शहर A और B में डेयरी बूथों की संख्या का योग ज्ञात कीजजए।
(A) 778 (B) 774 (C) 772
(D) 776 (E) None of these

227. Find the average of number of dairy booths in city P, Q and S.


शहर P, Q और S में डेयरी बूथों की औसत संख्या ज्ञात कीजजए।
(A) 318 (B) 336 (C) 324
(D) 334 (E) None of these

228. Number of dairy booths in city S is how much percent more or less than
number of dairy booths in city R?
शहर S में डेयरी बूथों की संख्या शहर R में डेयरी बूथों की संख्या से ककतने प्रनतशत अचधक या
कम है ?
(A) 43.56% (B) 41.17% (C) 42.58%
(D) 44.28% (E) None of these

https :
//www. https : https
https : //instagra
//youtube.c
facebo m.com/aas :
om/channe hisharoraso
l/UCYa4_Jr ok.com cial(?) //t.m
Orf8R5Kz2u
/aashis utm_mediu
e/stu

308
OtccXQ m=
haroras copy_link
ocial dified
229. The ratio of number of Amul dairy booths to other dairy booths in city Q
and R is respectively 3 : 5 and 8 : 9, then find the total number of Amul dairy
booths in city Q and R.
शहर Q और R में अन्य डेयरी बूथों से अमूल डेयरी बूथों की संख्या का अनुपात क्रमशः 3 : 5 और
8 : 9 है , तो शहर Q और R में अमलू डेयरी बथ
ू ों की कुल संख्या ज्ञात कीजजए।
(A) 254 (B) 242 (C) 226
(D) 268 (E) None of these

230. Number of dairy booths in city T is what percent of sum of number of


dairy booths in city Q and S?
शहर T में डेयरी बूथों की संख्या शहर Q और S में डेयरी बूथों की संख्या के योग का ककतना
प्रनतशत है ?
(A) 42.98% (B) 48.74% (C) 46.12%
(D) 44.44% (E) None of these

(231 - 235) Directions : Study the following chart carefully and answer the
questions given below.
The line graph given below shows the quantity of milk produced in five
different months in three different cities.
ननिे श : ननम्नललखखत चाटष का ध्यानपव
ू क
ष अध्ययन कीजजए और नीचे दिए गए प्रश्नों के उत्तर
िीजजए।
नीचे दिया गया लाइन ग्राि तीन अलग - अलग शहरों में पांच अलग - अलग महीनों में उत्पादित
िधू की मात्रा को िशाषता है ।

231. Find the average quantity of milk produced in city A in all five months.
सभी पांच महीनों में शहर A में उत्पादित िध
ू की औसत मात्रा ज्ञात कीजजए।
(A) 412 (B) 356 (C) 258
(D) 188 (E) None of these

https :
//www. https : https
https : //instagra
//youtube.c
facebo m.com/aas :
om/channe hisharoraso
l/UCYa4_Jr ok.com cial(?) //t.m
Orf8R5Kz2u
/aashis utm_mediu
e/stu

309
OtccXQ m=
haroras copy_link
ocial dified
232. Quantity of milk produced in city B in May is what percent of total
quantity of milk produced in city B in five months?
मई में शहर B में उत्पादित िध
ू की मात्रा पांच महीने में शहर B में उत्पादित िध
ू की कुल मात्रा का
ककतना प्रनतशत है ?
(A) 18.45% (B) 17.57% (C) 19.63%
(D) 16.42% (E) None of these

233. Quantity of milk produced in city C in July is how much percent more or
less than quantity of milk produced in city A in May?
जल ु ाई में शहर C में उत्पादित िध
ू की मात्रा मई में शहर A में उत्पादित िध
ू की मात्रा से ककतने
प्रनतशत अचधक या कम है ?
(A) 22.14% (B) 21.56% (C) 23.45%
(D) 24.74% (E) None of these

234. Quantity of milk produced in city A, B and C in March is respectively


14.28%, 12.5% and 8.33% more than quantity of milk produced in city A, B
and C in May, then find the total quantity of milk produced in city A, B and C
in March.
माचष में शहर A, B और C में उत्पादित िध ू की मात्रा क्रमशः शहर A, B और C में मई में
उत्पादित िधू की मात्रा से क्रमशः 14.28%, 12.5% और 8.33% अचधक है , तो माचष में शहर A,
B और C में उत्पादित िध ू की कुल मात्रा ज्ञात कीजजए।
(A) 867 (B) 855 (C) 836
(D) 841 (E) None of these

235. Quantity of milk produced in city D in August and September is


respectively 20% and 12.5% more than quantity of milk produced in city C in
August and September, then find the total quantity of milk produced in city
D in August and September.
अगस्त और लसतंबर में शहर D में उत्पादित िध ू की मात्रा अगस्त और लसतंबर में शहर C में
उत्पादित िध ू की मात्रा से क्रमशः 20% और 12.5% अचधक है , तो अगस्त और लसतंबर में शहर D
में उत्पादित िध
ू की कुल मात्रा ज्ञात कीजजए।
(A) 530 (B) 510 (C) 550
(D) 570 (E) None of these

(236 - 240) Directions : Study the following chart carefully and answer the
questions given below.
The bar graph given below shows the number of 1.5 ton and number of 2 ton
AC sold by five different companies.
ननिे श : ननम्नललखखत चाटष का ध्यानपूवक
ष अध्ययन कीजजए और नीचे दिए गए प्रश्नों के उत्तर
िीजजए।

https :
//www. https : https
https : //instagra
//youtube.c
facebo m.com/aas :
om/channe hisharoraso
l/UCYa4_Jr ok.com cial(?) //t.m
Orf8R5Kz2u
/aashis utm_mediu
e/stu

310
OtccXQ m=
haroras copy_link
ocial dified
नीचे दिया गया बार ग्राि पांच अलग - अलग कंपननयों द्वारा बेचे गए 1.5 टन की संख्या और 2
टन AC की संख्या को िशाषता है ।

236. Find the average of number of 1.5 ton AC sold by all five companies.
सभी पांच कंपननयों द्वारा बेचे गए 1.5 टन AC की औसत संख्या ज्ञात कीजजए।
(A) 394 (B) 368 (C) 372
(D) 386 (E) None of these

237. Number of 1.5 ton AC sold by SAMSUNG is how much percent more or
less than number of 2 ton AC sold by GODREJ?
SAMSUNG द्वारा बेचे गए 1.5 टन AC की संख्या, GODREJ द्वारा बेचे गए 2 टन AC की
संख्या से ककतने प्रनतशत अचधक या कम है ?
(A) 66% (B) 68% (C) 69%
(D) 67% (E) None of these

238. The ratio of number of AC sold by IFB to number of AC sold by


GODREJ of 1.5 ton and 2 ton is respectively 9 : 7 and 7 : 5, then find the total
number of AC sold by IFB of 1.5 ton and 2 ton.
IFB द्वारा बेचे गए AC की संख्या का GODREJ द्वारा 1.5 टन और 2 टन के बेचे गए AC की
संख्या से अनुपात क्रमशः 9 : 7 और 7 : 5 है , तो IFB द्वारा बेचे गए 1.5 टन और 2 टन AC की
कुल संख्या ज्ञात कीजजए।
(A) 600 (B) 800 (C) 700
(D) 900 (E) None of these

239. The ratio of 3 star to 5 star AC of 2 ton sold by DAIKIN and SAMSUNG is
respectively 3 : 5 and 3 : 4, then find the sum of number of 3 star 2 ton AC
sold by DAIKIN and 5 star 2 ton AC sold by SAMSUNG.
DAIKIN और SAMSUNG द्वारा बेचे गए 2 टन के 3 स्टार से 5 स्टार AC का अनुपात क्रमशः
3 : 5 और 3 : 4 है , तो DAIKIN द्वारा बेचे गए 3 स्टार 2 टन AC और SAMSUNG द्वारा बेचे
गए 5 स्टार 2 टन AC की संख्या का योग ज्ञात कीजजए।

https :
//www. https : https
https : //instagra
//youtube.c
facebo m.com/aas :
om/channe hisharoraso
l/UCYa4_Jr ok.com cial(?) //t.m
Orf8R5Kz2u
/aashis utm_mediu
e/stu

311
OtccXQ m=
haroras copy_link
ocial dified
(A) 290 (B) 280 (C) 260
(D) 270 (E) None of these

240. Number of 2 ton AC sold by DAIKIN is what percent of total number of 2


ton AC sold by all five companies?
DAIKIN द्वारा बेचे गए 2 टन AC की संख्या सभी पांच कंपननयों द्वारा बेचे गए 2 टन AC की
कुल संख्या का ककतना प्रनतशत है ?
(A) 17.23% (B) 15.89% (C) 14.52%
(D) 16.78% (E) None of these

(241 - 245) Directions : Study the following chart carefully and answer the
questions given below.
The table given below shows the data about number of bikes sold by three
different companies in five different years.
ननिे श : ननम्नललखखत चाटष का ध्यानपव ू क
ष अध्ययन कीजजए और नीचे दिए गए प्रश्नों के उत्तर
िीजजए।
नीचे िी गई ताललका पांच अलग - अलग वर्ों में तीन अलग - अलग कंपननयों द्वारा बेची गई
बाइकों की संख्या के बारे में डेटा िशाषती है ।

Company TVS Bikes : Honda Bikes % of Hero Bikes Total Bikes

2017 9:8 40% 680

2018 4:3 35% 700

2019 5:4 40% 720

2020 7:6 20% 650

2021 9:5 30% 840

241. Find the average of number of bikes sold by TVS in 2017, 2018, 2019
and 2020.
2017, 2018, 2019 और 2020 में TVS द्वारा बेची गई बाइक की औसत संख्या ज्ञात कीजजए।
(A) 247 (B) 245 (C) 249
(D) 243 (E) None of these

https :
//www. https : https
https : //instagra
//youtube.c
facebo m.com/aas :
om/channe hisharoraso
l/UCYa4_Jr ok.com cial(?) //t.m
Orf8R5Kz2u
/aashis utm_mediu
e/stu

312
OtccXQ m=
haroras copy_link
ocial dified
242. Number of bikes sold by Hero in 2019 is what percent of total number of
total number of bikes sold by Hero in all five years?
2019 में Hero द्वारा बेची गई बाइक की संख्या सभी पांच वर्ों में Hero द्वारा बेची गई बाइक
की कुल संख्या का ककतना प्रनतशत है ?
(A) 22.89% (B) 24.26% (C) 28.74%
(D) 26.96% (E) None of these

243. Number of bikes sold by TVS in 2021 is how much percent more than
number of bikes sold by Honda in 2021?
2021 में TVS द्वारा बेची गई बाइक की संख्या, 2021 में Honda द्वारा बेची गई बाइक की
संख्या से ककतने प्रनतशत अचधक है ?
(A) 70% (B) 80% (C) 60%
(D) 90% (E) None of these

244. Number of bikes sold by Honda and Hero in 2022 is respectively 10%
and 14.28% more than number of bikes sold by Honda and Hero in 2021,
then find the total number of bikes sold by Hero and Honda in 2022.
2022 में Honda और Hero द्वारा बेची गई बाइक की संख्या 2021 में Honda और Hero
द्वारा बेची गई बाइक की संख्या से क्रमशः 10% और 14.28% अचधक है , तो 2022 में Hero
और Honda द्वारा बेची गई बाइक की कुल संख्या ज्ञात कीजजए।
(A) 519 (B) 523 (C) 518
(D) 534 (E) None of these

245. Find the ratio between sum of number of bikes sold by TVS in 2018 and
2019 to sum of number of bikes sold by Honda in 2020 and 2021.
2018 और 2019 में TVS द्वारा बेची गई बाइक की संख्या का Honda द्वारा 2020 और 2021
में बेची गई बाइक की संख्या से अनुपात ज्ञात कीजजए।
(A) 15 : 16 (B) 13 : 14 (C) 11 : 12
(D) 10 : 9 (E) None of these

(246 - 270) Directions : Study the following chart carefully and answer the
questions given below.
The bar graph given below shows the number of jeans sold in three different
months by five different shopkeepers.
ननिे श : ननम्नललखखत चाटष का ध्यानपवू क
ष अध्ययन कीजजए और नीचे दिए गए प्रश्नों के उत्तर
िीजजए।
नीचे दिया गया बार ग्राि पांच अलग - अलग िक ु ानिारों द्वारा तीन अलग - अलग महीनों में बेची
गई जींस की संख्या को िशाषता है ।

https :
//www. https : https
https : //instagra
//youtube.c
facebo m.com/aas :
om/channe hisharoraso
l/UCYa4_Jr ok.com cial(?) //t.m
Orf8R5Kz2u
/aashis utm_mediu
e/stu

313
OtccXQ m=
haroras copy_link
ocial dified
246. Find the average of number of jeans sold by all five shopkeepers in
March.
माचष में सभी पांच िक
ु ानिारों द्वारा बेची गई जींस की औसत संख्या ज्ञात कीजजए।
(A) 742 (B) 735 (C) 712
(D) 728 (E) None of these

247. Number of jeans sold by A in January is what percent of total number of


jeans sold in January?
जनवरी में A द्वारा बेची गई जींस की संख्या, A द्वारा बेची गई जींस की कुल संख्या का ककतना
प्रनतशत है ?
(A) 11.56% (B) 14.87% (C) 12.89%
(D) 13.41% (E) None of these

248. Number of jeans sold by B in January is how much percent more or


less than number of jeans sold by B in February?
जनवरी में B द्वारा बेची गई जींस की संख्या िरवरी में B द्वारा बेची गई जींस की संख्या से
ककतने प्रनतशत अचधक या कम है ?
(A) 45.78% (B) 48.43% (C) 46.89%
(D) 47.12% (E) None of these

249. The ratio of number of girls jeans to boys jeans sold by E in January
and February is respectively 5 : 6 and 5 : 9, then find the total number of
boys jeans sold by E in January and February.
जनवरी और िरवरी में E द्वारा बेची गई लडककयों की जींस का लडकों की जींस से अनुपात
क्रमशः 5 : 6 और 5 : 9 है , तो जनवरी और िरवरी में E द्वारा बेची गई लडकों की जींस की कुल
संख्या ज्ञात कीजजए।
(A) 930 (B) 940 (C) 920
(D) 950 (E) None of these

https :
//www. https : https
https : //instagra
//youtube.c
facebo m.com/aas :
om/channe hisharoraso
l/UCYa4_Jr ok.com cial(?) //t.m
Orf8R5Kz2u
/aashis utm_mediu
e/stu

314
OtccXQ m=
haroras copy_link
ocial dified
250. Number of jeans sold by A and B in April is respectively 18.18% and
25% more than number of jeans sold by A and B in March, then find the sum
of number of jeans sold by A and B in April.
अप्रैल में A और B द्वारा बेची गई जींस की संख्या माचष में A और B द्वारा बेची गई जींस की
संख्या से क्रमशः 18.18% और 25% अचधक है , तो अप्रैल में A और B द्वारा बेची गई जींस की
संख्या का योग ज्ञात कीजजए।
(A) 1740 (B) 1770 (C) 1750
(D) 1760 (E) None of these

(251 - 255) Directions : Study the following chart carefully and answer the
questions given below.
A shopkeeper purchased some power banks and sold all of them. Everyday
he left with some unsold power banks which he sold on Saturday and
Sunday.
Note : He did not purchase anything on Saturday and Sunday.
(Sold power bank > Unsold power bank for all the days)
ननिे श : ननम्नललखखत चाटष का ध्यानपूवक ष अध्ययन कीजजए और नीचे दिए गए प्रश्नों के उत्तर
िीजजए।
एक िक ु ानिार ने कुछ पावर बैंक खरीिे और उन सभी को बेच दिया। हर दिन वह कुछ बबना बबके
पावर बैंक लेकर ननकलता था जजसे वह शननवार और रवववार को बेच िे ता था।
नोट : उसने शननवार और रवववार को कुछ भी नहीं खरीिा।
(बेचा गया पावर बैंक > पूरे दिन बबना बबका पावर बैंक)

251. What is the average number of sold power banks by the shopkeeper on
all the five days together?
िक
ु ानिार द्वारा सभी पांचों दिनों में बेचे गए पावर बैंकों की औसत संख्या ककतनी है ?
(A) 30 (B) 60 (C) 35
(D) 20 (E) None of these

252. Out of the total available power banks to sell on Saturday and

https :
//www. https : https
https : //instagra
//youtube.c
facebo m.com/aas :
om/channe hisharoraso
l/UCYa4_Jr ok.com cial(?) //t.m
Orf8R5Kz2u
/aashis utm_mediu
e/stu

315
OtccXQ m=
haroras copy_link
ocial dified
Sunday, shopkeeper could not sell 14 power banks. How many banks are
sold by him on Saturday and Sunday together?
शननवार और रवववार को बेचने के ललए कुल उपलब्ध पावर बैंक में से िक
ु ानिार 14 पावर बैंक नहीं
बेच सका। शननवार और रवववार को लमलाकर उसके द्वारा ककतने बैंक बेचे गए?
(A) 41 (B) 39 (C) 51
(D) 20 (E) None of these

253. What is the ratio of the difference between purchased power banks on
Tuesday and Friday to the total number of unsold power banks on Monday,
Wednesday and Thursday?
मंगलवार और शुक्रवार को खरीिे गए पावर बैंकों के बीच के अंतर का सोमवार, बुधवार और
गुरुवार को न बबके पावर बैंकों की कुल संख्या से अनुपात ककतना है ?
(A) 2 : 3 (B) 5 : 7 (C) 1 : 2
(D) 2 : 1 (E) None of these

254. If the ratio of the total sold power banks on Saturday and Sunday
together is 7 : 4, then what is the difference between the sold power banks
between Saturday and Tuesday? (Assuming he sold all the remaining power
banks on Saturday and Sunday)
यदि शननवार और रवववार को लमलाकर बेचे गए कुल पावर बैंकों का अनप ु ात 7 : 4 है , तो शननवार
और मंगलवार के बीच बेचे गए पावर बैंकों का अंतर ककतना है ? (यह मानते हुए कक उसने शननवार
और रवववार को शेर् सभी पावर बैंक बेच)े
(A) 15 (B) 5 (C) 10
(D) 20 (E) None of these

255. Total number of unsold power banks on Wednesday is what percent of


total number of purchased power banks on that day?
बुधवार को बबना बबके पावर बैंकों की कुल संख्या उस दिन खरीिे गए पावर बैंकों की कुल संख्या
का ककतना प्रनतशत है ?
(A) 50% (B) 60% (C) 55%
(D) 30% (E) None of these

(256 - 260) Directions : Study the following chart carefully and answer the
questions given below.
Some passengers want to travel from place A to place B and they have two
options while booking flights, wither they can book non - stop flight or flight
with at least on stop in five different airlines. Pie chart shows the number of
passengers who booked the non stop flights in various airlines.
Note : Number of passengers who booked with at least one stop in each
airline is 30 less than the number of passengers who booked non - stop
flights in the respective airlines.

https :
//www. https : https
https : //instagra
//youtube.c
facebo m.com/aas :
om/channe hisharoraso
l/UCYa4_Jr ok.com cial(?) //t.m
Orf8R5Kz2u
/aashis utm_mediu
e/stu

316
OtccXQ m=
haroras copy_link
ocial dified
ननिे श : ननम्नललखखत चाटष का ध्यानपूवक ष अध्ययन कीजजए और नीचे दिए गए प्रश्नों के उत्तर
िीजजए।
कुछ यात्री स्थान ए से स्थान बी तक यात्रा करना चाहते हैं और उनके पास फ्लाइट बुक करते समय
िो ववकल्प होते हैं, जजसमें वे पांच अलग-अलग एयरलाइनों में नॉन-स्टॉप फ्लाइट या कम से कम
स्टॉप वाली फ्लाइट बक ु कर सकते हैं। पाई चाटष उन याबत्रयों की संख्या दिखाता है जजन्होंने
ववलभन्न एयरलाइनों में नॉन स्टॉप फ़्लाइट बुक कीं।
नोट : प्रत्येक एयरलाइन में कम से कम एक स्टॉप के साथ बुक करने वाले याबत्रयों की संख्या
संबंचधत एयरलाइनों में नॉन-स्टॉप फ़्लाइट बुक करने वाले याबत्रयों की संख्या से 30 कम है ।

256. What is the ratio of the number of total booking in airline Q to the that in
airline T?
एयरलाइन Q में कुल बकु कंग की संख्या का एयरलाइन T में कुल बकु कंग से अनप
ु ात ककतना है ?
(A) 10 : 7 (B) 12 : 11 (C) 11 : 9
(D) 11 : 12 (E) None of these

257. What percentage of total passengers who booked flights in airline R is


the number of passengers who booked non - stop flights in the same
airline?
एयरलाइन R में फ्लाइट बुक करने वाले कुल याबत्रयों का प्रनतशत उसी एयरलाइन में नॉन-स्टॉप
फ्लाइट बुक करने वाले याबत्रयों की संख्या का ककतना प्रनतशत है ?
(A) 80% (B) 60% (C) 40%
(D) 50% (E) None of these

258. In airline Q, ratios of the number of males to that of females in non -


stop flights and that in flights with at least one stop are 7 : 5 and 3 : 2
respectively. What is the total number of male passengers who booked
tickets in that airline?

https :
//www. https : https
https : //instagra
//youtube.c
facebo m.com/aas :
om/channe hisharoraso
l/UCYa4_Jr ok.com cial(?) //t.m
Orf8R5Kz2u
/aashis utm_mediu
e/stu

317
OtccXQ m=
haroras copy_link
ocial dified
एयरलाइन Q में , नॉन-स्टॉप फ़्लाइट और कम से कम एक स्टॉप वाली फ़्लाइट में पुरुर्ों की संख्या
का मदहलाओं की संख्या से अनुपात क्रमशः 7 : 5 और 3 : 2 है । उस एयरलाइन में दटकट बुक
करने वाले पुरुर् याबत्रयों की कुल संख्या ककतनी है ?
(A) 195 (B) 250 (C) 163
(D) 190 (E) None of these

259. Out of the total passengers who booked flights in airline T, 1/6th booked
flights with more than one stop. How many passengers booked flights with
exactly one stop in that airline?
एयरलाइन T में फ़्लाइट बकु करने वाले कुल याबत्रयों में से, 1/6 ने एक से अचधक स्टॉप वाली
फ़्लाइट बुक कीं। ककतने याबत्रयों ने उस एयरलाइन में िीक एक स्टॉप वाली फ़्लाइट बुक की?
(A) 60 (B) 54 (C) 75
(D) 70 (E) None of these

260. Total passengers who booked non - stop flights in airlines R and S
together is what percentage of total passengers who booked flights in these
two airlines together?
एयरलाइन R और S में नॉन-स्टॉप फ़्लाइट बुक करने वाले कुल याबत्रयों की संख्या, इन िोनों
एयरलाइनों में लमलाकर फ़्लाइट बुक करने वाले याबत्रयों की कुल संख्या का ककतना प्रनतशत है ?
(A) 56.5% (B) 62.5% (C) 40%
(D) 36.5% (E) None of these

(261 - 265) Directions : Study the following chart carefully and answer the
questions given below.
Following bar graph represents the data regarding number of Hair Dryers
and sum of Hair Dryers and Hair Straighteners sold from different stores.
ननिे श : ननम्नललखखत चाटष का ध्यानपूवक ष अध्ययन कीजजए और नीचे दिए गए प्रश्नों के उत्तर
िीजजए।
ननम्नललखखत बार ग्राि हे यर ड्रायर की संख्या और ववलभन्न स्टोसष से बेचे गए हे यर ड्रायर और
हे यर स्ट्रे टनर के योग के बारे में डेटा का प्रनतननचधत्व करता है ।

https :
//www. https : https
https : //instagra
//youtube.c
facebo m.com/aas :
om/channe hisharoraso
l/UCYa4_Jr ok.com cial(?) //t.m
Orf8R5Kz2u
/aashis utm_mediu
e/stu

318
OtccXQ m=
haroras copy_link
ocial dified
261. What is the average number of Hair Dryers sold from store C, D and E
have taken together?
स्टोर C, D और E को लमलाकर बेचे गए हे यर ड्रायर की औसत संख्या ककतनी है ?
(A) 1150 (B) 900 (C) 1250
(D) 900 (E) None of these

262. The number of Hair Dryers sold from store B is approximately what
percent more/less than the number of Hair Straighteners sold from store E?
स्टोर B से बेचे गए हे यर ड्रायर की संख्या, स्टोर E से बेचे गए हे यर स्ट्रे टनर की संख्या से लगभग
ककतने प्रनतशत अचधक/कम है ?
(A) 44% (B) 35% (C) 47%
(D) 49% (E) None of these

263. In store C, If the price of each Hair Dryer is Rs. 350 and the price of each
Hair Straightener is Rs. 600, then find total earnings of store C on Hair
Dryers and Hair Straighteners.
स्टोर C में , यदि प्रत्येक हे यर ड्रायर की कीमत 350 रुपये है और प्रत्येक हे यर स्ट्रे टनर की कीमत
600 रुपये है , तो हे यर ड्रायर और हे यर स्ट्रे टनर पर स्टोर C की कुल आय ज्ञात कीजजए।
(A) Rs. 980000 (B) Rs. 1010000 (C) Rs. 972000
(D) Rs. 106400 (E) None of these

264. What is the respective ratio of the number of Hair Dryers sold from
store A and E together and the number of Hair Straighteners sold from store
B and E together?
स्टोर A और E से लमलाकर बेचे गए हे यर ड्रायर की संख्या और स्टोर B और E से लमलाकर बेचे
गए हे यर स्ट्रे टनर की संख्या का क्रमशः अनुपात ककतना है ?
(A) 3 : 2 (B) 6 : 5 (C) 4 : 3

https :
//www. https : https
https : //instagra
//youtube.c
facebo m.com/aas :
om/channe hisharoraso
l/UCYa4_Jr ok.com cial(?) //t.m
Orf8R5Kz2u
/aashis utm_mediu
e/stu

319
OtccXQ m=
haroras copy_link
ocial dified
(D) 2 : 5 (E) None of these

265. What is the difference between the number of Hair Dryers and Hair
Straighteners together sold from store A and number of Hair Straighteners
sold from store D?
स्टोर A से बेचे गए हे यर ड्रायर और हे यर स्ट्रे टनर की कुल संख्या और स्टोर D से बेचे गए हे यर
स्ट्रे टनर की संख्या के बीच ककतना अंतर है ?
(A) 1200 (B) 1600 (C) 1300
(D) 1500 (E) None of these

(266 - 270) Directions : Study the following chart carefully and answer the
questions given below.
The given table shows the data about the number of people who work in 5
NGOs. Some data are missing, answer accordingly.
ननिे श : ननम्नललखखत चाटष का ध्यानपव ू क
ष अध्ययन कीजजए और नीचे दिए गए प्रश्नों के उत्तर
िीजजए।
िी गई ताललका 5 NGO में काम करने वाले लोगों की संख्या के बारे में डेटा दिखाती है । कुछ
आँकडे लुप्त हैं, तिनुसार उत्तर िीजजए।
Total number Number of
NGO Male : Female
of people Females
A 1200 8:7 ___
B ___ 5:7 875
C ___ 11 : 5 250
D ___ 5:3 ___
E ___ 3:7 ___
266. Find the ratio of the number of females who work in NGO 'A' to the total
number of people who work in NGO 'C'.
NGO 'A' में काम करने वाली मदहलाओं की संख्या का NGO 'C' में काम करने वाले लोगों की
कुल संख्या से अनुपात ज्ञात कीजजए।
(A) 7 : 3 (B) 6 : 7 (C) 7 : 10
(D) 2 : 3 (E) None of these

267. If total number of people who work in NGO 'D' is 156% more than the
number of males who work in NGO 'B', then find the number of females who
work in NGO 'D'.
यदि NGO 'D' में काम करने वाले लोगों की कुल संख्या NGO 'B' में काम करने वाले पुरुर्ों की
संख्या से 156% अचधक है , तो NGO 'D' में काम करने वाली मदहलाओं की संख्या ज्ञात कीजजए।

https :
//www. https : https
https : //instagra
//youtube.c
facebo m.com/aas :
om/channe hisharoraso
l/UCYa4_Jr ok.com cial(?) //t.m
Orf8R5Kz2u
/aashis utm_mediu
e/stu

320
OtccXQ m=
haroras copy_link
ocial dified
(A) 700 (B) 600 (C) 1125
(D) 900 (E) None of these

268. If total number of people who work in NGO 'E' is 500 less than that in
NGO 'B', then find the difference between the number of females who work
in NGO 'B' and that in NGO 'E'.
यदि NGO 'E' में काम करने वाले लोगों की कुल संख्या NGO 'B' की तुलना में 500 कम है , तो
NGO 'B' और NGO 'E' में काम करने वाली मदहलाओं की संख्या के बीच अंतर ज्ञात कीजजए।
(A) 175 (B) 125 (C) 163
(D) 190 (E) None of these

269. Find the sum of the number of people who work in NGO 'A' and NGO
'C'.
NGO 'A' और NGO 'C' में काम करने वाले लोगों की संख्या का योग ज्ञात कीजजए।
(A) 6000 (B) 5000 (C) 2000
(D) 1700 (E) None of these

270. If the number of females who work in NGO 'D' is 600, then the number
of males who work in NGO 'A' is how much percent more/less than that in
NGO 'D'?
यदि NGO 'D' में काम करने वाली मदहलाओं की संख्या 600 है , तो NGO 'A' में काम करने वाले
पुरुर्ों की संख्या NGO 'D' की तुलना में ककतने प्रनतशत अचधक/कम है ?
(A) 25% (B) 36% (C) 42%
(D) 44% (E) None of these

(271 - 275) Directions : Study the following chart carefully and answer the
questions given below.
Below bar graph shows investment by three persons in different schemes
(in hundreds of Rs. ) during 2020.
ननिे श : ननम्नललखखत चाटष का ध्यानपव ू क
ष अध्ययन कीजजए और नीचे दिए गए प्रश्नों के उत्तर
िीजजए।
नीचे बार ग्राि 2020 के िौरान तीन व्यजक्तयों द्वारा ववलभन्न योजनाओं में ननवेश (सैकडों रुपये
में ) िशाषता है ।

https :
//www. https : https
https : //instagra
//youtube.c
facebo m.com/aas :
om/channe hisharoraso
l/UCYa4_Jr ok.com cial(?) //t.m
Orf8R5Kz2u
/aashis utm_mediu
e/stu

321
OtccXQ m=
haroras copy_link
ocial dified
271. What is the average investment (in Rs. ) by Jaya in all the schemes
taken together?
सभी योजनाओं को लमलाकर जया द्वारा औसत ननवेश (रुपये में ) ककतना है ?
(A) 34345 (B) 36845 (C) 30346
(D) 30346 (E) None of these

272. What is the ratio of investment by Raj in LIC and Reliance to that of
investment by Shekhar in Alliance and ICICI?
LIC और Reliance में राज द्वारा ननवेश का शेखर द्वारा Alliance और ICICI में ननवेश से
अनुपात ककतना है ?
(A) 41 : 36 (B) 42 : 37 (C) 14 : 13
(D) 21 : 22 (E) None of these

273. Which two schemes have same investment taken all three persons
together?
ककन िो योजनाओं में तीनों व्यजक्तयों का एक साथ समान ननवेश है ?
(A) Reliance and LIC (B) Reliance and Alliance (C) LIC and ICICI
(D) SIP and ICICI (E) None of these

274. If LIC and ICICI offer 5% and 6% interest per annum respectively, then
what is the difference between average interests (in Rs. ) earned for one year
by Raj and Jaya from both these schemes?
यदि LIC और ICICI क्रमशः 5% और 6% प्रनत वर्ष ब्याज प्रिान करते हैं, तो इन िोनों योजनाओं
से राज और जया द्वारा एक वर्ष के ललए अजजषत औसत ब्याज (रुपये में ) के बीच ककतना अंतर है ?
(A) 308 (B) 248 (C) 268
(D) 300 (E) None of these

https :
//www. https : https
https : //instagra
//youtube.c
facebo m.com/aas :
om/channe hisharoraso
l/UCYa4_Jr ok.com cial(?) //t.m
Orf8R5Kz2u
/aashis utm_mediu
e/stu

322
OtccXQ m=
haroras copy_link
ocial dified
275. Total investments by Raj in SIP, Alliance and Reliance is what
percentage of investment by Shekhar in LIC, Alliance and ICICI taken
together?
SPI, Alliance और Reliance में राज द्वारा कुल ननवेश LIC, Alliance और ICICI में शेखर
द्वारा कुल ननवेश का ककतना प्रनतशत है ?
(A) 104% (B) 108.25% (C) 106.8%
(D) 105.8% (E) None of these

(276 - 280) Directions : Study the following chart carefully and answer the
questions given below.
Below tabular data shows employees recruited from various companies
during 2015 and 2016.
ननिे श : ननम्नललखखत चाटष का ध्यानपूवक
ष अध्ययन कीजजए और नीचे दिए गए प्रश्नों के उत्तर
िीजजए।
नीचे सारणीबद्ध डेटा 2015 और 2016 के िौरान ववलभन्न कंपननयों से भती ककए गए कमषचाररयों
को िशाषता है ।
2015 2016
Company
Employee Male : Female Employee Male : Female

P 2850 3:2 3000 2:3

Q 2400 7:5 2700 2:1

R 3500 2:3 4500 4:5

S 3250 3:2 4050 5:4

T 3000 8:7 3600 5:7

U 3200 5:3 4500 3:2

276. What is the average number of employees recruited during 2016 taken
all the companies together?
सभी कंपननयों को लमलाकर 2016 के िौरान भती ककए गए कमषचाररयों की औसत संख्या ककतनी
है ?
(A) 3800 (B) 3500 (C) 3725
(D) 4500 (E) None of these

277. What is the ratio of male employees recruited during 2015 and 2016
from S and T taken together?
S और T से लमलाकर 2015 और 2016 के िौरान भती ककए गए पुरुर् कमषचाररयों का अनुपात
ककतना है ?
(A) 72 : 77 (B) 71 : 75 (C) 71 : 47
(D) 85 : 78 (E) None of these

https :
//www. https : https
https : //instagra
//youtube.c
facebo m.com/aas :
om/channe hisharoraso
l/UCYa4_Jr ok.com cial(?) //t.m
Orf8R5Kz2u
/aashis utm_mediu
e/stu

323
OtccXQ m=
haroras copy_link
ocial dified
278. Female employees recruited during 2016 from P and R taken together is
approximately what percent of female employees during 2015 from the same
companies?
2016 के िौरान P और R से भती की गई मदहला कमषचाररयों को लमलाकर समान कंपननयों से
2015 के िौरान मदहला कमषचाररयों का लगभग ककतना प्रनतशत है ?
(A) 132.7% (B) 124.8% (C) 130.5%
(D) 136.4% (E) None of these

279. If male employees from company S during 2017 are increased by 12%,
then what is the difference between male employees from company S from
2017 to 2015?
यदि 2017 के िौरान कंपनी S के पुरुर् कमषचाररयों में 12% की वद् र चध होती है , तो 2017 से 2015
तक कंपनी S के पुरुर् कमषचाररयों की संख्या में ककतना अंतर है ?
(A) 550 (B) 540 (C) 570
(D) 500 (E) None of these

280. Percentage increase in employees recruited from 2015 and 2016 is


second highest from which of the following company?
2015 और 2016 से भती ककए गए कमषचाररयों में प्रनतशत वद्
र चध ननम्न में से ककस कंपनी से
िस
ू री सबसे अचधक है ?
(A) Company S (B) Company R (C) Company P
(D) Company Q (E) None of these

(281 - 285) Directions : Study the following chart carefully and answer the
questions given below.
The given Pie chart shows the percentage distribution of the population of
five different villages of a block.
Note : Population of a village = Number of (males + females) in that village.
ननिे श : ननम्नललखखत चाटष का ध्यानपूवक
ष अध्ययन कीजजए और नीचे दिए गए प्रश्नों के उत्तर
िीजजए।
दिया गया पाई चाटष एक ब्लॉक के पांच अलग - अलग गांवों की जनसंख्या के प्रनतशत ववतरण को
िशाषता है ।
नोट : एक गाँव की जनसंख्या = उस गाँव में (पुरुर् + मदहलाएँ) की संख्या।

https :
//www. https : https
https : //instagra
//youtube.c
facebo m.com/aas :
om/channe hisharoraso
l/UCYa4_Jr ok.com cial(?) //t.m
Orf8R5Kz2u
/aashis utm_mediu
e/stu

324
OtccXQ m=
haroras copy_link
ocial dified
281. If the total population of all the given villages together is 1500, then find
the ratio of population of Ballasherpur to that of Gyanpur.
यदि दिए गए सभी गाँवों की कुल जनसंख्या लमलाकर 1500 है , तो बल्लाशेरपुर की जनसंख्या का
ज्ञानपुर की जनसंख्या से अनुपात ज्ञात कीजजए।
(A) 5 : 1 (B) 4 : 1 (C) 4 : 3
(D) 2 : 1 (E) None of these

282. The ratio of males to females in Dhampur is 3 : 2 and number of males


in Sadatnagar is 50% more than number of females in Sadatnagar. Total
number of males in Dhampur and Sadatnagar taken together are A and B
respectively. Which of the following can be the values of A and B?
धामपुर में पुरुर्षों का मरहलाओ ं से अनुपात 3 : 2 है और सदातनगर में पुरुर्षों की संख्या
सदातनगर में मरहलाओ ं की संख्या से 50% अत्रधक है । धामपुर और सदातनगर में
जमलाकर पुरुर्षों की कुल संख्या क्रमशः A और B है । जनम्नजलखित में से कौन सा A और
B का मान हो सकता है ?
I. 165, 110
II. 264, 176
III. 198, 124
IV. 297, 138
(A) Only I (B) Only I,IV (C) Only III,IV
(D) Only III (E) None of these

283. The population of Gyanpur is what percent more or less than the
population of Rampur?
ज्ञानपुर की जनसंख्या, रामपुर की जनसंख्या से ककतने प्रनतशत अचधक या कम है ?
(A) 20% (B) 55% (C) 75%
(D) 75% (E) None of these

https :
//www. https : https
https : //instagra
//youtube.c
facebo m.com/aas :
om/channe hisharoraso
l/UCYa4_Jr ok.com cial(?) //t.m
Orf8R5Kz2u
/aashis utm_mediu
e/stu

325
OtccXQ m=
haroras copy_link
ocial dified
284. Find the ratio of the difference between the population of Dhampur and
Rampur to the difference between the population of Sadatnagar and
Ballasherpur.
धामपुर और रामपुर की जनसंख्या के बीच के अंतर का सिातनगर और बल्लाशेरपुर की
जनसंख्या के अंतर से अनप
ु ात ज्ञात कीजजए।
(A) 4 : 1 (B) 2 : 3 (C) 5 : 1
(D) 4 : 3 (E) None of these

285. If the population Sadatnagar is 300, then find the population of


Dhampur.
यदि सिातनगर की जनसंख्या 300 है , तो धामपुर की जनसंख्या ज्ञात कीजजए।
(A) 300 (B) 400 (C) 40
(D) 360 (E) None of these

(286 - 290) Directions : Study the following chart carefully and answer the
questions given below.
Below tabular data shows the population details of six towns (in hundreds)
during 2016.
U = Urban population, R = Rural population
ननिे श : ननम्नललखखत चाटष का ध्यानपव
ू क
ष अध्ययन कीजजए और नीचे दिए गए प्रश्नों के उत्तर
िीजजए।
नीचे सारणीबद्ध डेटा 2016 के िौरान छह शहरों (सैकडों में ) की जनसंख्या का वववरण दिखाता है ।
U = शहरी आबािी, R = ग्रामीण आबािी

Town Population (in hundreds) U:R Literacy Rate

A 750 8:7 60%

B 800 5:3 80%

C 650 5:8 80%

D 750 3:2 90%

E 850 3:2 60%

F 600 5:3 80%


286. What is the ratio of Urban to Rural population taken of towns B and F
together?

https :
//www. https : https
https : //instagra
//youtube.c
facebo m.com/aas :
om/channe hisharoraso
l/UCYa4_Jr ok.com cial(?) //t.m
Orf8R5Kz2u
/aashis utm_mediu
e/stu

326
OtccXQ m=
haroras copy_link
ocial dified
शहरों B और F को लमलाकर शहरी से ग्रामीण आबािी का अनुपात ककतना है ?
(A) 5 : 3 (B) 4 : 3 (C) 5 : 1
(D) 1 : 3 (E) None of these

287. Literate population from towns A and E is approximately what


percentage less than that of literate population from towns C and D?
शहर A और E की साक्षर जनसँख्या शहर C और D की साक्षर जनसँख्या से लगभग ककतने
प्रनतशत कम है ?
(A) 25% (B) 20% (C) 45%
(D) 30% (E) None of these

288. If 60% of population from town D are female and out of which 80%
female live in Urban areas, then what percentage of male live in Rural areas?
यदि शहर D की जनसंख्या का 60% मदहलाएं हैं और जजनमें से 80% मदहलाएं शहरी क्षेत्रों में
रहती हैं, तो ग्रामीण क्षेत्रों में ककतने प्रनतशत परु
ु र् रहते हैं?
(A) 75% (B) 45% (C) 70%
(D) 60% (E) None of these

289. What is the approximate average of Urban population taken all towns
together (in hundreds)?
सभी नगरों को लमलाकर (सैकडों में ) शहरी जनसंख्या का अनुमाननत औसत क्या है ?
(A) 401 (B) 420 (C) 414
(D) 450 (E) None of these

290. If 20% and 25% of Rural population from towns A and B respectively
relocate to Urban area, then what is the approximate percentage change in
Urban population from towns A and B in 2017 compared to 2016?
यदि शहरों A और B से क्रमशः 20% और 25% ग्रामीण आबािी शहरी क्षेत्र में स्थानांतररत हो
जाती है , तो 2016 की तुलना में 2017 में शहरों A और B से शहरी आबािी में अनुमाननत प्रनतशत
पररवतषन ककतना है ?
(A) 18% (B) 16% (C) 20%
(D) 25% (E) None of these

(291 - 295) Directions : Study the following chart carefully and answer the
questions given below.
The following shows the angular distribution of students 1st, 2nd, 3rd, 4th
and 5th grades present on Monday.
Note : The total number of students present on Monday is 5400.
Total number of boys is 50% more than total number of girls in each grade.
ननिे श : ननम्नललखखत चाटष का ध्यानपव
ू क
ष अध्ययन कीजजए और नीचे दिए गए प्रश्नों के उत्तर
िीजजए।

https :
//www. https : https
https : //instagra
//youtube.c
facebo m.com/aas :
om/channe hisharoraso
l/UCYa4_Jr ok.com cial(?) //t.m
Orf8R5Kz2u
/aashis utm_mediu
e/stu

327
OtccXQ m=
haroras copy_link
ocial dified
ननम्नललखखत सोमवार को उपजस्थत पहली, िस ू री, तीसरी, चौथी और पांचवीं कक्षा के छात्रों के
कोणीय ववतरण को िशाषता है ।
नोट : सोमवार को उपजस्थत छात्रों की कुल संख्या 5400 है ।
प्रत्येक कक्षा में लडकों की कुल संख्या लडककयों की कुल संख्या से 50% अचधक है ।

291. Find the total number of students in 4th grade.


चौथी कक्षा में कुल छात्रों की संख्या ज्ञात कीजजए।
(A) 1000 (B) 500 (C) 1050
(D) 1450 (E) None of these

292. Find the ratio between the total number of boys in 1st grade and the
total number of girls in 4th grade.
पहली कक्षा में लडकों की कुल संख्या और चौथी कक्षा में लडककयों की कुल संख्या के बीच अनप
ु ात
ज्ञात कीजजए।
(A) 12 : 7 (B) 12 : 5 (C) 4 : 5
(D) 12 : 11 (E) None of these

293. Find the sum of the total number of boys present on Monday.
सोमवार को उपजस्थत लडकों की कुल संख्या का योग ज्ञात कीजजए।
(A) 3590 (B) 3240 (C) 4500
(D) 3646 (E) None of these

294. Find the difference between the total number of girls present in grade
1st and the total number of girls present in grade 5th.
कक्षा 1 में उपजस्थत लडककयों की कुल संख्या और कक्षा 5 में उपजस्थत लडककयों की कुल संख्या
के बीच का अंतर ज्ञात कीजजए।
(A) 35 (B) 39 (C) 30
(D) 25 (E) None of these

https :
//www. https : https
https : //instagra
//youtube.c
facebo m.com/aas :
om/channe hisharoraso
l/UCYa4_Jr ok.com cial(?) //t.m
Orf8R5Kz2u
/aashis utm_mediu
e/stu

328
OtccXQ m=
haroras copy_link
ocial dified
295. Find the average number of all the girls present on Monday.
सोमवार को उपजस्थत सभी लडककयों की औसत संख्या ज्ञात कीजजए।
(A) 492 (B) 430 (C) 480
(D) 432 (E) None of these

(296 - 300) Directions : Study the following chart carefully and answer the
questions given below.
Following table shows the number of employees working in two companies
in five departments and Ratio of male to female employees working in two
companies in five departments.
ननिे श : ननम्नललखखत चाटष का ध्यानपूवक ष अध्ययन कीजजए और नीचे दिए गए प्रश्नों के उत्तर
िीजजए।
ननम्नललखखत ताललका पांच ववभागों में िो कंपननयों में काम कर रहे कमषचाररयों की संख्या और
पांच ववभागों में िो कंपननयों में कायषरत पुरुर् से मदहला कमषचाररयों का अनुपात िशाषती है ।

Company A Company B
Department Number of Number of
Male : Female Male : Female
Employees Employees
IT 220 6:5 180 5:4
HR 135 5:4 120 5:1
Finance 145 15 : 14 125 13 : 12
Admin 128 17 : 15 165 6:5
Marketing 162 16 : 11 140 4:3
296. What is the average number of employees working in the Company B in
five departments together?
कंपनी B में पांच ववभागों में लमलाकर कायषरत कमषचाररयों की औसत संख्या ककतनी है ?
(A) 156 (B) 140 (C) 146
(D) 150 (E) None of these

297. The number of male employees working in HR and Finance


departments together in company A is what percentage more or less than
the number of female employees working in Admin and Marketing
departments together in company B?
कंपनी A में मानव संसाधन और ववत्त ववभागों में लमलाकर काम करने वाले पुरुर् कमषचाररयों की
संख्या, कंपनी B में प्रशासन और ववपणन ववभागों में लमलाकर काम करने वाली मदहला
कमषचाररयों की संख्या से ककतने प्रनतशत अचधक या कम है ?

https :
//www. https : https
https : //instagra
//youtube.c
facebo m.com/aas :
om/channe hisharoraso
l/UCYa4_Jr ok.com cial(?) //t.m
Orf8R5Kz2u
/aashis utm_mediu
e/stu

329
OtccXQ m=
haroras copy_link
ocial dified
(A) 115.5% (B) 11.11% (C) 09.09%
(D) 15% (E) None of these

298. What is the difference between the number of female employees


working in IT, HR and Finance departments together from company A and
the number of male employees working in Finance, Admin and Marketing
departments together from company B?
कंपनी A से IT, मानव संसाधन और ववत्त ववभागों में लमलाकर कायषरत मदहला कमषचाररयों की
संख्या और कंपनी B से ववत्त, प्रशासन और ववपणन ववभागों में लमलाकर कायषरत पुरुर्
कमषचाररयों की संख्या के बीच ककतना अंतर है ?
(A) 5 (B) 15 (C) 10
(D) 2 (E) None of these

299. What is the ratio of the number of male employees working in IT


department and Finance department together from both companies to the
number of female employees working in HR department and Admin
department together from both companies?
िोनों कंपननयों से IT ववभाग और ववत्त ववभाग में लमलाकर कायषरत पुरुर् कमषचाररयों की संख्या का
िोनों कंपननयों से मानव संसाधन ववभाग और प्रशासन ववभाग में लमलाकर कायषरत मदहला
कमषचाररयों की संख्या से अनपु ात ककतना है ?
(A) 71 : 43 (B) 73 : 33 (C) 72 : 43
(D) 62 : 43 (E) None of these

300. The number of employees working in Finance department from both


companies is what percentage of number of employees working in IT
department from both companies?
िोनों कंपननयों के ववत्त ववभाग में कायषरत कमषचाररयों की संख्या, िोनों कंपननयों के IT ववभाग में
कायषरत कमषचाररयों की संख्या का ककतना प्रनतशत है ?
(A) 65.5% (B) 67.5% (C) 66.6%
(D) 68.5% (E) None of these

https :
//www. https : https
https : //instagra
//youtube.c
facebo m.com/aas :
om/channe hisharoraso
l/UCYa4_Jr ok.com cial(?) //t.m
Orf8R5Kz2u
/aashis utm_mediu
e/stu

330
OtccXQ m=
haroras copy_link
ocial dified
(1 - 5)

Day ➔ Monday Tuesday Wednesday Thursday Friday Total


Zomato 270 480 450 400 350 1950
Swiggy 370 320 390 360 440 1880
Total 640 800 840 760 790 3830
1. Ans. (C)
Average of food delivered by Zomato = 1950/5 = 390

2. Ans. (B)
% of food orders delivered by Swiggy on Thursday = 360/1880× 100 =
19.14%

3. Ans. (A)
Required % = (480 - 270) /270 × 100 = 77.77%

4. Ans. (A)
Food orders delivered on Saturday = 350 × 8/7 + 440 × 12/11 = 400 + 480 =
880

5. Ans. (D)
Food orders delivered by Uber Eats on Wednesday & Tuesday
= 840/2 × 5/6 + 320 × 0.8 = 350 + 256 = 606

(6-10)

Month ➔ January February March April May Total


Keyboard sold 432 552 456 576 384 2400
6. Ans. (D)
Required % = (552 - 432) /432 × 100 = 27.77%

7. Ans. (B)
Wired keyboard sold in April = 576 × 7/12 = 336
Wired keyboard sold in May = 384 × 5/8 = 240
Difference = 336 - 240 = 96

https :
//www. https : https
https : //instagra
//youtube.c
facebo m.com/aas :
om/channe hisharoraso
l/UCYa4_Jr ok.com cial(?) //t.m
Orf8R5Kz2u
/aashis utm_mediu
e/stu

331
OtccXQ m=
haroras copy_link
ocial dified
8. Ans. (B)
Keyboards sold in June and July = 456 × 1.125 + 552 × 1.25 = 513 + 690 =
1203

9. Ans. (A)
Average = (432 + 552 + 456) /3 = 1440/3 = 480

10. Ans. (D)


Keyboards sold by Dell & Asus in January = 432 × (1.25 + 13/12) = 432 × 7/3 =
1008

(11 - 15)

Year➔ M N O P Q Total
January 342 318 350 375 353 1738
February 382 345 332 325 392 1776
March 315 365 389 378 387 1834
Total 1039 1028 1071 1078 1132 5348
11. Ans. (C)
% of Shirts sold by N in January = 318/1738 × 100 = 18.29%

12. Ans. (B)


Sum = 342 × 5/9 + 315 × 4/7 = 190 + 180 = 370

13. Ans. (A)


Ratio = (345 + 325) : (315 + 365) = 670 : 680 = 67 : 68

14. Ans. (A)


Shirts sold by M & N in April = 315 × 11/9 + 365 × 1.2 = 385 + 438 = 823

15. Ans. (D)


Required % = (378 - 318) /318 × 100 = 18.86%

https :
//www. https : https
https : //instagra
//youtube.c
facebo m.com/aas :
om/channe hisharoraso
l/UCYa4_Jr ok.com cial(?) //t.m
Orf8R5Kz2u
/aashis utm_mediu
e/stu

332
OtccXQ m=
haroras copy_link
ocial dified
(16 - 20)

Day ➔ Monday Tuesday Wednesday Thursday Friday Total


Jio 220 192 184 210 218 1024
Airtel 185 168 156 180 175 864
Total 405 360 340 390 393 1888
16. Ans. (D)
Average = (168 + 156 + 180) /3 = 504/3 = 168

17. Ans. (B)


Required % = 184/(220 + 192 + 184) × 100 = 184/596 × 100 = 30.87%

18. Ans. (B)


On Saturday, JioSim sold = 210 × 1.3 = 273
AirtelSim = 180 × 1.1 = 198
Difference = 273 - 198 = 75

19. Ans. (A)


Required % = (220 - 180) /180 × 100 = 22.22%

20. Ans. (D)


Vodafone Sim sold on Thursday & Friday = 180 × 1.4 + 175 × 9/7 = 252 + 225
= 477

(21-25)
Month ➔ January February March April Total
M 280 325 215 195 1015
N 185 287 320 240 1032
O 305 164 198 245 912
P 210 224 227 220 881
Total 980 1000 960 900 3760

https :
//www. https : https
https : //instagra
//youtube.c
facebo m.com/aas :
om/channe hisharoraso
l/UCYa4_Jr ok.com cial(?) //t.m
Orf8R5Kz2u
/aashis utm_mediu
e/stu

333
OtccXQ m=
haroras copy_link
ocial dified
21. Ans. (C)
Average person visited in February = (325 + 287 + 164 + 224) /4 = 1000/4 =
250

22. Ans. (B)


% of people visits city N in January = 185/1032 × 100 = 17.92%

23. Ans. (A)


Required % = (245 - 185) /185 × 100 = 32.43%

24. Ans. (A)


Person visited city M and city N in may = 195 × 1.2 + 240 × 1.375 = 234 + 330
= 564

25. Ans. (D)


Males visited city O and city P in April = 245 × 4/7 + 220 × 6/11 = 140 + 120 =
260

(26-30)
(2x - 1) + (x + 7) + 2x + (2x - 6) + (x + 4) = 100
8x + 4 = 100
8x = 96
x = 12
Month ➔ January February March April May Total
% of Chair manufactured 23 19 24 18 16 100
Chair manufactured 644 532 672 504 448 2800

26. Ans. (D)


Required % = (24 - 19) /19 × 100 = 26.31%

27. Ans. (A)


Chairs sold in February = (644 × 2/7 + 532) × 0.75 = (184 + 532) × 0.75 = 716 ×
0.75 = 537

https :
//www. https : https
https : //instagra
//youtube.c
facebo m.com/aas :
om/channe hisharoraso
l/UCYa4_Jr ok.com cial(?) //t.m
Orf8R5Kz2u
/aashis utm_mediu
e/stu

334
OtccXQ m=
haroras copy_link
ocial dified
28. Ans. (B)
Wooden Chairs manufactured in March = 672 × 5/8 = 420
Wooden Chairs manufactured in May = 448 × 4/7 = 256
Difference = 420 - 256 = 164

29. Ans. (A)


Ratio = 24 : (16 + 18) = 24 : 34 = 12 : 17

30. Ans. (D)


Chairs manufactured in June & July = 448 × (1.25 + 1.25 × 1.1)
= 448 × (1.25 + 1.375) = 448 × 2.625 = 1176

(31-35)
City➔ A B C D E Total
NEET 580 840 780 860 900 3960
JEE 920 670 590 700 820 3700
Total 1500 1510 1370 1560 1720 7660

31. Ans. (C)


Sum = 840 × 7/12 + 780 × 6/13 = 490 + 360 = 850

32. Ans. (B)


Students preparing for NEET & JEE in city F
= 900 × 1.23 + 820 × 1.1 = 1107 + 902 = 2009

33. Ans. (A)


Average of students preparing for NEET = 3960/5 = 792

34. Ans. (A)


% of students preparing for JEE in city A = 920/3700 × 100 = 24.86%

35.Ans. (D)
Ratio = (580 + 840) : (670 + 590) = 1420 : 1260 = 71 : 63

https :
//www. https : https
https : //instagra
//youtube.c
facebo m.com/aas :
om/channe hisharoraso
l/UCYa4_Jr ok.com cial(?) //t.m
Orf8R5Kz2u
/aashis utm_mediu
e/stu

335
OtccXQ m=
haroras copy_link
ocial dified
(36-40)

Day ➔ Monday Tuesday Wednesday Thursday Friday Total


Wireless 75 60 72 75 50 332
Wired 50 80 108 90 100 428
Total 125 140 180 165 150 760
36. Ans. (D)
Wired Earphones sold on Saturday = 90× 1.2 = 108
Wireless Earphones sold on Saturday = 75× 0.8 = 60
Difference = 108 - 60 = 48

37. Ans. (B)


Total Earphones sold by Vivo on Tuesday & Wednesday
= 140 × 9/7 + 180 × 11/9 = 180 + 220 = 400

38. Ans. (B)


Average = (50 + 80 + 90 + 100) /4 = 320/4 = 80

39. Ans. (A)


Required % = (165 - 125) /125× 100 = 32%

40. Ans. (D)


% of Wireless Earphones sold on Tuesday = 60/332 × 100 = 18.07%

(41-45)
Month ➔ January February March April May Total
Wheat 228 236 268 272 283 1287
Rice 240 245 234 226 265 1210
Total 468 481 502 498 548 2497

41. Ans. (C)


Average of Rice floor produced = 1210/5 = 242

42. Ans. (B)


Required % = (468 - 228) /468× 100 = 51.28%

https :
//www. https : https
https : //instagra
//youtube.c
facebo m.com/aas :
om/channe hisharoraso
l/UCYa4_Jr ok.com cial(?) //t.m
Orf8R5Kz2u
/aashis utm_mediu
e/stu

336
OtccXQ m=
haroras copy_link
ocial dified
43. Ans. (A)
Ratio = (268 + 272) : (234 + 226) = 540 : 460 = 27 : 23

44. Ans. (A)


Total Quantity of floor produced in June = 236 × 1.25 + 245 × 1.2 = 295 + 294
= 589

45. Ans. (D)


Revenue generated in May = 283 × 32 + 265 × 45 = 9056 + 11925 = Rs. 20981

(46-50)
School ➔ RPS NPS JPS KPS HPS Total
Boys 264 216 288 360 324 1452
Girls 168 168 216 216 180 948
Total 432 384 504 576 504 2400
46. Ans. (D)
Average = (384 + 504 + 576) /3 = 1464/3 = 488

47.Ans. (B)
Required % = 216/576 × 100 = 37.5%

48. Ans. (B)


Required % = (360 - 264) /360 × 100 = 26.66%

49. Ans. (A)


Sum = 324 × 5/9 + 180 × 7/12 = 180 + 105 = 285

50. Ans. (D)


Total Students in LBS = 360 × 10/9 + 216 × 13/12 = 400 + 234 = 634

https :
//www. https : https
https : //instagra
//youtube.c
facebo m.com/aas :
om/channe hisharoraso
l/UCYa4_Jr ok.com cial(?) //t.m
Orf8R5Kz2u
/aashis utm_mediu
e/stu

337
OtccXQ m=
haroras copy_link
ocial dified
(51 - 55)

Showroom ➔ A B C D Total
Sold 180 216 504 180 1080
Unsold 720 324 936 540 2520
Total 900 540 1440 720 3600

51. Ans. (A)


Ratio = (720 + 540) : 504 = 1260 : 504 = 5 : 2

52. Ans. (D)


Total Cars in E = 216 × 9/4 × 1/0.27 = 486/0.27 = 1800
Difference = 1800 - 1440 = 360

53. Ans. (C)


Unsold Cars in B, C & D = 324 + 936 + 540 = 1800

54. Ans. (B)


Required % = 216/(180 + 180) × 100 = 216/360 × 100 = 60%

55. Ans. (A)


Unsold Cars in F = 540 ×2 × 0.7 = 756
Difference = 756 - 720 = 36

(56-60)
Locality ➔ A B C D
CP 800 1000 1250 750
Profit P 12.5% 5% 8% 15%
Profit Q 10% 7.5% 10% 12%

https :
//www. https : https
https : //instagra
//youtube.c
facebo m.com/aas :
om/channe hisharoraso
l/UCYa4_Jr ok.com cial(?) //t.m
Orf8R5Kz2u
/aashis utm_mediu
e/stu

338
OtccXQ m=
haroras copy_link
ocial dified
56. Ans. (A)
Discount offered on Article A in shop P = 960 - 800 × 1.125 = 960 - 900 = 60
Rs.
Discount offered on Article B in shop Q = 1125 - 1000 × 1.075 = 1125 - 1075 =
50 Rs.
Sum = 60 + 50 = Rs. 110

57. Ans. (D)


MP of Article D in shop Q = 750 × 1.12/(0.8 × 0.84) = 840/(0.8 × 0.84) = Rs.
1250

58. Ans. (C)


Profit on Article E in shop Q = 1250 × 0.08 + 60 = 100 + 60 = Rs. 160
CP of Article E in shop Q = 1000 × 0.8 = Rs. 800
P% = 160/800 × 100 = 20%

59. Ans. (B)


Average SP of Article A& B in shop Q
= (800 × 1.1 + 1000 × 1.075) /2 = (880 + 1075) /2 = 1955/2 = 977.5 Rs.
Average SP of Article C & D in shop P
= (1250 × 1.08 + 750 × 1.15) /2 = (1350 + 862.5) /2 = 2212.5/2 = 1106.25 Rs.
Difference = 1106.25 - 977.5 = Rs. 128.75

60. Ans. (B)


Profit on Article A, B & C at shop P = 800 × 0.125 + 1000 × 0.05 + 1250 × 0.08
= 100 + 50 + 100 = Rs. 250

(61-65)
Document➔ A B C D E Total
Initial word count 960 720 800 1080 440 4000
Final word count 680 560 600 760 280 2880
Deleted word 280 160 200 320 160 1120

https :
//www. https : https
https : //instagra
//youtube.c
facebo m.com/aas :
om/channe hisharoraso
l/UCYa4_Jr ok.com cial(?) //t.m
Orf8R5Kz2u
/aashis utm_mediu
e/stu

339
OtccXQ m=
haroras copy_link
ocial dified
61. Ans. (C)
Average of deleted words in document A and E = (280 + 160) /2 = 440/2 = 220

62. Ans. (A)


Required % = (1080 - 200) /200 × 100 = 440%

63. Ans. (B)


Final words count in documents F = 1428 - 760 = 668

64. Ans. (C)


Ratio = 720 : 600 = 6 : 5

65. Ans. (D)


Meaningless words deleted from document A & E = 280 × 0.45 + 160 × 0.25 =
126 + 40 = 166

(66-70)

Year ➔ 2010 2011 2012 2013 2014 Total


Appeared 72000 40500 96000 62500 46800 317800
Qualified 52000 36000 56000 37500 36000 217500
Not Qualified 20000 4500 40000 25000 10800 100300

66. Ans. (C)


Required % = (36000 + 37500) /(40500 + 62500) × 100 = 73500/103000 × 100 =
71%

67. Ans. (B)


Average = (52000 + 56000 + 36000) /3 = 144000/3 = 48000

68. Ans. (A)


Ratio = 72000 : 36000 = 2 : 1

https :
//www. https : https
https : //instagra
//youtube.c
facebo m.com/aas :
om/channe hisharoraso
l/UCYa4_Jr ok.com cial(?) //t.m
Orf8R5Kz2u
/aashis utm_mediu
e/stu

340
OtccXQ m=
haroras copy_link
ocial dified
69. Ans. (C)
In 2013, Amount given to qualified students = 120 × 37500 = 4500000 = 45
lakh

70. Ans. (B)


Students appeared in 2012 are more than previous year by = 96000 - 40500 =
55500

(71-75)
Year➔ 2011 2012 2013 2014 2015 Total
Lee Jeans 4800 3600 2800 2400 3000 16600
Rapido Jeans 3200 4800 4400 4000 3200 19600
Pepe Jeans 4000 6000 5500 5000 4000 24500
Total 12000 14400 12700 11400 10200 60700

71. Ans. (C)


Pepe jeans sold in 2016 = 4000 × 11/16 = 2750
Difference = 2750 - 2400 = 350

72. Ans. (A)


Lee jeans sold in 2016 = 4800 × 0.5 = 2400

73. Ans. (B)


Ratio = 4800 : 5000 = 24 : 25

74. Ans. (C)


Average = (4000 + 5500) /2 = 9500/2 = 4750

75. Ans. (D)


Total jeans sold in 2017 = 2400 × 1.2 + 4000 × 1.5 + 5000 × 1.4 = 2880 + 6000 +
7000 = 15880

https :
//www. https : https
https : //instagra
//youtube.c
facebo m.com/aas :
om/channe hisharoraso
l/UCYa4_Jr ok.com cial(?) //t.m
Orf8R5Kz2u
/aashis utm_mediu
e/stu

341
OtccXQ m=
haroras copy_link
ocial dified
(76 - 80)
Academy➔ A B C D E Total
Kits manufactured 6600 4800 2500 4200 5400 23500
Kits distributed 5940 3600 2000 3570 3780 18890
Number of Students 22 18 8 17 21 86
Kits given to each student 270 200 250 210 180
Kits rejected 660 1200 500 630 1620 4610

76. Ans. (C)


Average = (6600 + 4800 + 5400) /3 = 16800/3 = 5600

77. Ans. (B)


Required % = (3780 - 3570) /3780 × 100 = 50/9%

78. Ans. (A)


Ratio = 270 : 210 = 9 : 7

79. Ans. (C)


Kits rejected by 5 academies = 4610

80. Ans. (B)


Kits received by each students of academy D = 3570/(17 + 4) = 3570/21 = 170
Difference = 250 - 170 = 80

(81-85)
Team➔ SRH KKR DC MI RR Total
Matches played 1500 600 900 1800 1200 6000

https :
//www. https : https
https : //instagra
//youtube.c
facebo m.com/aas :
om/channe hisharoraso
l/UCYa4_Jr ok.com cial(?) //t.m
Orf8R5Kz2u
/aashis utm_mediu
e/stu

342
OtccXQ m=
haroras copy_link
ocial dified
81. Ans. (C)
Matches played by KKR and SRH which are not declared as draw
= 600 × 0.8 + 1500 × 0.6 = 480 + 900 = 1380

82.Ans. (A)
Difference = 1200 × (3 - 2) /(3 + 2) = 1200 × 1/5 = 240

83. Ans. (B)


Matches played by MI & SRH = 1800 + 1500 = 3300
Matches played by DC & RR = 900 + 1200 = 2100
Difference = 3300 - 2100 = 1200

84. Ans. (C)


Matches lost by DC on Home ground = 900 × 0.75 × 0.2 = 135

85. Ans. (D)


Average = (900 + 12000 + 1800) /2 = 3900/3 = 1300

(86-90)

86. Ans. (C)


Participants in 1st tournament in Baseball = 53 - (15 + 18) = 53 - 33 = 20
Participants in 5th tournament in Baseball = 70 - (24 + 18) = 70 - 42 = 28
Average = (20 + 30 + 40 + 16 + 28) /5 = 134/5 = 26.8

87. Ans. (B)


Participants in 6th tournament in Football = x
18 + 1.75x + 30 + 1.2x + 18 + x = 145
3.95x = 145 - 66 = 79
x = 20
Participants in 2nd tournament = 25 + 20 × 1.75 + 30 = 55 + 35 = 90

88. Ans. (A)


Participants in 3rd tournament in Cricket = 90 - (30 + 40) = 90 - 70 = 20
Average participants in Cricket = (15 + 25 + 20 + 30 + 24 + 30) /6 = 144/6 = 24

https :
//www. https : https
https : //instagra
//youtube.c
facebo m.com/aas :
om/channe hisharoraso
l/UCYa4_Jr ok.com cial(?) //t.m
Orf8R5Kz2u
/aashis utm_mediu
e/stu

343
OtccXQ m=
haroras copy_link
ocial dified
89. Ans. (C)
Required % = (30 + 18) /(30 + 30) × 100 = 48/60 × 100 = 80%

90. Ans. (C)


Ratio = (30 + 40 + 16 + 30) : (15 + 25 + 30 + 24 + 30) = 116 : 124 = 29 : 31

(91 - 95)
Instrument ➔ Guitar Drum Flute Piano Sitar Mandolin Total
Male 1728 1980 2688 2112 1080 3900 13488
Female 1152 1620 1152 3168 1320 2100 10512
Total 2880 3600 3840 5280 2400 6000 24000

91. Ans. (C)


Average = (1152 + 3168 + 1320) /3 = 5640/3 = 1880

92. Ans. (A)


Required % = (1728 + 2688) /(1620 + 2100) × 100 = 4416/3720 × 100 = 118%

93. Ans. (B)


Ratio = (1152 + 3168) : (1980 + 1080) = 4320 : 3060 = 24 : 17

94. Ans. (C)


Males who learn to play Mandolin and Piano = 3900 + 2112 = 6012
Females who learn to play Drum and Sitar = 1620 + 1320 = 2940
Difference = 6012 - 2940 = 3072

95. Ans. (D)


Males who learn Flute and Mandolin = 2688 + 3900 = 6588
Female who learn Sitar and Guitar = 1320 + 1152 = 2472
Required % = (6588 - 2472) /2472 × 100 = 166%

(96 - 100)
Total Maximum Marks = 150 × 2 + 100 × 2 = 300 + 200 = 500

96. Ans. (C)


Marks of Bhanu in Hindi = 500 × 0.83 - (70 + 140 + 145) = 415 - 355 = 60

https :
//www. https : https
https : //instagra
//youtube.c
facebo m.com/aas :
om/channe hisharoraso
l/UCYa4_Jr ok.com cial(?) //t.m
Orf8R5Kz2u
/aashis utm_mediu
e/stu

344
OtccXQ m=
haroras copy_link
ocial dified
97. Ans. (B)
Marks of Aditi in English = 150 × 0.8/1.5 = 80

98.Ans. (A)
Marks of Diksha in Hindi = (305 - 75 - 80 - 40) × 5/11 = (305 - 195) × 5/11 = 110
× 5/11 = 50

99. Ans. (C)


Marks of Bhanu in Science & Math = 145 + 140 = 285
Marks of Diksha in Math & English = 135 + 80 = 215
Difference = 285 - 215 = 70

100. Ans. (B)


Marks scored by Eshan in Math = x
135 + 140 + (x + 43) + 135 + x = 563
2x = 563 - 453 = 110
x = 55

(101-105)
Bag➔ A B C D E Total
Rs. 5 coins 32 38 26 44 18 158
Rs. 10 coins 18 22 14 26 12 92
Total 50 60 40 70 30 250

101. Ans. (C)


Worth of denomination in Bag C = 26 × 5 + 14 × 10 = 130 + 140 = Rs. 270
Worth of denomination in Bag E = 18 × 5 + 12 × 10 = 90 + 120 = Rs. 210
Ratio = 270 : 210 = 9 : 7

102. Ans. (A)


Worth of Rs. 10 denomination in Bag D = 26 × 10 = Rs. 260
Worth of Rs. 5 denomination in Bag A = 32 × 5 = Rs. 160
Required % = (260 - 160) /160 × 100 = 62.5%

https :
//www. https : https
https : //instagra
//youtube.c
facebo m.com/aas :
om/channe hisharoraso
l/UCYa4_Jr ok.com cial(?) //t.m
Orf8R5Kz2u
/aashis utm_mediu
e/stu

345
OtccXQ m=
haroras copy_link
ocial dified
103. Ans. (B)
Average = (22 + 14 + 12) /3 = 48/3 = 16

104. Ans. (C)


Bags, in which Rs. 5 denomination coins is in between 30 to 40 = 2

105. Ans. (D)


Rs. 5 denomination coins in Bag A, C & E = 32 + 26 + 18 = 76
Rs. 10 denomination coins in bag A, C & E = 18 + 14 + 12 = 44
Difference = 76 - 44 = 32

(106 - 110)

Players➔ Virat Shikhar Rohit Rahul Pant Total


Total Runs 2880 2160 3040 1760 704 10544
Runs scored by Sixes 384 216 420 228 180 1428
Runs scored in Fours 480 376 432 192 204 1684
Run scored from boundaries 864 592 852 420 384 3112
Runs scored other than boundaries 2016 1568 2188 1340 320 7432

106. Ans. (C)


Difference = 7432 - (1428 + 1684) = 7432 - 3112 = 4320

107. Ans. (B)


Runs scored by Virat and Shikhar other than boundaries = 2016 + 1568 =
3584
Runs scored by Rohit, Rahul and Pant from boundaries = 852 + 420 + 384 =
1656
Difference = 3584 - 1656 = 1928

108. Ans. (A)


Difference = 1684 - 1428 = 256

109. Ans. (C)


% of Runs scored by Pant from boundaries = 384/704 × 100 = 54(6/11)%

https :
//www. https : https
https : //instagra
//youtube.c
facebo m.com/aas :
om/channe hisharoraso
l/UCYa4_Jr ok.com cial(?) //t.m
Orf8R5Kz2u
/aashis utm_mediu
e/stu

346
OtccXQ m=
haroras copy_link
ocial dified
110. Ans. (B)
Difference = 228 - 192 = 36

(111 - 115)

Coaching➔ P Q R S T Total
Students 300 200 400 250 100 1250

111. Ans. (C)


Average = (300 + 200 + 250) /3 = 750/3 = 250

112. Ans. (A)


Girls students preparing from institute R = 400 × 3/8 = 150

113. Ans. (B)


Students preparing from Institute F = 200/0.125 - 1250 = 1600 - 1250 = 350

114. Ans. (C)


Ratio = (200 - 150) : (250 - 150) = 50 : 100 = 1 : 2

115. Ans. (D)


Total Students = 1250 + 250 = 1500
% of Students in institute P = 300/1500 × 100 = 20%

(116-120)
116. Ans. (C)
Unsold Notebooks of company C & E
= 16200 × 1.25 × 0.1 + 13600 × 1.35 × 0.25 = 2025 + 4590 = 6615

117. Ans. (B)


Ratio = (15300 × 0.8) : (12600 × 0.75) = 12240 : 9450 = 136 : 105

118. Ans. (A)


Unsold Notebooks of company E = 12600 × 2 × 3/7 × 0.25 = 2700
Unsold Notebooks of company F = 12600 × 2 × 4/7 × 0.2 = 2880
Difference = 2880 - 2700 = 180

https :
//www. https : https
https : //instagra
//youtube.c
facebo m.com/aas :
om/channe hisharoraso
l/UCYa4_Jr ok.com cial(?) //t.m
Orf8R5Kz2u
/aashis utm_mediu
e/stu

347
OtccXQ m=
haroras copy_link
ocial dified
119. Ans. (C)
Average of Notebooks sold by company C, D && E = 16200/3× (8/9 × 0.9 +
0.75 + 7/9 × 0.75)
= 5400× (0.8 + 0.75 × 1.75/3) = 5400×32/15 = 11520

120. Ans. (B)


Notebooks sold by A & B = 15300 × 0.8 + 12600 × 0.5 = 12240 + 6300 = 18540
Notebooks sold by F & G = 12400 × 0.8 + 13600 × 0.95 = 9920 + 12920 =
22840
Required % = (22840 - 18540) /22840 × 100 = 18.82%

(121-125)

Shop ➔ A B C D E Total
Samsung 60 75 40 30 50 255
Nokia 40 60 25 20 40 185
Total 100 135 65 50 90 440
121. Ans. (C)
Required % = (135 - 90) /90 × 100 = 50%

122. Ans. (A)


In shop A, C & E, Samsung phones are more than Nokia phones by = 20 + 15
+ 10 = 45

123. Ans. (B)


Ratio = 65 : 50 = 13 : 10

124. Ans. (C)


Average of Samsung phones sold by shop A, B & D = (60 + 75 + 30) /3 =
165/3 = 55
Average of Nokia phones sold by shop B, D & E = (60 + 20 + 40) /3 = 120/3 =
40
Difference = 55 - 40 = 15

125. Ans. (D)


Samsung phones sold by A, B & C = 60 + 75 + 40 = 175

https :
//www. https : https
https : //instagra
//youtube.c
facebo m.com/aas :
om/channe hisharoraso
l/UCYa4_Jr ok.com cial(?) //t.m
Orf8R5Kz2u
/aashis utm_mediu
e/stu

348
OtccXQ m=
haroras copy_link
ocial dified
Nokia phones sold by C, D & E = 25 + 20 + 40 = 85
Difference = 175 - 85 = 90

(126-130)
Ward ➔ A B C Total
P 3600 4500 4200 12300
Q 4400 1800 4800 11000
R 1600 3200 1600 6400
Total 9600 9500 10600 29700
126. Ans. (C)
Average of votes received by P = 12300/3 = 4100

127. Ans. (B)


Total voters in ward C = 10600

128. Ans. (A)


Ratio = 1600 : 1600 = 1 : 1

129. Ans. (C)


Votes of Candidates who received maximum votes in Ward B = 4500

130. Ans. (C)


Ward in which candidate Q received maximum votes = Both A& C

(131-135)
Type➔ P Q R S T Total
Sold 324 264 240 900 1224 2952
Unsold 396 216 720 300 216 1848
Manufactured 720 480 960 1200 1440 4800

https :
//www. https : https
https : //instagra
//youtube.c
facebo m.com/aas :
om/channe hisharoraso
l/UCYa4_Jr ok.com cial(?) //t.m
Orf8R5Kz2u
/aashis utm_mediu
e/stu

349
OtccXQ m=
haroras copy_link
ocial dified
131. Ans. (C)
Ratio = 324 : 216 = 3 : 2

132. Ans. (A)


Difference = 396 - 264 = 132

133. Ans. (B)


Unsold units of type S & type T = 300 + 216 = 516
Required % = (516 - 240) /240 × 100 = 115%

134. Ans. (C)


Required % = 720/900 × 100 = 80%

135. Ans. (D)


Difference = 1224 - 324 = 900

(136-140)

College ➔ P Q R S T Total
BCA 81 1152 147 1750 400 3530
DCA 162 1728 147 2750 300 5087
MCA 243 960 294 500 300 2297
Total 486 3840 588 5000 1000 10914

136. Ans. (C)


Required % = 5000/1000 × 100 = 500%

137. Ans. (B)


Students studying in BCA from college R & S = 147 + 1750 = 1897
Students studying in MCA from college P & R = 243 + 294 = 537
Difference = 1897 - 537 = 1360

138. Ans. (A)


Required % = (2750 - 960) /960 × 100 = 186.45% = 187%

139. Ans. (C)

https :
//www. https : https
https : //instagra
//youtube.c
facebo m.com/aas :
om/channe hisharoraso
l/UCYa4_Jr ok.com cial(?) //t.m
Orf8R5Kz2u
/aashis utm_mediu
e/stu

350
OtccXQ m=
haroras copy_link
ocial dified
Boys studying DCA in college P and Q = 162 × 2/3 + (1728 - 988) = 108 + 740
= 848

140. Ans. (B)


Average of Students studying in BCA = 3530/5 = 706

(141 - 145)

Year ➔ 2014 2015 2016 2017 2018 Total


Expenditure 80 75 120 150 200 625
Revenue 100 120 150 180 220 770
Profit 20 45 30 30 20 145
Profit % 25% 60% 25% 20% 10%

141. Ans. (C)


Profit % is greatest in = 2015

142. Ans. (A)


Profit % is least in = 2018

143. Ans. (B)


Ratio = 45 : 30 = 3 : 2

144. Ans. (C)


Profit in years 2014 & 2018 = 20 + 20 = 40
Required % = 30/40× 100 = 75%

145. Ans. (D)


Profit in 2019 = 220 × 1.45 - 200 × 1.55 = 319 - 310 = 9
% decrease in Profit from 2018 to 2019 = (20 - 9) /20 × 100 = 55%

(146-150)

https :
//www. https : https
https : //instagra
//youtube.c
facebo m.com/aas :
om/channe hisharoraso
l/UCYa4_Jr ok.com cial(?) //t.m
Orf8R5Kz2u
/aashis utm_mediu
e/stu

351
OtccXQ m=
haroras copy_link
ocial dified
Locality ➔ M N P Q R Total
Blue Houses 41 15 36 28 35 155
Brown Houses 29 25 32 52 42 180
Pink Houses 30 12 54 33 16 145
Yellow Houses 37 40 27 21 31 156
Total 137 92 149 134 124 636

146 Ans. (C)


% of Brown houses in locality Q = 52/134× 100 = 38.80% = 39%

147. Ans. (A)


Ratio = (15 + 40) : (42 + 16) = 55 : 58

148. Ans. (A)


Brown houses in localities M & P = 29 + 32 = 61
Yellow & Blue houses in locality Q = 21 + 28 = 49
Difference = 61 - 49 = 12

149. Ans. (C)


Brown houses in localities N & R = 25 + 42 = 67

150. Ans. (B)


Yellow houses in localities M & Q = 37 + 21 = 58
House in locality P which are not Yellow or Pink = 149 - 81 = 68
Difference = 68 - 58 = 10 less

(151-155)
City ➔ A B C D E Total
Literate 2128 1872 2646 1525 2030 10201
Illiterate 1672 1728 1554 1525 1470 7949
Total 3800 3600 4200 3050 3500 18150

https :
//www. https : https
https : //instagra
//youtube.c
facebo m.com/aas :
om/channe hisharoraso
l/UCYa4_Jr ok.com cial(?) //t.m
Orf8R5Kz2u
/aashis utm_mediu
e/stu

352
OtccXQ m=
haroras copy_link
ocial dified
151. Ans. (C)
In city F, Literate persons = 2030 × 1.2 = 2436
Illiterate persons = 1470 × 0.8 = 1176
Difference = 2436 - 1176 = 1260

152. Ans. (B)


In city B, Literate Males = 1872 × 5/9 = 1040
Illiterate Females = 1728 × 5/12 = 720
Difference = 1040 - 720 = 320

153. Ans. (A)


Average = (1728 + 1554 + 1470) /3 = 4752/3 = 1584

154. Ans. (A)


% of Illiterate persons in city A = 1672/7949 × 100 = 21.03%

155. Ans. (D)


Required % = (2646 - 1872) /1872 × 100 = 41.34%

(156-160)

Shopkeeper ➔ P Q R S T Total
Dupattas 960 1240 1350 1180 1070 5800
Comforters 1320 1080 980 1240 1120 5740
Quilts 1200 1150 1040 1380 950 5720
Total 3480 3470 3370 3800 3140 17260
156. Ans. (D)
Average of Dupattas sold = 5800/5 = 1160

157. Ans. (D)


Required % = 1320/(980 + 1240 + 1120) × 100 = 1320/3340 × 100 = 39.52%

158. Ans. (B)

https :
//www. https : https
https : //instagra
//youtube.c
facebo m.com/aas :
om/channe hisharoraso
l/UCYa4_Jr ok.com cial(?) //t.m
Orf8R5Kz2u
/aashis utm_mediu
e/stu

352
OtccXQ m=
haroras copy_link
ocial dified

3
Silk Quilts sold by R = 1040 × 8/13 = 640
Woolen Quilts sold by S = 1380 × 2/5 = 552
Difference = 6400 - 552 = 88

159. Ans. (A)


Ratio = (980 + 1120) : (1150 + 950) = 2100 : 2100 = 1 : 1

160. Ans. (D)


Total items sold by M = 1180 × 1.1 + 1240 × 1.15 + 1380 × 0.9
= 1298 + 1426 + 1242 = 3966

(161-165)

City➔ Pune Delhi Mumbai Agra Chennai Total


April 600 550 850 670 720 3390
May 780 820 980 740 590 3910
June 920 730 680 910 870 4110
Total 2300 2100 2510 2320 2180 11410

161. Ans. (C)


Average person who visited Mumbai = 2100/3 = 700

162. Ans. (B)


% of persons who visited Chennai in May = 590/3910 × 100 = 15.08%

163. Ans. (A)


Required % = (980 - 740) /740 × 100 = 32.43%

164. Ans. (A)


Persons who visits Mumbai & Agra in July = 680 × 1.2 + 910 × 15/13
= 816 + 1050 = 1866

165. Ans. (D)


Sum = 850 × 9/17 + 980 × 9/14 = 450 + 630 = 1080

https :
//www. https : https
https : //instagra
//youtube.c
facebo m.com/aas :
om/channe hisharoraso
l/UCYa4_Jr ok.com cial(?) //t.m
Orf8R5Kz2u
/aashis utm_mediu
e/stu

354
OtccXQ m=
haroras copy_link
ocial dified
(166 - 170)
(2x - 2) + (x + 5) + (x + 7) + (2x + 2) + (x + 4) = 100
7x + 16 = 100
7x = 84
x = 12

Day➔ Monday Tuesday Wednesday Thursday Friday Total


Data Cable sold 88 68 76 104 64 400
166. Ans. (D)
Average = (88 + 68 + 76 + 104) /4 = 336/4 = 84

167. Ans. (B)


C - Type Data Cables sold on Thursday & Friday = 104 × 8/13 + 64 × 5/8 = 64
+ 40 = 104

168. Ans. (B)


Data Cables sold by Tarun on Monday & Tuesday
= 88 × 13/11 + 68 × 1.25 = 104 + 85 = 189

169. Ans. (A)


Required % = (104 - 76) /76 × 100 = 36.84%

170. Ans. (D)


Required % = 64/(76 + 104) × 100 = 64/180 × 100 = 35.55%

(171-175)
Month➔ January February March April May Total
2021 Comic 156 168 170 210 192 896
2022 Comic 198 207 210 230 215 1060
2021 Story 228 197 225 195 165 1010
2022 Story 208 287 260 270 285 1310
Total 790 859 865 905 857 4276

https :
//www. https : https
https : //instagra
//youtube.c
facebo m.com/aas :
om/channe hisharoraso
l/UCYa4_Jr ok.com cial(?) //t.m
Orf8R5Kz2u
/aashis utm_mediu
e/stu

355
OtccXQ m=
haroras copy_link
ocial dified
171. Ans. (C)
Sum = 168 × 5/8 + 207 × 4/9 = 105 + 92 = 197

172. Ans. (B)


Ratio = (168 + 192) : (165 + 285) = 360 : 450 = 4 : 5

173. Ans. (A)


Amount received from sell of Comic Books in May 2021 and Story Books in
May 2021
= 192 × 35 + 165 × 23 = 6720 + 3795 = Rs. 10515

174.Ans. (A)
Average of Story Books sold in 2021 = 1010/5 = 202

175. Ans. (D)


% of Story Books sold in March 2022 = 260/1310 × 100 = 19.84%

(176-200)
City➔ A B C D E Total
Male 1280 896 1080 1428 1225 5909
Female 830 1108 1040 712 1125 4815
Children 1090 796 880 1260 1150 5176
Total 3200 2800 3000 3400 3500 15900
176. Ans. (D)
Sum = 880 × 4/11 + 1260 × 7/18 = 320 + 490 = 810

177. Ans. (B)


Male & Females in city F & city G = (1080 + 1040) × 0.2 + (1280 + 830) × 0.3
= 2120 × 0.2 + 2110 × 0.3 = 424 + 633 = 1057

178. Ans. (B)


Average number of Female = 4815/5 = 963

179. Ans. (A)


Required % = (1428 - 1080) /1080× 100 = 32.22%

https :
//www. https : https
https : //instagra
//youtube.c
facebo m.com/aas :
om/channe hisharoraso
l/UCYa4_Jr ok.com cial(?) //t.m
Orf8R5Kz2u
/aashis utm_mediu
e/stu

356
OtccXQ m=
haroras copy_link
ocial dified
180. Ans. (D)
% of Children in city A = 1090/5176 × 100 = 21.05%

(181-185)

Company ➔ TATA FORCE FORD HONDA TOYOTA Total


2021 700 480 616 754 648 3198
2022 550 600 924 546 792 3412
Total 1250 1080 1540 1300 1440 6610
181. Ans. (C)
Cars sold by KIA in 2021 & 2022 = 480 × 1.3 + 600 × 1.18 = 624 + 708 = 1332

182. Ans. (B)


Cars sold by FORCE & FORD in 2020 = 480 × 5/6 + 616 × 9/11 = 400 + 504 =
904

183. Ans. (A)


Required % = 754/1540× 100 = 48.96%

184. Ans. (A)


Required % = (924 - 546) /546 × 100 = 69.23%

185. Ans. (D)


Ratio = 616 : 792 = 7 : 9

(186-190)

Month ➔ January February March April May Total


Peanut Oil 680 750 880 620 910 3840
Soyabean Oil 830 720 640 950 760 3900
Total 1510 1470 1520 1570 1670 7740

https :
//www. https : https
https : //instagra
//youtube.c
facebo m.com/aas :
om/channe hisharoraso
l/UCYa4_Jr ok.com cial(?) //t.m
Orf8R5Kz2u
/aashis utm_mediu
e/stu

357
OtccXQ m=
haroras copy_link
ocial dified
186. Ans. (D)
Average of Peanut oil produced = 3840/5 = 768

187. Ans. (B)


% of Soyabean oil produced in May = 760/3900 × 100 = 19.48%

188. Ans. (B)


Peanut oil &Soyabean oil produced in June = 910 × 14/13 + 760 × 1.25 = 980
+ 950 = 1930

189. Ans. (A)


Coconut oil produced in January & February = 680 × 0.75 + 750 × 0.8 = 510 +
600 = 1110

190. Ans. (D)


Required % = (910 - 720) /720 × 100 = 26.38%

(191-195)

Company ➔ LG Sony Panasonic Samsung Haier Total


A 152 160 87 130 147 676
B 114 96 116 104 105 535
C 114 144 147 126 168 699
Total 380 400 350 360 420 1910

191. Ans. (C)


MI LED TV sold by A & B = 147 × 9/7 + 105 × 11/7 = 189 + 165 = 354

192. Ans. (B)


LG & Sony LED TV sold by D = 114 × 7/6 + 144 × 1.125 = 133 + 162 = 295

193. Ans. (A)


Required % = (147 - 96) /96× 100 = 53.12%

https :
//www. https : https
https : //instagra
//youtube.c
facebo m.com/aas :
om/channe hisharoraso
l/UCYa4_Jr ok.com cial(?) //t.m
Orf8R5Kz2u
/aashis utm_mediu
e/stu

358
OtccXQ m=
haroras copy_link
ocial dified
194. Ans. (A)
% of Samsung LED TV sold by C = 126/699× 100 = 18.02%

195. Ans. (D)


Average of LED TV sold by B = 535/5 = 107

(196-200)

Day➔ Monday Tuesday Wednesday Thursday Friday Total


Within 3 KM 108 185 133 150 136 712
More than 3 KM 128 172 160 186 144 790
Total 236 357 293 336 280 1502

196. Ans. (D)


Food orders delivered by Swiggy on Thursday & Friday = 336 × 7/6 + 280 ×
8/7 = 392 + 320 = 712

197. Ans. (B)


Food orders delivered on Saturday = 136 × 1.125 + 144 × 13/12 = 153 + 156 =
309

198. Ans. (B)


Required % = (186 - 108) /108 × 100 = 72.22%

199. Ans. (A)


% of food orders delivered within 3 km on Thursday = 150/712 × 100 =
21.06%

200.Ans. (D)
Average of food orders delivered above 3 km = 790/5 = 158

https :
//www. https : https
https : //instagra
//youtube.c
facebo m.com/aas :
om/channe hisharoraso
l/UCYa4_Jr ok.com cial(?) //t.m
Orf8R5Kz2u
/aashis utm_mediu
e/stu

359
OtccXQ m=
haroras copy_link
ocial dified
(201 - 205)

Subject➔ Hindi English Math Science Computer Science Total


Priya 130 95 110 132 124 591
Jiya 128 138 120 135 144 665
Diya 140 117 108 115 90 570
Total 398 350 338 382 358 1826

201. Ans. (C)


% of marks scored by Priya = 591/(150 × 5) × 100 = 78.8%

202. Ans. (B)


Highest marks scored by = Jiya
Lowest marks scored by = Diya

203. Ans. (A)


Required % = (138 - 120) /120× 100 = 15%

204. Ans. (A)


Average of Marks scored by Diya = 570/5 = 114

205. Ans. (D)


Marks obtained in GK = 110 × 1.1 + 120 × 0.9 + 108 × 10/9 = 121 + 108 + 120 =
349

(206-210)
Shopkeeper➔ P Q R S T Total
Table Fan 360 350 350 400 360 1820
Ceiling Fan 216 300 250 320 320 1406
Total 576 650 600 720 680 3226

https :
//www. https : https
https : //instagra
//youtube.c
facebo m.com/aas :
om/channe hisharoraso
l/UCYa4_Jr ok.com cial(?) //t.m
Orf8R5Kz2u
/aashis utm_mediu
e/stu

360
OtccXQ m=
haroras copy_link
ocial dified
206. Ans. (D)
Average of Table fans sold = 1820/5 = 364

207. Ans. (B)


Ceiling fans sold by M = 360 × 11/9 + 320 × 1.125 = 440 + 360 = 800

208. Ans. (B)


Wall fans sold by Q & R = 300 × 7/6 + 250 × 6/5 = 350 + 300 = 650

209. Ans. (A)


Required % = (400 - 216) /400× 100 = 46%

210. Ans. (D)


Ratio = (350 + 400) : (300 + 250) = 750 : 550 = 15 : 11

(211-215)

City➔ P Q R S T Total
PhonePe 285 220 215 232 260 1212
Google Pay 240 290 252 270 265 1317
Total 525 510 467 502 525 2529
211. Ans. (C)
Google Pay &PhonePe users in city X = 285 × 1.4 + 240 × 1.125 = 399 + 270 =
669

212. Ans. (B)


% of PhonePe users in city Q = 220/1212 × 100 = 18.15%

213. Ans. (A)


Male Google Pay users in city R & S = 252 × 4/7 + 270 × 5/9 = 144 + 150 = 294

214. Ans. (A)


Paytm users in city Q & T = 220 × 13/11 + 260 × 15/13 = 260 + 300 = 560

215. Ans. (D)


Required % = (285 - 215) /215 × 100 = 32.55%

https :
//www. https : https
https : //instagra
//youtube.c
facebo m.com/aas :
om/channe hisharoraso
l/UCYa4_Jr ok.com cial(?) //t.m
Orf8R5Kz2u
/aashis utm_mediu
e/stu

361
OtccXQ m=
haroras copy_link
ocial dified
(216 - 230)

Month➔ January February March April May Total


A 540 480 510 470 430 2430
B 410 330 390 380 350 1860
Total 950 810 900 850 780 4290

216. Ans. (D)


Average of Bedsheets sold by A = 2430/5 = 486

217. Ans. (B)


% of Bedsheets sold by B in March = 390/1860 × 100 = 20.96% = 21%

218. Ans. (B)


In February, Cotton bedsheets sold by A = 480 × 5/8 = 300
Cotton bedsheets sold by B = 330 × 6/11 = 180
Difference = 300 - 180 = 120

219. Ans. (A)


Bedsheets sold in June = 430 × 1.2 + 350 × 8/7 = 516 + 400 = 916

220. Ans. (D)


Required % = (850 - 350) /850 × 100 = 58.82%

(221-225)
Shopkeeper➔ A B C D E Total
Balls 432 342 390 390 364 1918
Bats 288 258 260 390 336 1532
Total 720 600 650 780 700 3450

221. Ans. (C)


Items sold by F = 336 × 13/12 + 364 × 15/13 = 364 + 420 = 784

https :
//www. https : https
https : //instagra
//youtube.c
facebo m.com/aas :
om/channe hisharoraso
l/UCYa4_Jr ok.com cial(?) //t.m
Orf8R5Kz2u
/aashis utm_mediu
e/stu

362
OtccXQ m=
haroras copy_link
ocial dified
222.Ans. (B)
Sum = 390 × 6/13 + 432 × 5/9 = 180 + 240 = 420

223. Ans. (A)


Average = (432 + 342 + 390) /3 = 1164/3 = 388

224. Ans. (A)


% of Bats sold by C = 260/1532 × 100 = 16.97% = 17%

225. Ans. (D)


Required % = 260/780 × 100 = 33.33% = 33%

(226-230)
(x + 6) + (x + 9) + (x + 5) + 2x + (2x - 4) = 100
7x + 16 = 100
7x = 84
x = 12
City➔ P Q R S T Total
Dairy
288 336 272 384 320 1600
Booth

226. Ans. (D)


Dairy Booths in city A & B = 384 × 13/12 + 320 × 1.125 = 416 + 360 = 776

227. Ans. (B)


Average = (288 + 336 + 384) /3 = 1008/3 = 336

228. Ans. (B)


Required % = (384 - 272) /272 × 100 = 41.17%

229. Ans. (A)


Amul Dairy Booths in city Q & R = 336 × 3/8 + 272 × 8/17 = 126 + 128 = 254

230. Ans. (D)


Required % = 320/(336 + 384) × 100 = 320/720 × 100 = 44.44%

https :
//www. https : https
https : //instagra
//youtube.c
facebo m.com/aas :
om/channe hisharoraso
l/UCYa4_Jr ok.com cial(?) //t.m
Orf8R5Kz2u
/aashis utm_mediu
e/stu

363
OtccXQ m=
haroras copy_link
ocial dified
(231 - 235)

Month ➔ May June July August September Total


A 280 247 265 243 255 1290
B 232 258 272 278 280 1320
C 264 292 218 235 256 1265
Total 776 797 755 756 791 3875

231. Ans. (C)


Average of milk produced in city A = 1290/5 = 258

232. Ans. (B)


% of milk produced in city B = 232/1320 × 100 = 17.57%

233. Ans. (A)


Required % = (280 - 218) /280 × 100 = 22.14%

234. Ans. (A)


Milk produced in March = 280 × 8/7 + 232 × 1.125 + 264 × 13/12 = 320 + 261 +
286 = 867

235. Ans. (D)


Milk produced in ciy D in August & September = 235 × 1.2 + 256 × 1.125 =
282 + 288 = 570

(236-240)

Company➔ DAIKIN VOLTAS SAMSUNG LLOYD GODREJ Total


2 Ton 240 320 350 270 250 1430
1.5 Ton 400 400 420 360 350 1930
Total 640 720 770 630 600 3360
236. Ans. (D)
Average of 1.5 Ton AC sold = 1930/5 = 386

https :
//www. https : https
https : //instagra
//youtube.c
facebo m.com/aas :
om/channe hisharoraso
l/UCYa4_Jr ok.com cial(?) //t.m
Orf8R5Kz2u
/aashis utm_mediu
e/stu

364
OtccXQ m=
haroras copy_link
ocial dified
237. Ans. (B)
Required % = (420 - 250) /250 × 100 = 68%

238. Ans. (B)


AC sold by IFB = 350 × 9/7 + 250 × 7/5 = 450 + 350 = 800

239. Ans. (A)


Sum = 240 × 3/8 + 350 × 4/7 = 90 + 200 = 290

240. Ans. (D)


% of 2 Ton AC sold by DAIKIN = 240/1430 × 100 = 16.78%

(241-245)

Year➔ 2017 2018 2019 2020 2021 Total


TVS 216 260 240 280 378 1374
Honda 192 195 192 240 210 1029
Hero 272 245 288 130 252 1187
Total 680 700 720 650 840 3590

241. Ans. (C)


Average = (216 + 260 + 240 + 280) /4 = 996/4 = 249

242. Ans. (B)


% of Hero bikes sold in 2019 = 288/1187 × 100 = 24.26%

243. Ans. (B)


Required % = (378 - 210) /210× 100 = 80%

244. Ans. (A)


Bikes sold by Hero & Honda in 2022 = 210 × 1.1 + 252 × 8/7 = 231 + 288 = 519

245. Ans. (D)


Ratio = (260 + 240) : (240 + 210) = 500 : 450 = 10 : 9

https :
//www. https : https
https : //instagra
//youtube.c
facebo m.com/aas :
om/channe hisharoraso
l/UCYa4_Jr ok.com cial(?) //t.m
Orf8R5Kz2u
/aashis utm_mediu
e/stu

365
OtccXQ m=
haroras copy_link
ocial dified
(246 - 250)

Shopkeeper➔ A B C D E Total
January 580 950 590 900 880 3900
February 820 640 790 670 700 3620
March 770 680 840 630 720 3640
Total 2170 2270 2220 2200 2300 11160

246. Ans. (D)


Average of Jeans sold in March = 3640/5 = 728

247. Ans. (B)


In January, % of Jeans sold by A = 580/3900× 100 = 14.87%

248. Ans. (B)


Required % = (950 - 640) /640 × 100 = 48.43%

249. Ans. (A)


Boys Jeans sold by E in January & February = 880 × 6/11 + 700 × 9/14 = 480
+ 450 = 930

250. Ans. (D)


Jeans sold by A & B in April = 770 × 13/11 + 680 × 1.25 = 910 + 850 = 1760

(251-255)

Day➔ Monday Tuesday Wednesday Thursday Friday Total


Sold 35 25 35 30 50 175
Unsold 5 10 15 15 10 55
Total 40 35 50 45 60 230

https :
//www. https : https
https : //instagra
//youtube.c
facebo m.com/aas :
om/channe hisharoraso
l/UCYa4_Jr ok.com cial(?) //t.m
Orf8R5Kz2u
/aashis utm_mediu
e/stu

366
OtccXQ m=
haroras copy_link
ocial dified
251. Ans. (C)
Average number of sold power banks = 175/5 = 35

252. Ans. (A)


Power banks sold on Saturday & Sunday = 55 - 14 = 41

253. Ans. (B)


Ratio = (60 - 35) : (5 + 15 + 15) = 25 : 35 = 5 : 7

254. Ans. (C)


Power banks sold on Saturday = 55 × 7/11 = 35
Difference = 35 - 25 = 10

255. Ans. (D)


% of unsold Power banks on Wednesday = 15/50× 100 = 30%

(256-260)
Airlines➔ P Q R S T Total
Non - Stop Flight 120 180 90 60 150 600
Flight with stoppage 90 150 60 30 120 450
Total 210 330 150 90 270 1050

256. Ans. (C)


Ratio = 330 : 270 = 11 : 9

257. Ans. (B)


In Airline R, % of passengers who booked non - stop flights
= 90/150 × 100 = 60%

258. Ans. (A)


Male passengers who booked tickets in Airline Q = 180 × 7/12 + 150 × 3/5 =
105 + 90 = 195

259. Ans. (C)


In Airline T, Passengers who booked flights with exactly one stop

https :
//www. https : https
https : //instagra
//youtube.c
facebo m.com/aas :
om/channe hisharoraso
l/UCYa4_Jr ok.com cial(?) //t.m
Orf8R5Kz2u
/aashis utm_mediu
e/stu

367
OtccXQ m=
haroras copy_link
ocial dified
= 120 - 270 × 1/6 = 120 - 45 = 75

260. Ans. (B)


% of passengers who booked non - stop flights in airlines R & S
= (90 + 60) /(150 + 90) × 100 = 150/240 × 100 = 62.5%

(261-265)

Store➔ A B C D E Total
Hair Dryers 1300 1150 1600 1250 900 6200
Hair Straighteners 1200 850 750 1000 800 4600
Total 2500 2000 2350 2250 1700 10800

261. Ans. (C)


Average = (1600 + 1250 + 900) /3 = 3750/3 = 1250

262. Ans. (A)


Required % = (1150 - 800) /800 × 100 = 43.75% = 44%

263. Ans. (B)


Total Earning of C = 1600 × 350 + 750 × 600 = 560000 + 450000 = Rs. 1010000

264. Ans. (C)


Ratio = (1300 + 900) : (850 + 800) = 2200 : 1650 = 4 : 3

265. Ans. (D)


Difference = 2500 - 1000 = 1500

(266-270)

NGO ➔ A B C D E
Male 640 625 550 - -
Female 560 875 250 - -
Total 1200 1500 800 - -

https :
//www. https : https
https : //instagra
//youtube.c
facebo m.com/aas :
om/channe hisharoraso
l/UCYa4_Jr ok.com cial(?) //t.m
Orf8R5Kz2u
/aashis utm_mediu
e/stu

368
OtccXQ m=
haroras copy_link
ocial dified
266. Ans. (C)
Ratio = 560 : 800 = 7 : 10

267. Ans. (B)


In NGO D, Total People = 625 × 2.56 = 1600
Females in NGO D = 1600 × 3/8 = 600

268. Ans. (A)


Females in NGO E = (1500 - 500) × 7/10 = 700
Difference = 875 - 700 = 175

269. Ans. (C)


Sum = 1200 + 800 = 2000

270. Ans. (B)


Males in NGO D = 600 × 5/3 = 1000
Required % = (1000 - 640) /1000 × 100 = 36%

(271-275)

Scheme➔ SIP LIC Reliance Alliance ICICI Total


Raj 340 455 365 422 288 1870
Shekhar 285 345 380 295 425 1730
Jaya 278 285 365 393 385 1706
Total 903 1085 1110 1110 1098 5306

271. Ans. (C)


Average investment of Jaya = 170600/5 = 34120

272. Ans. (A)


Ratio = (455 + 365) : (295 + 425) = 820 : 720 = 41 : 36

273. Ans. (B)


Scheme which have same investment taken all 3 person together = Reliance
& Alliance

https :
//www. https : https
https : //instagra
//youtube.c
facebo m.com/aas :
om/channe hisharoraso
l/UCYa4_Jr ok.com cial(?) //t.m
Orf8R5Kz2u
/aashis utm_mediu
e/stu

369
OtccXQ m=
haroras copy_link
ocial dified
274. Ans. (C)
Interest received by Raj = 45500 × 0.05 + 28800 × 0.06 = 2275 + 1728 = Rs.
4003
Interest received by Jaya = 28500 × 0.05 + 38500 × 0.06 = 1425 + 2310 = Rs.
3735
Difference = 4003 - 3735 = 268

275. Ans. (D)


Required % = (340 + 422 + 365) /(345 + 295 + 425) × 100 = 1127/1065 × 100 =
105.8%

(276-280)

2015 2016 % increase in


Company
Male Female Total Male Female Total employees

P 1710 1140 2850 1200 1800 3000 5.26%


Q 1400 1000 2400 1800 900 2700 12.5%
R 1400 2100 3500 2000 2500 4500 28.57%
S 1950 1300 3250 2250 1800 4050 24.61%
T 1600 1400 3000 1500 2100 3600 20%
U 2000 1200 3200 2700 1800 4500 40.625%
Total 10060 8140 18200 11450 10900 22350

276. Ans. (C)


Average of employees recruited in 2016 = 22350/6 = 3725

277. Ans. (B)


Ratio = (1950 + 1600) : (2250 + 1500) = 3550 : 3750 = 71 : 75

278. Ans. (A)


Required % = (1800 + 2500) /(1140 + 2100) × 100 = 4300/3240 × 100 = 132.7%

https :
//www. https : https
https : //instagra
//youtube.c
facebo m.com/aas :
om/channe hisharoraso
l/UCYa4_Jr ok.com cial(?) //t.m
Orf8R5Kz2u
/aashis utm_mediu
e/stu

370
OtccXQ m=
haroras copy_link
ocial dified
279. Ans. (C)
Difference = 2250 × 1.12 - 1950 = 2520 - 1950 = 570

280. Ans. (B)


% increase employees recruited from 2015 to 2016 is 2nd highest in =
Company R

(281-285)

281. Ans. (C)


Ratio = 20 : 15 = 4 : 3

282. Ans. (A)


Let the total population of all 5 villages = 100x
Males in Dhampur = A = 100x × 0.30 × 3/5 = 18x
Males in Sadatnagar = B = (100x × 0.25) /(1 + 1.5) × 1.5 = 15x
A : B = 18x : 15x = 6 : 5
I) A : B = 165 : 110 = 6 : 5 (Possible)
II) A : B = 264 : 176 = 3 : 2(Not Possible)
III) A : B = 198 : 124 = 99 : 62 (Not Possible)
IV) A : B = 297 : 138 = 99 : 46 (Not Possible)
Only I

283. Ans. (B)


Required % = (15 - 10) /10 × 100 = 50%

284. Ans. (A)


Ratio = (30 - 10) : (25 - 20) = 20 : 5 = 4 : 1

285. Ans. (D)


Population of Dhampur = 300 × 30/25 = 360

(286-290)

https :
//www. https : https
https : //instagra
//youtube.c
facebo m.com/aas :
om/channe hisharoraso
l/UCYa4_Jr ok.com cial(?) //t.m
Orf8R5Kz2u
/aashis utm_mediu
e/stu

371
OtccXQ m=
haroras copy_link
ocial dified
Town ➔ A B C D E F Total
Urban 400 500 250 450 510 375 2485
Rural 350 300 400 300 340 225 1915
Literate 450 640 520 675 510 480 3275
Illiterate 300 160 130 75 340 120 1125
Total 750 800 650 750 850 600 4400

286. Ans. (A)


Ratio = (500 + 375) : (300 + 225) = 875 : 525 = 5 : 3

287. Ans. (B)


Literate population from town A & E = 450 + 510 = 960
Literate population from town C & D = 520 + 675 = 1195
Required % = (1195 - 960) /1195 × 100 = 19.66% = 20%

288. Ans. (C)


In Town D, Females = 750 × 0.6 = 450
Males = 750 × 0.4 = 300
Male lives in Rural Area = 300 - 450 × 0.2 = 300 - 90 = 210
% of Males live in Rural Area = 210/300 × 100 = 70%

289. Ans. (C)


Average of Urban Population = 2485/6 = 414.16 = 414

290. Ans. (B)


Urban Population from town A & B,
In 2016 = 400 + 500 = 900
In 2017 = 900 + 350 × 0.2 + 300 ×0.25 = 900 + 70 + 75 = 1045
% change in Urban population = (1045 - 900) /900 × 100 = 16%

https :
//www. https : https
https : //instagra
//youtube.c
facebo m.com/aas :
om/channe hisharoraso
l/UCYa4_Jr ok.com cial(?) //t.m
Orf8R5Kz2u
/aashis utm_mediu
e/stu

372
OtccXQ m=
haroras copy_link
ocial dified
(291 - 295)

Grade➔ 1st 2nd 3rd 4th 5th Total


Boys 720 540 585 630 765 3240
Girls 480 360 390 420 510 2160
Total 1200 900 975 1050 1275 5400

291. Ans. (C)


Students in 4th class = 1050

292. Ans. (A)


Ratio = 720 : 420 = 12 : 7

293. Ans. (B)


Total Boys present on Monday = 3240

294. Ans. (C)


Difference = 510 - 480 = 30

295. Ans. (D)


Average of Girls on Monday = 2160/5 = 432

(296 - 300)

Company A Company B
Department
Male Female Total Male Female Total
IT 120 100 220 100 80 180
HR 75 60 135 100 20 120
Finance 75 70 145 65 60 125
Admin 68 60 128 90 75 165
Marketing 96 66 162 80 60 140
Total 434 356 790 435 295 730

https :
//www. https : https
https : //instagra
//youtube.c
facebo m.com/aas :
om/channe hisharoraso
l/UCYa4_Jr ok.com cial(?) //t.m
Orf8R5Kz2u
/aashis utm_mediu
e/stu

373
OtccXQ m=
haroras copy_link
ocial dified
296. Ans. (C)
Average of employees in company B = 730/5 = 146

297. Ans. (B)


Male working in HR & Finance department in Company A = 75 + 75 = 150
Female working in Admin & Marketing department in Company B = 75 + 60 =
135
Required % = (150 - 135) /135 × 100 = 11.11%

298. Ans. (A)


Female working in IT, HR & Finance department in company A = 100 + 60 +
70 = 230
Male working in Finance, Admin & Marketing department in company B = 65
+ 90 + 80 = 235
Difference = 235 - 230 = 5

299. Ans. (C)


Ratio = (120 + 75 + 100 + 65) : (60 + 60 + 20 + 75) = 360 : 215 = 72 : 43

300. Ans. (B)


Employees working in Finance department in both companies = 145 + 125 =
270
Employees working in IT department in both companies = 220 + 180 = 400
Required % = 270/400 × 100 = 67.5%

https :
//www. https : https
https : //instagra
//youtube.c
facebo m.com/aas :
om/channe hisharoraso
l/UCYa4_Jr ok.com cial(?) //t.m
Orf8R5Kz2u
/aashis utm_mediu
e/stu

374
OtccXQ m=
haroras copy_link
ocial dified
https :
//instagram.com/aashisharorasocia
l(?) utm_medium = copy_link

https: //t.me/studified
https : //t.me/studified

https :
//youtube.com/channel/UCYa
4_JrOrf8R5Kz2uOtccXQ

https :
//www.facebook.com/aa
shisharorasocial

https :
//www. https : https
https : //instagra
//youtube.c
facebo m.com/aas :
om/channe hisharoraso
l/UCYa4_Jr ok.com cial(?) //t.m
Orf8R5Kz2u
/aashis utm_mediu
e/stu

375
OtccXQ m=
haroras copy_link
ocial dified
Direction: What will come in the place of the question mark ‘?’ in the
following question?
01. 33, ?, 176, 224, 259, 283.
(A) 113 (B) 112 (C) 111
(D) 117 (E) 115

02. 15, ? , 1279, 7679, 30719, 61439.


(A) 154 (B) 159 (C) 156
(D) 155 (E) Major

03. 10, 34, 66, 108, 162, ?


(A) 222 (B) 333 (C) 234
(D) 213 (E) 230

04. 80, 45, 50, 80, 165, ?


(A) 417.5 (B) 416.5 (C) 415.5
(D) 412.5 (E) 420.5

05. 100,152,382,1339, ?, 33153.25


(A) 6027.5 (B) 6022.5 (C) 6029.5
(D) 6037.5 (E) 6020.5

06. In the given question, two equations numbered l and II are given. Solve
both the equations and mark the appropriate answer.
I. x2 – 6x – 16 = 0
II. y2 + 6y + 8 = 0
(A) if x > y (B) if x < y (C) if x ≥ y
(D) if x ≤ y (E) if x = y or there is no relation between x and y

07. I. x2 + 2x - 15 = 0
II. y2 - 3y - 28 = 0
(A) if x > y (B) if x < y (C) if x ≥ y
(D) if x ≤ y (E) if x = y or there is no relation between x and y

08. I. x2 – 2x – 15 = 0\
II. y2 + 6y – 16 = 0

https://
www.fa https://inst
https
https://yout
ube.com/ch
cebook agram.com
/aashisharo
://t.
annel/UCYa
4_JrOrf8R5 .com/a rasocial(?)
utm_mediu
me/s
Kz2uOtccX
ashisha m= tudifi

376
Q
rorasoc copy_link
ial ed
(A) if x > y (B) if x < y (C) if x ≥ y
(D) if x ≤ y (E) if x = y or there is no relation between x and y

09. 864.02 ÷ 3.99 + 38.05 ÷ 18.98 × 110.01 = ?


(A) 444 (B) 111 (C) 131
(D) 436 (E) 123

10. (63.99) 1/3 × 124.989 + 407.05 ÷ 10.98 = ?


(A) 634 (B) 531 (C) 431
(D) 537 (E) 123

11. 39.87 + 56.83 – 19.11 × 3.99 + 7.84 = ?


(A) 22 (B) 29 (C) 28
(D) 24 (E) 21

12. Directions: Answer the questions based on the information given below.
The table given below shows the number of 'HR' employees, the ratio of the
number of 'HR' to 'Finance' employees and the difference between the
number of 'HR' and 'Finance' employees in four different companies - 'A', 'B',
'C' and 'D‘

Note: The number of 'HR' employees of company 'C' is more than the number
of 'Finance' employees of company 'C'.

If in company 'A', the number of female 'HR' employees is 40% of the total
number of 'HR' employees in the company and in the 'Finance' department,
the ratio of the number of male to female employees is 3 : 4, then the total
number of male employees (HR + Finance) in company 'A', are how much
more/less than the total number of employees (HR + Finance) in company 'D',
if the number of male to female employees in company 'D' is 3 : 1?
(A) 934 (B) 966 (C) 431
(D) 937 (E) 960

https://
www.fa https://inst
https
https://yout
ube.com/ch
cebook agram.com
/aashisharo
://t.
annel/UCYa
4_JrOrf8R5 .com/a rasocial(?)
utm_mediu
me/s
Kz2uOtccX
ashisha m= tudifi

377
Q
rorasoc copy_link
ial ed
13. 60% of total ‘HR’ employees in company ‘B’ are ‘males’ and the rest are
‘females’. Similarly, 75% of total ‘Finance’ employees in the company are
‘males’ and the rest are ‘females’. The total number of ‘female’ employees in
both departments is approximately what percentage of the total number of
‘male’ employees in company ‘B’?
(A) 41% (B) 46% (C) 42%
(D) 45% (E) 44%

14. In company 'C', if the ratio of 'P' and 'Q' is swapped and after swapping
the number of 'senior' employees in 'HR' department is 40% of the total
number of employees in 'HR' department and the number of the 'junior'
employees in 'Finance' department is 50% more than the number of 'senior'
employees in the 'Finance' department, then find the number of 'senior'
employees in company 'C'?
(A) 102 (B) 240 (C) 370
(D) 60 (E) 360

15. In company 'D', if the number of 'graduate' HR employees is 66.66% more


than the number of 'post graduate' HR employees and in the 'finance'
department, the number of 'graduate' finance employees is 40% less than
that of the 'post graduate' finance employees, then find the total number of
'post graduate' employees in company 'D'?
(A) 900 (B) 960 (C) 930
(D) 950 (E) 980

16. If X is divided in the ratio of 3 : Q and Q part of X is removed and added to


Y. What will be the number of HR in company C?
(A) 29 (B) 50 (C) 75
(D) 55 (E) 10

17. What is the difference between the average number of 'HR' employees in
company 'A', 'B', and 'D' together and the average number of 'finance'
employees in company 'A', 'B', and 'D' together?
(A) 20 (B) 50 (C) 30
(D) 55 (E) 10

(18 – 22) Directions: Answer the questions based on the information given
below. The first pie chart shows the percentage distribution of the total
number of vehicles manufactured(Buses +cars) by five different companies
and the second pie chart given below shows the percentage distribution of
the total number of buses manufactured by the same five companies. Total

https://
www.fa https://inst
https
https://yout
ube.com/ch
cebook agram.com
/aashisharo
://t.
annel/UCYa
4_JrOrf8R5 .com/a rasocial(?)
utm_mediu
me/s
Kz2uOtccX
ashisha m= tudifi

378
Q
rorasoc copy_link
ial ed
number of vehicles manufactured by all five companies together = 196000
and the total number of buses manufactured by all by all five companies
together = 90000

Note : (1) Total number of vehicles manufactured by a company= Number of


buses manufactured by the company + Number of cars manufactured by the
same company

18. The total number of cars manufactured by company 'A' and company 'B'
together is how much percent more/less than the number of buses
manufactured by company 'D' and company 'E' together?
(A) 1.25% (B) 5.65% (C) 1.51%
(D) 1.65% (E) 1.23%

https://
www.fa https://inst
https
https://yout
ube.com/ch
cebook agram.com
/aashisharo
://t.
annel/UCYa
4_JrOrf8R5 .com/a rasocial(?)
utm_mediu
me/s
Kz2uOtccX
ashisha m= tudifi

379
Q
rorasoc copy_link
ial ed
19. Out of the total buses manufactured by company 'E', the manufacturing
cost for each bus was Rs. 20000 and the manufacturing cost for each car was
Rs. 15000, then find the difference between the total manufacturing cost of
both types of vehicles? (where k = 1000)
(A) 256000k (B) 233000k (C) 237000k
(D) 273000k (E) 275000k

20. If another company 'Z' manufactured 20% more vehicles than that of
company 'D' and the ratio of the number of buses to cars manufactured by
company 'Z' is 2 : 3, then find the number of total cars manufactured by
company 'Z'?
(A) 35280 (B) 33222 (C) 34280
(D) 33270 (E) 34580

21. Out of the total cars manufactured by company 'D' and company 'E', 30%
and 40% of the respective 'car - parts' were damaged then find the total
number of damaged 'car - parts' for the number of cars manufactured by both
the companies.
(A) 20030 (B) 10020 (C) 101020
(D) 20020 (E) 10030

22. If another company 'Z' manufactured 60% of the total buses manufactured
by company 'B', then find the ratio of the total number of buses
manufactured by company 'Z' and the total number of buses manufactured
by company 'C'?
(A) 9:20 (B) 4:5 (C) 12:29
(D) 8:20 (E) 10:11

(23-27 Directions: Answer the questions based on the information given


below
There are three types of coins i.e. 'Gold', 'Silver', and 'Copper' in each of
three bags A, B, and C. The total number of coins in all the bags is 1510. The
number of coins in bag 'A' is 580 and the total number of coins in bag 'B' is
510. The ratio between the number of copper coins in bag 'A', the number of
copper coins in bag 'B', and the number of 'Gold' coins in bag 'C' is 3 : 2 : 4
respectively. The number of silver coins in bag 'B' is 60 more than the
number of copper coins in bag 'B'. The number of 'Gold' coins in bag 'B' is
25% less than the number of silver coins in bag 'A'. The total number of 'gold'
coins in all three bags is 570. The ratio of the total number of copper coins in
all three bags and the total number of silver coins in all three bags is 20 : 27.

https://
www.fa https://inst
https
https://yout
ube.com/ch
cebook agram.com
/aashisharo
://t.
annel/UCYa
4_JrOrf8R5 .com/a rasocial(?)
utm_mediu
me/s
Kz2uOtccX
ashisha m= tudifi

380
Q
rorasoc copy_link
ial ed
23. Which of the following statement/statements is correct?
(A) The number of copper coins in bag 'C' = The number of silver coins in bag
'B‘
(B) The number of 'Gold' coins in bag 'A' = The number of copper coins in
Bag 'B'
(C) The number of 'Gold' coins bag B' = The number of 'Gold' coins in bag 'C‘
(D) Both (1) and (2)
(E) 5. Both (2) and (3) .

24. Find the difference between the total number of 'gold' and 'copper' coins
in bag 'B'?
(A) 65 (B) 90 (C) 60
(D) 23 (E) 80

25. Find the ratio between the total number of silver coins in bag 'A' and the
total number of 'Gold' coins in bag 'C'?
(A) 6:5 (B) 7:6 (C) 1:5
(D) 2:5 (E) 6:7

26. The number of 'Gold' coins in bag 'A' is what percent more/less than the
number of 'copper' coins in bag 'C'?
(A) 20% (B) 50% (C) 30%
(D) 40% (E) 10%

27. Find the total number of silver coins in bag 'B' and bag 'C' together?
(A) 220 (B) 230 (C) 260
(D) 250 (E) 140

28. Riya spends 20% of her monthly salary on rent. From the remaining, she
spends Rs.5000 on flight ticket. Out of the remaining she invested the
amount in insurance premiums and savings in the ratio of 3: 2. If the amount
spent on rent is Rs. 4000 less than the amount invested in insurance
premiums, then find the monthly salary of Riya.
(A) Rs. 25000 (B) Rs. 20000 (C) Rs. 15000
(D) Rs. 55000 (E) Rs. 35000

https://
www.fa https://inst
https
https://yout
ube.com/ch
cebook agram.com
/aashisharo
://t.
annel/UCYa
4_JrOrf8R5 .com/a rasocial(?)
utm_mediu
me/s
Kz2uOtccX
ashisha m= tudifi

381
Q
rorasoc copy_link
ial ed
29. A box contains some balls. 5 blue balls, 6 black balls and 4 pink balls. If 2
balls are drawn random from the box. Find the probability that one balls is
black and other is pink
(A) 1/2 (B) 3/5 (C) 6/7
(D) 8 / 35 (E) 2/11

30. A sum of Rs. P is invested at compound interest at 20% for 3 years and
the amount received from it was Rs.3456. If Rs. 3P is invested at simple
interest at 15% for 12 years, then find the interest received.
(A) Rs. 10500 (B) Rs. 20800 (C) Rs. 10400
(D) Rs. 10800 (E) Rs. 10300

31. A mixture contains 60 litres of pure milk. 10 litres of milk are taken out
and 25 litres of water are added to it and after that 60% of the mixture is taken
out. Another mixture of 40 litres contains 60% milk and the rest syrup. If two
mixtures are mixed then find the final ratio of Milk, water and syrup.
(A) 22:5:8 (B) 5:6:7 (C) 1:5:9
(D) 6:1:25 (E)11:1:3

32. Train A starts from Mumbai at 5.00 AM for Delhi. Train B starts from Delhi
at 7.00 AM for Mumbai. Distance between Delhi and Mumbai is 720km. Speed
of train B is 60km/hr. Train A can cross a 150m pole in 5 sec. Find the time at
which both trains will meet?
(A) 10 A.M (B) 10 P.M (C) 9 A.M
(D) 12 A.M (E) 12 P.M

33. Marked price of watch is 150 percent of marked price of bag and cost
price of watch is equal to marked price of bag and watch is sold at 25% profit
and bag is sold for 25% loss and the discount given on watch and bag are
Rs.120 and Rs.180 respectively, then what is the cost price of bag?
(A) Rs. 300 (B) Rs. 200 (C) Rs. 340
(D) Rs. 400 (E) Rs. 500

34. Volume of a cone is 40 cubic metre more than volume of a cuboid. Radius
and height of cone are 7 and 12 metres respectively and the length of the
cuboid is equal to the height of the cone and the breadth and height of the
cuboid are R and (R-2). Find the breadth of cuboid?
(A) 1 (B) 3 (C) 7
(D) 8 (E) 11

https://
www.fa https://inst
https
https://yout
ube.com/ch
cebook agram.com
/aashisharo
://t.
annel/UCYa
4_JrOrf8R5 .com/a rasocial(?)
utm_mediu
me/s
Kz2uOtccX
ashisha m= tudifi

382
Q
rorasoc copy_link
ial ed
35. In the question, two quantities I and II are given. You have to solve both
the quantities and establish the correct relation between Quantity - I and
Quantity - II and choose the correct option accordingly.
Area of a rhombus is 240 cm2 and one of the diagonals of the rhombus is 24
cm.
Quantity I : If the area of a rectangle is 20 cm2 more than the area of the
rhombus and the length of the rectangle is equal to one of the diagonals of
the rhombus, then find the breadth of the rectangle. Quantity II : Find the side
of the rhombus
(A) Quantity I > Quantity II
(B) Quantity I < Quantity II
(C) Quantity I ≥ Quantity II
(D) Quantity I ≤ Quantity II
(E) Quantity I = Quantity II⇒

36. In the question, two quantities I and II are given. You have to solve both
the quantities and establish the correct relation between Quantity - I and
Quantity - II and choose the correct option accordingly.
'M', 'N', and 'P' are three two-digit natural numbers where the unit digit of
each number is 'odd' and the ten's digit is 'even'. 'M' is the largest possible
number and 'P' is the smallest possible number.
Quantity I : If all the numbers are divided by '2', then find the sum of
reminders.
Quantity II : If the average of 'M', 'N', and 'P' is 51, then find the sum of digits
of 'N‘
A) Quantity I > Quantity II
(B) Quantity I < Quantity II
(C) Quantity I ≥ Quantity II
(D) Quantity I ≤ Quantity II
(E) Quantity I = Quantity II

37. The question consists of two statements numbered “I” and “II” given
below it. You have to decide whether the data provided in the statements are
sufficient to answer the question or not and choose the correct option
accordingly.
'A', 'B' and 'C' started a partnership with Rs. (x - 500), Rs. (x + 100) and Rs. (x
+ 700) respectively. Find the value of 'x', if the investment time is equal for all
three partners.
Statment (I) : The ratio between the amount invested by 'A' and 'C' was 5 : 9.
Statment (II) : If the total profit was 80% more than the value of 'x' then the
profit share of 'C' was Rs. (x - 1400).

https://
www.fa https://inst
https
https://yout
ube.com/ch
cebook agram.com
/aashisharo
://t.
annel/UCYa
4_JrOrf8R5 .com/a rasocial(?)
utm_mediu
me/s
Kz2uOtccX
ashisha m= tudifi

383
Q
rorasoc copy_link
ial ed
(A) The data in statement I alone is sufficient to answer, while the data in
statement II alone is not sufficient to answer the question.
(B) The data in statement II alone is sufficient to answer, while the data in
statement I alone is not sufficient to answer the question.
(C) The data either in statement I alone or in statement II alone are sufficient
to answer the question.
(D) The data given in both statements I and II together are not sufficient to
answer the question.
(E)The data in both statements I and II together are necessary to answer the
question

238. The question consists of two statements numbered “I” and “II” given
below it. You have to decide whether the data provided in the statements are
sufficient to answer the question or not and choose the correct option
accordingly.
The ratio of the cost prices of articles 'A', 'B' and 'C' is 8 : 5 : 3.The
percentage by which article 'A', article 'B' and article 'C' are marked up is
80%, 60%, and 40% respectively. Find the cost price of article 'C' ? Statement
(I) : The ratio of the selling prices of article 'A', article 'B', and article 'C' is 10 :
7 : 13.
Statement (II) : Total profit earned on the three articles is Rs. 120 less than
the total discount given on the three articles together

(A) The data in statement I alone is sufficient to answer, while the data in
statement II alone is not sufficient to answer the question.
(B) The data in statement II alone is sufficient to answer, while the data in
statement I alone is not sufficient to answer the question.
(C) The data either in statement I alone or in statement II alone are sufficient
to answer the question.
(D) The data given in both statements I and II together are not sufficient to
answer the question.
(E) The data in both statements I and II together are necessary to answer the
question.

239. The question consists of two statements numbered “I” and “II” given
below it. You have to decide whether the data provided in the statements
are sufficient to answer the question or not and choose the correct option
accordingly.
A path of 'x' meter width is carved around a rectangular field. If the length of
the rectangular field is 3m more than the breadth of the field, then find the
value of 'x'?.

https://
www.fa https://inst
https
https://yout
ube.com/ch
cebook agram.com
/aashisharo
://t.
annel/UCYa
4_JrOrf8R5 .com/a rasocial(?)
utm_mediu
me/s
Kz2uOtccX
ashisha m= tudifi

384
Q
rorasoc copy_link
ial ed
Statement (I) : There is a 'square' whose side is 2m more than the breadth of
the rectangular field and the area of the square is 50 m2 more than the area of
the rectangular field. Statement (II) : Total cost of construction of a path
around the rectangular field is Rs. 1920 and the rate of the construction of
the path is Rs.40/m2.
(A) The data in statement I alone is sufficient to answer, while the data in
statement II alone is not sufficient to answer the question.
(B) The data in statement II alone is sufficient to answer, while the data in
statement I alone is not sufficient to answer the question.
(C) The data either in statement I alone or in statement II alone are sufficient
to answer the question.
(D) The data given in both statements I and II together are not sufficient to
answer the question.
(E) The data in both statements I and II together are necessary to answer the
question.

240. A boat has speed a of 15 m/s in still water, but while going upstream its
speed is half the speed as compared to the speed of the boat going
downstream. The boat is going downstream and a ship while going
downstream crosses the boat in 15 seconds. The length of the ship is
double the length of the boat. Find the time the ship will take to cover a
distance of 144 kilometers while going upstream if the length of the ship
is 100 meters.
(A) 2 hours (B) 3 hours (C) 1.6 hours
(D) 1 hours (E) 1.33 hours

https://
www.fa https://inst
https
https://yout
ube.com/ch
cebook agram.com
/aashisharo
://t.
annel/UCYa
4_JrOrf8R5 .com/a rasocial(?)
utm_mediu
me/s
Kz2uOtccX
ashisha m= tudifi

385
Q
rorasoc copy_link
ial ed
https://instagram.com/aashisharor
asocial(?) utm_medium = copy_link

https://t.me/studified
https://t.me/studified

https://youtube.com/channel/
UCYa4_JrOrf8R5Kz2uOtccXQ

https://www.facebook.c
om/aashisharorasocial

https://
www.fa https://inst
https
https://yout
ube.com/ch
cebook agram.com
/aashisharo
://t.
annel/UCYa
4_JrOrf8R5 .com/a rasocial(?)
utm_mediu
me/s
Kz2uOtccX
ashisha m= tudifi

386
Q
rorasoc copy_link
ial ed

You might also like